You are on page 1of 267

Tailieumontoan.

com


Điện thoại (Zalo) 039.373.2038

30 ĐỀ TOÁN
LUYỆN THI VÀO LỚP 10 CHUYÊN

Tài liệu sưu tầm, ngày 8 tháng 12 năm 2020


1
Website:tailieumontoan.com

TUYỂN TẬP 30 ĐỀ THI

VÀO LỚP 10 CHUYÊN MÔN TOÁN

LỜI NÓI ĐẦU

Nhằm đáp ứng nhu cầu về của giáo viên toán THCS và học sinh về các chuyên đề toán

THCS, website tailieumontoan.com giới thiệu đến thầy cô và các em tuyển tập 30 đề thi vào lớp 10

chuyên môn toán có đáp án chi tiết, công phu. Chúng tôi đã sưu tầm được một tập đề thi lẻ có đáp

án rất hay không rõ tác giả muốn tập hợp lại để các thầy cô và học sinh dễ dùng.

Các vị phụ huynh và các thầy cô dạy toán có thể dùng có thể dùng chuyên đề này để giúp
con em mình học tập. Hy vọng tuyển tập đề thi chuyên toán này thể giúp ích nhiều cho học sinh
phát huy nội lực giải toán nói riêng và học toán nói chung.
Chúc các thầy, cô giáo và các em học sinh thu được kết quả cao nhất từ chuyên đề này!

Liên hệ tài liệu word toán sđt và zalo: 039.373.2038 TÀI LIỆU TOÁN HỌC
2
Website:tailieumontoan.com

SỞ GIÁO DỤC VÀ ĐÀO TẠO KÌ THI TUYỂN SINH VÀO LỚP 10


ĐẠI HỌC QUỐC GIA HÀ NỘI NĂM HỌC 2011-2012
MÔN THI: TOÁN (Vòng 1)
ĐỀ THI CHÍNH THỨC Thời gian: 120 phút ( Không kể thời gian giao đề)

Đề số 1

Câu 1 (3,0 điểm).


1). Giải hệ phương trình

  x  1 y  x  y  3
 2

 .

 ( y  2) x 2
 y  x  1

2). Giải phương trình

3 x2  7
x  .
x 2  x  1

Câu 2. (3,0 điểm).


1). Chứng minh rằng không tồn tại các bộ ba số nguyên  x; y; z thỏa mãn đẳng thức:

x4  y 4  7 z4  5 .

2). Tìm tất cả các cặp số nguyên  x; y thỏa mãn đẳng thức
4 4
 x  1   x  1   y 3 .
  90 . Đường phân giác của góc
Câu 3. (3,0 điểm). Cho hình bình hành ABCD với BAD

BCD cắt đường tròn ngoại tiếp tam giác BCD tại O khác C . Kẻ đường thẳng d đi qua
A và vuông góc với CO . Đường thẳng d lần lượt cắt các đường thẳng CB; CD tại E; F .

1). Chứng minh rằng OBE  ODC .


2). Chứng minh rằng O là tâm đường tròn ngoại tiếp tam giác CEF .
3). Gọi giao điểm của OC và BD là I , chứng minh rằng
IB.BE.EI  ID.DF.FI .
Câu 4. (1,0 điểm). Với x; y là những số thực dương, tìm giá trị nhỏ nhất của biểu thức

x3 4y3
P  .
x3  8 y 3 y 3   x  y
3

______________________Hết____________________

Liên hệ tài liệu word toán sđt và zalo: 039.373.2038 TÀI LIỆU TOÁN HỌC
3
Website:tailieumontoan.com

SỞ GIÁO DỤC VÀ ĐÀO TẠO KÌ THI TUYỂN SINH VÀO LỚP 10


ĐẠI HỌC QUỐC GIA HÀ NỘI NĂM HỌC 2012-2013
MÔN THI: TOÁN (Vòng 1)
ĐỀ THI CHÍNH THỨC Thời gian: 120 phút ( Không kể thời gian giao đề)

Đề số 2

Câu 1. (3,0 điểm).

1). Giải phương trình: x  9  2012 x  6  2012  x  9x  6 .



x 2  y 2  2 y  4
2). Giải hệ phương trình  .

2 x  y  xy  4

Câu 2. (3,0 điểm).

1). Tìm tất cả các cặp số nguyên  x; y thỏa mãn đẳng thức

x  y  1xy  x  y  5  2 x  y .
2). Giả sử  x; y là các số thực dương thỏa mãn điều kiện  x 1 
y  1  4 . Tìm giá trị

nhỏ nhất của biểu thức

x2 y 2
P  .
y x

Câu 3. (3,0 điểm). Cho tam giác nhọn ABC nội tiếp đường tròn tâm O . Gọi M là một điểm
 ( M khác B; C và AM không đi qua O ). Giả sử P là một điểm thuộc đoạn
trên cung nhỏ BC

 tại điểm N khác M .


thẳng AM sao cho đường tròn đường kính MP cắt cung nhỏ BC

1). Gọi D là điểm đối xứng với điểm M qua O . Chứng minh rằng ba điểm N ; P; D thẳng

hàng.

2). Đường tròn đường kính MP cắt MD tại điểm Q khác M . Chứng minh rằng P là tâm

đường tròn nội tiếp tam giác AQN .

Câu 4. (1,0 điểm). Giả sử a; b; c là các số thực dương thỏa mãn: a  b  3  c ; c  b  1 ;

2 ab  a  b  c ab  1
a  b  c . Tìm giá trị nhỏ nhất của biểu thức: Q .
a  1b  1c  1

______________________Hết____________________
Liên hệ tài liệu word toán sđt và zalo: 039.373.2038 TÀI LIỆU TOÁN HỌC
4
Website:tailieumontoan.com

SỞ GIÁO DỤC VÀ ĐÀO TẠO KÌ THI TUYỂN SINH VÀO LỚP 10


ĐẠI HỌC QUỐC GIA HÀ NỘI NĂM HỌC 2013-2014
MÔN THI: TOÁN (Vòng 1)
ĐỀ THI CHÍNH THỨC Thời gian: 120 phút ( Không kể thời gian giao đề)

Đề số 3

Câu 1. (3,0 điểm).


1). Giải phương trình
3x  1  2  x  3 .
2). Giải hệ phương trình

 1 1 9

 xy  

 x y 2
 .

 1 3  1  1

   x    xy 


 4 2  y  xy

Câu 2. (3,0 điểm).


1). Cho các số thực a; b; c  0 thỏa mãn a  bb  cc  a  8 abc . Chứng minh rằng

a b c 3 ab bc ca
      .
a  b b  c c  a 4 a  bb  c b  cc  a c  aa  b

2). Có bao nhiêu số nguyên dương có 5 chữ số abcde sao cho abc  10d  e chia hết cho
101 ?
Câu 3. (3,0 điểm). Cho tam giác nhọn ABC nội tiếp đường tròn (O) với AB  AC. Đường
 cắt (O) tại điểm D khác A. Gọi M là trung điểm của AD và E là
phân giác của góc BAC
điểm đối xứng với D qua tâm O . Giả sử đường tròn ngoại tiếp tam giác ABM cắt đoạn
thẳng AC tại điểm F khác A.
1). Chứng minh rằng tam giác BDM và tam giác BCF đồng dạng.
2). Chứng minh rằng EF vuông góc với AC.
Câu 4. (1,0 điểm). Giả sử a; b; c; d là các số thực dương thỏa mãn điều kiện
abc  bcd  cda  dab  1. Tìm giá trị nhỏ nhất của biểu thức
P  4  a 3  b 3  c 3   9d 3 .

______________________Hết____________________

Liên hệ tài liệu word toán sđt và zalo: 039.373.2038 TÀI LIỆU TOÁN HỌC
5
Website:tailieumontoan.com

SỞ GIÁO DỤC VÀ ĐÀO TẠO KÌ THI TUYỂN SINH VÀO LỚP 10


ĐẠI HỌC QUỐC GIA HÀ NỘI NĂM HỌC 2014-2015
MÔN THI: TOÁN (Vòng 1)
ĐỀ THI CHÍNH THỨC Thời gian: 120 phút ( Không kể thời gian giao đề)

Đề số 4

Câu 1. (3,0 điểm).


1). Giải phương trình
 
 
1  x  1  x 2  2 1  x 2   8 .
 

2). Giải hệ phương trình



 2 2
x  xy  y  1
 2 .

 x  xy  2 y 2
 4


Câu 2. (3,0 điểm).
1). Giả sử x; y; z là các số thực dương thỏa mãn điều kiện x  y  z  xyz . Chứng minh rằng

x 2y 3z xyz 5 x  4 y  3z
   .
1  x2 1  y2 1  z2  x  y y  z z  x
2). Tìm nghiệm nguyên của phương trình
x 2 y 2  x  y   x  y  3  xy .

Câu 3. (3,0 điểm). Cho tam giác ABC nhọn với AB  BC và D là điểm thuộc cạnh BC sao
 . Đường thẳng qua C và song song với AD , cắt trung trực
cho AD là phân giác của BAC
của AC tại E . Đường thẳng qua B song song với AD , cắt trung trực của AB tại F .
1). Chứng minh rằng tam giác ABF đồng dạng với tam giác ACE .
2). Chứng minh rằng các đường thẳng BE; CF ; AD đồng quy tại một điểm, gọi điểm đó là
G.
3). Đường thẳng qua G song song với AE cắt đường thẳng BF tại Q . Đường thẳng QE ,
cắt đường tròn ngoại tiếp tam giác GEC tại P khác E . Chứng minh rằng các điểm
A; P; G; Q; F cùng thuộc một đường tròn.

Câu 4. (1,0 điểm). Giả sử a; b; c là các số thực dương thỏa mãn đẳng thức ab  bc  ca  1 .
5
Chứng minh rằng : 2abc a  b  c    a4 b2  b4 c 2  c 4 a2 .
9

______________________Hết____________________

Liên hệ tài liệu word toán sđt và zalo: 039.373.2038 TÀI LIỆU TOÁN HỌC
6
Website:tailieumontoan.com

SỞ GIÁO DỤC VÀ ĐÀO TẠO KÌ THI TUYỂN SINH VÀO LỚP 10


ĐẠI HỌC QUỐC GIA HÀ NỘI NĂM HỌC 2010-2011
MÔN THI: TOÁN (Vòng 1)
ĐỀ THI CHÍNH THỨC Thời gian: 120 phút ( Không kể thời gian giao đề)

Đề số 5
(Đề dự bị )

Câu 1. (3,0 điểm).


1). Với a; b; c là những số thực thỏa mãn điều kiện ab  bc  ca  abc  a  b  c ;
3  ab  2 a  b ; 3  bc  2b  c ; 3  ca  2c  a . Chứng minh rằng
1 1 1
   1.
3  ab  2 a  b 3  bc  2b  c 3  ac  2c  a
2). Giải hệ phương trình
xy  x  y  2
 .
 3
x  y 3  6  8 x 2 y 2

Câu 2. (3,0 điểm).
1). Với mỗi số thực a ta ký hiệu  a là số nguyên lớn nhất không vượt quá a giải phương
trình
 2  1
2  3x   x     x     x  1 .
 3   3 

2). Chứng minh rằng từ 52 số nguyên bất kỳ luôn có thể chọn ra được hai số có tổng hoặc
hiệu chia hết cho 100 .
Câu 3. (3,0 điểm). Cho tam giác nhọn ABC , đường cao AH , H thuộc BC . P thuộc AB
 . Giao điểm của CP và AH là Q . Trung trực của PQ cắt
sao cho CP là phân giác góc BCA
AH và BC lần lượt tại E; F .
1). PE giao AC tại K . Chứng minh rằng PK vuông góc AC .
2). FQ giao CE , CA lần lượt tại M ; N . Chứng minh rằng bốn điểm E; K ; N ; M thuộc một
đường tròn.
3). Chứng minh rằng bốn điểm P; E; C ; F thuộc một đường tròn.
Câu 4. (1,0 điểm). Giả sử 0  a; b; c  1 . Chứng minh rằng
a b  c  b c  a c a  b 6
   .
bc 1  a ca 1  b ab 1  c 1  3 abc

______________________Hết____________________

Liên hệ tài liệu word toán sđt và zalo: 039.373.2038 TÀI LIỆU TOÁN HỌC
7
Website:tailieumontoan.com

SỞ GIÁO DỤC VÀ ĐÀO TẠO KÌ THI TUYỂN SINH VÀO LỚP 10


ĐẠI HỌC QUỐC GIA HÀ NỘI NĂM HỌC 2011-2012
MÔN THI: TOÁN (Vòng 1)
ĐỀ THI CHÍNH THỨC Thời gian: 120 phút ( Không kể thời gian giao đề)

Đề số 6
(Đề dự bị )

Câu 1. (3,0 điểm).

1). Giải phương trình

x8 2 2x 1.

2). Giải hệ phương trình


x  3 y  xy  3
 .
 2
x  y 2  xy  3

Câu 2. (3,0 điểm).

1). Với x; y là các số nguyên, chứng minh x 5 y  xy 5 chia hết cho 30.

2). Giả sử a; b là các số thực dương thỏa mãn điều kiện a  b  2 . Tìm giá trị nhỏ nhất của
a3 b3
biểu thức P  2
 2
.
b  1 a  1
Câu 3. (3,0 điểm). Cho tam giác ABC vuông tại A . Gọi CT là đường phân giác trong của

tam giác ( T thuộc cạnh AB ).

1). Chứng minh rằng đường tròn ( K ) đi qua C ; T và tiếp xúc với AB có tâm K thuộc BC .

2). Gọi giao điểm của AC và ( K ) là D khác C , giao điểm của DB và ( K ) là E khác D .
  BCE
Chứng minh rằng ABD .

3). Gọi giao điểm của CE và AB là M . Chứng minh rằng M là trung điểm của đoạn thẳng

BT .

Câu 4. (1,0 điểm). Giả sử a; b; c là các số thực dương. Chứng minh rằng

1  a2  1  b2  1  c 2  a  b  b  c  c  a .

______________________Hết____________________

Liên hệ tài liệu word toán sđt và zalo: 039.373.2038 TÀI LIỆU TOÁN HỌC
8
Website:tailieumontoan.com

SỞ GIÁO DỤC VÀ ĐÀO TẠO KÌ THI TUYỂN SINH VÀO LỚP 10


ĐẠI HỌC QUỐC GIA HÀ NỘI NĂM HỌC 2012-2013
MÔN THI: TOÁN (Vòng 1)
ĐỀ THI CHÍNH THỨC Thời gian: 120 phút ( Không kể thời gian giao đề)

Đề số 7
(Đề dự bị )

Câu 1. (3,0 điểm).


1). Giải phương trình
x  2  x  2  2 x 2  4  2  3  x .
2). Giải hệ phương trình
x 2  xy  y 2  1

 2
x  2 xy  y 2  3 x  y  2.

Câu 2. (3,0 điểm).

1). Tìm tất cả các cặp số nguyên  x; y thỏa mãn

x 2  xy  y 2  x 2 y 2  5 .

2). Với x; y; z là các số thực thỏa mãn x  y  z  xy  yz  zx  6 . Tìm giá trị nhỏ nhất của

biểu thức

P  4  x4  4  y 4  4  z4 .

Câu 3. (3,0 điểm). Cho tam giác nhọn ABC nội tiếp đường tròn (O) . M ; N là hai điểm thuộc

 sao cho MN song song với AC và tia BM nằm giữa hai tia BA; BN . BM giao
cung nhỏ AC

 sao cho PQ vuông góc với BC . QN giao AC


AC tại P . Gọi Q là một điểm thuộc cung nhỏ BC

tại R.

1). Chứng minh rằng bốn điểm B; P; R; Q cùng thuộc một đường tròn.

2). Chứng minh rằng BR vuông góc với AQ.


  BPQ
3). Gọi F là giao của AQ và BN . Chứng minh rằng AFB   ABR
.

Câu 4. (1,0 điểm). Cho a; b; c  0 . Chứng minh rằng

11a 3  b3 11b3  c 3 11c 3  a 3


 2  2  2  a  b  c .
4 a 2  ab 4b  bc 4c  ca

……….HẾT……….

Liên hệ tài liệu word toán sđt và zalo: 039.373.2038 TÀI LIỆU TOÁN HỌC
9
Website:tailieumontoan.com

SỞ GIÁO DỤC VÀ ĐÀO TẠO KÌ THI TUYỂN SINH VÀO LỚP 10


ĐẠI HỌC QUỐC GIA HÀ NỘI NĂM HỌC 2013-2014
MÔN THI: TOÁN (Vòng 1)
ĐỀ THI CHÍNH THỨC Thời gian: 120 phút ( Không kể thời gian giao đề)

Đề số 8
(Đề dự bị )

Câu 1. (3,0 điểm).


1). Giải hệ phương trình

 x2  y 2  2


 3 .

 2 x   x  y  2  xy 


2). Với a; b; c là 3 số thực đôi một phân biệt, chứng minh rằng
2a  b2b  c 2b  c2c  a 2c  a2a  b 2a  b 2b  c 2c  a
3     
a  bb  c b  cc  a c  aa  b ab bc ca
Câu 2. (3,0 điểm).
1). Với mỗi số thực x ta định nghĩa  x là số nguyên lớn nhất không vượt quá x . Chứng
minh rằng với mọi số nguyên n ta luôn có
 n  1   n  2   2n  1 
    n.
 3   3   6 
2). Với a; b; c là các số thực dương thỏa mãn đẳng thức ab  bc  ca  abc . Tìm giá trị nhỏ
nhất của biểu thức
a2 b2 c2
M   .
b3 c3 a3
Câu 3. (3,0 điểm). Cho tam giác ABC nhọn nội tiếp đường tròn O . H là trực tâm của
tam giác ABC . AD là đường kính của O . E thuộc AC sao cho HE  BC .
1). Chứng minh rằng các đường thẳng BH và DE cắt nhau trên O.
2). Gọi F là giao điểm của các đường thẳng EH và AB. Chứng minh rằng A là tâm
đường tròn bàng tiếp ứng với đỉnh D của tam giác DEF.
3). Gọi I là tâm đường tròn nội tiếp của tam giác DEF . Chứng minh rằng BE, CF và IH
đồng quy.
Câu 4. (1,0 điểm). Với a; b; c là các số thực dương, chứng minh rằng

a 2  b2  3ab  b2  c 2  bc  a 2  c 2 .

……….HẾT……….

Liên hệ tài liệu word toán sđt và zalo: 039.373.2038 TÀI LIỆU TOÁN HỌC
10
Website:tailieumontoan.com

SỞ GIÁO DỤC VÀ ĐÀO TẠO KÌ THI TUYỂN SINH VÀO LỚP 10


ĐẠI HỌC QUỐC GIA HÀ NỘI NĂM HỌC 2011-2012
MÔN THI: TOÁN (Vòng 2)
ĐỀ THI CHÍNH THỨC Thời gian: 120 phút ( Không kể thời gian giao đề)

Đề số 9

Câu 1 (3,0 điểm).


1). Giải hệ phương trình

x  y  xy  3

 .

 2 xy  1  2  x 2 2
y  2


2). Giải phương trình
1 1
 4x  1   5x .
2x  1 x2

Câu 2 (3,0 điểm).


1). Một số có 10 chữ số được gọi là tốt nếu số đó chia hết cho 11111 và tất cả các chữ số
đều khác nhau. Hỏi có tất cả bao nhiêu số tốt.
2). Tìm giá trị nhỏ nhất của biểu thức
f  x; y ; z  4 x 2  2 y 2  z 2  4 xy  2 yz  6 y  12 x .


Câu 3 (3,0 điểm). Cho tam giác ABC nội tiếp đường tròn (O) . P di chuyển trên cung BC
chứa A của (O) . I là tâm đường tròn nội tiếp tam giác ABC . Q là tâm đường tròn nội
tiếp tam giác PBC .
1). Chứng minh rằng B; I ; Q; C cùng nằm trên một đường tròn.
2). Trên tia BQ; CQ lần lượt lấy các điểm M ; N sao cho BM  BI ; CN  CI . Chứng minh
rằng MN luôn đi qua một điểm cố định.
Câu 4 (1,0 điểm). Với 0  a; b; c  1 . Chứng minh rằng
1 1 1 3
   .
1  a  b 1  b  c 1  c  a 1  2 3 abc

……….HẾT……….

Liên hệ tài liệu word toán sđt và zalo: 039.373.2038 TÀI LIỆU TOÁN HỌC
11
Website:tailieumontoan.com

SỞ GIÁO DỤC VÀ ĐÀO TẠO KÌ THI TUYỂN SINH VÀO LỚP 10


ĐẠI HỌC QUỐC GIA HÀ NỘI NĂM HỌC 2012-2013
MÔN THI: TOÁN (Vòng 2)
ĐỀ THI CHÍNH THỨC Thời gian: 120 phút ( Không kể thời gian giao đề)

Đề số 10

Câu 1. (3,0 điểm).

1). Giải phương trình

x2  8  3 x3  8 .

2). Giải hệ phương trình


x 2  xy  y 2  1
 .
 2
x  2 xy  2 y 2  5x  y  3

Câu 2. (3,0 điểm).

1). Tìm các cặp số nguyên  x; y thỏa mãn

5x 2  13 y 2  6 xy  4 3x  y .

2). Giả sử a; b; c là 3 số dương có tổng là 3. Tìm giá trị nhỏ nhất của biểu thức

1 1 1 1
P    2 .
ab bc ca a  b2  c 2
  90 . Giả sử O là điểm nằm trong
Câu 3. (3,0 điểm). Cho hình bình hành ABCD có BAD

ABD sao cho OC không vuông góc với BD. Vẽ đường tròn tâm O đi qua C. BD cắt (O)

tại hai điểm M , N sao cho B nằm giữa M và D. Tiếp tuyến tại C của (O) cắt AD , AB lần

lượt tại P , Q.

1). Chứng minh rằng bốn điểm M ; N ; P; Q cùng thuộc một đường tròn.

2). CM cắt QN tại K , CN cắt PM tại L. Chứng minh rằng KL  OC.


Câu 4. (1,0 điểm). Tồn tại hay không 9 số nguyên a1 , a2 , , a9 sao cho tập các giá trị của

tổng ai  a j ( 1  i  j  9 ) có chứa 36 số nguyên liên tiếp? Giải thích tại sao.

……….HẾT……….

Liên hệ tài liệu word toán sđt và zalo: 039.373.2038 TÀI LIỆU TOÁN HỌC
12
Website:tailieumontoan.com

SỞ GIÁO DỤC VÀ ĐÀO TẠO KÌ THI TUYỂN SINH VÀO LỚP 10


ĐẠI HỌC QUỐC GIA HÀ NỘI NĂM HỌC 2013-2014
MÔN THI: TOÁN (Vòng 2)
ĐỀ THI CHÍNH THỨC Thời gian: 120 phút ( Không kể thời gian giao đề)

Đề số 11

Câu 1. (3,0 điểm).


1). Tìm tất cả các cặp số nguyên  x; y sao cho
x 3  y 3  6 xy  1  0.
2). Với x là số thực ta ký hiệu  x là các số nguyên lớn nhất không vượt quá x . Chứng
minh rằng nếu n là số nguyên dương lớn hơn 1 thì
 1    2    3      n2  1   1 nn  14n  1 .
       
  6
Câu 2. (3,0 điểm).
1). Giải hệ phương trình

 3 3 2
x  y  3 x  6 x  3 y  4
 2 .

 2 x  xy  y 2
 11


2). Giả sử  x; y là các số thực thỏa mãn
  
 x  3  x 2  y  3  y 2   9 .
  
Tìm giá trị nhỏ nhất của biểu thức P  x 2  xy  y 2 .
Câu 3. (3,0 điểm). Cho hình thang cân ABCD nội tiếp đường tròn (O) với AB song song
CD và AB  CD . M là trung điểm CD . P là điểm di chuyển trên đoạn MD ( P khác M , D
). AP cắt (O) tại Q khác A , BP cắt (O) tại R khác B , QR cắt CD tại E . Gọi F là điểm đối
xứng với P qua E.
1). Chứng minh rằng tâm đường tròn ngoại tiếp tam giác AQF luôn thuộc một đường
thẳng cố định khi P di chuyển.
2). Giả sử EA tiếp xúc (O). Chứng minh rằng khi đó QM vuông góc với CD.
Câu 4. (1,0 điểm). Cho một bảng ô vuông 2013  2014, mỗi ô vuông con có thể tô một trong
hai màu xanh hoặc đỏ. Biết rằng ban đầu tất cả các ô đều được tô màu xanh. Cho phép
mỗi lần ta chọn một hàng hoặc một cột và thay đổi màu của tất cả các ô thuộc hàng hoặc
cột đó. Hỏi sau một số hữu hạn lần đổi màu ta có thể thu được bảng gồm đúng 1000 ô
vuông con màu đỏ hay không?

……….HẾT……….

Liên hệ tài liệu word toán sđt và zalo: 039.373.2038 TÀI LIỆU TOÁN HỌC
13
Website:tailieumontoan.com

SỞ GIÁO DỤC VÀ ĐÀO TẠO KÌ THI TUYỂN SINH VÀO LỚP 10


ĐẠI HỌC QUỐC GIA HÀ NỘI NĂM HỌC 2014-2015
MÔN THI: TOÁN (Vòng 2)
ĐỀ THI CHÍNH THỨC Thời gian: 120 phút ( Không kể thời gian giao đề)

Đề số 12

Câu 1. (3,0 điểm).


1). Giải hệ phương trình

 2 2
x  y  xy  3
 2 .

2 x  3 xy  1  4 x


2). Giải phương trình
x  3  3x  1  x  1 .
Câu 2. (3,0 điểm).
1). Tìm các bộ ba số nguyên dương x; y; z thỏa mãn
x 3  y 3  z 3  2001 .
2). Với a; b; c là các số thực không âm, chứng minh rằng

a 2  2b 2 ab  b2  2c 2 bc  c 2  2 a 2 ca  2 a  b  c .

Câu 3. (3,0 điểm). Cho tam giác ABC nhọn không cân nội tiếp đường tròn (O) . D là
điểm thuộc cạnh BC ( D khác B và D khác C ). Trung trực của CA; AB lần lượt cắt đường
thẳng AD tại E; F . Đường thẳng qua E song song với AC cắt tiếp tuyến qua C của (O)
tại M. Đường thẳng qua qua F song song với AB cắt tiếp tuyến qua B của (O) tại N .
1). Chứng minh rằng đường thẳng MN tiếp xúc với (O) .
FN BN
2). Giả sử  . Chứng minh rằng AD là phân giác của tam giác ABC .
EM CM
Câu 4. (1,0 điểm). Trên bảng ô vuông kích thước 100 100 ta viết vào mỗi ô một số nguyên
dương một cách tùy ý sao cho hiệu 2 số ở hai ô kề nhau bất kỳ đều nhỏ hơn hoặc bằng 1
(hai ô được gọi là kề nhau nếu chúng có một cạnh chung). Chứng minh rằng tồn tại một số
nguyên dương k nào đó được viết vào ít nhất 51 ô.

………HẾT……….

Liên hệ tài liệu word toán sđt và zalo: 039.373.2038 TÀI LIỆU TOÁN HỌC
14
Website:tailieumontoan.com

SỞ GIÁO DỤC VÀ ĐÀO TẠO KÌ THI TUYỂN SINH VÀO LỚP 10 CHUYÊN
TỈNH AN GIANG NĂM HỌC 2014-2015
MÔN THI: TOÁN (Vòng 2)
ĐỀ THI CHÍNH THỨC Thời gian: 150 phút ( Không kể thời gian giao đề)

Đề số 13

Câu 1. (1,0 điểm). Cho A  x  2 3 x  3  x  2 3 x  3 . Tính A khi x  5 .


 1 5

 2x  
 y 2
Câu 2. (1,5 điểm). Giải hệ phương trình  .

 1

 2y   5

 x
1
Câu 3. (2,0 điểm). Chứng minh rằng với mọi số nguyên n  1 ta luôn có 2 n .
n1 n
1 1 1 1
Từ đó chứng minh    ...   2 n 2 .
1 2 3 n

Câu IV (1,5 điểm). Tìm a; b; c biết rằng phương trình x 3  ax 2  bx  c  0 có tập nghiệm là
S  1; 1 .

 bằng 60 nội tiếp trong


Câu 4. (3,0 điểm). Cho tam giác ABC có ba góc nhọn và góc A
đường tròn tâm O , bán kính R . Các đường cao BD và CE của tam giác cắt nhau tại H .
1). Chứng minh rằng AD.AC  AE.AB .
2). Chứng minh rằng BC  2.DE .
3). Kéo dài BH cắt đường tròn tâm O tại H  . Chứng minh H và H  đối xứng qua AC và
hai đường tròn ngoại tiếp hai tam giác AHC ; ABC có cùng bán kính.
Câu 5. (1,0 điểm).

Cầu Vàm Cống bắc ngang qua Sông Hậu nối liền hai tỉnh Cần Thơ và Đồng Tháp thiết kế
theo kiểu dây giăng như hình vẽ. Chiều cao từ sàn cầu đến đỉnh trụ đỡ AB  120 (m), dây
giăng AC  258 (m), chiều dài sàn cầu từ B đến C là 218 (m). Hỏi góc nghiêng của sàn
cầu BC so với mặt nằm ngang (giả thiết xem như trụ đỡ AB thẳng đứng).

……....HẾT……….
Liên hệ tài liệu word toán sđt và zalo: 039.373.2038 TÀI LIỆU TOÁN HỌC
15
Website:tailieumontoan.com

SỞ GIÁO DỤC VÀ ĐÀO TẠO KÌ THI TUYỂN SINH VÀO LỚP 10 CHUYÊN
ĐẠI HỌC SƯ PHẠM HÀ NỘI NĂM HỌC 2015-2016
MÔN THI: TOÁN (Vòng 1)
ĐỀ THI CHÍNH THỨC Thời gian: 150 phút ( Không kể thời gian giao đề)

Đề số 14

2
a b  
   1 1  1 
 b a  a b 
Câu 1. (2,5 điểm). Cho biểu thức P  với a  0; b  0; a  b .
a 2 b2  a b 
   
b2 a 2  b a 

1
1). Chứng minh P 
ab
2). Giả sử a; b thay đổi sao cho 4 a  b  ab  1 . Tìm giá trị nhỏ nhất của P .
Câu 2. (2,0 điểm). Cho hệ phương trình
x  my  2  4 m
 (với m là tham số).
mx  y  3m  1

1). Giải phương trình khi m  2 .


2). Chứng minh hệ luôn có nghiệm với mọi giá trị của m. giả sử  x0 ; y0  là một nghiệm của
hệ phương trình. Chứng minh đẳng thức
x0 2  y0 2  5 x0  y0   10  0 .

Câu 3. (1,5 điểm). Cho a; b là các số thực khác 0. Biết rằng phương trình
2 2
a  x  a  b  y  b  0 có nghiệm duy nhất. Chứng minh a  b .

 và góc ACB
Câu 4. (3,0 điểm). Cho tam giác ABC có các góc ABC  nhọn, góc BAC
  60 0 .

Các đường phân giác trong BB1 ; CC1 của tam giác ABC cắt nhau tại I .
1). Chứng minh tứ giác AB1 IC1 nội tiếp.

2). Gọi K là giao điểm thứ hai khác B của đường thẳng BC với đường tròn ngoại tiếp tam
giác BC1 I . Chứng minh tứ giác CKIB1 nội tiếp.
3). Chứng minh AK  B1C1
Câu 5. (1,0 điểm). Tìm số thực không âm a và b thỏa mãn
 2     
a  b  3 b2  a  3   2 a  1 2b  1  .
 4  4   
2  2 

……....HẾT……….

Liên hệ tài liệu word toán sđt và zalo: 039.373.2038 TÀI LIỆU TOÁN HỌC
16
Website:tailieumontoan.com

SỞ GIÁO DỤC VÀ ĐÀO TẠO KÌ THI TUYỂN SINH VÀO LỚP 10 CHUYÊN
ĐẠI HỌC SƯ PHẠM HÀ NỘI NĂM HỌC 2015-2016
MÔN THI: TOÁN (Vòng 2)
ĐỀ THI CHÍNH THỨC Thời gian: 150 phút ( Không kể thời gian giao đề)

Đề số 15

Câu 1. (2,5 điểm).


1). Cho a  0; a  1 . Rút gọn biểu thức
 a 1 
S  6  4 2 . 3 20  14 2  3 a  3 a  3a  1 :   1 .
 2( a  1) 
x y
2). Cho x; y thỏa mãn 0  x  1; 0  y  1 và  1.
1 x 1 y

Tính giá trị của biểu thức P  x  y  x 2  xy  y 2 .


Câu 2. (2,0 điểm). Một xe tải có chiều rộng là 2,4m và chiều cao là 2,5m muốn đi qua một
cái cổng có hình Parabol. Biết khoảng cách giữa hai chân cổng là 4m và khoảng từ đỉnh
cổng (đỉnh Parabol) tới chân cổng là 2 5m (bỏ qua độ dày của cổng).
1). Trong mặt phẳng tọa độ Oxy , gọi Parabol  P : y  ax 2 với a  0 là hình chiếu biểu diễn
cổng mà xe tải muốn đi qua. Chứng minh a  1
2). Hỏi xe tải có thể đi qua cổng được không? Tại sao?
Câu 3. (1,5 điểm). Cho hai số nguyên a; b thỏa mãn a 2  b2  1  2 ab  a  b . Chứng minh
a và b là hai số chính phương liên tiếp.
Câu 4. (3,0 điểm). Cho tam giác nhọn ABC ( AB  AC ), M là trung điểm của cạnh BC , O là
tâm của đường tròn ngoại tiếp tam giác. Các đường cao AD; BE; CF của tam giác ABC
đồng quy tại H . Các tiếp tuyến với (O) tại B và C cắt nhau tại S . Gọi X , Y lần lượt là giao
điểm của đường thẳng EF với các đường thẳng BS; AO . Chứng minh rằng

1). MX  BF .
2). Hai tam giác SMX và DHF đồng dạng.
EF BC
3).  .
FY CD
Câu 5. (1,0 điểm). Trong mặt phẳng tọa độ Oxy . Cho tam giác ABC có các đỉnh là các
điểm nguyên (một điểm gọi là điểm nguyên nếu hoành độ và tung độ của điểm đó là các
số nguyên). Chứng minh rằng hai lần diện tích của tam giác ABC là một số nguyên.

.....….....HẾT……….

Liên hệ tài liệu word toán sđt và zalo: 039.373.2038 TÀI LIỆU TOÁN HỌC
17
Website:tailieumontoan.com

SỞ GIÁO DỤC VÀ ĐÀO TẠO KÌ THI TUYỂN SINH VÀO LỚP 10 CHUYÊN
TỈNH PHÚ THỌ NĂM HỌC 2015-2016
MÔN THI: TOÁN (Chuyên Tin)
ĐỀ THI CHÍNH THỨC Thời gian: 150 phút ( Không kể thời gian giao đề)

Đề số 16

Câu 1. (2,0 điểm).


1). Giải phương trình x 2  3 x  2  0 .
x  y  z  1


2). Tìm các số thực x; y; z thỏa mãn:  y  z  x  3 .

z  x  y  5
Câu 2. (2,0 điểm).
1 1
1). Phép toán T được định nghĩa như sau aTb   với a và b là các số thực khác 0 tùy
a b
ý.
1 1 1
Thí dụ: 2 T 3    . Tính giá trị biểu thức P  5 T 6T 7 T 8 .
2 3 6
2). Cho a và b là các số thực thỏa mãn các điều kiện 6 a 2  20 a  15  0 ; 15b2  20b  6  0 ;
b3 6
ab  1 . Chứng minh rằng  .
ab2  9 ab  1
3
2015

Câu 3. (2,0 điểm).


1). Tìm tất cả các số tự nhiên n sao cho n  2015 và n  2199 đều là các số chính phương.
2). Bạn Nam viết một chương trình để máy tính in ra các số nguyên dương liên tiếp theo
thứ tự tăng dần từ 1 đến 1000 dưới dạng sau
12345678910111213141516...9989991000 .
Trong dãy số trên, tính từ trái qua phải, chữ số thứ 11 là chữ số 0, chữ số thứ 15 là chữ số
2. Hỏi chữ số thứ 2016 trong dãy số trên là chữ số nào?
Câu 4. (3,0 điểm). Cho hình vuông ABCD tâm O , M là điểm di động trên cạnh AB . Trên
cạnh AD lấy điểm E sao cho AM  AE , trên cạnh BC lấy điểm F sao cho BM  BF .
 , đường thẳng OB
1). Chứng minh rằng đường thẳng OA là phân giác trong của góc MOE
 . Từ đó suy ra ba điểm O; E; F thẳng hàng.
là phân giác trong của góc MOF
2). Gọi H là chân đường vuông góc kẻ từ M tới đường thẳng EF . Chứng minh bốn điểm
A; B; H ; O cùng nằm trên một đường tròn.
3). Chứng minh rằng khi điểm M di động trên cạnh AB thì đường thẳng MH luôn đi qua
một điểm cố định.
Câu 5. (1,0 điểm). Cho x là một số thực tùy ý. Tìm giá trị nhỏ nhất của hàm số:
f  x  x  1  2 x  2  3 x  3  4 x  4 .

……....HẾT……….

Liên hệ tài liệu word toán sđt và zalo: 039.373.2038 TÀI LIỆU TOÁN HỌC
18
Website:tailieumontoan.com

SỞ GIÁO DỤC VÀ ĐÀO TẠO KÌ THI TUYỂN SINH VÀO LỚP 10 CHUYÊN
ĐẠI HỌC QUỐC GIA HÀ NỘI NĂM HỌC 2011-2012
MÔN THI: TOÁN (Vòng 2)
ĐỀ THI CHÍNH THỨC Thời gian: 150 phút ( Không kể thời gian giao đề)

Đề số 17

Câu 1. (3,0 điểm).


1). Giải phương trình :  x3 x  
1 x  1  1 .

 2 2 2 2
2). Giải hệ phương trình: x  y  2 x y

.




 x  y1  xy  4 x 2 y 2

Câu 2. (3,0 điểm).


1). Với mỗi số thực a ta gọi phần nguyên của a là số nguyên lớn nhất không vượt quá a
và ký hiệu là  a . Chứng minh rằng với mọi số nguyên dương n , biểu thức
2
 1 1
n   3 n    không biểu diễn được dưới dạng lập phương của một số nguyên dương.
 27 3 

2). Với x; y; z là các số thực dương thỏa mãn đẳng thức xy  yz  zx  5 , Tìm giá trị nhỏ
3x  3 y  2 z
nhất của biểu thức : P  .
6  x 2  5  6  y 2  5  6  z 2  5

 và CDA
Câu 3. (3,0 điểm). Cho hình thang ABCD với BC song song AD . Các góc BAD  là

các góc nhọn. Hai đường chéo AC và BD cắt nhau tại I . P là điểm bất kỳ trên đoạn thẳng
BC ( P không trùng với B; C ). Giả sử đường tròn ngoại tiếp tam giác BIP cắt đoạn thẳng
PA tại M khác P và đường tròn ngoại tiếp tam giác CIP cắt đoạn thẳng PD tại N khác P
1). Chứng minh rằng năm điểm A; M ; I ; N ; D cùng nằm trên một đường tròn. Gọi đường
tròn này là ( K ) .
2). Giả sử các đường thẳng BM và CN cắt nhau tại Q , chứng minh rằng Q cũng nằm
trên đường tròn ( K ) .
PB BD
3). Trong trường hợp P; I ; Q thẳng hàng, chứng minh rằng  .
PC CA
Câu 4. (1,0 điểm). Giả sử A là một tập con của tập các số tự nhiên  . Tập A có phần tử
nhỏ nhất là 1, phần tử lớn nhất là 100 và mỗi x thuộc A ( x  1 ) luôn tồn tại a; b cũng
thuộc A sao cho x  a  b ( a có thể bằng b ). Hãy tìm một tập A có số phần tử nhỏ nhất.

……....HẾT……….
Liên hệ tài liệu word toán sđt và zalo: 039.373.2038 TÀI LIỆU TOÁN HỌC
19
Website:tailieumontoan.com

SỞ GIÁO DỤC VÀ ĐÀO TẠO KÌ THI TUYỂN SINH VÀO LỚP 10 CHUYÊN
ĐẠI HỌC QUỐC GIA HÀ NỘI NĂM HỌC 2013-2014
MÔN THI: TOÁN (Vòng 2)
ĐỀ THI CHÍNH THỨC Thời gian: 150 phút ( Không kể thời gian giao đề)

Đề số 18

Câu 1. (3,0 điểm).

1). Giải hệ phương trình



xy  x  y  2

 .
9 xy 3 x  y  6  26 x  2 y
 3 3

2). Giải phương trình

 x4 2  
4  x  2  2x .

Câu 2. (3,0 điểm).

1). Tìm hai chữ số cuối cùng của số

A  41106  57 2012 .

1 5
2). Tìm giá trị lớn nhất của hàm số y  3 2 x  1  x 5  4 x 2 , với x .
2 2

Câu 3. (3,0 điểm). Cho tam giác nhọn ABC ( AB  AC ) nội tiếp đường tròn (O) . Giả sử

 sao cho MN song song với BC và tia AN nằm giữa


M ; N là hai điểm thuộc cung nhỏ BC

hai tia AM ; AB . Gọi P là hình chiếu vuông góc của điểm C trên AN và Q là hình chiếu

vuông góc của điểm M trên AB .

1). Giả sử CP cắt QM tại điểm T . Chứng minh T nằm trên đường tròn (O) .

2). Gọi giao điểm của NQ và (O) là R khác N . Giả sử AM cắt PQ tại S . Chứng minh rằng

bốn điểm A; R; Q; S cùng thuộc một đường tròn.

Câu 4. (1,0 điểm). Với mỗi số n nguyên lớn hơn hoặc bằng 2 cố định, xét các tập n số thực

đôi một khác nhau X  x1 ; x2 ;...; xn  . Kí hiệu C X  là số các giá trị khác nhau của tổng

xi  x j ( 1  i  j  n ). Tìm giá trị lớn nhất và giá trị nhỏ nhất của C X  .

……....HẾT……….

Liên hệ tài liệu word toán sđt và zalo: 039.373.2038 TÀI LIỆU TOÁN HỌC
20
Website:tailieumontoan.com

SỞ GIÁO DỤC VÀ ĐÀO TẠO KÌ THI TUYỂN SINH VÀO LỚP 10 CHUYÊN
ĐẠI HỌC QUỐC GIA HÀ NỘI NĂM HỌC 2012-2013
MÔN THI: TOÁN (Vòng 2)
ĐỀ THI CHÍNH THỨC Thời gian: 150 phút ( Không kể thời gian giao đề)

Đề số 19

Câu 1. (3,0 điểm).


1). Giải hệ phương trình
x 3  y 3  1  y  x  xy
 .

7 xy  y  x  7

2). Giải phương trình

x  3  1  x2  3 x  1  1  x .
Câu 2. (3,0 điểm).
1). Tìm các cặp số nguyên  x; y thỏa mãn

5 x 2  8 y 2  20412 .

2). Với  x; y là các số thực dương thỏa mãn x  y  1, tìm giá trị nhỏ nhất của biểu thức
1 1
P     1  x 2 y 2 .
 x y 

Câu 3. (3,0 điểm). Cho tam giác nhọn ABC nội tiếp đường tròn (O) có trực tâm H . Gọi P
là điểm nằm trên đường tròn ngoại tiếp tam giác HBC ( P khác B, C và H ) và nằm trong
tam giác ABC . PB cắt (O) tại M khác B, PC cắt (O) tại N khác C . BM cắt AC tại E,
CN cắt AB tại F . Đường tròn ngoại tiếp tam giác AME và đường tròn ngoại tiếp tam
giác ANF cắt nhau tại Q khác A.
1). Chứng minh rằng ba điểm M ; N ; Q thẳng hàng.
 . Chứng minh rằng khi đó PQ đi qua trung điểm của
2). Giả sử AP là phân giác góc MAN
BC.
Câu 4. (1,0 điểm). Giả sử dãy số thực có thứ tự x1  x2    x192 thỏa mãn các điều kiện
x1  x2    x192  0 và x1  x2    x192  2013.

Chứng minh rằng


2013
x192  x1  .
96
……....HẾT……….

Liên hệ tài liệu word toán sđt và zalo: 039.373.2038 TÀI LIỆU TOÁN HỌC
21
Website:tailieumontoan.com

SỞ GIÁO DỤC VÀ ĐÀO TẠO KÌ THI TUYỂN SINH VÀO LỚP 10 CHUYÊN
ĐẠI HỌC QUỐC GIA HÀ NỘI NĂM HỌC 2013-2014
MÔN THI: TOÁN (Vòng 2)
ĐỀ THI CHÍNH THỨC Thời gian: 150 phút ( Không kể thời gian giao đề)

Đề số 20

Câu 1. (3,0 điểm).


1). Giả sử x; y là những số thực dương phân biệt thỏa mãn
y 2y2 4y4 8 y8
   4.
x  y x 2  y 2 x 4  y 4 x8  y 8
Chứng minh rằng 5 y  4 x .
2). Giải hệ phương trình

2 x 2  3 y 2  xy  12

 .

6 x  x 2
y  12  6 y  y 2
x


Câu 2. (3,0 điểm).
1) Cho x; y là những số nguyên lớn hơn 1 sao cho 4 x 2 y 2  7 x  7 y là số chính phương.
Chứng minh rằng x  y.
2) Giả sử x; y là những số thực không âm thỏa mãn x 3  y 3  xy  x 2  y 2 . Tìm giá trị lớn
nhất và nhỏ nhất của biểu thức
1 x 2 x
P  .
2 y 1 y
Câu 3. (3,0 điểm). Cho tam giác ABC nội tiếp đường tròn (O) và điểm P nằm trong tam
giác thỏa mãn PB = PC. D là điểm thuộc cạnh BC ( D khác B và D khác C ) sao cho P
nằm trong đường tròn ngoại tiếp tam giác DAB và đường tròn ngoại tiếp tam giác DAC .
Đường thẳng PB cắt đường tròn ngoại tiếp tam giác DAB tại E khác B. Đường thẳng
PC cắt đường tròn ngoại tiếp tam giác DAC tại F khác C.
1). Chứng minh rằng bốn điểm A; E; P; F cùng thuộc một đường tròn.
2). Giả sử đường thẳng AD cắt đường tròn (O) tại Q khác A , đường thẳng AF cắt đường
thẳng QC tại L. Chứng minh rằng tam giác ABE đồng dạng với tam giác CLF .
3). Gọi K là giao điểm của đường thẳng AE và đường thẳng QB . Chứng minh rằng
  PAB
QKL   PAC
  QLK .
Câu 4. (1,0 điểm). Cho tập hợp A gồm 31 phần tử và dãy gồm m tập con của A thỏa
mãn đồng thời các điều kiện sau
i). Mỗi tập thuộc dãy có ít nhất 2 phần tử;
ii). Nếu hai tập thuộc dãy có chung nhau ít nhất 2 phần tử thì số phần tử của hai tập này
khác nhau.
Chứng minh rằng m  900.

….…..HẾT………
Liên hệ tài liệu word toán sđt và zalo: 039.373.2038 TÀI LIỆU TOÁN HỌC
22
Website:tailieumontoan.com

SỞ GIÁO DỤC VÀ ĐÀO TẠO KÌ THI TUYỂN SINH VÀO LỚP 10 CHUYÊN
TỈNH PHÚ THỌ NĂM HỌC 2015-2016
MÔN THI: TOÁN (Chuyên toán)
ĐỀ THI CHÍNH THỨC Thời gian: 150 phút ( Không kể thời gian giao đề)

Đề số 21

Câu 1. (1,5 điểm).


1). Chứng minh rằng nếu số nguyên n lớn hơn 1 thoả mãn n2  4 và n2  16 là các số
nguyên tố thì n chia hết cho 5.
2). Tìm nghiệm nguyên của phương trình x 2  2 y  x  y  2  x  1 .
Câu 2. (2,0 điểm).

1). Rút gọn biểu thức A 



2 3 5  

2 3 5  .
2 2  3 5 2 2  3 5
2). Tìm m để phương trình  x  2 x  3 x  4 x  5  m có 4 nghiệm phân biệt.
Câu 3. (2,0 điểm).
1). Giải phương trình x 2  x  4  2 x  1 1  x .
 3
x  xy  10 y  0 2

2). Giải hệ phương trình  2 .


x  6 y  10
2

Câu 4 (3,5 điểm). Cho đường tròn O; R và dây cung BC  R 3 cố định. Điểm A di động

 sao cho tam giác ABC nhọn. Gọi E là điểm đối xứng với B qua AC và
trên cung lớn BC

F là điểm đối xứng với C qua AB . Các đường tròn ngoại tiếp các tam giác ABE và

ACF cắt nhau tại K ( K  A ). Gọi H là giao điểm của BE và CF .


 và tứ giác BHCK nội tiếp.
1). Chứng minh KA là phân giác trong góc BKC

2). Xác định vị trí điểm A để diện tích tứ giác BHCK lớn nhất, tính diện tích lớn nhất của

tứ giác đó theo R .

3). Chứng minh AK luôn đi qua một điểm cố định.


1 1 1
Câu 5. (1,0 điểm). Cho 3 số thực dương x; y; z thỏa mãn 2
 2  2  1 . Tìm giá trị nhỏ
x y z
nhất của biểu thức
y2 z2 z2 x2 x2 y 2
P   .
xy2  z2  y z2  x2  z x2  y 2 

……....HẾT……….

Liên hệ tài liệu word toán sđt và zalo: 039.373.2038 TÀI LIỆU TOÁN HỌC
23
Website:tailieumontoan.com

SỞ GIÁO DỤC VÀ ĐÀO TẠO KÌ THI TUYỂN SINH VÀO LỚP 10 CHUYÊN
ĐẠI HỌC KHTN HÀ NỘI NĂM HỌC 2015-2016
MÔN THI: TOÁN (Vòng 1)
ĐỀ THI CHÍNH THỨC Thời gian: 120 phút ( Không kể thời gian giao đề)

Đề số 22

Câu 1. (3,0 điểm).


1). Giả sử a; b là hai số thực phân biệt thỏa mãn a 2  3a  b2  3b  2 .
a). Chứng minh rằng a  b  3 .
b). Chứng minh rằng a 3  b3  45 .

2 x  3 y  5 xy
2). Giải hệ phương trình  2 .
2 2
4 x  y  5 xy

Câu 2. (3,0 điểm).
1). Tìm các số nguyên  x; y không nhỏ hơn 2 sao cho xy  1 chia hết cho  x  1 y  1 .
2). Với x; y là những số thực thỏa mãn đẳng thức x 2 y 2  2 y  1  0. Tìm giá trị lớn nhất và
xy
nhỏ nhất của biểu thức P  .
3y  1
Câu 3. (3,0 điểm). Cho tam giác nhọn ABC không cân có tâm đường tròn nội tiếp là điểm
I . Đường thẳng AI cắt BC tại D . Gọi E; F lần lượt là các điểm đối xứng của D qua
IC ; IB .
1). Chứng minh rằng EF song song với BC .
2). Gọi M ; N ; J lần lượt là trung điểm các đoạn thẳng DE; DF ; EF . Đường tròn ngoại tiếp
tam giác AEM cắt đường tròn ngoại tiếp tam giác AFN tại P khác A . Chứng minh rằng
bốn điểm M ; N ; P; J cùng nằm trên một đường tròn.
3). Chứng minh rằng ba điểm A; J ; P thẳng hàng.
Câu 4. (1,0 điểm).
1). Cho bảng ô vuông 2015 x2015. Kí hiệu ô (i ; j ) là ô ở hang
thứ i , cột thứ j . Ta viết các số nguyên dương từ 1 đến 2015
vào các ô của bảng theo quy tắc sau:
i). Số 1 được viết vào ô (1,1)
ii). Nếu số k được viết vào ô (i ; j ) ( i  1 ) thì số k  1 được
viết vào ô (i  1; j  1) .
iii). Nếu số k được viết vào ô (1; j ) thì số k  1 được viết vào ô ( j  1; 1) (xem hình 1).
Khi đó số 2015 được viết vào ô m; n .
Hãy xác định m và n.
2). Giả sử a; b; c là các số thực dương thỏa mãn ab  bc  ac  abc  4.
Chứng minh rằng a 2  b2  c 2  a  b  c  2 ab  bc  ac .

……....HẾT……….
Liên hệ tài liệu word toán sđt và zalo: 039.373.2038 TÀI LIỆU TOÁN HỌC
24
Website:tailieumontoan.com

SỞ GIÁO DỤC VÀ ĐÀO TẠO KÌ THI TUYỂN SINH VÀO LỚP 10 CHUYÊN
ĐẠI HỌC KHTN HÀ NỘI NĂM HỌC 2015-2016
MÔN THI: TOÁN (Vòng 2)
ĐỀ THI CHÍNH THỨC Thời gian: 150 phút ( Không kể thời gian giao đề)

Đề số 23

Câu 1. (3,0 điểm).


1) Với a; b; c là các số thỏa mãn
3 3 3 3
3a  3b  3c  24  3a  b  c  3b  c  a  3c  a  b .

Chứng minh rằng a  2bb  2cc  2 a  1 .



2 x  2 y  xy  5
2) Giải hệ phương trình  .
27  x  y  y  7  26 x 3  27 x 2  9 x
3

Câu 2. (3,0 điểm).
1) Tìm số tự nhiên n để n  5 và n  30 là số chính phương (số chính phương là bình
phương của một số nguyên)
2) Tìm  x; y nguyên thỏa mãn đẳng thức 1  x  y  3  x  y .
3) Giả sử x; y; z là các số thực lớn hơn 2. Tìm giá trị nhỏ nhất của biểu thức:
x y z
P   .
y  z4 z x4 x y4

Câu 3. (3,0 điểm). Cho tam giác ABC nhọn không cân với AB  AC . Gọi M là trung điểm
của đoạn thẳng BC . Gọi H là hình chiếu vuông góc của B trên đoạn AM . Trên tia đối của
tia AM lấy điểm N sao cho AN  2 MH .
1) Chứng minh rằng BN  AC .
2) Gọi Q là điểm đối xứng với A qua N . Đường thẳng AC cắt BQ tại D .Chunwgs minh
rằng bốn điểm B; D; N ; C cùng thuộc một đường tròn,gọi đường tròn này là (O) .
3) Đường tròn ngoại tiếp tam giác AQD cắt (O) tại G và D . Chứng minh rằng NG song
song với BC .
Câu 4. (1,0 điểm). Ký hiệu S là tập hợp gồm 2015 diểm phân biệt trên một mặt phẳng. Giả
sử tất cả các điểm của S không cùng nằm trên một đường thẳng. Chứng minh rằng có ít
nhất 2015 đường thẳng phân biệt mà mỗi đường thẳng đi qua ít nhất hai điểm của S .

……....HẾT……….

Liên hệ tài liệu word toán sđt và zalo: 039.373.2038 TÀI LIỆU TOÁN HỌC
25
Website:tailieumontoan.com

SỞ GIÁO DỤC VÀ ĐÀO TẠO KÌ THI VÀO LỚP 10 CHUYÊN


TỈNH NAM ĐỊNH NĂM HỌC 2015-2016
MÔN THI: TOÁN (ĐỀ CHUNG)
ĐỀ THI CHÍNH THỨC Thời gian: 120 phút ( Không kể thời gian giao đề)

Đề số 24
(Đề thi có 01 trang)

Câu 1. (2,0 điểm). 1). Với giá trị nào của x thì biểu thức x  1  x  3 xác định .
2). Tính giá trị của biểu thức A  x  3  3  x khi x  2 2 .
3). Tìm tọa độ của các điểm có tung độ bằng 8 và nằm trên đồ thị hàm số y  2 x 2 .
.
4). Cho tam giác ABC vuông tại A có AB  3; BC  5 . Tính cos ACB

.1 2  x  x 1  x 
Câu 2. (1,5 điểm). Cho biểu thức Q      (với x  0; x  1 ).
 x  1 x  1  x  1 x  x 
1). Rút gọn biểu thức Q . 2). Tìm các giá trị của x để Q  1 .
Câu 3. (2,5 điểm). 1). Cho phương trình x 2  2 m  1 x  m2  6  0 (1) (với m là tham số).

a). Giải phương trình với m  3 .

b). Với giá trị nào của m thì phương trình (1) có các nghiệm x1 ; x2 thỏa mãn x12  x2 2  16 .

 x  2  x  y  3  y

2). Giải hệ phương trình  2 .
x   x  32 x  y  5  x  16

Câu 4. (3,0 điểm). Cho tam giác ABC vuông tại A ( AB  AC ), đường cao AH . Đường
tròn tâm I đường kính AH cắt các cạnh AB; AC lần lượt tại M ; N . Gọi O là trung điểm
của đoạn BC ; D là gia điểm của MN và OA .
1). Chứng minh rằng:

a). AM.AB  AN .AC . b). Tứ giác BMNC là tứ giác nội tiếp.

2). Chứng minh rằng:


1 1 1
a). ADI ∽AHO . b).   .
AD HB HC

3). Gọi P là giao điểm của BC và MN ; K là giao điểm thứ hai của AP và đường tròn đường

  90 0 .
kính AH . Chứng ming rằng BKC
5
Câu 5. (1,0 điểm). 1). Giải phương trình 3x2  6 x  6  3  2  x  7 x  19 2  x .
2). Xét các số thực dương a; b; c thỏa mãn abc  1 . Tìm giá trị lớn nhất của biểu thức
a b c
T  4  4 .
b  c  a a  c  b a  b4  c
4 4 4

___________________Hết_________________

Liên hệ tài liệu word toán sđt và zalo: 039.373.2038 TÀI LIỆU TOÁN HỌC
26
Website:tailieumontoan.com

SỞ GIÁO DỤC VÀ ĐÀO TẠO KÌ THI VÀO LỚP 10 CHUYÊN


TỈNH BÌNH ĐỊNH NĂM HỌC 2015-2016
MÔN THI: TOÁN (Chuyên Tin)
ĐỀ THI CHÍNH THỨC Thời gian: 120 phút ( Không kể thời gian giao đề)

Đề số 25
(Đề thi có 01 trang)

Câu 1. (2,0 điểm).

1). Rút gọn biểu thức P  2  3  6 2 .


2 x  y  3
2). Giải hệ phương trình  .
x  y  6

Câu 2. (2,0 điểm). Cho phương trình mx 2  2 m  1 x  1  3m  0 (1) ( m là tham số).

1). Chứng tỏ rằng phương trình (1) luôn có nghiệm với mọi m .
2). Trong trường hợp m  0 . Gọi x1 ; x2 là hai nghiệm của phương trình (1), tìm giá trị nhỏ
nhất của biểu thức A  x12  x2 2 .

Câu 3. (2,0 điểm). Trong một phòng có 80 người họp, được sắp ngồi trên các dãy ghế có
chỗ ngồi bằng nhau. Nếu ta bớt đi 2 dãy ghế thì mỗi dãy ghế còn lại phải xếp thêm 2
người ngồi thì vừa đủ chỗ.
Hỏi lúc đầu có mấy dãy ghế và mỗi dãy ghế được xếp bao nhiêu chỗ ngồi.
Câu 4. (3,0 điểm). Cho điểm M nằm ngoài đường tròn (O) . Vẽ các tiếp tuyến MA; MB (
A; B là các tiếp điểm) và cát tuyến MCD không đi qua O ( C nằm giữa M và D ) với

đường tròn (O) . Đoạn thẳng MO cắt AB và (O) theo thứ tự tại H và I . Chứng minh rằng

1). Tứ giác MAOB nội tiếp đường tròn.


2). MC.MD  MA 2 .
3). OH .OM  MC.MD  MO 2 .
3x2
Câu 5. (1,0 điểm). Cho x; y; z là các số thực thỏa mãn điều kiện:  y 2  z 2  yz  1 . Tìm
2
giá trị nhỏ nhất và giá trị lớn nhất của biểu thức B  x  y  z .

-------------- HẾT--------------

Liên hệ tài liệu word toán sđt và zalo: 039.373.2038 TÀI LIỆU TOÁN HỌC
27
Website:tailieumontoan.com

SỞ GIÁO DỤC VÀ ĐÀO TẠO KÌ THI VÀO LỚP 10 CHUYÊN


TỈNH HẢI DƯƠNG NĂM HỌC 2015-2016
MÔN THI: TOÁN (Chuyên)
ĐỀ THI CHÍNH THỨC Thời gian: 120 phút ( Không kể thời gian giao đề)

Đề số 26
(Đề thi có 01 trang)

Câu 1 (2,0 điểm).


1). Cho a  b  29  12 5  2 5 . Tính giá trị của biểu thức

A  a 2 a  1  b2 b  1  11ab  2015 .
2). Cho x; y là hai số thực thỏa mãn xy  (1  x 2 )(1  y 2 )  1 . Chứng minh rằng

x 1  y 2  y 1  x2  0 .

Câu 2. (2,0 điểm).


1). Giải phương trình 2 x  3  4 x 2  9 x  2  2 x  2  4 x  1 .

 2 2
2 x  y  xy  5 x  y  2  y  2 x  1  3  3 x
2). Giải hệ phương trình :  .
 2
x  y  1  4 x  y  5  x  2 y  2


Câu 3. (2,0 điểm).
1). Tìm các số nguyên x; y thỏa mãn x 4  x 2  y 2  y  20  0 .

2). Tìm các số nguyên k để k 4  8 k 3  23k 2  26 k  10 là số chính phương.


Câu 4. (3,0 điểm). Cho đường tròn (O; R) và dây BC cố định không đi qua tâm. Trên tia

đối của tia BC lấy điểm A ( A khác B ). Từ A kẻ hai tiếp tuyến AM và AN với đường

tròn (O) ( M và N là các tiếp điểm). Gọi I là trung điểm của BC .

1). Chứng minh A; O; M ; N ; I cùng thuộc một đường tròn và IA là tia phân giác của góc

.
MIN

2 1 1
2). Gọi K là giao điểm của MN và BC . Chứng minh   .
AK AB AC

3). Đường thẳng qua M và vuông góc với đường thẳng ON cắt (O) tại điểm thứ hai là P .

Xác định vị trí của điểm A trên tia đối của tia BC để AMPN là hình bình hành.
3
Câu 5. (1,0 điểm) Cho a; b là các số dương thỏa mãn điều kiện a  b  4 ab  12 . Chứng

1 1
minh bất đẳng thức :   2015ab  2016 .
1 a 1 b

……....HẾT……….
Liên hệ tài liệu word toán sđt và zalo: 039.373.2038 TÀI LIỆU TOÁN HỌC
28
Website:tailieumontoan.com

SỞ GIÁO DỤC VÀ ĐÀO TẠO KÌ THI VÀO LỚP 10 CHUYÊN


TỈNH NGHỆ AN NĂM HỌC 2015-2016
MÔN THI: TOÁN (Chuyên)
ĐỀ THI CHÍNH THỨC Thời gian: 120 phút ( Không kể thời gian giao đề)

Đề số 27
(Đề thi có 01 trang)

Câu 1. (7,0 điểm).


1) Giải phương trình x2  5x  4  2 x  5  2 x  4  x2  4 x  5 .

 1  1


 x   y    2
2) Giải hệ phương trình  y  x .


 2 2
2 x y  xy  4 xy  2 x  y

Câu 2. (2,0 điểm). Cho a; b là các số nguyên dương thỏa mãn a 2  b2  ab . Tính giá trị biểu
a2  b2
thức A  .
2 ab
Câu 3. (2,0 điểm). Cho a; b; c là các số thực. Chứng minh
2
3  a  b  c
a 2
 1b  1c  1 
2 2
.
4
Câu 4. (7,0 điểm). Cho đường tròn ( O; R ) có BC là dây cố định ( BC < 2 R ); E là điểm
chính giữa cung nhỏ BC . Gọi A là điểm di động trên cung lớn BC và AB < AC ( A khác B
). Trên đoạn AC lấy điểm D khác C sao cho ED  EC . Tia BD cắt đường tròn ( O; R ) tại
điểm thứ hai là F .
1). Chứng minh D là trực tâm của tam giác AEF .
2). Gọi H là trực tâm của tam giác DEC ; DH cắt BC tại N . Đường tròn ngoại tiếp tam
giác BDN cắt đường tròn ( O; R ) tại điểm thứ hai là M . Chứng minh đường thẳng DM
luôn đi qua một điểm cố định.
Câu 5. (2,0 điểm). Cho tập hợp A gồm 21 phần tử là các số nguyên kahsc nhau thỏa mãn
tổng của 11 phần tử bất kỳ lớn hơn tổng của 10 phần tử còn lại. Biết các số 101 và 102
thuộc A . Tìm tất cả các phần tử của A .

……....HẾT……….

Liên hệ tài liệu word toán sđt và zalo: 039.373.2038 TÀI LIỆU TOÁN HỌC
29
Website:tailieumontoan.com

SỞ GIÁO DỤC VÀ ĐÀO TẠO KÌ THI VÀO LỚP 10 CHUYÊN


TỈNH THÁI BÌNH NĂM HỌC 2015-2016
MÔN THI: TOÁN (Chung)
ĐỀ THI CHÍNH THỨC Thời gian: 120 phút ( Không kể thời gian giao đề)

Đề số 28
(Đề thi có 01 trang)

2x  2 x x 1 x2  x
Câu 1. (3,0 điểm). Cho biểu thức P    với ( 0  x  1 ).
x x x x x x
1). Rút gọn biểu thức P .
2). Tính giá trị của thức P khi x  3  2 x .
7
3). Chứng minh rằng: với mọi giá trị của x để biểu thức P có nghĩa thì biểu thức chỉ
P
nhận một giá trị nguyên.
3
Câu 2. (2,0 điểm). Cho phương trình x 2 – 2 mx  m – 1  0 ( m là tham số).

1). Giải phương trình khi m  1 .


2). Tìm m để phương trình có hai nghiệm phân biệt trong đó có một nghiệm bằng bình
phương nghiệm còn lại.
9 2x
Câu 3. (1,0 điểm). Giải phương trình 2
 1  0 .
x 2x2  9

Câu 4. (3,5 điểm). Cho tam giác ABC vuông tại A , đường cao AH . Đường tròn đường
kính AH , tâm O , cắt các cạnh AB và AC lần lượt tại E và F . Gọi M là trung điểm của
cạnh HC .
1). Chứng minh AE.AB  AF.AC .
2). Chứng minh rằng MF là tiếp tuyến của đường tròn đường kính AH .
  HBO
3). Chứng minh HAM .

4). Xác định điểm trực tâm của tam giác ABM .
Câu 5. (0,5 điểm). Cho các số dương a; b; c thỏa mãn ab  bc  ca  3 . Chứng minh rằng
1 1 1 3
 2  2  .
a 1 b 1 c 1 2
2

……....HẾT……….

Liên hệ tài liệu word toán sđt và zalo: 039.373.2038 TÀI LIỆU TOÁN HỌC
30
Website:tailieumontoan.com

SỞ GIÁO DỤC VÀ ĐÀO TẠO KÌ THI VÀO LỚP 10 CHUYÊN


TỈNH THÁI BÌNH NĂM HỌC 2015-2016
MÔN THI: TOÁN (Chuyên)
ĐỀ THI CHÍNH THỨC Thời gian: 150 phút ( Không kể thời gian giao đề)

Đề số 29
(Đề thi có 01 trang)

Câu 1. (1,5 điểm). Cho phương trình 2 x 2  mx  1  0 (với m là tham số).


1). Tìm m sao cho phương trình trên có hai nghiệm x1 ; x2 thỏa mãn x12  4 x22  0 .
2). Chứng minh rằng với mọi m phương trình trên có nghiệm x thỏa mãn x  1 .
Câu 2. (2,0 điểm).
17 1
1). Giải phương trình sau 18 x 2  2 x  9 x  0 .
3 3
2). Tìm các số nguyên  x; y với x  0; y  0 thỏa mãn
x 2  3 y 2  4 xy  4 x  10 y  12  0 .
Câu 3. (1,0 điểm). Giải hệ phương trình sau
 2
 x  y  1  1  4  x  y   3 x  y
 .
4 x 2  2 xy  1

Câu 4. (1,0 điểm). Cho x; y thỏa mãn x 2  y 2  4 x  2  0 . Chứng minh rằng
10  4 6  x 2  y 2  10  4 6 .
Câu 5. (2,5 điểm). Cho tam giác ABC nhọn, nội tiếp đường tròn (O) . Đường thẳng AO
cắt đường tròn (O) tại M ( M  A ). Đường thẳng qua C vuông góc với AB cắt đường tròn
(O) tại N ( N  C ). Gọi K là giao điểm MN với BC .
1). Chứng minh tam giác KCN cân.
2). Chứng minh OK vuông góc với BM .
3). Hai tiếp tuyến của đường tròn (O) tại M và N cắt nhau tại P . Chứng minh ba điểm
P ; B; O thẳng hàng.
  60 0 .
Câu 6. (1,0 điểm). Cho tam giác ABC có độ dài cạnh AB  3a; AC  4 a và góc BAC
Qua A kẻ AH vuông góc với BC tại H . Tính độ dài đoạn AH theo a .
Câu 7. (1,0 điểm). Cho ba số dương a; b; c thỏa mãn abc  1 . Chứng minh rằng
b2 c 2 a2 9 9
    .
a b c 2 ab  bc  ca 2

-------------- HẾT--------------

Liên hệ tài liệu word toán sđt và zalo: 039.373.2038 TÀI LIỆU TOÁN HỌC
31
Website:tailieumontoan.com

SỞ GIÁO DỤC VÀ ĐÀO TẠO KÌ THI VÀO LỚP 10 CHUYÊN


TỈNH THÁI NGUYÊN NĂM HỌC 2015-2016
MÔN THI: TOÁN (Chuyên)
ĐỀ THI CHÍNH THỨC Thời gian: 150 phút ( Không kể thời gian giao đề)

Đề số 30
(Đề thi có 01 trang)

Câu 1 (1,5 điểm). Không dùng máy tính, chứng minh


1 1 1 1
   ...   21,5 .
1 3 3 5 5 7 2013  2015
Có thể thay giá trị 21,5 bằng một giá trị khác lơn hơn được không? Vì sao?
Câu 2 (1,0 điểm). Rút gọn biểu thức

A  x  2  2 x  3  x  1  4 x  3 với x  4 .

Câu 3 (1,0 điểm). Giải phương trình  x 2  3x  2 x 2  7 x  12  35 .

Câu 4 (1,5 điểm). Cho ba số a; b; c khác không, đôi một khác nhau thỏa mãn điều kiện
ab bc ca
  . Tính giá trị của biểu thức
c a b
 a  b  c
B  1  1  1   .
 b  c  a

Câu 5 (2,0 điểm). Tìm tất cả các số nguyên n sao cho a  n4  2n3  2n2  n  7 là số chính
phương.
 và ABC
Câu 6 (3,0 điểm). Cho tứ giác ABCD . Các đường phân giác của hai góc BAD  cắt
 và ADC
nhau tại M . Các đường phân giác của hai góc BCD  cắt nhau tại N . Giả sử đường

thẳng BM cắt đường thẳng CN tại P , đường thẳng AM cắt đường thẳng DN tại Q .
1). Chứng minh rằng bốn điểm M ; N ; P; Q cùng nằm trên một đường tròn.
2). Ký hiệu I ; K ; J ; H lần lượt là tâm các đường tròn nội tiếp các tam giác MAB ; NCD ;
PBC ; QAD . Các đường thẳng MI ; NK ; PJ ; QH cắt đường tròn đi qua bốn điểm
M ; N ; P ; Q lần lượt tại các điểm I1 ; K1 ; J1 ; H1 . Chứng minh rằng I1 K1  J1 H1 .

……....HẾT……….

Liên hệ tài liệu word toán sđt và zalo: 039.373.2038 TÀI LIỆU TOÁN HỌC
32
Website:tailieumontoan.com

LỜI GIẢI – NHẬN XÉT – BÀI TẬP TƯƠNG TỰ

ĐỀ SỐ 1.
Câu 1.
1). Hệ phương trình tương đương với

 x  1 y   x  1  2  y
 2 


 x  1 y 2  1  2  y (1)
  .



 y  2 x 2   y  2)  x  1 
 y  2 x 2  1  x  1 (2)


+) Nếu x  1 , suy ra  x  1 y 2  1  0 nên từ (1)  2  y  0
 y  2   y  2 x 2  1  0 ;
Do đó từ (2)  x  1  0  x  1 (mâu thuẫn).
+) Nếu x  1 , tuơng tự suy ra x  1 (mâu thuẫn).
+) Nếu x  1  y  2 (thỏa mãn).
Đáp số: x  1; y  2
Nhận xét: Bài toán sử dụng kỹ thuật đánh giá theo miền nghiệm khi đoán trước được
nghiệm của hệ phương trình.
Nhắc lại kiến thức và phương pháp:
• Cho hai biểu thức f  x; y và g  x; y , trong đó g  x; y  0 .
Xét biểu thức: P  f  x; y.g  x; y .
Có hai trường hợp sau xảy ra đó là P  0  f  x; y  0 và P  0  f  x; y  0 .
• Kỹ thuật nhẩm nghiệm.
Ý tưởng: Bài toán này không phải là một hệ phương trình đồng bậc, nếu là đồng bậc hai
thì ta có thể giải quyết bằng cách đưa về hệ số bất định. Nhưng một điều đáng lưu ý ở bài
toán này đó chính là các biểu thức x  1; y  2 được gắn với hai đại lượng không âm. Nên
nhiều khả năng sẽ xảy ra  x  1 y 2   y  2 x 2  0 . Xét các trường hợp thì thấy  x; y  1; 2
là nghiệm duy nhất của hệ phương trình. Hoặc ta có thể sử dụng kỹ thuật nhẩm nghiệm
như sau, đó là giả sử x  k , bây giờ ta sẽ thay thử các giá trị của k , tất nhiên sẽ lấy các giá
trị k nguyên và đẹp. Và cũng cho ta được nghiệm  x; y  1; 2 . Với cặp nghiệm này, thực
x  y  3 x  1  y  2  0
chất bài toán quy về giải hệ phương trình    .
 y  x  1  y  2  x  1
Vì thế ta tách hệ phương trình ban đầu, và nhóm nhân tử như sau:
• Hệ phương trình đã cho
 x  1 y 2  x  1  y  2  0  x  1 y 2  1  2  y 1

  
 y  2 x 2  y  2  x  1  y  2 x 2  1  x  1
   2
• Đến đây, ta sẽ đánh giá miền nghiệm:
1   x  1 y 2  1  0  2  y  0  y  2

TH1. Nếu x  1   .
2   y  2 x 2  1  0  y  2

Hệ bất phương trình này vô nghiệm.
Liên hệ tài liệu word toán sđt và zalo: 039.373.2038 TÀI LIỆU TOÁN HỌC
33
Website:tailieumontoan.com
1   x  1 y 2  1  0  2  y  0  y  2

TH2. Nếu x  1   .
2   y  2 x 2  1  0  y  2

Hệ bất phương trình vô nghiệm
• Vậy x  1; y  2 là nghiệm duy nhất của hệ phương trình.
Bài toán kết thúc.
Bài tập tương tự:

2  x  1 y 2  x  y  4
1. Giải hệ phương trình  .




2
y  3 x  y  x  2
Đáp số:  x; y  1; 3 .

2). Điều kiện x  0 .


3
Phương trình tương đương 2  x  1 x   x2  7 .
x
Chia hai vế cho x  0 , ta được
 1 3 7  3  1 3 4  3  3 2 
2 1   x   x    x    2 1   x    0   x   2 x     0 .
 x  x x  x   x  x x  x  x x 

3 3 x  1
+ Giải x   2  x   4  x 2  4 x  3  0   .
x x x  3
3 2 3 4
+ Giải x    x   2  x 3  3x  4  0
x x x x
  x  1 x 2  x  4  0  x  1 .
Đáp số x  1; x  3 .
Nhận xét: Bài toán sử dụng phương pháp ẩn phụ không hoàn toàn, sau đó nâng lũy thừa
tìm nghiệm của phương trình ban đầu.
Nhắc lại kiến thức và phương pháp:
• Phương pháp đặt ẩn phụ không hoàn toàn: xét một phương trình bậc hai có dạng
mf  x.t 2  ng  x.t  k  0 (*), trong đó t là ẩn phụ được biểu diễn dưới dạng t  h  x .
Khi đó, ta có
2
t  ng  x  4 kmf  x , với t bắt buộc là một số chính phương. Nên ta tìm được
 
nghiệm của (*) đó là
ng  x  t ng  x  t
t  h  x ; t   h  x .
mf  x mf  x

 f  x ; g  x  0

• Cách giải phương trình f  x  g  x   .




f  x   g 2
 x 
Ý tưởng: Trước hết, ta cần quy đồng mẫu số bài toán, như vậy ta sẽ được phương trình có
dạng f  x. g  x  h  x và nếu nâng lũy thừa hai vế, ta sẽ thu được một phương trình bậc
5. Và phương trình bậc 5 nếu không có nghiệm nguyên thì sẽ rất khó để giải quyết. Vậy
3
nên ta cần nghĩ đến hướng tư duy khác, đó là bài toán xuất hiện căn thức x nên ta
x

Liên hệ tài liệu word toán sđt và zalo: 039.373.2038 TÀI LIỆU TOÁN HỌC
34
Website:tailieumontoan.com
2
 3
mong muốn sẽ tạo được lượng k  x   để có thể đưa về phương trình bậc hai, sau đó
 x 
3
đặt t  x  để sử dụng phương pháp ẩn phụ không hoàn toàn. Tức là sẽ chia cả hai vế
x
của phương trình, ta được:
3 x2  7 3
• Ta có x    2  x  1 x   x 2  7
x 2( x  1) x
 1 3 7 3  1 3 4
 2 1   x   x   x   2 1   x    0 (*).
 x x x x  x x x
3  1 4
• Đặt t  x   0 , khi đó ta có (*)  t 2  2 1  t   0 .
x   x  x
2 2
 1 4  1
Có 't  1     1   nên suy ra được
 x   x  x 

 1
t  1   1 
1 t  2  x 3 2 i 
 x x  x
 2   .
 1 1  t
t  1   1    x 3  2
 x x
x  ii
 x x
3 x  1
• Giải ( i ), ta có ( i )  x   4  x 2  4 x  3  0   .
x  x  3
3 4
• Giải ( ii ), ta có ( ii )  x   2  x 3  3x  4  0  x  1 .
x x
Bài toán kết thúc.
Bài tập tương tự:
1. Giải phương trình:  x  3 4  x12  x  28  x .
Đáp số: x  4  
2  1 ; x  31  3 .

2. Giải phương trình: x3  x  2x2  x  2 .


1 5 1  65
Đáp số: x  ; x .
2 8
Câu 2.
1). Giả sử tồn tại các số nguyên x , y , z thỏa mãn
x 4  y 4  7 z 4  5  x 4  y 4  z 4  8 z 4  5 (1) .
Ta có a 4  0,1 (mod 8) với mọi số nguyên a
x 4  y 4  z 4  0,1,2,3 (mod 8)
  4 .
8 z  5  5(mod 8)

Mâu thuẫn với (1).
Vậy không tồn tại  x; y; z thỏa mãn đẳng thức.
Nhận xét. Để giải bài toán trên cần sử dụng phương pháp phản chứng: “Giả sử xảy rồi
biến đổi thấy điều mâu thuẫn với giả sử”.
Nhắc lại kiến thức và phương pháp.

Liên hệ tài liệu word toán sđt và zalo: 039.373.2038 TÀI LIỆU TOÁN HỌC
35
Website:tailieumontoan.com
• Thêm cùng một lượng vào hai vế của đẳng thức ta được một đẳng thức mới tương
đương với đẳng thức ban đầu
x4  y 4  7 z4  5  x4  y 4  z4  8z4  5 .
• Lũy thừa bậc bốn của một số nguyên khi chia cho 8 dư 0 hoặc 1 tức là a 4  0, 1 mod 8
với mọi số nguyên a
x 4  0, 1 mod 8
 x 4  y 4  z 4  0, 1, 2, 3 mod 8
 4
 y  0, 1 mod 8   4 .
 8 z  5  5 mod 8
z  0, 1 mod 8 
4

• Hai vế của một đẳng thức có số dư khi chia cho cùng một số nhận được số dư khác
nhau thì đẳng thức này không thể tồn tại.
Ta thấy VT  x 4  y 4  z 4 chia 8 dư 0 hoặc 1 hoặc 2 hoặc 3 .
Mà VP  8 z 4  5 chia 8 dư 5 do đó không thể tồn tại đẳng thức x 4  y 4  z 4  8 z 4  5 hay
x4  y 4  z4  8z4  5 .
Vậy không tồn tại các bộ ba số nguyên  x; y; z thỏa mãn đẳng thức
x4  y 4  7 z4  5 .

2). Phương trình tương đương với


 2  2
 x  1   x  1   x  1   x  1   y  2 x  24 x  y  8 x  8 x  y .
2 2 3 2 3 3 3

  
3 3 3
+) Nếu x  1  8 x 3  8 x 3  8 x  2 x  1  2 x  y 3  2 x  1 (mâu thuẫn vì y nguyên).
+) Nếu x  1 và  x; y là nghiệm, ta suy ra x;  y cũng là nghiệm, mà x  1  mâu
thuẫn.
+) Nếu x  0  y  0 (thỏa mãn).
Vậy x  y  0 là nghiệm duy nhất.
Nhận xét. Để giải bài toán trên cần sử dụng phương pháp biến đổi tương đương đưa về
xét khoảng giá trị của nghiệm.
Nhắc lại kiến thức và phương pháp.
• Hằng đẳng thức A 2  B2   A  B A  B
4 4  2 2  2 2
VT   x  1   x  1   x  1   x  1   x  1   x  1 
  
2
• Hằng đẳng thức  A  B  A 2  2 AB  B2
VT   x 2  2 x  1  x 2  2 x  1 x 2  2 x  1  x 2  2 x  1

 2 x 2  24 x  8 x 3  8 x .
Khi đó phương trình đã cho tương đương với phương trình 8 x 3  8 x  y 3
Giải phương trình nghiệm nguyên bằng cách xét khoảng giá trị của nghiệm
• Không tồn tại lũy thừa bậc ba nào giữa hai lập phương (lũy thừa bậc ba) liên tiếp.
3 3 3
+) Nếu x  1  8 x 3  8 x 3  8 x  2 x  1  2 x  y 3  2 x  1
3 3
(mâu thuẫn vì y nguyên và 2x và 2 x  1 là hai lập phương liên tiếp).
3 3 3
+) Nếu x  1  8 x 3  8 x 3  8 x  2 x  1  2 x  y 3  2 x  1

Liên hệ tài liệu word toán sđt và zalo: 039.373.2038 TÀI LIỆU TOÁN HỌC
36
Website:tailieumontoan.com
3 3
(mâu thuẫn vì y nguyên và 2x và 2 x  1 là hai lập phương liên tiếp).
+) Nếu x  0  y  0 (thỏa mãn).
Vậy x  y  0 là nghiệm duy nhất.

Câu 3.

 , suy ra OBD
1). Tứ giác OBCD nội tiếp và CO là phân giác góc BCD   OCD
  OCB
  ODB
,

nên tam giác OBD cân tại O , do đó OB  OD (1).


  OBE
Tứ giác OBCD nội tiếp ODC  (cùng bù với góc OBC
 ) (2).

Trong tam giác CEF có CO vừa là đường cao vừa là đường phân giác nên tam giác
CEF cân tại C .
  AFC
Do AB  CF  AEB   EAB
 , suy ra tam giác ABE cân tại B , nên BE  BA  CD (3).

Từ (1), (2) và (3), suy ra OBE  ODC (c – g – c).


Nhận xét. Có ba trường hợp bằng nhau cơ bản của hai tam giác. Ở bài này, chúng ta sử
dụng trường hợp bằng nhau “cạnh-góc-cạnh” từ đó ta sẽ đi tìm ra các cạnh và góc bằng
nhau.
Nhắc lại kiến thức và phương pháp.
• Hai góc nội tiếp cùng chắn một cung của một đường tròn thì bằng nhau.
  ODB
+ BCO  (hai góc nội tiếp cùng chắn cung OB
 của đường tròn ngoại tiếp tam giác
BCD ).
  CBD
+ OCD  (hai góc nội tiếp cùng chắn cung DO
 của đường tròn ngoại tiếp tam giác
BCD ).
  OCD
Mà BCO  (vì CO là tia phân giác của BCD ), suy ra OBD
  ODB
.
• Tam giác có hai góc bằng nhau là tam giác cân.
Tam giác OBD có OBD   ODB
 (chứng minh trên) nên OBD cân tại O .
• Tam giác cân có hai cạnh bên bằng nhau.
Tam giác OBD cân tại O suy ra OB  OD .
• Tứ giác nội tiếp có góc ngoài tại một đỉnh bằng góc trong tại đỉnh không kề với nó.

Liên hệ tài liệu word toán sđt và zalo: 039.373.2038 TÀI LIỆU TOÁN HỌC
37
Website:tailieumontoan.com
 là góc ngoài tại đỉnh B
Tứ giác BCDO nội tiếp đường tròn ngoại tiếp BCD có EBD
 là góc trong tại đỉnh D không kề B suy ra EBD
và CDO   CDO
.
• Tam giác có đường cao cũng là đường phân giác thì tam giác đó cân.
Tam giác CEF có CO vừa là đường cao vừa là đường phân giác nên tam giác CEF
cân tại C .
• Tam giác cân có hai góc kề cạnh đáy bằng nhau.
  CFE
Tam giác CFE cân tại C , suy ra CEF   AFC
 hay AEB .
• Một đường thẳng cắt hai đường thẳng song song tạo ra hai góc ở vị trí đồng vị bằng
nhau.
  EAB
AFC   EAB
 (hai góc ở vị trí đồng vị của AB  FC ), suy ra AEB  , nên EBA cân tại

B (tam giác có hai góc bằng nhau), do đó BE  BA mà ABCD là hình bình hành nên
AB  CD suy ra BE  CD .
Xét OBE và ODC có: OB  OD ; EBD   CDO
 ; BE  CD suy ra OBE  ODC (c – g
– c).

2). Từ OBE  ODC  OE  OC .


Mà CO là đường cao tam giác cân CEF , suy ra OE  OF .
Từ đó OE  OC  OF , vậy O là tâm đường tròn ngoại tiếp tam giác CEF .
Nhận xét. Đường tròn ngoại tiếp tam giác cách đều ba đỉnh của tam giác do đó ta chứng
minh điểm O cách đều các điểm E ; C ; F hay OE  OC  OF .
Nhắc lại kiến thức và phương pháp.
• Hai tam giác bằng nhau có các cặp cạnh và cặp góc tương ứng bằng nhau.
OBE  ODC  OE  OC .
• Tam giác cân có đường cao cũng là đường trung trực của cạnh tương ứng
CO là đường cao của tam giác cân CFE nên CO là đường trung trực của FE .
• Một điểm thuộc trung trực của một đoạn thẳng thì cách đều hai mút của đoạn thẳng.
Điểm O thuộc đường trung trực CO của đoạn thẳng FE nên OE  OF , suy ra
OE  OC  OF .
• Điểm cách đều ba đỉnh của một tam giác là tâm đường tròn ngoại tiếp tam giác đó.
Ta có OE  OC  OF suy ra O là tâm đường tròn ngoại tiếp CEF .

3). Theo trên, ta có BE  CD mà CE  CF  BC  DF .


 , nên
Ta có CI là đường phân giác góc BCD
IB CB DF
   IB.BE  ID.DF .
ID CD BE
Mà CO là trung trực EF và I  CO , suy ra IE  IF .
Từ hai đẳng thức trên, suy ra IB.BE.EI  ID.DF.FI .
Nhận xét. Chứng minh một đẳng thức ta kết hợp các đẳng thức đã cho, đã chứng minh
được để ghép vào đẳng thức cần chứng minh.
Nhắc lại kiến thức và phương pháp.

Liên hệ tài liệu word toán sđt và zalo: 039.373.2038 TÀI LIỆU TOÁN HỌC
38
Website:tailieumontoan.com
Ta có BE  CD (chứng minh trên) và CE  CF ( ECF cân tại C ) suy ra
CB DF
CE  BE  CF  CD  BC  DF suy ra  .
CD BE
• Đường phân giác trong của một tam giác chia cạnh dối diện thành hai đoạn thẳng có
tỷ số bằng với tỷ số hai cạnh tương ứng của tam giác.
 của tam giác CBD nên IB  CB , suy ra ta
Ta có CI là phân giác của góc trong BCD
ID CD
IB DE
được   BE.BI  DE.DI .
ID BE
• Nhắc lại kiến thức.
Ta có I nằm trên trung trực CO của đoạn thẳng FE nên suy ra IE  IF nhân vế theo vế
với đẳng thức BE.BI  DE.DI ta được BE.BI .IE  DE.DI .IF

x3 x2
Câu 4. Ta chứng minh  (1) .
x3  8 y 3 x2  2 y 2

x3 x4 2
 3 3
 2
  x 2  2 y 2   x  x 3  8 y 3   4 x 2 y 2  4 y 4  8 xy 3
x  8y x2  2 y 2 
 x 2  y 2  2 xy (đúng).

y3 y2
Ta chứng minh 3
 (2).
y 3   x  y x2  2 y 2

y3 y4  3 2
  x 2  2 y 2   y  y 3   x  y    x 2  2 y 2   y 4  y  x  y
3
 3
 2
 2y2   
y 3   x  y x 2

  x 2  y 2  x 2  3 y 2   y  x  y
3

1 2
Ta có x 2  y 2   x  y
2
x 2  3 y 2  x 2  y 2  2 y 2  2 xy  2 y 2  2 y  x  y
1
  x 2  y 2  x 2  3 y 2  
2 3
 x  y .2 y x  y  y  x  y  (2) (đúng).
2
Từ (1) và (2), suy ra P  1 .
Dấu “=” xảy ra khi và chỉ khi x  y .
Vậy Pmin  1 .
Nhận xét: Bài toán sử dụng phương pháp dự đoán điểm rơi, từ đó phát hiện tư duy bất
đẳng thức phụ cần thiết để tìm giá trị nhỏ nhất của bài toán.
Nhắc lại kiến thức và phương pháp:
• Bất đẳng thức Cosi cho hai số thực dương x  y  2 xy .
1 2
• Hệ quả của bất đẳng thức Cosi, đó là: x 2  y 2   x  y .
2
Ý tưởng: Đây là một bài toán chứa các biểu thức đồng bậc, nên điểm rơi của bài toán sẽ có
dạng x  ky . Từ đó thay ngược lại biểu thức P , ta có:

Liên hệ tài liệu word toán sđt và zalo: 039.373.2038 TÀI LIỆU TOÁN HỌC
39
Website:tailieumontoan.com

k3 4 k 4
P 3
 k  .
k 8 1   k  1
3 3
k 8 1   k  1
3

3
Các biểu thức k 3  8; 1   k  1 đều xuất hiện trong căn, nên ta mong muốn nó là một số

m  3
phương trình và bằng nhau, do đó k 3  8  1   k  1 và k 3  8  m2 nên dễ thấy 
3
.

k  1

Tức là điểm rơi của bài toán tại x  y và giá trị nhỏ nhất của P là 1 . Việc dự đoán điểm rơi
này là cần thiết, nó giúp ta có nhiều sự lựa chọn hơn trong các việc đánh giá như
1 2
x  y  2 xy hay x 2  y 2  x  y . Với điểm rơi đó, hai mẫu số trong đã bằng nhau P
2
nên nếu có đánh giá P , ta cần đánh giá hai căn về hai biểu thức cùng mẫu, ví dụ như cùng
mẫu số x  2 y; 2 x  y; x 2  2 y 2 ; 2 x 2  y 2 . Bây giờ, quan sát từng căn thức một:
x3 x
• Với f  x; y  3 3
x 3 , trong căn thức mẫu số có bậc ba, trên tử là bậc
x  8y x  8y3
nhất, vậy để đồng hóa bậc thì ta cần đánh giá căn thức về một biểu thức dạng bậc
không trên bậc nhất . Hơn nữa lại xuất hiện 8y 3 nên ta sẽ chọn đánh giá x  2 y để tối
thiểu hóa ẩn y , tức là ta sẽ cần chứng minh:
x 1 2
  x  x  2 y  x 3  8 y 3  y  y  x x  2 y  0 .
3
x  8y 3
x  2y
Nhưng điều này chưa hoàn toàn đúng, vì cần phải có điều kiện y  x , vậy nên hướng
tư duy như trên chưa đúng. Tức là ta sẽ lựa chọn biểu thức x 2  2 y 2 thay vì x  2 y như
ta đã chọn, vì thế:
x x 2
3 3
 2 2
 xx2  2 y 2   x2 x3  8 y 3 
x  8y x  2y
2
 xy 2  x  y  0 .

4y3 4y
• Với g  x; y  3
y 3
, với hướng tư duy tương tự, chúng ta sẽ
y   x  y
3
y   x  y
3

có:
4y3 2y2
  x 2  y 2  x 2  3 y 2   y  x  y .
3
3
 2 2
y 3   x  y x  2y

Điều này luôn đúng do theo bất đẳng thức Cosi, ta có:

 1 2
x  y   x  y
 2 2
  x 2  y 2  x 2  3 y 2   y  x  y .
3
 2

 2 2 2 2 2 2
x  3 y  x  y  2 y  2 xy  2 y


Bài toán kết thúc.
Bài tập tương tự:
1. Cho a; b là hai số thực dương thỏa mãn a  b  2 . Tìm giá trị nhỏ nhất của biểu thức
2 a 2  3b 2 2 b 2  3 a 2
P  .
2 a 3  3b3 2b3  3a 3
2. Cho a; b là hai số thực dương. Chứng minh rằng:

Liên hệ tài liệu word toán sđt và zalo: 039.373.2038 TÀI LIỆU TOÁN HỌC
40
Website:tailieumontoan.com
 2     
a  b  3 b2  a  3   2 a  1 2b  1  .
 4  4   
2  2 

ĐỀ SỐ 2.

Câu 1.
1). Điều kiện x  6.
Phương trình đã cho tương đương với  x  9  2012  
x  6 1  0 .
2
+ Giải x  9  2012  0  x  2012  9  4048135 .

+ Giải x  6  1  0  x  5 .
Vậy phương trình có hai nghiệm: x  4048135; x  5 .
Nhận xét: Bài toán sử dụng phương pháp nhóm nhân tử chung và nâng lũy thừa tìm
nghiệm của phương trình.
Nhắc lại kiến thức và phương pháp:
• Cách giải phương trình dạng
 f x m 
   f  x  m 2
 
f  x  m . 
g  x  n  0  
 g  x  n
 
 g  x  n 2
.
 
Ý tưởng: Bài toán cho hết sức đơn giản, với sự xuất hiện của hai căn thức
x  9; x  6; x  9x  6 nên không khó để nhóm được nhân tử chung như sau:
x  9  2012 x  6  2012  x  9x  6
 x9  x  9x  6  2012 x  6  2012  0
 
 x  9 1  x  6  2012 1  x  6  0  

 1 x  6  
x  9  2012  0 .
Bài toán kết thúc.
Bài tập tương tự:
1. Giải phương trình 2 x  6  x  7  2  x 2  13x  42 .
Đáp số: x  3; x  5 .
2. Giải phương trình x  4  2 x  3  2  x 2  7 x  12 .
Đáp số: x  0; x  2 .
 2 2
x   y  1  5
2). Cách 1: Hệ đã cho tương đương với  .
x  y  1  x   y  1  5


u  x   y  1
Đặt 
2 2
 x 2   y  1   x  y  1  2 x  y  1  u2  2 v .


 v  x  y  1

Liên hệ tài liệu word toán sđt và zalo: 039.373.2038 TÀI LIỆU TOÁN HỌC
41
Website:tailieumontoan.com

u2  2 v  5
Thu được 

u  v  5

u  3  v  2
 u2  2 5  u  5  u2  2u  15  0   .
u  5  v  10


x   y  1  3  x  y  1
+ Giải  

.
 


 x y  1  2  x  2, y  0

x   y  1  5
+ Giải  (vô nghiệm).
 


 x y  1  10

Vậy hệ phương trình có hai nghiệm 1; 1 và 2; 0 .



x 2  y 2  2 y  4
Cách 2: Hệ tương đương với  .

 4 x  2 y  2 xy  8

Cộng vế với vế hai phương trình ta thu được
2 x  y  2
 x  y  4  x  y  12  0  
 x  y  6
.

x  y  2 x  y  2 x  1  y  1
+ Giải     .
2 x  y  xy  4 x  x 2  x  2  x  2  y  0

x  y  6 x  y  6
+ Giải    (vô nghiệm).
2 x  y  xy  4 x  x 6  x  10

Vậy hệ phương trình có nghiệm  x; y  1; 1 , 2; 0 .

Nhận xét: Bài toán sử dụng phương pháp ẩn phụ sau đó từ ẩn phụ tìm ngược lại nghiệm
của hệ phương trình.
Ý tưởng: Sự xuất hiện của y 2  2 y ở phương trình một của hệ, làm ta nghĩ đến hằng đẳng
2 2
thức y 2  2 y  1   y  1 hay nói cách khác, từ phương trình một ta có: x 2   y  1  5 .

Đây là phương trình có dạng là tổng các bình phương, dễ làm ta suy đoán đến hệ phương
trình đối xứng loại I, tức là đặt ẩn phụ theo định lý Vi-et ( đặt tổng và tích ) như sau:
u  x  y  1 và v  x  y  1 .

Nhưng đây cũng chỉ là suy đoán ban đầu, bây giờ ta sẽ đi xét phương trình hai để xuất
hiện u, v .
Thật vậy, ta có phương trình hai trong hệ tương đương với:
2 x  y  xy  4  x  y  1  x  y  1  5 .


u2  2 v  5 u  3  v  2
Do đó hệ phương trình đã cho trở thành  

.

u  v  5 u  5  v  10

Liên hệ tài liệu word toán sđt và zalo: 039.373.2038 TÀI LIỆU TOÁN HỌC
42
Website:tailieumontoan.com
Thế ngược lại tìm hệ của phương trình ban đầu.
Hoặc, ta có thể suy luận như sau: ta đi kết hợp cả hai phương trình trong hệ, vẫn với sự
xuất hiện x 2  y 2 ở phương trình một, đồng thởi có tích ở phương trình xy ta sẽ liên tưởng
2
đến hằng đẳng thức  x  y . Vì thế lấy phương trình hai nhân 2 rồi cộng với phương trình

một ta được:
2 x  y  2
 x  y  4  x  y  12  0  
 x  y  6
.

Thế ngược lại một trong hai phương trình trong hệ ban đầu để tìm nghiệm của hệ ban
đầu.
Bài toán kết thúc.
Bài tập tương tự:

x 2  y 2  4 y  1
1. Giải hệ phương trình  .

3 x  xy  y  3

Đáp số:  x; y  1; 0 , 2;  1 .



x 2  2 x  y 2  4 y  0
2. Giải hệ phương trình  .

3 x  xy  2 y  0

Đáp số:  x; y  0; 0 , 1;  1 .

Câu 2.
1). Phương trình tương đương với
x  y  1xy  x  y  2  x  y  1  3
  x  y  1 xy  x  y  2  3 .

 x  y  1 là ước của 3.

x  y  1  1 x  y  0
+ Giải    (vô nghiệm).
xy  x  y  2  3 xy  5

x  y  1  1 x  y  2 x  1


+ Giải     .
xy  x  y  2  3 xy  1  y  1

x  y  1  3 x  y  2 x  1


+ Giải     .
xy  x  y  2  1 xy  1  y  1

x  y  1  3 x  y  4
+ Giải    (vô nghiệm).
xy  x  y  2  1 xy  5

Vậy  x; y  1;  1 , 1; 1 .

Nhận xét. Bài toán nghiệm nguyên giải bằng phương pháp đưa về phương trình ước số
Nhắc lại kiến thức và phương pháp.
Liên hệ tài liệu word toán sđt và zalo: 039.373.2038 TÀI LIỆU TOÁN HỌC
43
Website:tailieumontoan.com
• Phân tích đưa về phương trình ước số
x  y  1xy  x  y  5  2  x  y
  x  y  1 xy  x  y  3  2  2  x  y

  x  y  1 xy  x  y  3  2  x  y  1

  x  y  1 xy  x  y  2  x  y  1  3

  x  y  1 xy  x  y  2  3

• Phân tích số thành tích của 2 ước số


3  1.3  3.1 .
• Cho mỗi thừa số chứa biến ở vế này đồng nhất với thừa số ở vế kia

2). Ta có  x 1  
y  1  4  xy  x  y  3.

Theo bất đẳng thức Cô si


x  y x 1 y 1
3  xy  x  y     xy2.
2 2 2
x2 

 y  2 x 2
 P  x  y  x y 2 .
2
y
Theo bất đẳng thức Cô si 2 
y 
 y x
 x  2 y
x 

Vậy giá trị nhỏ nhất của P là 2 khi x  y  1 .
Nhận xét: bài toán sử dụng bất đẳng thức Cosi dựa trên điểm rơi đã được suy đoán cũng
như kết hợp với điều kiện bài toán để tìm giá trị nhỏ nhất.
Nhắc lại kiến thức và phương pháp:
• Bất đẳng thức Cosi cho hai số thực dương x  y  2 xy .
2
a 2 b2 a  b
• Mở rộng ra một đánh giá   , a ; b; x ; y  0 .
x y xy
Ý tưởng: Đây là một bài toán có sự đối xứng rõ ràng giữa x , y nên ta mạnh dạn dự đoán
2
điểm rơi tại x  y  k . Thay ngược lại giả thiết bài toán, ta có  
k  1  4  k  1 . Với điểm
rơi x  y  1 , ta sẽ dễ dàng đánh giá hơn khi vận dụng bất đẳng thức Cosi, chính vì vậy,
khi đi khai khác giả thiết, ta suy ra:  x 1 
y  1  4  xy  x  y  1  4
x  y x 1 y 1
 xy  x  y  3  3     xy2.
2 2 2
x2 y 2
Vậy nên, bây giờ ta sẽ đánh giá biểu thức P theo   f  x  y  2 . Hiển nhiên có con
y x
số 2 vì điểm rơi x  y  1 . Vậy nên ta cần tìm f  x  y thỏa mãn f  x  y  2 . Mà như bên

Liên hệ tài liệu word toán sđt và zalo: 039.373.2038 TÀI LIỆU TOÁN HỌC
44
Website:tailieumontoan.com
x2 y 2
trên, ta đã tìm được x  y  2 do đó, ta cần chứng minh rằng   x  y . Đánh giá này
y x
ta có các cách như sau:
• Biến đổi tương đương, ta có:
x2 y2 x  x  y y  x  y 2
x y0    0   x  y  0 .
y x y x
• Sử dụng bất đẳng thức Cosi, ta có:
 x 2 x2

 y  y  2 .y  2 x
 y x2 y 2
  xy.
 2 2 y x
 y  x  2 y .x  2 y
 x x
• Sử dụng đánh giá mở rộng như đã nêu, ta có:
2
x 2 y 2  x  y
  xy.
y x xy
Bài toán kết thúc.
Bài tập tương tự:
1. Cho a; b là hai số thực dương thỏa mãn ab  1 . Tìm giá trị nhỏ nhất của biểu thức
a2 b2
P   2 a 2  b2  .
b a
2. Cho a; b; c là các số thực dương thỏa mãn a  b  c  3 . Tìm giá trị nhỏ nhất của biểu
a b c
thức P  2
 2
 .
1 b 1 c 1  a2
Câu 3.

1). Vì MP là đường kính suy ra PN  MN (1).


Vì MD là đường kính suy ra DN  MN (2).
Từ (1) và (2), suy ra N ; P; D thẳng hàng.

Liên hệ tài liệu word toán sđt và zalo: 039.373.2038 TÀI LIỆU TOÁN HỌC
45
Website:tailieumontoan.com
Nhận xét. Chứng minh ba điểm thẳng hàng ta quy về chứng minh chúng cùng thuộc một
đường thẳng.
Nhắc lại kiến thức và phương pháp.
• Góc nội tiếp chắn nửa đường tròn là góc vuông.
 là góc nội tiếp chắn nửa đường tròn được chia đôi bởi đường kính PM
+ Góc PNM
  90 hay PN  NM .
của đường tròn đường kính PM nên PNM
 là góc nội tiếp chắn nửa đường tròn được chia đôi bởi đường kính DM
+ Góc DNM
  90 hay DN  NM
của đường tròn (O) nên DNM
• Từ một điểm nằm ngoài một đường thẳng đã kẻ được một và chỉ có một đường thẳng
vuông góc với đường thẳng đó.
Ta có từ N ta kẻ được PN và DN vuông góc với MN , suy ra PN  DN hay ba điểm
P ; N ; D thẳng hàng.
  MAD
2). Tứ giác APQD nội tiếp ( PQD   90 0 ),

  PDQ
suy ra PAQ   NDM
 (3).
  NAM
Xét (O) , ta có NDM  (4).
  NAP
Từ (3) và (4) PAQ  (*).
 , suy ra AP là phân giác của góc NAQ

  AMD
Xét (O) , ta có AND .

Xét đường tròn đường kính MP có QMP   ANP


  QNP   QNP
 , nên NP là phân giác của

 (**).
góc ANQ
Từ (*) và (**), suy ra P là tâm đường tròn nội tiếp tam giác ANQ.
Nhận xét. Chứng minh một điểm là tâm đường tròn nội tiếp một tam giác ta chứng minh
điểm đó là giao điểm của hai đường phân giác trong của tam giác đó.
Nhắc lại kiến thức và phương pháp.
• Tứ giác có tổng hai góc đối diện bằng 180 là tứ giác nội tiếp.
  90
  90 (góc nội tiếp chắn nửa đường tròn (O) ) và DQP
Tứ giác APQD có DAP
(góc ngoài tại đỉnh đối diện bằng góc trong không kề với nó của tứ giác nội tiếp) suy
  DQP
ra DAP   90  90  180 do đó tứ giác APQD là tứ giác nội tiếp.

• Hai góc nội tiếp cùng chắn một cung của một đường tròn thì bằng nhau.
  QAP
Tứ giác APQD là tứ giác nội tiếp nên QDP  (hai góc nội tiếp cùng chắn cung QP

  NDM
của đường tròn (O) ) hay PAQ  .

  NAM
+ NDM  (hai góc nội tiếp cùng chắn cung MN
 của đường tròn (O) ).

+ AND  (hai góc nội tiếp cùng chắn cung AD


  AMD  của đường tròn (O) ).

Liên hệ tài liệu word toán sđt và zalo: 039.373.2038 TÀI LIỆU TOÁN HỌC
46
Website:tailieumontoan.com

+ QMP  (hai góc nội tiếp cùng chắn cung QP


  QNP  của đường tròn đường kính PM ) ,

  NAP
suy ra PAQ  hay AP là phân giác của NAP   QNP
 và AND  hay ND là phân

.
giác của QNA
• Giao điểm của hai đường phân giác của hai góc trong một tam giác là tâm đường tròn
nội tiếp tam giác đó.
 , ta có AP
 và ND là phân giác của QNA
Tam giác ANQ có AP là phân giác của NAP
và ND cắt nhau tại P nên suy ra P là tâm đường tròn nội tiếp của ANQ .

Câu 4. Ta có
abc  ab  ac  a  abc  bc  ba  b abc  ca  cb  c
Q
a  1b  1c  1
a b  1c  1  bc  1a  1  c a  1b  1
Q
a  1b  1c  1
a b c
Q   .
a 1 b 1 c 1
Ta chứng minh rằng
a b c 1 2 3 5
      .
1  a 1  b 1  c 1  1 1  2 1  3 12
Bất đẳng thức trên tương đương với
 3 c   b 2   a 1 

 1  3 1  c   1  b 1  2   1  a 1  1 0
         

3c b2 a 1
   0
4 1  c 3b  1 2 1  a

   
 1 1    1 1 
 3  c      3  c  b  2 
  
 4 c  1 3b  1   3b  1 2 1  a
   
1
 3  c  b  2  a  1 0
  2 1  a

3b  4c  1 2a  3b  1 1
 3  c   b  1  c   a  b  c  0.
12 b  1c  1 6 b  1a  1 2 a  1

3b  4c  1
Vì c  3; 0  b  c  3  c  0 (1).
12 b  1c  1

2a  3b  1
Vì b  1  c; 0  a  b  b  1  c   0 (2).
6 b  1a  1

Liên hệ tài liệu word toán sđt và zalo: 039.373.2038 TÀI LIỆU TOÁN HỌC
47
Website:tailieumontoan.com
1
Vì a  b  c; 0  a  a  b  c   0 (3).
2 a  1

Từ (1), (2) và (3), suy ra điều phải chứng minh.


5
Vậy giá trị nhỏ nhất của Q là khi a  1; b  2; c  3 .
12

ĐỀ SỐ 3.

Câu 1.
1
1). Điều kiện:   x  2 .
3
Phương trình đã cho tương đương với 2 x  3  2 3x  12  x  9

3  x  0

 3 x 2  5 x  2  3  x  
 2 2
3 x  5 x  2  x  6 x  9

x  1

2
 4 x  11x  7  0   .
x  7
 4
7
Đối chiếu với điều kiện ta được được nghiệm: x  1; x  .
4
Nhận xét: Bài toán sử dụng phương pháp nâng lũy thừa (bình phương) hai vế tìm nghiệm
của phương trình.
Nhắc lại kiến thức và phương pháp:

 
2
• Phương trình dạng f  x  g  x  m  0  f  x  g  x  m 2

 f  x   g  x  2 f  x g  x  m 2  2 f  x g  x  m 2  f  x  g  x

 m 2  f  x  g  x  x  x1


 
2  .

 4 f  x g  x   m2  f  x  g  x  x  x2

  
• Phương trình trên có cách giải khác như sau:
f  x  g  x  m  f  x  m  g  x
m  f  x m  f  x
   
 f x  m2  2 m g x  g x

      2m g  x  g  x  m2  f  x

 m  f  x ; g  x  m 2  f  x

  x  x1
  

.

4 m g  x   g  x  m  f  x
2 2
2
 x  x2


Ý tưởng: Đây là một bài phương trình cơ bản, dạng toán một vế chứa hai căn thức vế còn
lại là một hằng số thì phương pháp nâng lũy thừa hai vế là một phương pháp tối ưu nhất.
Bài toán kết thúc.
Bài tập tương tự:

Liên hệ tài liệu word toán sđt và zalo: 039.373.2038 TÀI LIỆU TOÁN HỌC
48
Website:tailieumontoan.com

1. Giải phương trình 3x  1  x  1  8 .


Đáp số: x  8 .
2. Giải phương trình 7x  4  x  1  3 .
Đáp số: x  3 .

 1   1 9
 x     y   


 y   x 2
2). Hệ phương trình tương đương với  .
 1 3  1   1  1 

   x  y    x  y  y  x   2
 4 2    

 1

u x
 y
Đặt  .

 1

 vy

 x

 9 
 9
uv  
 v  u

  2
Hệ phương trình trở thành  2  .
 9 

 1 3 
 9 3u 
  u  uv  2    u  u
   2 
4 2
 4
 2
9 3u 9u 9
Suy ra    u2  u2  3u   0
4 2 2 4

 1 3
2  3 
 x 
 3   u  y 2
 u    0   2  
 2   1
v  3 
 y 3
 

 x

 3y  1

 xy  1  3y y 2 y  1  x 
2.
2
 2   3 x  x   y   3  y  3 y  2  0  

xy  1  3 x 2 2 y 

  y  2  x  1
1 
Hệ phương trình có nghiệm  x; y   ; 1 , 1; 2 .
 2 
Nhận xét: Bài toán sử dụng phương pháp đặt hai ẩn phụ, đưa về hệ phương trình bậc hai
cơ bản giải bằng phương pháp thế. Sau đó từ nghiệm ẩn phụ suy ngược lại nghiệm của hệ
phương trình.
Ý tưởng: Hình thức bài toán khá phực tạp vì sự xuất hiện của phân thức, quan sát ta thấy
1
ở cả hai phương trình của hệ đều xuất hiện biểu thức x  . Ta sẽ nghĩ đến chuyện thế
y
1 9  1 1
x    y   xuống phương trình hai nhưng còn đại lượng xy  chưa biết xử lý
y 2  x xy
như thế nào. Có lẽ tác giá đã gợi mở theo con đường đặt ẩn phụ, nếu đặt
1 1 1
u  x  ; v  y  thì bây giờ ta chỉ cần biểu diễn xy  qua u; v thì hệ phương trình đã
y x xy

 1

u  x   uy  xy  1
 y
cho sẽ được giải quyết. Ta có 

 1

 v  y   vx  xy  1

 x

Liên hệ tài liệu word toán sđt và zalo: 039.373.2038 TÀI LIỆU TOÁN HỌC
49
Website:tailieumontoan.com
2 1
 uvxy   xy  1  uv  xy  2.
xy

 9

uv 
 2
Khi đó, hệ phương trình đã cho tương đương với  .

 1 3
  u  uv  2

4 2

Hệ phương trình trên là hệ phương trình cơ bản, hoàn toàn giải quyết được bằng phương
pháp thế.
Bài toán kết thúc.
Bài tập tương tự:
x 1  y   2

 2 2

1. Giải hệ phương trình  .



1  xy  x 2 2
y  3 x 2


 7 5   7 5 
Đáp số:  x; y  1; 1 , 1;  1 ,  ;  ,  ; .
 4 7   4 7 

 2 2

 x  1  y  1  27 xy
2. Giải hệ phương trình  2 .

 x  1 y 2  1  10 xy


1   1 
 2 
  
Đáp số:  x; y   ; 2  3  , 2; 2  3 , 2  3; .
2 
Câu 2.
1). Cách 1: Đẳng thức cần chứng minh tương đương với
a  b  b  c  c  a  3
1  
 1  
 1  
a  b b  c b  c  c  a c  a  a  b  4
ac ba cb 3
   
a  bb  c b  cc  a c  aa  b 4
3
 ac a  c  ba b  a  cb c  b  a  bb  cc  a
4
 ac a  c  ba b  a  cb c  b  6 abc
 ac a  c  b2 a  c  ba 2  abc  c 2 b  abc  8 abc

 a  cac  b2  ab  bc  8 abc


 a  cc  bb  a  8 abc , điều phải chứng minh.
 
x  a 1  x  b
 a  b  ba
 
b c
Cách 2: Đặt  y   1  y  .
 b  c  bc
 
z  c 1  z  a
 c  a  ca
 
1
Từ điều kiện suy ra xyz  , ta được
8
xyz  1  x1  y1  z  1   x  y  z   xy  yz  zx  xy
 2 xyz  1   x  y  z   xy  yz  zx

Liên hệ tài liệu word toán sđt và zalo: 039.373.2038 TÀI LIỆU TOÁN HỌC
50
Website:tailieumontoan.com
3
 xyz  xy  yz  zx .
4
Nhận xét: bài toán sử dụng phép biến đổi tương đương hoặc ẩn phụ để chứng minh đẳng
thức đã cho.
Ý tưởng: Nhìn đẳng thức cần chứng minh ( gọi là (*)) khá là cồng kềnh, tuy nhiên nếu tinh
ý một chút, ta thấy rằng bên vế trái (*) có tổng của ba thừa số, đồng thời vế phải (*) xuất
hiện tổng hoán vị của tích hai thừa số . Vì thế nếu chuyển vế ta sẽ nhóm được nhân tử
chung là:
a  b  b  c  c  a  3
1    1    1  
a  b b  c b  c  c  a c  a  a  b  4
ac ba cb 3
    ( i ).
a  bb  c b  cc  a c  aa  b 4
Với biểu thức ( i ), hướng tối ưu nhất có lẽ là quy đồng mẫu số và biến đổi tương đương,
kết hợp với giải thiết a  bb  cc  a  8 abc thì ta có:
3
( i )  ac a  c  ba b  a  cb c  b  a  bb  cc  a
4
 ac a  c  ba b  a  cb c  b  6 abc
 ac a  c  b2 a  c  ba 2  abc  c 2 b  abc  8 abc

 a  cac  b2  ab  bc  8 abc .


Hoặc, ta có thể đi với hướng tư duy ẩn phụ hóa để đơn giản bài toán hơn một chút. Vẫn là
a b c
hướng phát hiện như bên trên, ta sẽ đặt ẩn phụ các thừa số là x  ; y ; z .
ab bc ca
1 3
Khi đó giả thiết  xyz  và ta cần chứng minh x  y  z   xy  yz  zx (**). Nếu chỉ
8 4
dựa vào giả thiết để chứng minh (**) là chưa đủ, ta cần phải khéo léo kết hợp với giải thiết
b c a
bài toán như sau 1  x  ; 1 y  ; 1 z  . Và từ đó suy ra được đẳng thức
ba bc ca
xyz  1  x1  y1  z . Khai triển tích số, ta sẽ được điểu phải chứng minh.
Bài toán kết thúc.

2). Ta có abcde  abc00  de  abc  100  de


 abc 101  1  de  abc  101  abc  de
Suy ra abcde chia hết cho 101  abc  de  abc  10d  e chia hết cho 101 .
99999 9
Ta có 101  m  99999  m   990 
101 101
Vậy số có 5 chữ số lớn nhất chia hết cho 101 là 990  101
999
Ta có 101  n  9999  n   99
101
Vậy số có 5 chữ số nhỏ nhất chia hết cho 101 là 100  101
Số các số có 5 chữ số thỏa mãn yêu cầu của bài toán là: 990  100  1  891.
Đáp số: 891 số.
Nhận xét. Bài toán chứng minh đẳng thức từ những điều kiện đã cho.
Liên hệ tài liệu word toán sđt và zalo: 039.373.2038 TÀI LIỆU TOÁN HỌC
51
Website:tailieumontoan.com
Nhắc lại kiến thức và phương pháp.
• Cấu tạo số
abcde  abc 00  de  abc  100  de  abc 101  1  de  abc  101  abc  de
• Tính chất chia hết của một tích: Trong một tích có một thừa số chia hết cho một số thì
tích chia hết cho số đó.
Ta có 101  101  abc  101  101
• Tính chất chia hết của một tổng: Tổng của hai số hạng, trong đó có một số hạng chia
hết cho một số thì số hạng còn lại chia hết cho số đó.
 
 abc  101  101

Ta có  , suy ra de  abc   101

abcde   abc.101  de  abc
    101
 
 abc  10d  e  101 .
• Số lớn nhất có năm chữ số chia hết cho 101.
99999 9
Ta có 101  m  99999  m   990  suy ra số có năm chữ số lớn nhất chia hết
101 101
cho 101 là 990  101 .
• Số bé nhất có năm chữ số chia hết cho 101 .
999
Ta có 101  n  9999  n   99 suy ra số có năm chữ số nhỏ nhất chia hết cho 101 là
101
100  101
cd
• Số các số của dãy số viết theo quy luật được tính theo công thức s   1 trong đó
h
c là số cuối, d là số đầu, h khoảng cách giữa 2 số liên tiếp của dãy.
Số các số có 5 chữ số thỏa mãn yêu cầu của bài toán là:
990.101  100.101
 1  990  101  1  891 .
101
Câu 3.
E
A
F

M O

N
C
B

Liên hệ tài liệu word toán sđt và zalo: 039.373.2038 TÀI LIỆU TOÁN HỌC
52
Website:tailieumontoan.com

1). Ta có góc nội tiếp bằng nhau   BCF


BDM  (1) và   BFA
BMA  suy ra
180 0  BMA  hay BMD
  180 0  BFA   BFC
 (2).

Từ (1) và (2) , suy ra BDM và BCF đồng dạng (g - g).


Nhận xét. Đây là bài toán tương đối cơ bản và thường gặp trong các bài toán chứng minh
tam giác đồng dạng ứng dụng của góc nội tiếp.
Nhắc lại kiến thức và phương pháp.
• Các góc nội tiếp cùng chắn một cung của một đường tròn thì bằng nhau.
  BCF
+ BDM  (hai góc nội tiếp cùng chắn cung AB của đường tròn (O) ).
  BFA
+ BMA  (hai góc nội tiếp cùng chắn cung AB của đường tròn ngoại tiếp tam giác
ABM )
  180 0  BFA
 180 0  BMA   BMD
  BFC
.
• Hai tam giác có hai cặp góc tương ứng bằng nhau thì đồng dạng.
  BCF
+ BDM và BCF có BDM  và BMD
  BFC
 , suy ra BDM ∽ BCF (g – g).

 suy ra DB  DC vậy DE vuông góc với BC tại trung điểm N


2). Từ AD là phân giác BAC
của BC .
DM BD
Từ 1). BDM ∽ BCF , ta có  .
CF BC
DA 2 DM 2 BD CD DE
Vậy ta có biến đổi sau     (3).
CF CF BC CN CE
  FCE
Ta lại có góc nội tiếp ADE  (4).

Từ (3) và (4) , suy ra EAD ∽ EFC suy ra EFC  EAD


  90 .

Vậy EF  AC .
Nhận xét. Với bài toán này ta đưa về chứng minh EF tạo với AC một góc vuông. Dựa vào
các góc đã biết và kết nối bởi tam giác đồng dạng.
Nhắc lại kiến thức và phương pháp.
• Hai góc nội tiếp bằng nhau chắn hai cung thì hai cung đó bằng nhau và hai dây cung
của cung đó bằng nhau.
Từ AD là phân giác BAC suy ra BAD
  DAC suy ra DB  DC kết hợp với OB  OC (
 R ) suy ra DO hay DE là trung trực của BC hay DE vuông góc với BC tại trung
điểm N của BC .
• Các góc nội tiếp cùng chắn một cung của đường tròn thì bằng nhau.
  FCE
ADE  (hai góc nội tiếp cùng chắn cung AE của đường tròn (O) )

• Các dữ liệu được suy ra, biến đổi từ những điều đã chứng minh.
DM BD 2 DM 2 BD DA CD
BDM ∽ BCF      
CF BC CF 2CN CF CN
• Hệ thức lượng trong tam giác vuông “Cạnh huyền  Đường cao = Tích hai cạnh góc
vuông”.
Áp dụng hệ thức lượng vào tam giác CDE vuông tại C , đường cao CN ta có
CD DE DA DE
CN .DE  CE.CD   kết hợp với chứng minh trên ta được  kết hợp
CN CE CF CE
  EFC
với trên, ta suy ra EAD ∽ EFC (g – c – g)  EAD .

Liên hệ tài liệu word toán sđt và zalo: 039.373.2038 TÀI LIỆU TOÁN HỌC
53
Website:tailieumontoan.com
• Góc nội tiếp chắn nửa đường tròn là góc vuông.
 chắn nửa đường tròn (O) được chia bởi đường kính ED nên EAD
Góc EAD   90 suy
  90 hay EF vuông góc với AC.
ra EFC

Câu 4.
Với  là số thực dương ta có
d3 a3 b3 dab d 3 b3 c3 dbc d 3 c3 a3 dca a 3  b3  c 3 abc
 3 3 2 ;  3 3 2 ;  3 3 2 ;  2 .
3 3 3  3 3 3  3 3 3  3 2 
Cộng bốn đẳng thức trên ta thu được
 2 1  1
d 3   3  2 a 3  b3  c 3   2 dab  dbc  dca  abc .
 3 
3  
2 1 4 4
Ta tìm   0 sao cho  2   2     3  4 3  3  6
3 3
3 9 3
3
1 1 1 1 3 1
Chọn    x   , ta được  x     x    6
2  x  2  x  2  x 
1 1  3 1 3  1
  x 3  3    x     x    6  x 6  12 x 3  1  0 .

2 x  2  x 2   x
Ta có nghiệm dương là
1 
x  3 6  35 ; x  3 6  35     3 6  35  3 6  35 
2  
Với  xác định như trên ta thu được
4 3 1
d3   a  b3  c 3   2
9 
9 36
 9d 3  4  a 3  b 3  c 3   2  2
.
 3 3 
 6  35  6  35 
 
 1
Đẳng thức xảy ra khi a  b  c  3 ; d= 3 3
3   3 2
36
Vậy giá trị nhỏ nhất của P là 2
.
3 
 6  35  3 6  35 
 
ĐỀ SỐ 4.

Câu 1.
 
1). Giải phương trình  
1  x  1  x 2  2 1  x 2   8 .
 
Điều kiện: 1  x  1 .
Đặt t  1  x  1  x
 t2  1  x  2 1  x1  x  1  x  2  2 1  x2 .

Khi đó phương trình đã cho trở thành: t.t 2  8


 t3  8  t  2  2  2 1  x2  4  1  x2  1  x  0 .
Liên hệ tài liệu word toán sđt và zalo: 039.373.2038 TÀI LIỆU TOÁN HỌC
54
Website:tailieumontoan.com
Vậy phương trình đã cho có nghiệm duy nhất là x  0 .
Nhận xét: Bài toán sử dụng phương pháp đặt ẩn phụ giải phương trình.
Nhắc lại kiến thức và phương pháp:
2
• Hằng đẳng thức quen thuộc u  v  u2  2uv  v 2 .
• Với f  x  a; b  , đặt t  a  f  x  b  f  x , khi đó

t2  a  b
t2  a  b  2 a  f xb  f x  a  f xb  f x  2
.
Ý tưởng: Nhận thấy ở hai căn thức, ta có tổng bình phương của chúng là một hằng số, mặt
khác tích của chúng có liên quan đến biểu thức còn lại trong phương trình.
2 2
• Ta có  1  x    1 x   1  x  1  x  2 ;
2 1  x2  2 1  x1  x .
2 2
• Do đó 2  2 1  x 2   1 x   1 x   2 1  x1  x
2
  1  x  1 x  .
• Đặt t  1  x  1  x , phương trình đã cho trở thành: t 3  8
t2 22 1  x2  4  1  x2  1  x  0 .
Bài toán kết thúc.
Bài tập tương tự:
1. Giải phương trình 6  2 4  x 2  3  2  x  2  x  .

Đáp số: x  2 .
2. Giải phương trình 7  2 2  4 x3  x  2 2  4 x  4 3  x .

54 3
Đáp số: x   .
4


 2 2
x  xy  y  1
2). Giải hệ phương trình  .
 2 2
x  xy  2 y  4


Thế 1  x 2  xy  y 2 xuống vế phải phương trình thứ hai của hệ phương trình, ta có

x 2  xy  2 y 2  4 x 2  xy  y 2 
x  y
 3 x 2  5 xy  2 y 2  0   x  y3 x  2 y  0   .
 3x  2 y

Với y  x , thế vào phương trình thứ nhất trong hệ, ta được
x  1  y  1
x2  1   .
 x  1  y  1

3x
Với 3x  2 y  y  , thế vào phương trình thứ nhất trong hệ, ta được:
2

Liên hệ tài liệu word toán sđt và zalo: 039.373.2038 TÀI LIỆU TOÁN HỌC
55
Website:tailieumontoan.com
 2 3
x  y
7 2  7 7
x 1 

.
4 x   2 3
 y 
 7 7

Vậy hệ phương trình đã cho có bốn nghiệm kể trên.
Nhận xét: Bài toán sử dụng phương pháp thế và tách ghép phương trình đẳng cấp bậc hai
tìm nhân tử chung.
Nhắc lại kiến thức và phương pháp:
• Phương trình đẳng cấp bậc hai có dạng a.x 2  b.xy  c.y 2  0 .
• Nhóm nhân tử chung, đưa về dạng
 x  my
a.x 2  b.xy  c.y 2  0   x  m.y  x  n.y   0   .
 x  ny

Hai trường hợp này, thế vào một trong hai phương trình còn lại của hệ, sẽ được
 x2  t  x   t
phương trình có dạng  .
 y 2  u  y   u
Chú ý: nếu t , u  0 thì phương trình sẽ vô nghiệm.
Ý tưởng: Nhận thấy ở vế trái của mỗi phương trình đều có dạng của phương trình đẳng
cấp bậc hai a.x 2  b.xy  c.y 2 , nếu thế một trong hai phương trình còn lại ta cũng sẽ thu
được một phương trình bậc đẳng cấp bậc hai bằng 0. Từ đó tìm mối quan hệ giữa x , y . Thế
ngược lại phương trình thứ nhất của hệ. Tìm nghiệm.
• Ở vế phải phương trình thứ hai có 4  4.1 mà từ phương trình một 1  x 2  xy  y 2 . Vậy
nên phương trình hai trở thành:

x 2  xy  2 y 2  4 x 2  xy  y 2 
 3x 2  5xy  2 y 2  0   x  y 3x  2 y   0 .
• Có thể dễ dàng giải phương trình theo cách đồng nhất hệ số ở bước làm ra nháp như
sau:

3x 2  5xy  2 y 2  3  x  my  x  ny   3 x 2  mxy  nxy  mny 2 . 
3m  3n  5
Đồng nhất hệ số hai phương trình, ta có  .
3mn  2
2
Đây là phương trình hai ẩn, đã biết cách giải theo Vi-et. Tìm được m  1; n  .
3
Bài toán kết thúc.
Bài tập tương tự:

x 2  xy  y 2  7
1. Giải hệ phương trình  2 .
 2
3x  xy  5 y  5


 5 4   5 4 
Đáp số:  x; y   2; 1 , 2;  1 ,  ;   ,  ;  .
 3 3   3 3 
x 3  x 2 y  y 3  3
2. Giải hệ phương trình  2 .
x  xy 2  3 y 3  3

Đáp số:  x; y   1; 1 , 2;  1 ,  3 3; 0 .
Liên hệ tài liệu word toán sđt và zalo: 039.373.2038 TÀI LIỆU TOÁN HỌC
56
Website:tailieumontoan.com
Câu 2.
1 1 1
1). Đặt a  ; b  ; c   ab  bc  ca  1 , và cần chứng minh:
x y z
a 2b 3c 5bc  4ca  3ab
  
1 a 2
1 b 2
1 c 2
  bb  cc  a
a
a 2b 3c 5bc  4ca  3ab
    .
a  ba  c b  cb  a c  ac  b a  bb  cc  a
Biến đổi rút gọn ta có điều luôn đúng.
Nhận xét: Đặt ẩn phụ để bài toán được giải dễ dàng hơn.
Nhắc lại kiến thức và phương pháp:
1 1 1
• Từ giả thiết ta có x  y  z  xyz     1.
yz zx xy
1 1 1
• Đặt ẩn phụ a  ; b  ; c  ta được giả thiết ab  bc  ca  1 .
x y z
Ta cần chứng minh
a 2b 3c 5bc  4ca  3ab
   .
1 a 2
1 b 2
1 c 2 a  bb  cc  a
• Biến đổi riêng một mẫu ta có
1 1 1 1  1 1  1 1 
1  a2            a  ba  c  .
yz zx xy x 2  x y  x z 
Tương tự ta có 1  b2  b  ab  c  và 1  c 2  c  bc  a .
Ta thấy ở vế phải các mẫu đã đưa được về dạng tích mà mẫu số chung đúng bằng
mẫu số của phân thức ở vế trái. Do đó ta chỉ cần quy đồng, rút gọn thông thường.
a 2b 3c a 2b 3c
VT      
1 a 2
1 b 2
1 c 2  a  b  a  c   b  c  b  a   c  a c  b
a b  c   2b c  a  3c a  b 5bc  4ca  3ab
   VP (điều phải chứng minh).
a  bb  cc  a   bb  cc  a
a

2). Cách 1: Phương trình đối xứng 2 biến x; y nên đặt u  x  y; v  xy ,


v3
ta có: v 2 u  u  3  v  u  .
v2  1
Suy ra v 2  1 v  3  v 2  1 v 2  9  v 2  1 v 2  1  10  v 2  1 10 .
Cách 2: uv 2  v  u  3  0 , ta phải có:
v  1  4u(u  3)  0  u  3  u  0, 1, 2, 3 .
Đáp số: 0; 3 , 3; 0 và 1; 1
Nhận xét: Ta thấy x  y và xy được lặp lại trong phương trình nên ta sẽ đặt ẩn phụ để
phương trình đơn giản hơn.
Nhắc lại kiến thức và phương pháp:
• Sau khi đặt ẩn phụ ta được phương trình nghiệm nguyên v 2 u  u  3  v .
3v
• Biểu diễn ẩn này theo ẩn kia u  là một số nguyên.
v2  1

Liên hệ tài liệu word toán sđt và zalo: 039.373.2038 TÀI LIỆU TOÁN HỌC
57
Website:tailieumontoan.com
• Một phân thức là một số nguyên khi và chỉ khi tử số chia hết cho mẫu số hay mẫu số
là ước của tử số.
Áp dụng vào bài toán:
3  vv2  1  v2  9v2  1  v2  1  10v2  1  v2  1  U10
mà v 2  1  1 nên ta có v 2  1  1; 2 ; 5 ; 10 , suy ra v 2  0 ; 1; 4 ; 9 .

Khi đó v  0 ;  1;  2 ;  3 .
Từ đây ta tìm ra các giá trị của u rồi từ đó tìm ra x , y để đi đến kết luận bài toán.

Câu 3.

1). Tam giác ABF và tam giác ACE lần lượt cân tại F , E và

  A  ABF ∽ ACE .
  ECA
FBA
2
Nhận xét:
Để giải bài toán trên cần sử dụng dấu hiệu nhận biết, tính chất tam giác cân kết hợp với
trường hợp đồng dạng đặc biệt của các tam giác đặc biệt (cụ thể là tam giác cân). Cùng với
các kiến thức liên quan đến tính chất của đường phân giác, các đường thẳng song song.
Nhắc lại kiến thức và phương pháp:
• Tam giác có một đỉnh nằm trên đường trung trực của cạnh đối diện là tam giác cân tại
đỉnh đó.
- Tam giác ABF có F nằm trên đường trung trực của AB nên tam giác ABF cân tại F
.
- Hoàn toàn tương tự với tam giác ACE .
• Đường thẳng c cắt hai đường thẳng a , b song song với nhau tạo ra các góc so le trong
bằng nhau.
  BAD
- Ta có AB cắt hai đường thẳng BF  AD nên FBA  (hai góc so le trong)

  CAD
- Hoàn toàn tương tự có ECA .
 nên BAD
Mà AD là phân giác của BAC   CAD do đó FBA
  ACE.
• Hai tam giác cân có góc ở đỉnh hoặc kề đáy bằng nhau thì đồng dạng với nhau.

Liên hệ tài liệu word toán sđt và zalo: 039.373.2038 TÀI LIỆU TOÁN HỌC
58
Website:tailieumontoan.com
  ACE
Tam giác ABF và tam giác ACE là hai tam giác cân có hai góc ở đỉnh FBA 
nên đồng dạng với nhau.

2). Giả sử G là giao điểm của BE và CF .


GF BF AB DB
Ta có     GD  FB , và FB  AD ta có G  AD .
GC CE AC DC
Nhận xét: Một trong các cách để chứng minh ba đường thẳng đồng quy là ta sẽ chứng
minh giao điểm của hai đường thẳng nằm trên đường thẳng thứ ba.
Một trong các cách để chứng minh ba điểm thẳng hàng là ta sẽ sử dụng tiên đề Ơ-clit.
Đối với bài toán này, ta sẽ sử dụng các tính chất nêu trên, kết hợp với định lý Ta-lét; định
lý Ta-lét đảo; tính chất đường phân giác trong tam giác và tam giác đồng dạng.
Nhắc lại kiến thức và phương pháp:
BF FG
• Định lý Ta-lét: BF  CE   .
CE GC
BF AB
• Tam giác đồng dạng: ABF ∽ ACE   .
CE AC
• Tính chất đường phân giác trong tam giác.
AB BD
AD là đường phân giác trong tam giác ABC ta có  .
AC DC
• Định lý Ta-lét đảo.
GF DB
Kết hợp những chứng minh trên ta được   GD / / BF .
GC DC
• Tiên đề Ơ-clit. Từ một điểm nằm ngoài đường thẳng đã cho có một và chỉ một đường
thẳng song song với đường thẳng đó.
Theo chứng minh trên ta có GD  BF ; theo giả thiết ta có AD  BF . Suy ra AD trùng GD
hay điểm G nằm trên AD .
  QGA
3). Chứng minh BQG   GAE
  GAC
  CAE   GAB
  BAF   GAF
 , nên AGQF nội tiếp, và

QPG   GFQ
  GCE  , suy ra tứ giác FQGP nội tiếp.

Nhận xét: Chứng minh các điểm cùng thuộc một đường tròn ta có thể chứng minh các
điểm đó cách đều một điểm hoặc chứng minh bốn điểm một cùng thuộc một đường tròn
bằng cách chứng minh tứ giác nội tiếp (qua ba điểm không thẳng hàng có một và chỉ một
đường tròn). Ở bài này ta sử dụng hướng tiếp cận thứ hai.
Nhắc lại kiến thức và phương pháp:
+ Một số điều suy ra từ giả thiết và các chứng minh trên.
- ABF ∽ ACE  FAB   EAC

-  nên BAD
AD là phân giác của góc BAC   DAC
.

+ Hai đường thẳng song song.


  QGA
- BF  AD  QGB  (hai góc so le trong)

  GAE
- QG  AE  QGA  (hai góc so le trong)

Suy ra   GAE
QGB   GAC
  CAE
 kết hợp với suy ra ở phần trên ta có
  GAE
QGB   GAC
  CAE
  GAB
  FAB
  FAG
.

-   GCE
BF  CE  QFG  (hai góc so le trong)

+ Tứ giác nội tiếp.


Liên hệ tài liệu word toán sđt và zalo: 039.373.2038 TÀI LIỆU TOÁN HỌC
59
Website:tailieumontoan.com
- Tứ giác có góc trong và góc ngoài tại đỉnh đối diện bằng nhau là tứ giác nội tiếp.
 bằng góc QGB
Tứ giác AFQG có góc trong FAG  là góc ngoài tại đỉnh đối diện nên

AFQG là tứ giác nội tiếp.


- Tứ giác nội tiếp có góc trong và góc ngoài tại đỉnh đối diện bằng nhau.
  ECG
Tứ giác PRGC nội tiếp đường trong ngoại tiếp tam giác EGC nên có QPG  kết

  QFG
hợp với chứng minh ở trên, ta có QPG .

- Tứ giác có hai đỉnh liên tiếp cùng nhìn một dưới một góc bằng nhau là tứ giác nội tiếp.
  QFG
Tứ giác PFQG có QPG  nên hai đỉnh P và F cùng nhìn cạnh QG dưới một góc

bằng nhau do đó PFQG là tứ giác nội tiếp.


Suy ra AFQG; PFQG là các tứ giác nội tiếp nên các điểm A; F ; Q; G; P cùng thuộc một
đường tròn.

Câu 4.
Cách 1: Ta có a4 b2  b4 c 2  2a2 b3 c ; b4 c 2  c 4 a2  2ab2 c 3 ; a4 b2  c 4 a2  2a3 bc 2 .
Nên a4 b2  b4 c 2  c 4 a2  abc a2 b  b2 c  c 2 a .
1 ab  bc  ca
Suy ra a4 b2  b4 c 2  c 4 a2   a4 b2  b4 c 2  c 4 a2 
9 9
ab  bc  ca
 
 abc a2 b  b2 c  c 2 a 
9
A

 1 1 1  1  1  1 
A  abc a2 b  b2 c  c 2 a      abc a2 b    b2 c    c 2 a  
 9 a 9b 9c   9b   9c   9 a 

2 2 2  2 abc
A  abc  a  b  c  a  b  c 
 3 3 3  3
1 2 abc
 a4 b2  b4 c 2  c 4 a2   a  b  c (1).
9 3
Mặt khác a2 b2  b2 c 2  c 2 a2  ab2 c  a2 bc  abc 2

 a2 b2  b2 c 2  c 2 a2  2 ab2 c  a2 bc  abc 2 
  2
 3 ab2 c  a2 bc  abc 2  ab  bc  ca  3abc a  b  c 
1 4 4abc
  abc a  b  c    a  b  c (2).
3 9 3
Từ (1) và (2), suy ra điều phải chứng minh.
1
Dấu “=” xảy ra khi và chỉ khi a  b  c  .
3
Cách 2: Đặt ab  x; bx  y; ac  z với x , y , z  0 .
Vì ab  bc  ca  1  x  y  z  1 ;
5
Suy ra 2  xy  yz  zx   a4 b2  b4 c 2  c 4 a2 .
9
Áp dụng bất đẳng thức Cô-si, ta có

Liên hệ tài liệu word toán sđt và zalo: 039.373.2038 TÀI LIỆU TOÁN HỌC
60
Website:tailieumontoan.com

 a4 b2  b4 c 2  2 a2 b2 bc  2 x 2 y


 5
 4 2 4 2 2 2 2
b c  c a  2b c ac  2 y z  2  xy  yz  zx   x y  y z  z x .

 9
2 2 2
 

c 4 a2  a4 b2  2 a2 c 2 ba  2 z 2 x


Áp dụng BĐT Cô-si, ta có
 2
x y  1 y  2 yx
 9 3

1 2
 y 2 z  z  yz  x 2 y  y 2 z  z 2 x  1  x  y  z  2  xy  yz  zx
 9 3 9 3

1 2
z 2 z  x  zx
 9 3

4 4 2 4 4
  x  y  z  3.  xy  yz  zx   xy  yz  zx
9 9 9 3
5
 x y  y z  z x   2  xy  yz  zx .
2 2 2
9
1
Dấu “=” xảy ra khi và chỉ khi a  b  c  .
3
Nhận xét: Bài toán sử dụng phương pháp kỹ thuật chọn điểm rơi, từ điểm rơi tìm được
khéo léo áp dụng bất đẳng thức Cosi và tổng các đại lượng không âm để suy ra điều phải
chứng minh.
Nhắc lại kiến thức và phương pháp:
• Tổng các đại lượng không âm:
2 2 2
a  b  b  c  a  c  0  a2  b2  c 2  ab  bc  ca .
• Bất đẳng thức Cosi cho hai số thực dương a  b  2 ab .
Ý tưởng: Quan sát thấy, đây vẫn là một bất đẳng thức đối xứng, vai trò giữa các biến là
như nhau vì thế, điểm rơi ban đầu ta sẽ chọn được là a  b  c , kết hợp với giả thiết
1
ab  bc  ca  1 thì tại a  b  c  dấu đẳng thức sẽ xảy ra. Với điểm rơi này, ta sẽ linh
3
hoạt hơn trong việc chứng minh bất đẳng thức bằng cách áp dụng bất đẳng thức Cosi.
Nhưng bất đẳng thức cần chứng minh (tạm gọi là P ) là một bất đẳng thức khá cồng kềnh,
bậc lại lớn vì thế ta sẽ nghĩ đến ẩn phụ để giảm bớt biến của P . Chú ý:
abc a  b  c  ab.ac  ab.bc  ac.bc vì thế nếu đặt x  ab , y  bc , z  ca thì giả thiết trở thành:
x  y  z  1 cũng như:
5 5
P  2  xy  yz  zx 
 a4 b2  b4 c 2  c 4 a2   x 2 a2  y 2 b2  z 2 c 2
9 9
Lại thấy, ở vế phải của P xuất hiện đại lượng  x a ; y b  và a2 b2  x 2 chính vì thế ta cần
2 2 2 2

đánh giá  x 2 a 2 ; y 2 b2  để xuất hiện tích a2 b2 khi đó bất đẳng thức P sẽ đồng bộ theo các
biến x; y; z . Vì thế ta sẽ nghĩ đến chuyện áp dụng bất đẳng thức Cosi cho hai số thực
dương là:
x 2 a 2  y 2 b2  2 xyab  2 x 2 y

 y 2 b2  z 2 c 2  2 yzbc  2 y 2 z  x 2 a 2  y 2 b2  z 2 c 2  x 2 y  y 2 z  z 2 x .

 2 2
z c  x 2 a 2  2 zxca  2 z 2 x

Liên hệ tài liệu word toán sđt và zalo: 039.373.2038 TÀI LIỆU TOÁN HỌC
61
Website:tailieumontoan.com
5
Khi đó, ta chỉ cần chứng minh: x 2 y  y 2 z  z 2 x   2  xy  yz  zx (*).
9
Tiếp tục, với điểm rơi x  y  z ta sẽ có đánh giá sau:
2 2 2
 x  y   y  z   z  x  0  x 2  y 2  z 2  xy  yz  zx
2 2
  x  y  z  3 xy  yz  zx  2  xy  yz  zx  .
3
Do đó, ta sẽ tách bất đẳng thức (*) thành dạng:
2 1 1
 2  xy  yz  zx   x 2 y  y 2 z  z 2 x  .
x2 y  y 2 z  z2 x 
3 9 9
1
Công việc cuối cùng là chỉ cần chứng minh x 2 y  y 2 z  z 2 x  thì bài toán của ta đã hoàn
9
1 1
tất. Vì a  b  c   x  y  z  , với điểm rơi này cân bằng với từng đại lượng
3 3
x 2 y ; y 2 z; z 2 x ta có:
 2
x y  1 y  2 yx
 9 3

1 2
 y z  z  yz  x 2 y  y 2 z  z 2 x  1  x  y  z  2  xy  yz  zx
2
 9 3 9 3

1 2
z z  x  zx
2
 9 3

2 2 1 1
 x2 y  y 2 z  z 2 x    xy  yz  zx   x 2 y  y 2 z  z 2 x  .
9 3 9 9
Từ đó suy ra điều phải chứng minh.
Bài toán kết thúc.
Bài tập tương tự:
1. Cho các số thực dương x; y; z có tích bằng 1 . Chứng minh rằng:
x4 y y4 z z4 x 3
   .
x 1 y 1 z 1 2
2 2 2

2. Cho các số thực dương x; y; z có tổng bằng 1 . Chứng minh rằng:


5
x 2 y  y 2 z  z 2 x   2  xy  yz  zx
9

ĐỀ SỐ 5.

Câu 1.
1). Ta có điều kiện ab  bc  ca  abc  a  b  c  1  a1  b1  c  1 .

1 1 1
VT   
1  1  a  1  a1  b 1  1  b  1  b1  c 1  1  c  1  c1  a

1 1 a 1  a1  b
  
1  1  a  1  a1  b 1  a  1  a1  b  1 1  a1  b  1  1  a
1  1  a  1  a1  b
 1
1  1  a  1  a1  b

Liên hệ tài liệu word toán sđt và zalo: 039.373.2038 TÀI LIỆU TOÁN HỌC
62
Website:tailieumontoan.com

Nhận xét: Bài toán kết hợp giả thiết cũng nhưng cách phân tích nhân tử ở đẳng thức suy
ra điều phải chứng minh.
Ý tưởng: Trước hết, quan sát giả thiết của bài toán, một đẳng thức khá cồng kềnh tuy
nhiên rất đối xứng, vì thế ta sẽ nhóm nhân tử lại như sau:
gt  ab 1  c  b c  1  1  c  a c  1  1
 1  cab  1  a  b  1  1  a1  b1  c  1
Để đơn giản hơn nữa, ta sẽ đặt x  1  a; y  1  b; z  1  c , suy ra xyz  1 . Chính vì thế ta
cần biểu diễn mẫu của các phân số trong đẳng thức cần chứng minh theo x; y; z hay chính
là việc thế a  1  x; b  1  y; c  1  z vào đẳng thức bài cho, ta có:
1 1 1
 
3  ab  2 a  b 3  1  x1  y  2 1  x  1  y 1  x  xy
1 1 1
 VT    .
1  x  xy 1  y  yz 1  z  zx
Kết hợp với giải thiết xyz  1 ta sẽ đưa VT về cùng mẫu như sau:
1 1 1
VT   
1  x  xy 1  y  yz 1  z  zx
1 x xy
  
1  x  xy x  xy  xyz xy  xyz  x 2 yz
1 1 xy 1  x  xy
     1 (điều phải chứng minh).
1  x  xy x  xy  1 xy  1  x 1  x  xy
Bài toán kết thúc.
3
2). Ta có x 3  y 3  6  x 3  y 3  3xy  x  y   x  y
2
 2  32
8 x y  8.
2 2
  (do x  y  0 )
 x  y 
2
 x  y
x  y  2
  x  y  32  
5
 x  y 1.
xy  1
Nhận xét: Bài toán sử dụng phương pháp thế, sau đó đưa về phương trình có chứa mối
liên hệ giữa x; y và thế ngược lại một trong hai phương trình của hệ, tìm nghiệm của hệ
ban đầu.
Ý tưởng: Đây là một hệ phương trình đối xứng, đồng thời lại xuất hiện các tổng x  y và
tích xy nên ta nghĩ đến việc đặt ẩn phụ ( u  x  y; v  xy ) nhưng việc phân tích x  y
3 3

theo u; v sẽ gặp khó khăn. Vì thế hướng đi này không khả thi, quan sát kỹ một chút, ta
thấy rằng phương trình một có hằng số 2  xy  x  y đồng thời hằng số 6 xuất hiện ở
phương trình hai do đó ta nghĩ đến chuyện thế, đồng thời chú ý đến hằng đẳng thức
3
 x  y  x 3  y 3  3 xy  x  y nên phương trình hai trong hệ tương đương với:
3 2 5
 x  y  8 x 2 y 2 mà xy  suy ra  x  y  32  x  y  2 . Khi đó hệ đã cho trở thành:
xy
xy  1
  x  y 1.
x  y  2

Liên hệ tài liệu word toán sđt và zalo: 039.373.2038 TÀI LIỆU TOÁN HỌC
63
Website:tailieumontoan.com
Bài toán kết thúc.
Bài tập tương tự:

2 xy 2 x  y  1

1. Giải hệ phương trình  3 3 2 2
.
8 x  y  6  32 x y


xy 2 x  4 y  1

2. Giải hệ phương trình  3 3 2 2
.
x  8 y  6  32 x y

Câu 2.
2  1  
1). Ta chứng minh đẳng thức  3x   x     x     x
 3  3
Ký hiệu  x  m; x  a  0  a  1; x  m  a .
1
+) Xét trường hợp 0  a 
3
2  2 1 1  2  1
 0  3a  1;  a   1  a   1   3 x  3m;  x    m; x    m
3 4 3 3 3  3   3 
 3m  m  m  m (đúng).
1 2
+) Xét trường hợp  a 
3 3
 1  3a  2
2 2 1
1  a   1;  a   1   3 x  3m  1
3 3 3
 2   1
 x    m  1;  x    m .
 3   3 
 2  1
Suy ra  3x  3m  1  m  1  m  m   x     x     x
 3   3 
2
+) Xét trường hợp  a 1
3
4 2 5 1 4
 2  3a  3;  a   ; 1 a  
3 3 3 3 3
 2   1 
  3 x  3m  2;  x    m  1;  x    m  1 .
 3   3 
Ta thu được [3x]   3m  3m  2  m  1  m  1  m
 2  1
  x     x     x .
 
3   3 
1 2
*) Phương trình đã cho tương đương với  3x  1  1  3x  2   x  .
3 3
Nhận xét. Để giải bài toán trên cần sử dụng phương pháp biến đổi tương đương đưa về
xét khoảng giá trị của nghiệm.
Nhắc lại kiến thức và phương pháp.
• Các ký hiệu cơ bản: “  x là số nguyên lớn nhất không vượt quá x và x là phần lẻ
hay phần thập phân. Do đó x   x  x ”. Ví dụ: Cho số 2,49. Ta có  2,49  2 và
2,49  0,49 .

Liên hệ tài liệu word toán sđt và zalo: 039.373.2038 TÀI LIỆU TOÁN HỌC
64
Website:tailieumontoan.com
• Xét các khoảng giá trị của nghiệm như bài giải đã làm theo đúng phương pháp thông
thường khi làm bài toán phần nguyên.

2). Chúng ta xét các số dư của 52 số trong phép chia cho 100 và chia các số thành các
nhóm theo số dư như sau 0 ,1,99 , ,49,51 ,50. Vì có 51 nhóm nếu tồn tại 2 số có
số dư thuộc cùng một nhóm. Nếu hai số đó đồng dư thì hiệu của chúng chia hết 100 . Nếu
hai số có số dư khác nhau thuộc cùng một nhóm có tổng số dư bằng 100 thì tổng của
chúng chia hết cho 100.
Nhận xét. Áp dụng kiến thức về số dư trong phép chia số nguyên và nguyên lý Dirichlet
(Đi - rich - lê)
Nhắc lại kiến thức và phương pháp.
• Phép chia một số nguyên a cho một số tự nhiên b khác 0 ta được thương là số nguyên
p (p cùng dấu với a) và số dư là số tự nhiên r. Ta có r luôn nhỏ hơn b. Hay r lớn nhất
có thể tồn tại là r  b  1 .
Phép chia một số tự nhiên cho 100 ta có thể nhận được các số dự sau r  0;1; 2; 3;; 99
(có 100 phần tử).
• Tư duy bài toán: Bài toán yêu cầu chứng minh từ 52 số nguyên bất kỳ luôn có thể chọn
ra được hai số có tổng hoặc hiệu chia hết cho 100 .
Từ các số dư r có thể tồn tại trên, chọn và ghép đôi hai số thành nhóm các số dư
0 ,1,99 , ,49,51 ,50 (có 51 nhóm).
• Nguyên lý Dirichlet (phát biểu cho bài toán này): “Có n cái lồng và cần nhốt n  1 con
chim và lồng. Ta luôn có ít nhất một cái lồng chứa 2 con chim.”
Vì ta đang xét với 52 số nguyên bất kỳ nên ta sẽ nhận được 52 số dư bất kỳ khi chia
cho 100 nên sẽ luôn có 1 số dư thuộc cùng một nhóm với một trong 51 số dư còn lại.
+ Nếu số dư đó cùng nhóm có tổng bằng 100 thì chia hết cho 100.
+ Nếu số dư đó cùng nhóm có số bằng nhau khi chia cho 100 (hay đồng dư với nhau
khi chia cho 100) thì có hiệu chia hết cho 100.

Câu 3.

1). Ta có tam giác EPQ cân tại E và CQ là phân giác góc  , nên
BCA
  EQP
EPQ   HQC
  90 0  HCQ
  90 0  PCK
.

  PCK
Do đó EPQ   90 0 , nên PK  AC .

Liên hệ tài liệu word toán sđt và zalo: 039.373.2038 TÀI LIỆU TOÁN HỌC
65
Website:tailieumontoan.com
Nhận xét. Chứng minh hai đường thẳng vuông góc ta đi chứng minh góc đó là góc còn lại
của một tam giác có tổng hai góc bằng 90 .
Nhắc lại kiến thức và phương pháp.
• Điểm thuộc trung trực của một đoạn thẳng cách đều hai mút của đoạn thẳng đó.
Điểm E thuộc trung trực của đoạn thẳng PQ nên EP  EQ .
• Tam giác có hai cạnh bằng nhau là tam giác cân.
Tam giác EPQ có EP  EQ , suy ra tam giác EPQ cân tại E .
• Tam giác cân có hai góc kề cạnh đáy bằng nhau.
  EQP
Tam giác EPQ cân tại E nên ta có EPQ .
• Hai góc đối đỉnh thì bằng nhau.
 và HQC
Ta có PQE  là hai góc đối đỉnh nên PQE
  HQC
.

• Tổng hai góc nhọn trong một tam giác vuông bằng 90 .
  HCQ
Tam giác HQC vuông tại H , nên HQC   90
  90  HCQ
 HQC   90  PCK
 ( CK là phân giác của góc ACB
 ).
  90 0  PCK
Từ các chứng minh trên ta có EPQ   EPQ
  PCK  90 0 .
• Tam giác có tổng hai góc bằng 90 thì góc còn lại là góc vuông.
  PCK
Tam giác KPC có EPQ   90 0 nên PKC
  90 hay PK  AC .

2). Trong tam giác EFC có CQ  EF (do EF là trung trực PQ ); EQ  FC nên FQ  EC.
  90 0 , nên tứ giác EKNM nội tiếp đường tròn đường tròn đường kính EN .
Từ đó EMN
  HCK
Ta có tứ giác EKCH nội tiếp đường tròn đường kính EC nên PEQ .

Chú ý: EF là phân giác góc 


PEQ và CQ là phân giác góc  , do đó
HCK
  1 PEQ
PEF   1 HCK
  PCF
 . Do đó tứ giác PECF nội tiếp.
2 2
Nhận xét. Chứng minh bốn điểm cùng thuộc một đường tròn ta chứng minh bốn điểm đó
tạo thành một tứ giác nội tiếp. Để chứng minh tứ giác do bốn điểm đó tạo thành là tứ giác
nội tiếp ta chứng minh hai đỉnh liên tiếp của tứ giác đó cùng nhìn một cạnh dưới hai góc
bằng nhau.
Nhắc lại kiến thức và phương pháp.
• Đường thẳng nối một đỉnh của tam giác với giao điểm của hai đường cao từ hai đỉnh
còn lại là đường cao thứ ba của tam giác.
Tam giác EFC có CQ  FE (do FE là trung trực của PQ ) và EQ  CF (do AH  BC )
  90 .
suy ra FQ  CE suy ra EMN
• Tứ giác có tổng hai góc đối diện bằng 180 là tứ giác nội tiếp.
  EKN
+ Tứ giác EKNM có EMN   90  90  180 suy ra EKNM là tứ giác nội tiếp
đường tròn đường kính EN .
+ Tứ giác EKCH có EKC  CHE
  90  90  180 suy ra EKCH là tứ giác nội tiếp đường
tròn đường kính EC .
• Tứ giác nội tiếp có góc ngoài tại đỉnh đối diện bằng góc trong không kề với nó.

Liên hệ tài liệu word toán sđt và zalo: 039.373.2038 TÀI LIỆU TOÁN HỌC
66
Website:tailieumontoan.com
 là góc ngoài tại đỉnh E và có góc KCH
Góc PEQ  là góc trong tại đỉnh C không kề với
  KCH
E của tứ giác nội tiếp EKCH nên PEQ .

• Đường phân giác của một góc chia góc đó thành hai góc bằng nhau và bằng một nửa
góc ban đầu.
+ EF là phân giác góc PEQ   1 PEQ
 nên PEF ;
2
+ CQ là phân giác góc HCK   1 HCK
 nên PCF ;
2
  1 PEQ
Suy ra PEF   1 HCK
  PCF .
  HCK
  PEQ
2 2
• Hai đỉnh liên tiếp của tứ giác đó cùng nhìn một cạnh dưới hai góc bằng nhau thì tứ
giác đó là tứ giác nội tiếp.
Tứ giác PECF có hai đỉnh E và C cùng nhìn cạnh PF dưới hai góc bằng nhau
  HCK
PEQ  nên PECF là tứ giác nội tiếp hay bốn điểm P ; E ; C ; F cùng thuộc một

đường tròn

Câu 4. Ta chứng minh đẳng bất đẳng thức:


xy yz zx 6 3 xyz
Với x; y; z  1 thì    (*).
1  z 1  x 1  y 1  3 xyz
Dấu bằng xảy ra khi và chỉ khi x  y  z .
Bất đẳng thức tương đương với
   z  x  1  6 xyz  3
x  y   y  z   3
  1     1
 1  z   1  y 
  1  x  1  3 xyz
 1 1 1  3  9 3 xyz
 1  x  y  z.    .
 1  z 1  x 1  y  1  3 xyz

+) Ta có 1  x  y  z  1  3 3 xyz (1).
+) Với x; y  1 ta chứng minh
1 1 2
  (2)
1  x 1  y 1  xy
2   x  y 2
 
1   x  y  xy 1  xy


 2   x  y  2 xy  xy  x  y  2  2  x  y  2 xy  2 xy 1  xy   x  y  
xy  1  0
2
  xy  1 x y   0 (bất đẳng thức hiển nhiên đúng).
+) Với x; y; z  1 ta chứng minh
1 1 1 3
   (3)
1  x 1  y 1  z 1  3 xyz

1 1 1 1 4
P     .
1  x 1  y 1  z 1  3 xyz 1  3 xyz

Áp dụng kết quả (2) ta thu được


Liên hệ tài liệu word toán sđt và zalo: 039.373.2038 TÀI LIỆU TOÁN HỌC
67
Website:tailieumontoan.com
2 2 4 4
P    .
1  xy 1  z xyz 3 1  xyz xyz
4 3 1  xyz
3

Từ (1) và (3), suy ra (*) đúng.


Trở lại bài toán: Bất đẳng thức đã cho tương đương với
1 1 1 1 1 1 6
  
b c  c a  a b
3
abc .
1 1 1 1
1 1 1 1 3
a b c abc
1 1 1
Với ; ;  1 . Áp dụng (*), suy ra điều phải chứng minh.
a b c
Dấu “=” xảy ra khi a  b  c .
Nhận xét: Bài toán sử dụng bất đẳng thức Cosi và bất đẳng thức bổ đề để chứng minh bất
đẳng thức.
Nhắc lại kiến thức và phương pháp:
• Bất đẳng thức Cosi cho hai số thực dương: a  b  2 ab .
• Bất đẳng thức Cosi cho ba số thực dương: a  b  c  3 abc .
3

1 1 2
• Bất đẳng thức bổ đề: 1  x  1  y  , x; y  0, xy  1 .
1  xy

Ý tưởng: Đây là một bài toán rất khó, ngoài việc biểu diễn theo bất đẳng thức Cosi đòi hỏi
người làm cần kết hợp với bất đẳng thức bổ đề. Nếu chưa được tiếp xúc sẽ không thể làm
được bài này. Và hướng tiếp cận bổ đề như sau:
1 1 2 2   x  y 2
   
1  x 1  y 1  xy 1   x  y  xy 1  xy

 2   x  y  2 xy  xy  x  y  2  2  x  y  2 xy


 2 xy 1  xy   x  y   
xy  1  0
2
  
xy  1 x y   0 (bất đẳng thức hiển nhiên đúng với xy  1 ).
Bài toán kết thúc.
Bài tập tương tự:
Với 0  a; b; c  1 . Chứng minh rằng
1 1 1 3
  
1  a  b 1  b  c 1  c  a 1  2 3 abc .

ĐỀ SỐ 6.
Câu 1.
1). Điều kiện 8  x  2 .
+ Nếu 1  x  2  x  8  9  3
2  x  2  1  1  x  8  2 2  x  3  2  1  loại.
+ Nếu 8  x  1, tương tự ta có
x  8  2 2  x  9  2 1  1  loại.

Liên hệ tài liệu word toán sđt và zalo: 039.373.2038 TÀI LIỆU TOÁN HỌC
68
Website:tailieumontoan.com
Với x  1 , thỏa mãn phương trình.
Đáp số x  1 .
Nhận xét: bài toán sử dụng phương pháp nhẩm nghiệm và đánh giá theo miền nghiệm để
chứng minh nó có nghiệm duy nhất.
Ý tưởng: Đây một bài toán đơn giản nhưng đòi hỏi đi theo phương pháp đánh giá thì cần
đoán trước nghiệm của phương trình. Đầu tiên ta sẽ ưu tiên nghiệm nguyên trước, với
nghiệm nguyên thì các biểu thức chứa căn phải là một số chính phương, tức là
x  8  k 2  
 .

2  x  h 2  

x  8; 2  
Với điều kiện chặn của thì ta sẽ thử một vài giá trị nguyên của x và thấy rằng tại
x  1 thỏa mãn phương trình. Công việc còn lại là ta sẽ đi chứng minh nó là nghiệm duy
 x  8;1  1; 2
 thì phương trình bài cho vô nghiệm.
nhất. Nó là nghiệm duy nhất nếu với
Đi xét từng trường hợp ta có:
x  8;1 x8 2 2x  9 2 1 1
• Với suy ra .
x  1; 2 x8 2 2x  9 2 1 1
• Với suy ra .
x1
Cả hai trường hợp trên đều chứng minh là nghiệm duy nhất của phương trình.
Bài toán kết thúc.
Bài tập tương tự:
7 x  8  x  3x  4
3. Giải phương trình
x  9  2 1  x  1.
4. Giải phương trình

2). Ta có x  3 y  xy  3   x  31  y  0 .
+ Với x  3  9  y 2  3 y  3  y 2  3 y  6  0 (vô nghiệm).
x  1
+ Với y  1  x 2  1  x  3  x 2  x  2  0   .
 x  2
Đáp số  x; y  1; 1 , 2; 1 .
Nhận xét: Phân tích nhân tử ở phương trình một, sau đó thế vào phương trình hai tìm
nghiệm của hệ.
Ý tưởng: Đây là một bài toán rất dễ, khi ý tưởng tác giả đã cho lộ ngay từ phương trình
x  3 y  xy  3   x  31  y  0
một, vì đi từ đó ta có .
Xét từng trường hợp một, thế vào phương trình hai giải phương trình bậc hai sẽ tìm được
nghiệm của hệ phương trình.
Bài toán kết thúc.
Bài tập tương tự:
2 x 2  y  xy  2 x

1. Giải hệ phương trình x 2  y 2  x  y  4 .

x 2  y  xy  x

2. Giải hệ phương trình x 2  y 2  x  y  4 .

Liên hệ tài liệu word toán sđt và zalo: 039.373.2038 TÀI LIỆU TOÁN HỌC
69
Website:tailieumontoan.com
Câu 2.
1). A  x y  xy  xy  x  y 
5 5 4 4

+ Nếu x hoặc y chẵn, suy ra A  2 , trái lại nếu x; y cùng lẻ, nên  x 4  y 4   2  A  2  A
luôn chia hết cho 2.
+ Nếu x hoặc y chia hết cho 3 , suy ra A  3 nếu x; y cùng không chia hết cho 3 , nên
x 2  y 2  1(mod 3)  x 4  y 4  1(mod 3)   x 4  y 4   3  A luôn chia hết cho 3.
+ Nếu x hoặc y chia hết cho 5, suy ra A  5 .
Nếu x; y cùng không chia hết cho 5, suy ra x 2 , y 2  1,4(mod 5)  x 4 , y 4  1(mod 5)  A  5 ,
nên A luôn chia hết cho 5.
Vậy A 2.3.5  30 .
Nhận xét. Một số vừa chia hết cho 2, 3 và 5 thì chia hết cho 30 vì 2, 3 và 5 là các số nguyên
tố cùng nhau
Nhắc lại kiến thức và phương pháp.
• Trong một tích có một thừa số là số chẵn thì tích đó chia hết cho 2.
Nếu x hoặc y là số chẵn thì A  2 .
• Hiệu của hai số lẻ là một số chẵn. Số chẵn luôn chia hết cho 2.
Nếu x và y đều là số lẻ thì x 4 hoặc y 4 đều là số lẻ thì x 4  y 4 là số chẵn suy ra A  2 ,
suy ra A luôn chia hết cho 2 .
• Trong một tích có một số chia hết cho 3 thì tích đó chia hết cho 3.
Nếu x hoặc y là số chia hết cho 3 thì A  3 .
• Một số nguyên dương khi chia cho 3 dư 0, 1 hoặc 2 nên lũy thừa bậc bốn của số đó
chia 3 dư 0 hoặc 1.
Nếu x; y cùng không chia hết cho 3 thì x 4  y 4  1(mod 3)   x 4  y 4   3 , suy ra A luôn
chia hết cho 3 .
• Trong một tích có một số chia hết cho 5 thì tích đó chia hết cho 5.
Nếu x hoặc y là số chia hết cho 5 thì A  5 .
• Một số nguyên khi chia cho 5 thì dư 0, 1, 2, 3 hoặc 4 nên bình phương của số đó chia 5
dư 0, 1 hoặc 4 do đó lũy thừa bậc bốn của số đó chia 5 dư 0 hoặc 1.
Nếu x; y cùng không chia hết cho 5 thì x 4  y 4  1 (mod 5)   x 4  y 4   5 , suy ra A luôn
chia hết cho 5 .
• Một số nguyên dương chia hết cho các số nguyên tố cùng nhau thì số nguyên dương
này chia hết cho tích của các số nguyên tố cùng nhau đó.

 A2



Ta có  A  3 (chứng minh trên) mà 2, 3 và 5 là các số nguyên tố cùng nhau nên


A  5


A  2.3.5 hay A  30 .

2). Theo bất đẳng thức Cosi


a3 b1 b1 3
   a.
b  1
2
8 8 4

Liên hệ tài liệu word toán sđt và zalo: 039.373.2038 TÀI LIỆU TOÁN HỌC
70
Website:tailieumontoan.com
b3 a 1 a 1 3
   b.
a  1
2
8 8 4

ab 1 3 1 1 1
P   a  b  P  a  b   .
4 2 4 2 2 2
Dấu “=” xảy ra khi a  b  1.
1
Vậy Pmin  .
2
Nhận xét: Bài toán sử dụng kỹ thuật đánh giá qua bất đẳng thức Cosi khi đoán được điểm
rơi, hay còn gọi là kỹ thuật chọn điểm rơi trong bất đẳng thức Cosi.
Nhắc lại kiến thức và phương pháp:
• Bất đẳng thức Cosi cho ba số thực dương a  b  c  3 3 abc .
Ý tưởng: Trước hết quan sát thấy đây là một bất đẳng thức đối xứng khi vai trò của hai
biến như nhau vì thế ta khẳng định a  b , đồng thời kết hợp với giả thiết sẽ có điểm rơi
của bài toán là a  b  1 . Mặt khác, ở biểu thức P cả hai phân số đều có dạng bậc ba trên
bậc hai và sử dụng giả thiết thì ta cần tìm số k sao cho
a3 b3
P 2
 2
 k a  b  h  2 k  h . Với điểm rơi đã dự đoán a  b  1 , ta thấy rằng
b  1 a  1
a3 b3 1
  . Vì vậy, để sử dụng được bất đẳng thức Cosi cho ba số thì ta cần tạo
b  1
2
a  1
2
8

1
ra thêm hai lượng . Do đó, ta sẽ đánh giá như sau:
8
a3 b1 b1 3 b3 a 1 a 1 3
   a ;    b
b  1
2
8 8 4 a  1
2
8 8 4

a3 b3 1 1 1
P   a  b   .
b  1
2
a  1
2
2 2 2

Bài toán kết thúc.


Bài tập tương tự:
1. Cho x; y là hai số thực dương thỏa mãn x; y  4 . Tìm giá trị nhỏ nhất của biểu thức
x y 12
P   .
2 y x xy
2. Cho x; y là hai số thực dương thỏa mãn x  y  2 . Tìm giá trị nhỏ nhất của biểu
thức P  x 3  y 3  2  x 2  y 2  .
Câu 3.

Liên hệ tài liệu word toán sđt và zalo: 039.373.2038 TÀI LIỆU TOÁN HỌC
71
Website:tailieumontoan.com

A
D

M
E
C
B F K

1). K  tiếp xúc AB tại T , nên KT  AB , suy ra KT / / AB .


  KTC
Chú ý tam giác KTC cân nên KCT   TCA
  TCB
 . nên K thuộc BC.

Nhận xét. Chứng minh một điểm thuộc một đoạn thẳng ta quy về chứng minh ba điểm
thẳng hàng.
Nhắc lại kiến thức và phương pháp.
• Đường thẳng tiếp xúc với đường tròn (tiếp tuyến) vuông góc với bán kính của đường
tròn tại tiếp điểm.
Đường tròn ( K ) tiếp xúc AB tại T , nên KT  AB tại T
• Đường thẳng c vuông góc với a và b nên a và b là hai đường thẳng song song.
  90 ) suy ra KT  AC .
Ta có KT  AB và AB  AC (do BAC
• Hai đường thẳng song song tạo ra các góc so le trong bằng nhau.
  TCA
Ta có KT  AC suy ra KTC  nên suy ra
 mà CT là phân giác của góc ACB

  TCB
ACT   TCB
 từ đây suy ra KTC .

• Tam giác có hai cạnh bằng nhau là tam giác cân. Tam giác cân có hai góc kề đáy bằng
nhau.
  KCT
Tam giác KTC có KT  KC  R suy ra KTC cân tại K do đó KTC  kết hợp với
  KCT
chứng minh từ ý (3) ta có TCB .

• Hai góc có cùng số đo, chung một cạnh, cạnh còn lại của mỗi góc cùng nằm một phía
so với cạnh chung thì trùng nhau.
  TCB
Ta có KTC  suy ra KC  CB hay ba điểm K ; C ; B hay K nằm trên BC .

2). Gọi ( K ) giao BC tại F khác C. Ta thấy tứ giác FEDC nội tiếp và chú ý K thuộc BC nên
  90 0 .
FEC

Liên hệ tài liệu word toán sđt và zalo: 039.373.2038 TÀI LIỆU TOÁN HỌC
72
Website:tailieumontoan.com
  90 0  ADB
Từ đó ABD   90 0  EFC .
  BCE
Nhận xét. Chứng minh hai góc bằng nhau đưa về biến đổi số đo hai góc cùng bằng một
lượng.
Nhắc lại kiến thức và phương pháp.
• Dây cung đi qua tâm đường tròn là đường kính.
Ta có dây cung FC đi qua tâm K nên FC là đường kính của đường tròn đã cho.
• Góc nội tiếp chắn nửa đường tròn là góc vuông.
 là góc nội tiếp chắn nửa đường tròn đã cho được chia đôi bởi đường kính
Ta có FEC
  90 do đó CEF
FC nên suy ra FEC   ECF
  90  ECF
  90  CEF
 mà ABD vuông tại
  ADB
A nên ta có ABD   90  ABD
  90  ADB

• Góc ngoài tại đỉnh đối diện bằng góc trong không kề với nó của một tứ giác nội tiếp.
 là góc ngoài tứ giác nội tiếp EDCF nên ADE
Góc ADE   EFC
 kết hợp với (7) ta được
  ABD
ECF .

  BCM
3). Từ trên, suy ra MBE  do đó MBE ∽ MCB  ME.MC  MB2 .
Từ đó chú ý MT tiếp xúc ( K ) , suy ra MT 2  ME.MC  MB2 .
Vậy M là trung điểm BT .
Nhận xét. Chứng minh một điểm là trung điểm của một đoạn thẳng ta chứng minh điểm
đó nằm trên đoạn thẳng và cách đều hai đầu mút của đoạn thẳng.
Nhắc lại kiến thức và phương pháp.
• Tam giác có hai cặp góc bằng nhau thì đồng dạng
- Xét MBE và MCB có:
 chung;
+ BMC
  BCM
+ MBE  (chứng minh phần 2),
ME MB
suy ra MBE ∽ MCB (g – g), suy ra   ME.MC  MB2 .
MB MC
- Xét MTE và MCT có:
 chung;
+ TMC
  TCM
+ MTE  (góc tạo bởi tia tiếp tuyến - dây cung và góc nội tiếp cùng chắn một
cung của đường tròn ( K ) ),
MT MC
suy ra  MTE ∽ MCT (g – g), suy ra   ME.MC  MT 2 .
ME MT
Từ đây suy ra MB2  MT 2  MB  MT mà ba điểm M ; B ; T thẳng hàng nên suy ra M
là trung điểm của BT .

Câu 4. Theo bất đẳng thức Cô si:


1  a2  1  b2  2 1  a2 1  b2  2 4 1  a2 1  b2 .
Theo bất đẳng thức Bunhia cốpxki:
1  a 1  b   1  a b
2 2 2 2
 1  ( a  b)2

 1  a2  1  b2  2 a  b
Tương tự: 1  b2  1  c 2  2 b  c  1  c 2  1  a 2  2 c  a

Liên hệ tài liệu word toán sđt và zalo: 039.373.2038 TÀI LIỆU TOÁN HỌC
73
Website:tailieumontoan.com
Cộng cả ba bất đẳng thức trên rồi chia cho 2 ta có
1  a2  1  b2  1  c 2  a  b  b  c  c  a
Dấu “=” xảy ra khi a  b  c  1.
Nhận xét: Bài toán kết hợp cả hai bất đẳng thức quen thuộc là Cosi và Bunhiacopxki để
chứng minh bất đẳng thức.
Nhắc lại kiến thức và phương pháp:
• Bất đẳng thức Cosi cho hai số thực dương: a  b  2 ab .
• Bất đẳng thức Bunhiacopxki cho bộ số a; b; c; d :
 a 2  b2 c 2  d 2  .
2
ac  bd
Ý tưởng: Bất đẳng thức bài cho đối xứng, vai trò các biến là hoàn toàn như nhau suy ra
điểm rơi hay nói cách khác tại a  b  c  1 thì dấu đẳng thức xảy ra. Sẽ có bạn hỏi việc dự
đoán trước điểm rơi để làm gì. Thực ra công việc này rất quan trọng giúp ta phát hiện ra
được cần áp dụng bất đẳng thức gì và áp dụng như thế nào. Như bên trên đã nói, vai trò
a; b; c như nhau nên ta chỉ cần chứng minh 1  a 2  1  b2  k a  b (*) và các biểu thức
còn lại đánh giá tương tự. Với điểm rơi a  b  1 để bất đẳng (*) thức xảy ra dấu “=” khi và
chỉ khi 2  2  2 k  k  2 . Và vì thế ta sẽ đi chứng minh: 1  a2  1  b2  2 a  b .
Áp dụng bất đẳng thức Cosi ta có: 1  a 2  1  b2  2 4 1  a 2 1  b2  do đó, ta chỉ cần

chứng minh 1  a 2 1  b2   a  b . Đây là một kết quá quen thuộc, ta có thể biến đổi
2

tương đương hoặc áp dụng bất đẳng thức Bunhiacopxki như sau:
 1  a 1  b 
2
1.a  b.1 2 2

 1  a 2  1  b2  2 4 1  a 2 1  b2   2 a  b (điều phải chứng minh).


Bài toán kết thúc.
Bài tập tương tự:
1. Cho các số thực dương a; b; c . Chứng minh rằng
 a  b  c
2

 3 a bc  b ca  c ab . 
2. Cho các số thực dương a; b; c . Chứng minh rằng
1  a2 1  b2 1  c2
  3.
bc ac ab

ĐỀ SỐ 7.
Câu 1.
1). Điều kiện x  2 .
Đặt t  x  2  x  2  0
 t 2  x  2  x  2  2 x2  4  2 x  2 x2  4
Phương trình đã cho tương đương x  2  x  2  2x  2 x2  4  6
 t 2  t  6  0  t  2 hoặc t  3 (loại).
Với t  2 ta có x  2  x  2  2
Do điều kiện x  2 ta có x  2  x  2  4  0  2 .
Liên hệ tài liệu word toán sđt và zalo: 039.373.2038 TÀI LIỆU TOÁN HỌC
74
Website:tailieumontoan.com
Suy ra phương trình có nghiệm duy nhất x  2 .
Nhận xét: Bài toán sử dụng phương pháp đặt ẩn phụ, đưa phương trình ban đầu về
phương trình bậc hai tìm ẩn, sau đó dùng phương pháp nâng lũy thừa tìm nghiệm của
phương trình ban đầu.
Nhắc lại kiến thức và phương pháp:
• Cách giải phương trình bậc hai tổng quát a.t 2  b.t  c  0 .
2
• Hằng đẳng thức a2  b2  a  ba  b và a  b  a2  2ab  b2 .
• Phương trình có dạng f  x  g  x  m , với m là số thực dương thì có hai cách nâng
lũy thừa như sau:
Cách 1. Bình phương hai vế của phương trình, ta có:

 f  x  0; g  x  0

f  x  g  x  m   

 f  x  g  x  2 f  x . g  x  m 2



 f  x  0; g  x  0  x  x1
 
2  .
4 f  x.g  x   m  f  x  g  x  x  x2
2

Cách 2. Chuyển g  x sang VP rồi bình phương, ta có:


m  g x
  
f  x  m  g  x  
 f  x  m2  2 m g  x  g  x

 




m  g  x  m  g  x
  .
         
2



2 m g x  m 2
 f x  g x 


4 m 2
g   
x  m 2
 f    
x  g x
Chú ý:
• Nếu f  x  g  x  k ; k là hằng số thì ta có thể sử dụng cách liên hợp như sau:
f  x  g  x  m ( i ).

  f  x  g  x   
f  x  g  x  m f  x  g  x 
k
 f  x  g  x  m  
f  x  g  x  f  x  g  x 
m
( ii ).
2
k  k
Lấy ( i ) + ( ii ), ta được 2 f  x  m   4 f  x  m   .
m  m
• Nếu c  a  b và x  c suy ra x  a  x  b  c  a  c  b .
Dấu “=” xảy ra khi và chỉ khi x  c .
Ý tưởng: Bài toán xuất hiện ba căn thức bậc hai ở VT là x2, x  2 và x 2  4 , áp dụng
hằng đẳng thức a2  b2  a  ba  b dễ thấy được rằng x 2  4  x  2. x  2 , hay nói
cách khác căn thức cuối cùng chính là tích của hai căn thức còn lại. Đồng thời nếu chuyển
2 2
2 3  x từ VP sang VT thì sẽ xuất hiện 2x , mà 2 x  x  2  x  2   x2   x2  do
đó VT của phương trình ban đầu có:
2 2
 x2   x2   2. x  2. x  2  x  2  x  2  6  0 .

Liên hệ tài liệu word toán sđt và zalo: 039.373.2038 TÀI LIỆU TOÁN HỌC
75
Website:tailieumontoan.com
2
  x2  x2  x  2  x  2  6  0 (*).

• Đặt t  x  2  x  2  0 thì phương trình (*) được viết lại thành


t  0 t  0
2   t2.
t  t  6  0 t  2t  3  0
 
x  2
• Với t  2 suy ra x  2  x  2  2  
2 x  2 x 2  4  4


 x  2
  2  x2.

 x 4 2x

Đến đây có thể đánh giá như lời giải là:
x2 x2  x2  4  0 2 x2.
• Vì x  2   x  2  4 nên giải phương trình x  2  x  2  2 theo chú ý như sau:

x2  x2 2 42  x2  x2 


 x2  x2 22 x2 4 x2.
Bài toán kết thúc.
Bài tập tương tự:
1. Giải phương trình 2 x  1  2 x  1  2 x 2  1  3  2 x .
Đáp số: phương trình vô nghiệm thực.
2. Giải phương trình 2 x 2  5  2 x 2  x  2  5 x  1  5 x  2 .
17
Đáp số: x  .
16
2). Cộng từng vế hai phương trình ta có 2 x 2  xy  3x  y  1
 x  2 x  y  x   2 x  y  1  0
  x  12 x  y  1  0 .
TH1: x  1  y 2  y  0  y  0 hoặc y  1 (thỏa mãn).
TH2: 2 x  y  1  y  1  2 x , suy ra
x  0  y  1
2 2 2 
x  x 1  2 x  1  2 x  1  7 x  5x  0   .
x  5  y   3
 7 7
 5 5
Đáp số  x; y  1; 0 , 1; 1 , 0; 1 ,  ;   .
 7 7 
Nhận xét: Bài toán sử dụng phương pháp hằng số biến thiên tìm ra được một phương
trình biển diễn mối liên hệ giữa hai biến và từ đó thế ngược lại một trong hai phương
trình, tìm nghiệm của hệ.
Ý tưởng: Đây là hệ phương trình bậc hai, trước hết ta sẽ đi tìm nhân tử ở từng phương
trình một trong hệ, nếu công việc này thất bại. Ta sẽ nghĩ đến việc kết hợp cả hai phương
trình. Và điều tối ưu ta nghĩ tới sẽ là xét đenta theo ẩn x hoặc y từng phương trình (bạn
đọc từ làm) khi đó không tìm được nhân tử x; y . Chính vì thế, còn hướng duy nhất đó là
kết hợp hai phương trình của hệ, giả sử tồn tại k   thỏa mãn phương trình:
 
k x 2  xy  y 2  1  x 2  2 xy  y 2  3 x  y  2  0

Liên hệ tài liệu word toán sđt và zalo: 039.373.2038 TÀI LIỆU TOÁN HỌC
76
Website:tailieumontoan.com

 1  k  x 2  2  k  y  3 x   k  1 y 2  y  2  k  0 ( i ).
 
Và ta coi ( i ) là phương trình bậc hai ẩn x đồng thời khi xét x nó phải là một số chính
phương. Ta có:
2
x  2  k y  3  4 1  k   k  1 y 2  y  2  k 
   
 
 5 k 2  4 k y 2  10 k  8 y  4 k 2  4 k  1 .

Đế x là số chính phương khi hệ số y 2 phải là một chính phương, tức là ta đi giải


phương trình nghiệm nguyên 5k 2  4 k  m2 . Không khó để ta thấy rằng k  1  m  1 thỏa
2
mãn. Hay nói cách khác: x   y  1 . Khi đó phương trình (i)
 3  y  y 1
x  1
 2
 x 2   y  3 x  1  y  0   .
 3 y y 1
x   2y
 2
Việc còn lại là thế x  1 hoặc x  y  2 vào phương trình một để tìm nghiệm của hệ
phương trình.
Bài toán kết thúc.
Bài tập tương tự:
x 3  y 3  3 y 2  9
1. Giải hệ phương trình  2 .
x  y 2  x  4 y

 1  13 5  13 
Đáp số:  x; y   ;  .
 2 2 
x 2  2 xy  2 y 2  3 x  0
2. Giải hệ phương trình  .
xy  y 2  3 y  1  0

 1  5 

Đáp số:  x; y  3  2 2; 1  2 , 3  5; 
.
2 
Câu 2.
1). Dễ thấy với x  0 hoặc y  0 không thỏa mãn.
Xét x , y  1 do vai trò như nhau, giả sử x  y
Khi đó ta có x 2  xy  y 2  3x 2
Suy ra x 2 y 2  x 2  xy  y 2  5  8 x 2  y 2  8  y  1, 2 .
+ Nếu y  1  x 2  x  6  x 2  x  6 .
+ Nếu y  1  x 2  x  6  x 2  x  6 .
+ Nếu y  2  x 2  2 x  4  4 x 2  5  x   loại.
+ Nếu y  2  x 2  2 x  4  4 x 2  5  x   loại.
Đáp số:  x; y  6; 1 , 6;  1 , 1; 6 , 1;  6 .
Nhận xét. Bài toán nghiệm nguyên giải bằng phương pháp giới hạn.
Nhắc lại kiến thức và phương pháp.

Liên hệ tài liệu word toán sđt và zalo: 039.373.2038 TÀI LIỆU TOÁN HỌC
77
Website:tailieumontoan.com
• Khi thay thế x bởi y và ngược lại ta thấy đẳng thức không đổi thì vai trò của các biến
là như nhau.
Dễ thấy với x  0 hoặc y  0 không thỏa mãn.
Xét x , y  1 do vai trò như nhau, giả sử x  y .
Khi đó ta có x 2  xy  y 2  3x 2
• Thay thế vế này bởi vế kia để áp dụng tính chất của vế còn lại và biến đổi theo tính
chất, suy ra
x 2 y 2  x 2  xy  y 2  5  8 x 2  y 2  8  y  1, 2 .
• Tìm và chọn ra các giá trị của biến
+ Nếu y  1  x 2  x  6  x 2  x  6 .
+ Nếu y  1  x 2  x  6  x 2  x  6 .
+ Nếu y  2  x 2  2 x  4  4 x 2  5  x   loại.
+ Nếu y  2  x 2  2 x  4  4 x 2  5  x   loại.
2). Trước hết ta chứng minh với x; y; z; t  bất kì thì
2 2
x2  y 2  z2  t 2   x  z   y  t (*).
Thật vậy, bất đẳng thức (*) tương đương với
x2  y 2  z2  t 2  2 x2  y2 z2  t 2   x2  2xz  z2  y2  2 yt  t 2
  x 2  y 2  z 2  t 2   xz  yt

Đúng vì theo bất đẳng thức Bunhia cốp xki

x2  y2 z2  t 2    xz  yt


2
  xz  yt    xz  yt  .

Áp dụng (*) ta có P  4  x 4  4  y 4  4  z 4
2
  2  2
2

 x2  y 2   4  z4

2
  2  2  2
2

 x2  y 2  z2 
2

 36  x 2  y 2  z 2  .
2 2 2 2 2 2
Ta có  x  1   y  1   z  1   x  y   y  z   z  x  0
 3 x 2  3 y 2  3 z 2  3  2 x  2 y  2 z  2 xy  2 yz  2 zx
 3 x 2  3 y 2  3 z 2  3  2.6  12  x 2  y 2  z 2  3 .
Từ đó P  36  9  3 5 .
Dấu “=” xảy ra x  y  z  1 .
Vậy min  3 5 .
Nhận xét: Bài toán sử dụng kết quả mở rộng (hay phát triển từ bất đẳng thức
Bunhiacopxki) kết hợp với kỹ thuật chọn điểm rơi (để đánh giá tổng các đại lượng không
âm) để tìm giá trị nhỏ nhất của biểu thức.
Nhắc lại kiến thức và phương pháp:
Liên hệ tài liệu word toán sđt và zalo: 039.373.2038 TÀI LIỆU TOÁN HỌC
78
Website:tailieumontoan.com
• Bất đẳng thức Bunhiacopxki cho bốn số x; y; z; t

  
( x 2  y 2 z 2  t 2  xz  yt .

• Phát triển tử bất đẳng thức trên, ta có:


2 x2  y2 z2  t 2   xz  yt  2 x2  y2 z2  t 2   2xz  2 yt
  x 2  y 2   2  x 2  y 2  z 2  t 2   2  z 2  t 2    x  z   y  t 
2 2

 2 2 2
  x 2  y 2  z 2  t 2    x  z   y  t 
 
2 2
 x2  y 2  z2  t 2   x  z   y  t  (*).
Ý tưởng: Bài toán ở cả biểu thức cực trị cũng như giả thiết, vai trò của các biến là như nhau
do đó điểm rơi sẽ xảy ra tại x  y  z  k  0 . Và thay ngược lại giả thiết, ta có:
3 k 2  2 k  6  0
  k  1  x  y  z  1 . Với điểm rơi này, ta sẽ đánh giá thoải mái hơn. Đầu

k  0

tiên, xét tới biểu thức P có dạng bậc bốn vì thế ta đánh giá để bậc bé nhất có thể. Áp dụng
bất đẳng thức (*), xét cho hai căn thức đầu của P , có:
2 2
4  x4  4  y 4   2  2
2

 x2  y 2  
 42  x2  y 2 
2 2

42  x2  y 2  
 4  z 4  36  x 2  y 2  z 2  .
2

Suy ra P  36  x 2  y 2  z 2  , và bây giờ từ giả thiết ta chỉ cần tìm được x 2  y 2  z 2  m
coi như bài toán đã kết thúc. Thật vậy, với điểm rơi ban đầu tìm được ta sẽ có các đại
lượng không âm là:
2 2 2 2 2 2
x  1   y  1   z  1  0 và  x  y   y  z   z  x  0 .
Cộng hai đánh giá trên theo từng vế và khai triển bình phương ta được:
3 x 2  3 y 2  3 z 2  3  2 x  2 y  2 z  2 xy  2 yz  2 zx
 3 x 2  3 y 2  3 z 2  3  12  x 2  y 2  z 2  3 .
2

Do đó P  36  x 2  y 2  z 2   3 5  Pmin  3 5  x  y  z  1 .

Bài toán kết thúc.


Bài tập tương tự:
1. Cho x; y là hai số thực dương. Chứng minh rằng
2
9  x 2  9  y 2  36   x  y .
2. Cho x; y; z là các số thực dương thỏa mãn xyz  x  y  z . Tìm giá trị nhỏ nhất của
biểu thức
P  9  x2  9  y 2  9  z2 .

Liên hệ tài liệu word toán sđt và zalo: 039.373.2038 TÀI LIỆU TOÁN HỌC
79
Website:tailieumontoan.com
Câu 3. M
A
1). Tứ giác BMNQ nội tiếp suy ra
  BQN
BMN   180.
P
  BMN
Mà BPR  (do MN  BC ).
  BQN
Từ đó BPR   1800 , suy ra tứ giác
N
BPRQ nội tiếp. Tức là B; P ; R; Q cùng
O
thuộc một đường tròn. R
Nhận xét. Chứng minh bốn điểm cùng F
thuộc một đường tròn đưa về chứng E
minh tứ giác tạo bởi bốn điểm là tứ giác B D G C
nội tiếp bằng dấu hiệu nhận biết có
tổng hai góc đối diện bằng 180 .
Nhắc lại kiến thức và phương pháp.
Q
• Tứ giác nội tiếp một đường tròn có
tổng hai góc đối diện bằng 180 .
Tứ giác BMNQ nội tiếp đường tròn
  BQN
(O) nên ta có BMN   180.

• Một đường thẳng cắt hai đường thẳng song song tạo ra các góc đồng vị bằng nhau.
  BMN
AC  MN nên ta có BPR  (hai góc đồng vị).

• Tứ giác có tổng hai góc đối diện bằng 180 là tứ giác nội tiếp.
  BQN
Ta có BPR   180 , suy ra tứ giác BPRQ hay bốn điểm B ; P ; R ; Q cùng thuộc

một đường tròn.

2). Gọi PQ giao BC tại D, AQ giao BR tại E ta có các biến đổi góc sau
  DQB
EQD   PRB
  AQB   ACB  RBC .
  EBD
  BDQ
Vậy tứ giác BEDQ nội tiếp, suy ra BEQ   900  BR  AQ .

Nhận xét. Chứng minh hai đường thẳng vuông góc ta chứng minh góc tạo bởi hai đường
thẳng đó bằng 90 .
Nhắc lại kiến thức và phương pháp.
• Hai góc nội tiếp cùng chắn một cung thì bằng nhau.
  PRB
+ PQB  của đường tròn ngoại tiếp tứ giác
 (hai góc nội tiếp cùng chắn cung BP
BPRQ ).
  ACB
+ AQB  (hai góc nội tiếp cùng chắn cung AB  của đường tròn (O) )

• Góc ngoài tại một đỉnh của tam giác bằng tổng hai góc trong không kề với nó.
 là góc ngoài tại đỉnh B của tam giác BRC nên BRA
Góc BRA   RBC
  RCB
.
• Tứ giác có hai đỉnh liên tiếp cùng nhìn một cạnh dưới hai góc bằng nhau là tứ giác nội
tiếp.
  AQB
Ta có PQB   PRB
  ACB
  RBC
  RCB
  ACB
  RBC

Liên hệ tài liệu word toán sđt và zalo: 039.373.2038 TÀI LIỆU TOÁN HỌC
80
Website:tailieumontoan.com
  RBC
 PQA  suy ra tứ giác EDQB có hai đỉnh B và Q cùng nhìn cạnh ED dưới hai
góc bằng nhau nên EDQB là tứ giác nội tiếp.
  BDQ
BEQ  (hai góc nội tiếp cùng chắn cung BQ
 của đường tròn ngoại tiếp tứ giác

EDQB ).
  90 (do PQ vuông góc với BC tại D ), suy ra BEQ
Mà BDQ   90 hay BR  AQ .

  BRQ
3). Ta có BPQ   RBN
  RNB
  EBF
  BAE
  900  BFE
  900  ABE

  ABE
 1800  BFE   AFB
  ABR
.
  BPQ
Do đó AFB   ABR
.

Nhận xét. Chứng minh tổng của hai góc bằng một góc khác, trong bài này ta đưa về chứng
minh hiệu của hai góc bằng góc còn lại a  b  c  c  a  b .
Nhắc lại kiến thức và phương pháp.
Với phần này, ta chỉ sử dụng những kiến thức và các dữ kiện đã cho ở đề bài và tìm ra ở
các phần trước đó.
  BRQ
+ BPQ  (hai góc nội tiếp cùng chắn cung BQ
 của đường tròn ngoại tiếp tứ giác

BPRQ ).
  RBN
+ BRQ   RNB
 (tính chất góc ngoài tam giác).
  BAE
+ RNB  (hai góc nội tiếp cùng chắn cung BQ
 của đường tròn (O) )
  BRQ
suy ra BPQ   RBN
  RNB
  EBF
  BAE
  90 BFE
  90 ABE

   
   ABE
 1800  BFE 
  AFB
  ABR
  AFB
  BPQ
  ABR
.

11a3  b3
Câu 4. Ta chứng minh  3a  b
4 a 2  ab

 11a 3  b3  3a  b 4 a 2  ab 
 11a3  b3  12 a 3  4 a 2 b  3a 2 b  ab2
 a 2 b  ab2  a 3  b3
 ( a  b)( a  b)2  0 đúng.
11b3  c 3 11c 3  a3
Tương tự  3b  c ;  3c  a
4b2  bc 4c 2  ca
Cộng cả ba bất đẳng thức ta có
11a3  b3 11b3  c 3 11c 3  a3
   2 a  2b  2c
4 a2  ab 4b2  bc 4c 2  ca
Dấu bằng xảy ra  a  b  c.
Nhận xét: bài toán sử dụng kỹ thuật chọn điểm rơi kết hợp với phương pháp hàm số biến
thiên để chứng minh bất đẳng thức.
Ý tưởng: Đây là một bất đẳng thức đối xứng vì vai trò các biến là như nhau. Vì vậy, điểm
rơi bài toán sẽ xảy ra tại a  b  c . Khi đó sẽ đưa ta đến các đẳng thức luôn đúng như
2 2 2
a  b ; b  c ; c  a  0 , và lợi dụng điều đó ta sẽ đi biến đổi tương đương các bất đẳng
thức. Tiếp theo quan sát vế trái của bất đẳng thức có dạng bậc ba chia bậc hai, đồng thời

Liên hệ tài liệu word toán sđt và zalo: 039.373.2038 TÀI LIỆU TOÁN HỌC
81
Website:tailieumontoan.com
11a 3  b3
vế trái có xuất hiện bậc nhất do đó ta chỉ cần đánh giá  m.a  n.b (*) và các biểu
4 a 2  ab
thức còn lại lập luận tương tự sẽ có điều phải chứng minh. Và công việc cuối cùng là tìm
m; n .
• Trước hết, với điểm rơi a  b thế vào (*) ta sẽ có: m  n  2 .
• Quy đồng biểu thức (*), ta được: 11a3  b3  m.a  n.b4a2  ab
 4 ma 3  4na 2 b  ma 2 b  nab2  11a 3  b3
 4 m  11 a 3  m  4n a 2 b  nab2  b3  0
3
Quan sát hệ số của b3 đồng thời bất đẳng thức a  b sẽ tìm ta nghĩ đến chuyện a; b

4 m  11  1
cùng hệ số hay nói cách khác 

m  n  2


m  3 11a 3  b3

2
 suy ra (*)  2  3a  b  a  ba  b  0 .

n  1
 4 a  ab
11b3  c 3 11c 3  a 3
Tương tự, ta cũng có  3b  c ;  3c  a .
4b2  bc 4c 2  ca
Bài toán kết thúc.
Bài tập tương tự:
1. Cho a; b; c là các số thực dương. Chứng minh rằng
a 2 a  2b b 2 b  2 c  c 2 c  2 a 3
    a  b  c .
a  b
2
b  c 
2
c  a
2
4

2. Cho a; b; c là các số thực dương. Chứng minh rằng


a3 b3 c3 abc
  
2
a b 2 2
b c 2 2
c a 2
2

ĐỀ SỐ 8.
Câu 1. (3 điểm)
1). Từ hệ phương trình đã cho ta có
 
2x 3   x  y x 2  y 2  xy  x 3  y 3  x  y .

Do đó hệ đã cho tương đương với


x  y
  x  y  1 .
 2
x  y 2  2

Vậy hệ phương trình có nghiệm  x; y  1; 1 , 1;  1 .
Nhận xét: Bài toán sử dụng phương pháo thế để đưa về phương trình đẳng cấp có mối
liên hệ giữa các biến sau đó thế ngược lại một trong hai phương trình của hệ ban đầu để
tìm nghiệm.
Nhắn lại kiến thức và phương pháp:
• Hằng đẳng thức dạng:
a 3  b3  a  ba 2  ab  b2  và a 3  b3  a  ba 2  ab  b2  .
• Phương trình dạng: x 3  y 3  x 3  y 3  0   x  y x 2  xy  y 2   0 .

Liên hệ tài liệu word toán sđt và zalo: 039.373.2038 TÀI LIỆU TOÁN HỌC
82
Website:tailieumontoan.com
2
 y 3y 2
Vì x 2  xy  y 2   x    0.
 2  4
Ý tưởng: Cả hai phương trình đều xuất hiện hằng số 2. Đồng thời ở phương trình thứ hai
có dáng dấp là một phương trình đẳng cấp bậc ba vì vế trái chứa 2x 3 còn vế phái chứa tích
của một hàm số bậc nhất và một hàm số bậc hai vì vậy ta sẽ thực hiện phép thế 2  x 2  y 2
vào vế phải của phương trình hai, ta sẽ có: 2x 3   x  y x 2  y 2  xy (*). Với phương trình
này (*) nếu bạn nào tinh ý ra sẽ phát hiện được hằng đẳng thức
x 3  y 3   x  y x 2  xy  y 2  . Vì thế (*)  2x 3  x 3  y 3  x  y . Không thì khai triển tích ra
ta cũng sẽ có được điều đó. Việc còn lại chỉ là thế x  y ngược lại phương trình thứ nhất
trong hệ và tìm nghiệm.
Bài toán kết thúc.
Bài tập tương tự:
x  3 y  4
1. Giải hệ phương trình  2 .
2 x   x  y2  x

 5
Đáp số:  x; y  1; 1 , 1;  .
 3

x 2  3 y 2  4
2. Giải hệ phương trình  3 .

 2 x   x  y2  xy

Đáp số:  x; y  1; 1 , 1;  1 .
2a  b 2b  c 2c  a
2). Đặt x  ;y ; z .
ab bc ca

x  1  3a ; x  2  3b
 ab ab

3 b 3c
Khi đó  y  1  ; y2  ;
 bc bc

z  1  3c ; z  2  3a
 ab ab
suy ra  x  1 y  1 z  1   x  2 y  2 z  2
 9  3 xy  yz  zx  3 x  y  z
 3  xy  yz  zx  x  y  z , điều phải chứng minh.
Nhận xét: Bài toán sử dụng phép đặt ẩn phụ sau đó khai thác giả thiết của bài toán đưa về
điều luôn đúng hoặc điều cần chứng minh.
Ý tưởng: Biểu thức cần chứng minh khá là cồng kềnh, tuy nhiên nếu để ý một chút thì biểu
thức đó chỉ gồm ba số hạng, vì vế trái chỉ là tổng của các tích các số hạng. Vậy nên để cho
đơn giản hóa, ta đặt các biểu thức bên vế phải lần lượt là x; y; z , tức là đặt:
2a  b 2b  c 2c  a
x ;y ; z (*).
ab bc ca
Khi đó, biểu thức cần chứng minh có dạng: 3  xy  yz  zx  x  y  z .

Liên hệ tài liệu word toán sđt và zalo: 039.373.2038 TÀI LIỆU TOÁN HỌC
83
Website:tailieumontoan.com
Việc còn lại là từ phép đặt (*), ta mong muốn tìm được mối quan hệ giữa x; y; z . Thực
chất, với phép đặt (*) để tìm mối liên hệ giữa x; y; z , thì ta sẽ coi (*) là một hệ phương

 2 a  b  x a  b



trình với ba ẩn a , b , c như sau: 2b  c  y b  c .




 2c  a  z c  a

Nhưng với hệ trên, ta sẽ gặp rất nhiều khó khăn tuy nhiên không phải là không làm được
nhưng sẽ tốn thời gian rất nhiều. Vậy nên, ta hướng tới lối tư duy như sau. Xét riêng với
2a  b
x , đây là hàm số bậc nhất trên bậc nhất với ẩn a; b vì thế ý tưởng của ta là tách
ab
3a 3b
riêng hai biến a; b ra và đưa nó về dạng là x  1  ; x2 . Và làm tương tự với
ab ab
y ; z ta sẽ được:
3b 3c 3c 3a
y  1 ; y2 ; z  1 ; z2 .
bc bc ca ca
Suy ra đẳng thức  x  1 y  1 z  1   x  2 y  2 z  2 . Và từ đẳng thức này, khai triển
hai tích ta sẽ có điều phải chứng minh như sau:
 9  3 xy  yz  zx  3 x  y  z  3  xy  yz  zx  x  y  z .
Bài toán kết thúc.
Bài tập tương tự:
1 1 1 1 1 1
1. Với a; b; c là các số thực khác 0 thỏa mãn    2 và 2  2  2  2 . Chứng
a b c a b c
minh rằng a  b  c  abc .
2. Chứng minh rằng nếu a 2
 bcb  abc  b2  aca  abc thì ta sẽ có
1 1 1
a  b  c    với a; b; c ; a  b khác 0.
a b c

Câu 2.
 3k  1  3k  2   6k  1
1). Ta có n  3k  VT        3 k  n.
 3   3   6 
 3k  2   3k  3   6 k  3 
n  3 k  1  VT       3 k  1  n.
 3   3   6 
 3k  3   3k  4   6 k  5 
n  3 k  2  VT       3 k  2  n, điều phải chứng minh.
 3   3   6 
Nhận xét. Bài toán về phần nguyên.
Nhắc lại kiến thức và phương pháp.
• Một số khi chia cho 3 chỉ dư 0, 1 hoặc 2 hay nói cách khác một số nguyên bất kỳ được
viết dưới dạng n  3k hoặc n  3k  1 hoặc n  3k  2 .
• Xét lần lượt với các giá trị của n ta có:
+ Với n  3k ta có:
 3k  1  3k  2   6 k  1  1  2  1
VT       k    k    k   .
 3   3   6   
3   
3   6 

Liên hệ tài liệu word toán sđt và zalo: 039.373.2038 TÀI LIỆU TOÁN HỌC
84
Website:tailieumontoan.com
+ Với mỗi số thực x ta định nghĩa  x là số nguyên lớn nhất không vượt quá x hay với
mỗi số x ta có x   x  m với m là phần thập phân ( 0  m  1 ).
Ta có VT  k  k  k  3k  n  VP (đúng).
Hoàn toàn tương tự biến đổi như trên bài và có được điều cần chứng minh.

1 1 1
2). Ta có ab  bc  ca  abc     1 .
c a b
2 2
a 1 1 3 b 1 1 3 c2 1 1 3
Ta có 3    ; 3    ; 3    .
b a a b c b b c a c c a
Cộng ba bất đẳng thức trên ta có:
a2 b2 c2 1 1 1
      1  M min  1 (khi a  b  c  3 ).
b3 c3 a3 a b c
Nhận xét: Bài toán sử dụng bất đẳng thức Cosi cho ba số thực dương giữa các biến và kết
hợp với giả thiết tìm giá trị nhỏ nhất của biểu thức.
Nhắc lại kiến thức và phương pháp:
• Bất đẳng thức Cosi cho ba số thực dương a  b  c  3 3 abc .
• Cho f a , b , c  m và P  f a , b , c suy ra P  m .
Ý tưởng: đây là một dạng bất đẳng thức đối xứng, điểm rơi sẽ xảy ra tại tâm là a  b  c ,
thế vào giả thiết ta sẽ có được a  b  c  3 . Việc định hình được điểm rơi sẽ giúp ta vận
dụng các bất đẳng thức một cách linh hoạt. Ở biểu thức P là biểu thức chứa các phân số,
vì thế để tương đồng trong đánh giá, ta sẽ đưa giả thiết về dạng phân số như sau:
1 1 1 1 1 1
   1 . Mặt khác, với điểm rơi a  b  c  3 thì P  1 , mà 1    vậy nên ta cần
a b c a b c
2 2 2
a b c 1 1 1
chứng minh cho P  3  3  3    (*).
b c a a b c
Bây giờ, ta sẽ đánh giá Cosi cho vế trái của (*) nhưng phải xuất hiện dạng phân thức như
a2
vế phải, xét riêng với phân thức .
b3
• Nếu muốn hạ bậc của b hay làm mất biến b ta cần áp dụng Cosi cho 4 số thực dương
a2 b b b a2 b
là     44 . Nhưng sự xuất hiện của không thỏa mãn vế phải (*). Số
b 3
9 9 9 9.9.9 9
b
được chọn là để cân bằng Cosi với điểm rơi đã chọn.
9
• Nếu muốn hạ bậc a hay làm mất biến a ta cần áp dụng Cosi cho 3 số thực dương là
a2 1 1 a2 1 1 3 1
   3 3 . .  và thỏa mãn điều kiện của vế phải (*). Số được chọn là để
b 3
a a b a a b
3
a
cân bằng điểm rơi a  b .
a2
1 1 3 b2 1 1 3 c 2 1 1 3
Vì thế, ta sẽ có được:
   ,    ,    .
b3 a a b c 3 b b c a3 c c a
a2 b2 c 2 1 1 1
Suy ra 3  3  3     1  Pmin  1 khi a  b  c  3.
b c a a b c
Bài toán kết thúc.
Bài tập tương tự:
Liên hệ tài liệu word toán sđt và zalo: 039.373.2038 TÀI LIỆU TOÁN HỌC
85
Website:tailieumontoan.com

1) Cho x; y ; z là các số thực dương thỏa mãn xy  yz  zx  3 xyz . Tìm giá trị nhỏ nhất
1 1 1
của biểu thức P  2
 2 2.
x y z
2) Cho x; y ; z là các số thực dương thỏa mãn xy  yz  zx  xyz . Tìm giá trị nhỏ nhất của
x y z
biểu thức P  2
 2 2.
y z x
Câu 3.
A

P F
H
O
E
I B

D
  900 , mà
1). Gọi DE cắt (O) tại P khác D. Do AD là đường kính của (O) , suy ra APD
  900 ( do HE  BC  HA ), nên tứ giác APEH nội tiếp.
AHE
  AEH
Ta có APH  (góc nội tiếp)
 (góc nội tiếp)
 ( HE  BC )  APB
 ACB
 PH  PB .
Nhận xét. Chứng minh hai đường thẳng cắt nhau trên đường tròn cũng gần giống với việc
chứng minh ba đường thẳng đồng quy bằng cách chứng minh một đường thẳng đi qua
giao điểm của hai đường còn lại. Do đó ta Gọi giao điểm của DE với (O) là P . Cần chứng
minh BH đi qua P hay BH trùng PH .
Nhắc lại kiến thức và phương pháp.
• Góc nội tiếp chắn nửa đường tròn là góc vuông.
 là góc nội tiếp chắn nửa đường tròn (O) được chia bởi đường kính AD nên
APD
  90 hay APE
APD   90 .

• Đường thẳng a song song với đường thẳng b, đường thẳng c vuông góc với đường
thẳng a. Suy ra đường thẳng c vuông góc với đường thẳng b.
Có HE  BC và BC  HA ( H là trực tâm của tam giác ABC ) nên suy ra HE  HA hay
  90 .
AHE
• Tứ giác có hai góc vuông đối diện là tứ giác nội tiếp.

Liên hệ tài liệu word toán sđt và zalo: 039.373.2038 TÀI LIỆU TOÁN HỌC
86
Website:tailieumontoan.com
  90 và AHE
Ta thấy tứ giác APEH có APE   90 là hai góc vuông đối diện nên tứ

giác APEH là tứ giác nội tiếp.


• Các góc nội tiếp một đường tròn cùng chắn một cung thì bằng nhau.
 là hai góc nội tiếp cùng chắn cung AH của đường tròn ngoại tiếp tứ
 và AEH
+ APH
  AEH
giác APEH nên APH .
 và ACB
+ APB  là hai góc nội tiếp cùng chắn cung AB của đường tròn (O) nên

  ACB
APB .

• Một đường thẳng cắt hai đường thẳng song song tạo ra các góc đồng vị bằng nhau.
  ACB
Có HE  BC suy ra AEH  (hai góc đồng vị)

• Hai góc có cùng số đo, có một cạnh chung, hai cạnh còn lại của hai góc cùng nằm về
một phía so với cạnh chung thì hai cạnh còn lại trùng nhau.
  APB
Từ chứng minh ở trên, ta có APH  , suy ra PH  PB hay BH đi qua P . Do đó BH

và DE cắt nhau tại điểm P trên đường tròn.

  AHP
2). Ta có HP  AC , suy ra AEH   AEP
.

Suy ra EA là phân giác ngoài đỉnh E của DEF .


Tương tự FA là phân giác ngoài đỉnh F của DEF
Suy ra A là tâm đường tròn bàng tiếp ứng với đỉnh D của DEF .
Nhận xét. Bài toán được quy về bài toán chứng minh một tia là tia phân giác của một góc
và sử dụng dấu hiệu nhận biết tâm đường tròn bàng tiếp tam giác là giao điểm của hai
đường phân giác góc ngoài tại hai đỉnh của tam giác.
Nhắc lại kiến thức và phương pháp.
• Hai góc cùng phụ với một góc thì bằng nhau.
AC  BH (hay AC  PH ) và AH  FE (chứng minh trên) nên   AEH
AHP  (
 ).
 90  PHE
• Các góc nội tiếp một đường tròn cùng chắn một cung thì bằng nhau.
 là hai góc nội tiếp cùng chắn cung AP của đường tròn ngoại tiếp tứ
 và AHP
AEP
  AHP
giác APEH nên AEP .

• Một tia tạo với một cạnh của tam giác và tia đối của một cạnh tam giác tại một đỉnh
hai góc bằng nhau thì tia đó là phân giác của góc ngoài tam giác.
  AEH
Từ chứng minh ở trên, ta có PEA  nên EA là tia phân giác của góc ngoài tại

đỉnh E của tam giác DEF .


Hoàn toàn tương tự ta chứng minh được FA là tia phân giác của góc ngoài tại đỉnh F
của tam giác DEF .
• Giao điểm của hai tia/đường phân giác của hai góc ngoài tại hai đỉnh của tam giác là
tâm đường tròn bàng tiếp ứng với đỉnh còn lại của tam giác.

Liên hệ tài liệu word toán sđt và zalo: 039.373.2038 TÀI LIỆU TOÁN HỌC
87
Website:tailieumontoan.com
FA và EA là hai tia phân giác của góc ngoài tại đỉnh F và E của tam giác DEF
(chứng minh trên) nên A là tâm đường tròn bàng tiếp tam giác ứng với đỉnh D của
tam giác DEF .

3). Do I là tâm nội tiếp nên EI là tia phân giác trong.


Mà EA là tia phân giác ngoài, suy ra EI  AC  EI  HB .
Tương tự FI  HC ; EF  BC , suy ra IEF và HBC có cạnh tương ứng song song, nên
BE; CF và IH đồng quy.
Nhận xét. Bài toán chứng minh ba đường thẳng a; b; c đồng quy bằng phương pháp chỉ ra
hai giao điểm của a và b; b và c trùng nhau. Áp dụng tính chất: Hai điểm cùng nằm trên
một đoạn thẳng cùng cách hai đầu mút của đoạn thẳng các đoạn tương ứng bằng nhau thì
trùng nhau.
Nhắc lại kiến thức và phương pháp.
• Đường nối đỉnh và tâm đường tròn nội tiếp của tam giác là phân giác trong của tam
giác.
I là tâm đường tròn nội tiếp tam giác DEF nên EI là phân giác của tam giác DEF .
• Phân giác trong và phân giác ngoài của tam giác tại một đỉnh tạo với nhau một góc
vuông (tính chất phân giác của hai góc kề bù).
• EA là phân giác ngoài (chứng minh trên) và EI là phân giác trong (chứng minh trên)
tại đỉnh E của tam giác DEF nên EI  AC .
• Hai đường thẳng vuông góc với một đường thẳng thứ ba thì song song với nhau.
BH  AC ( H là trực tâm tam giác ABC ) và EI  AC (chứng minh trên) nên suy ra
EI  HB . Hoàn toàn chứng minh tương tự ta có FI  HC ; EF  BC .
• Hai tam giác có ba cạnh tương ứng song song thì đồng dạng.
Tam giác IEF và tam giác HBC có EI  HB; FI  HC ; EF  BC , nên IEF ∽ HBC suy
IF IE EF
ra  
HC HB BC
• Định lý Ta-lét.
EI IM
Gọi giao điểm của BE và HI là M ta có EI  HB suy ra  . Gọi giao điểm của
BH HM
FI IN
CF và HI là N ta có FI  HC suy ra  .
CH HN
• Hai điểm cùng nằm trên một đoạn thẳng cùng cách hai đầu mút của đoạn thẳng các
đoạn tương ứng bằng nhau thì trùng nhau.
IM IN
Từ chứng minh trên, ta có  mà M ; N cùng thuộc IH nên suy ra M trùng
HM HN
N . Hay ba đường thẳng BE ; IH ; CF đồng quy.

Liên hệ tài liệu word toán sđt và zalo: 039.373.2038 TÀI LIỆU TOÁN HỌC
88
Website:tailieumontoan.com
F
H O
E
M
N
I
B
C

Câu 4. (1 điểm)
Ta đặt các đoạn thẳng có độ dài a; b; c như sau

Ta có AB  a 2  b2  3ab ; BC  b2  c 2  bc ; AC  c 2  a 2 .
Vì AB  BC  AC nên ta có điều phải chứng minh.

ĐỀ SỐ 9.

Câu 1.
1
1). Điều kiện  xy  2 .
2
Ta có
2 xy  1  1
2 xy  1  2  x 2 y 2   2  x2 y 2  x2 y 2  2  x2 y 2
2
x2 y 2  2  x2 y 2
2 2.
2

xy  1
Từ hệ suy ra xy  1 và thu được hệ tương đương   x  y  1.

x  y  2

Nhận xét: bài toán sử dụng phương pháp đánh giá phương trình hai (cụ thể là bất đẳng
thức Cosi ) tìm mối quan hệ giữa các biến, sau đó thế ngược lại vào phương trình một tìm
nghiệm của hệ phương trình.
Ý tưởng: Quan sát hai phương trình của hệ, ta thấy rằng phương trình hai là phương trình
xy
chứa căn thức và chỉ có một ẩn vì thế ta sẽ đi khai thác nó. Nhưng vấn đề là khai thác
như thế nào, nếu bình phương hai vế hai lần sẽ đưa ta đến phương trình bậc bốn như vậy

Liên hệ tài liệu word toán sđt và zalo: 039.373.2038 TÀI LIỆU TOÁN HỌC
89
Website:tailieumontoan.com

càng đưa về bậc cao sẽ càng khó làm. Do đó, ta sẽ nghĩ đến hướng đánh giá cụ thể ở đây là
sử dụng bất đẳng thức Cosi vì ta nhẩm được nghiệm xy = 1.
Với xy  1 , thì 2 xy  1  1 và 2  x 2 y 2  1 vậy theo bất đẳng thức Cô-si để dấu bằng xảy
ra như sau:

x2 y 2  1
2 xy  1  1  2 2 xy  1  2 xy  1  xy  x 2 y 2 
2
x2 y 2  1 3  x2 y 2
 2 xy  1  2  x 2 y 2    2  xy  1
2 2
Với xy =1 thế vào phương trình một của hệ, ta có: x + y = 2 và xy = 1
Bài toán kết thúc.
Bài tập tương tự:

 x2  y 2  3



 2 2
1. Giải hệ phương trình  2 xy  1  2  x y  2 .

x  y  xy  1


 4 xy  1  2  4 x 2 y 2  2
2. Giải hệ phương trình  .

1 1
2). Phương trình tương đương với   5x  4 x  1 .
2x  1 x2
1 1
Giả sử x  1  2 x  1  x  2  
2x  1 x2
 VT  0  5 x  4 x  1  x  1 (mâu thuẫn).
Giả sử 0  x  1 , lập luận tương tự thu được x  1 (mâu thuẫn). Thử lại thu được nghiệm
duy nhất x  1 .
Nhận xét: bài toán sử dụng phương pháp đánh giá miền nghiệm để tìm nghiệm của
phương trình.
Ý tưởng: Quan sát bài toán, ta thấy có vẻ nó hơi rắc rối một chút là vì xuất hiện tới bốn căn
thức mà căn lại còn ở dưới mẫu thì nó sẽ rất khó khăn trong việc giải bằng các công cụ mà
ta thường làm như ẩn phụ, nâng lũy thừa …Và hướng cuối cùng ta nghĩ đến là đánh giá.
Để đánh giá được nó, ta cần tìm nghiệm trước, vẫn là ưu tiên hướng nghiệm nguyên đồng
thời cân bằng căn thức với căn thức, phân thức với phân thức ta có:
1 1
  x1; 4 x  1  5x  x  1 .
2x  1 x2
Và bây giờ, ta sẽ đi chứng minh x  1 là nghiệm duy nhất của phương trình. Hay nói cách
khác, với x  1 hoặc x  1 nó sẽ vô nghiệm. Biến đổi phương trình đã cho về dạng:
1 1 x  2  2x  1
 4x  1   5x   5x  4 x  1
2x  1 x2 2 x  1 x  2
x  2  2 x  1 
  x  2  2x  1  0
• Với x  1 , ta có  
 , suy ra phương trình vô nghiệm.

5 x  4 x  1 
  5x  4 x  1  0

Liên hệ tài liệu word toán sđt và zalo: 039.373.2038 TÀI LIỆU TOÁN HỌC
90
Website:tailieumontoan.com

x  2  2 x  1 
 
 x  2  2 x  1  0 , suy ra phương trình vô
• Với 1  x  0 , ta có:  

5 x  4 x  1
 
 5x  4 x  1  0


nghiệm.
Vậy x  1 là nghiệm duy nhất của phương trình.
Bài toán kết thúc.
Bài tập tương tự:
1 1
1. Giải phương trình  3x  1   4x .
2x  1 x2
1 1
2. Giải phương trình  3x  1   4x .
2x2  x  1 x2  x  2
Câu II.
1). Giả sử n  abcdefghij là một số tốt .
Ta có n  a  b  c    j (mod 9) vì các chữ số của n khác nhau nên
9.10
n  0  1  2  9  (mod 9)  0 (mod 9) .
2
Vậy n chia hết cho 9.
Ta có 11111,9  1 suy ra n chia hết cho 99999 .
Ký hiệu x  abcde ; y  fghij ta có n  10 5.x  y , suy ra n   x  ymod 99999 ,

 x  y  0 mod 99999 .
Mà 0  x  y  2.99999  x  y  99999
 a f b g chdi  e j 9.
Có tất cả 5! cách sắp xếp 5 cặp 0; 9 , 1; 8 , 2; 7  , 3; 6 , 4; 5 (mỗi cách sắp xếp ta được
một số n ). Với mỗi cặp có 2 cách sắp xếp  số các bộ số tốt bằng t  55.5! . Ta phải trừ
 1 
trường hợp a  0 đứng đầu bằng 4!2 4 . Vậy số tốt bằng d  32.5! 4!2 4  32.5!1    3456 .
  10 
Nhận xét. Áp dụng kiến thức số học về cấu tạo số, phép chia hết, đồng dư thức,…
Nhắc lại kiến thức và phương pháp.
• Một số tự nhiên khi chia cho 9 có cùng số dư với tổng của các chữ số của số đó khi chia cho 9.
Ta có n  a  b  c    j (mod 9) .
• Vì n là số tự nhiên có 10 chữ số khác nhau nên các chữ số của n chính là 10 chữ số tự nhiên 0,
1, 2, 3, 4, 5, 6, 7, 8, 9.
9.10
Do đó ta có n  0  1  2    9   9.5 suy ra n  0 mod 9 .
2
• Hai số có ước chung lớn nhất bằng 1 là hai số nguyên tố cùng nhau.
Ta có 9 và 11111 có ước chung lớn nhất bằng 1 nên là hai số nguyên tố cùng nhau.
• Một số tự nhiên chia hết cho hai số nguyên tố cùng nhau thì chia hết cho tích hai số đó.
Ta có n chia hết cho 9 (chứng minh trên) và n chia hết cho 11111 (theo đề bài) mà 9 và
11111 là hai số nguyên tố cùng nhau (chứng minh trên) suy ra n chia hết cho 99999
• Phân tích cấu tạo số.
Ký hiệu x  abcde ; y  fghij ta có n  10 5.x  y
• Áp dụng các phép toán đồng dư.

Liên hệ tài liệu word toán sđt và zalo: 039.373.2038 TÀI LIỆU TOÁN HỌC
91
Website:tailieumontoan.com
Ta có 10 5  1mod 9999  10 5 x  y  x  y mod 9999 mà n  0 mod 99999 (chứng minh
trên), suy ra  x  y  0 mod 99999
• Suy luận.
Vì x; y là các số có 5 chữ số nên 0  x  99999 và 0  y  99999 (do các chữ số khác nhau
nên x; y không thể vượt quá 99999) suy ra 0   x  y  2.99999 mà  x  y  99999 do đó
x  y  99999 .
Từ đây ta có abcde  fghij  99999 suy ra a  f  b  g  c  h  d  i  e  j  9 .
Các cặp số tự nhiên có tổng bằng 9 là 0; 9 , 1; 8 , 2; 7  , 3; 6 , 4; 5 ứng với 5 tổng
trên ta có 5! cách lập và mỗi cặp có 2 cách xây dựng nên có tất cả 2 5.5! số tốt được tạo
thành, trường hợp số 0 đứng ở hàng trăm triệu không có nghĩa có 2 4.4! nên sẽ có
2 5.5! 2 4.4!  3456 số tốt.

2 2
2). Ta có f  x; y; z  2 x  y  3   y  z  9  9 .
Vậy fmin  9 khi x  y  z  1 .
Nhận xét. Dạng toán tìm giá trị nhỏ nhất đưa về tổng các lũy thừa bậc chẵn cộng với một
số khác 0.
Nhắc lại kiến thức và phương pháp.
• Tách, nhóm các hạng tử
f  x; y; z  4 x 2  2 y 2  z 2  4 xy  2 yz  6 y  12 x
 4 x 2  y 2  9  4 xy  12 x  6 y   y 2  2 yz  z 2   9 .
2
• Hằng đẳng thức bình phương của một tổng hoặc hiệu a  b  a 2  2 ab  b2 .
2
Ta có y 2  2 yz  z 2   y  z .
• Hằng đẳng thức bình phương của tổng hoặc hiệu mở rộng cho nhiều hạng tử
2
 a  b  c  a 2  b2  c 2  2 ab  2 ac  2bc .
2
Ta có 4 x 2  y 2  9  4 xy  12 x  6 y  2 x  y  3 .
2 2
Khi đó ta được f  x; y; z  2 x  y  3   y  z  9  9 .

 2

2 x  y  3  0
• Đánh giá  với mọi giá trị của x; y; z .
 2

 

 y  z  0
2 2 2 2
Khi đó 2 x  y  3   y  z  0  2 x  y  3   y  z  9  9 .

 2

2 x  y  3  0
Dấu “=” xảy ra khi và chỉ khi 
 2


 y  z  0

 2 x y3 0
  y  z  3  2x .


 y  z  0
Câu III.

Liên hệ tài liệu word toán sđt và zalo: 039.373.2038 TÀI LIỆU TOÁN HỌC
92
Website:tailieumontoan.com

O
M
Q I

N
B

  180 0  IBC
1). Ta có BIC   ICB

 ACB
ABC  
180  BAC 
BAC
 180 0    180 0   90 0 
2 2 2 2
 
 BAC  2 BIC  180

  90 0  BPC  BPC
Tương tự BQC   2 BQC
  180 .
2
  BPC
Tứ giác BPAC nội tiếp, suy ra BAC   BQC
  BIC
 , nên 4 điểm B; I ; Q; C thuộc một

đường tròn.
Nhận xét. Chứng minh bốn điểm cùng thuộc một đường tròn ta chứng minh bốn điểm
này tạo thành một tứ giác nội tiếp.
Nhắc lại kiến thức và phương pháp.
• Tổng ba góc trong một tam giác bằng 180 .
  IBC
Ta có BIC   180 0  BIC
  ICB   180 0  IBC .
  ICB
• Tâm đường tròn nội tiếp là giao điểm của ba đường phân giác trong của tam giác.
I là tâm đường tròn nội tiếp tam giác ABC nên:

  ABC .
 , suy ra IBC
+ BI là phân giác của góc ABC
2

  ACB .
 , suy ra ICB
+ CI là phân giác của góc ACB
2
 
  180 0  IBC
Từ đây ta có BIC   180 0  ABC  ACB
  ICB
2 2
 
ABC  ACB 0 
180  CAB 
BAC
 180 0   180 0   90 0  hoàn toàn tương tự ta có
2 2 2

  90 0  BPC .
BQC
2
• Hai góc nội tiếp cùng chắn một cung của một đường tròn thì bằng nhau.

Liên hệ tài liệu word toán sđt và zalo: 039.373.2038 TÀI LIỆU TOÁN HỌC
93
Website:tailieumontoan.com
  BPC
BAC  (hai góc nội tiếp cùng chắn cung BC
 của đường tròn ngoại tiếp tứ giác
BAPC )
  180  2 BQC
 2 BIC   180  BIC
  BQC
.

• Tứ giác có hai đỉnh liên tiếp cùng nhìn một dưới một góc bằng nhau là tứ giác nội tiếp.
Tứ giác QICB có hai đỉnh Q và I cùng nhìn cạnh BC dưới hai góc bằng nhau
  BQC
BIC  suy ra QICB là tứ giác nội tiếp hay bốn điểm B; I ; Q; C cùng thuộc một

đường tròn.

2). Gọi đường tròn  B; BI  giao C ; CI  tại K khác I thì K cố định.


 là góc ở tâm chắn cung IM
Góc IBM  là góc nội tiếp chắn cung IM
 và IKM  , suy ra

  1 IBM
IKM  (1).
2
  1 ICN
Tương tự IKN  (2).
2
Theo câu 1) B; I ; Q; C thuộc một đường tròn, suy ra
  IBQ
IBM   ICQ
  ICN  (3).
  IKN
Từ (1), (2) và (3), suy ra IKM   KM  KN .
Vậy MN đi qua K cố định.
Nhận xét. Chứng minh một đường thẳng đi qua một điểm cố định ta chứng minh hai
đường thẳng cùng đi qua một điểm cố định và chứng minh hai đường thẳng này trùng
nhau.
Nhắc lại kiến thức và phương pháp.
• Số đo góc ở tâm gấp đôi số đo góc nội tiếp cùng chắn một cung của một đường tròn.
 là góc ở tâm chắn cung IM
+ Góc IBM  là góc nội tiếp chắn cung IM
 và IKM  của

  1 IBM
đường tròn  B; BI  , suy ra IKM .
2

+ Góc ICN là góc ở tâm chắn cung IN  và IKN
 là góc nội tiếp chắn cung IN
 của đường

  1 ICN
tròn C ; CI  , suy ra IKN .
2
• Áp dụng nhắc lại kiến thức.
  QCI
QBI  (hai góc nội tiếp cùng chắn cung QI
 của đường tròn ngoại tiếp tứ giác
  ICN
QICB ) hay IBM .

Từ các đẳng thức chứng minh trên ta được IKM   IKN


.
• Hai tia chung gốc cùng nằm trên nửa mặt phẳng bờ là đường thẳng chứa một tia chung gốc
khác hai tia đã cho. Hai tia này cùng tạo với tia chung gốc khác kia hai góc bằng nhau thì hai
tia đã cho trùng nhau.
  IKN
IKM  và IM ; IN cùng thuộc một nửa mặt phẳng bờ IK nên IM  IN .

Mà IM và IN cùng đi qua điểm K cố định nên MN đi qua K cố định.

Câu IV. Ta cần chứng minh

Liên hệ tài liệu word toán sđt và zalo: 039.373.2038 TÀI LIỆU TOÁN HỌC
94
Website:tailieumontoan.com
1 1 1 3
F  a ; b; c      0
1  a  b 1  b  c 1  c  a 1  2 3 abc
1 1 2
Với a; b  0 và ab  x , ta có đẳng thức   (1)
x  a x  b x  ab
2x  a  b 2
 2 
x  x a  b  ab x  ab
 2 x 2  x a  b  2 x ab  ab ( a  b)  2 x 2  2 x a  b  2 ab  2 x ab  ab a  b  x a  b  2 ab

   ab  x  0
 2 ab x  ab  a  b
2
  ab  x a  b   0 (hiển nhiên đúng).

+) vì c  1  1  ab , áp dụng kết quả (1) ta suy ra:


1 1 2
 
1  c  b 1  c  a 1  c  ab
Đẳng thức xay ra khi và chỉ khi a=b.
Ta cũng có
1 1

1  a  b 1  2 ab
Từ đó suy ra
1 1 1 3
F  a ; b; c     
1  ab  ab 1  ab  c 1  c  ab 1  2 3 abc
F  
ab ; ab ; c .

Xét bộ a; b; c bất kỳ có a  b ta có F a; b; c  F  ab ; ab ; c . 


Suy ra giá trị lớn nhất chỉ đạt được khi a  b  c , hay F a; b; c  F a; a; a  0 (điều phải
chứng minh).

ĐỀ SỐ 10.

Câu 1.
1). Điều kiện: x  2 .
a  x  2

Đặt  ( a  0; b  0 ).
b  x 2  2 x  4

Ta có 2 a 2  b2  2 x  4  x 2  2 x  4  x 2  8 ,
suy ra 2 a 2  b2  3ab  a  b2a  b  0
x  1
+ TH1: a  b  x  2  x 2  2 x  4  x 2  3x  2  0   .
x  2
+ TH2: 2 a  b  4  x  2  x 2  2 x  4  x 2  6 x  4  0  x  3  13 .
Nhận xét: Bài toán sử dụng phương pháp đặt ẩn phụ rồi đưa về phương trình đẳng cấp
bậc hai, tìm mối quan hệ giữa hai ẩn phụ sau đó nâng lũy thừa tìm nghiệm của phương
trình ban đầu.
Nhắc lại kiến thức cũ và phương pháp:

Liên hệ tài liệu word toán sđt và zalo: 039.373.2038 TÀI LIỆU TOÁN HỌC
95
Website:tailieumontoan.com

• Hằng đẳng thức u3  v 3  u  vu2  uv  v 2  .


• Cách giải phương trình bậc hai dạng a.x 2  b.xy  c.y 2  0 (*).
Làm nháp: Chia cả hai vế của phương trình (*) cho y 2 ( vì là làm nháp nên ta cứ coi là
2
 x x
y  0 ), khi đó (*)  a.   b.  c  0 .
 y  y
x
Coi đây là phương trình bậc hai ẩn t  (đã biết cách giải) thì nghiệm t tìm được sẽ
y
 x  t1 y
biểu hiện mối quan hệ giữa x , y . Cụ thể là  .
 x  t2 y
Ý tưởng: Bài toán này có thể giải bằng cách lũy thừa hai vế, đưa về phương trình bậc bốn
và vấn đề ta gặp phải sẽ là nghiệm của phương trình bậc bốn. Ta đặt dấu hỏi cho nó ?
Nhưng nếu để ý 8  2 3 nên trong căn thức sẽ xuất hiện ngay hằng đẳng thức
x 3  8  x 3  2 3   x  2 x 2  2 x  4 . Vậy nên trong căn bậc hai đã xuất hiện hai tích. Do đó
để xuất hiện phương trình đẳng cấp thì ta cần làm công việc sau, đó là đồng nhất hệ số
a  1
 a  1
của biểu thức dạng a  x  2 x  4  b  x  2  x  8  b  2 a  0  
2 
2
.
4 a  2b  8 b  2

Đến đây, ta viết lại phương trình ban đầu như sau:
x 2  8  3 x 3  8   x 2  2 x  4  2  x  2  3  x 2  2 x  4 x  2 ( i ).

 2
u  x  2 x  4  0
• Đặt  nên phương trình trên trở thành:

v  x  2  0


u  v
( i )  u2  3uv  2 v 2  0  u  vu  2 v  0  
u  2 v
2
• Hoặc nhận thấy x 2  2 x  4   x  1  3  0 nên phương trình ( i ) tương đương với:
x2 x2
2. 2
3 2  1  0 . Có thể đặt căn thức là ẩn phụ để giải phương trình
x  2x  4 x  2x  4
bậc hai.
• Hai hướng trên đều cho ta:
 x  2
 x  1
TH1. Với x2  2x  4  x  2   2  .
 x  3 x  2  0 x  2

 

 x   2
TH2. Với x2  2x  4  2 x  2   2  x  3  13 .

x  6 x  4  0

Bài toán kết thúc.
Bài tập tương tự:
1. Giải phương trình x 2  3x  5  2 x 3  x 2  6 x  4 .
1  13
Đáp số: x  .
2
3 4
2. Giải phương trình x 2  3x  1   x  x2  1 .
3

Liên hệ tài liệu word toán sđt và zalo: 039.373.2038 TÀI LIỆU TOÁN HỌC
96
Website:tailieumontoan.com
Đáp số: x  1 .

2). Cộng hai phương trình ta có


2 x 2  xy  y 2  5 x  y  2  0   x  y  22 x  y  1  0 .

x  y  2 x  y  2

+ TH1:  2    x y1
 2 xy  1
x  xy  y  1 
 

2 x  y  1
+ TH2:  2
2
 x 2  x 2 x  1  2 x  1  1
 2
x  xy  y  1

 x  0  y  1
 3x2  3x  0  
x  1  y  1
.

Đáp số:  x; y  1; 1 , 0;  1 .
Nhận xét: Kết hợp giữa hai phương trình của hệ, đưa về một phương trình xét  chính
phương, từ đó tìm mối quan hệ giữa hai biến rồi thế ngược lại một trong hai phương trình
của hệ tìm nghiệm.
Nhắc lại kiến thức và phương pháp:
• Cách giải phương trình bậc hai tổng quát a.t 2  b.t  c  0 a  0 .
 f  x; y  0 
• Đưa hệ phương trình đã cho về dạng 
 g  x; y  0



• Giả sử tồn tại k   sao cho f  x; y  k.g  x; y  0 ( i ). Coi phương trình ( i ) là phương
trình bậc hai ẩn x hoặc ẩn y , sau đó xét  sao cho  là một số chính phương, từ đó
sẽ tìm được k   cũng như mối quan hệ giữa x; y .
Ý tưởng: Hai phương trình của hệ khá tách biệt, ở phương trình một rõ ràng không thể
tìm được mối quan hệ giữa x; y , nên ta xét phương trình hai. Nó rắc rối hơn một chút,
thậm chí còn xuất hiện đầy đủ các biến số. Do đó ta thử cách xét  phương trình hai xem
thế nào. Bây giờ coi nó là phương trình bậc hai ẩn x , ta có:
x 2  2 xy  2 y 2  5 x  y  3  x 2  2 y  5 x  2 y 2  y  3  0
 x  2 y  5  4 2 y 2  y  3  12 y 2  24 y  13 .
2

Rõ ràng  không thể chính phương, nên ta đã thất bại ở lối tư duy này. Vậy còn lại đó
chính là cách kết hợp cả hai phương trình để đưa ra một phương trình có  chính
phương. Ta làm như sau:
x 2  xy  y 2  1  0
• Đưa hệ đã cho về dạng  2
x  2 xy  2 y  5 x  y  3  0
2

• Giả sử tồn tại k   sao cho:
x 2  xy  y 2  1  k  x 2  2 xy  2 y 2  5 x  y  3  0
 1  k x 2  2 yk  y  5 k x  1  2 k y 2  ky  3 k  1  0 ( i ).
Coi ( i ) là phương trình bậc hai ẩn x , khi đó
2
x  2 ky  y  5 k   4  k  1 1  2 k  y 2  ky  3 k  1
 
 12 k  3 y  6 k  24 k  y  25 k  4  k  13 k  1 .
2 2 2 2

Liên hệ tài liệu word toán sđt và zalo: 039.373.2038 TÀI LIỆU TOÁN HỌC
97
Website:tailieumontoan.com
Để x là một số chính phương thì trước hết hệ số của y 2 cũng phải là số chính
phương, tức là chúng ta cần đi giải phương trình nghiệm nguyên 12 k 2  3  m2 . Dễ
k  1 
dàng tìm được  2
2
nên thử lại, ta sẽ có được x  9 y 2  18 y  9  3 y  3 .
m  9

• Vậy nên, dựa vào nghiệm của phương trình bậc hai, suy ra:

 x  5  y  x

( i )  2 x 2   y  5 x  y 2  y  2  0   4
 5  y  x
x 
 4
 5  y  3y  3
x  2x  y  1  0
 4
 
x  y  2  0
.
 5  y  3  3y 
x 
 4
• Từ đó thế ngược lại vào một trong hai phương trình của hệ để tìm nghiệm của hệ
phương trình.
Bài toán kết thúc.
Bài tập tương tự:
x 2  3 xy  2 x  1
1. Tìm nghiệm dương của hệ phương trình  2 .
2 y  2 xy  3 x  y  1

 5 1 5  1  7  41 7  41 
Đáp số:  x; y   ; ,  ; ,
 4 4   2 2 
 
 7  41 ; 7  41  .
 2 
 2 

 2

2. Giải hệ phương trình 



 x  y  x  y  y 2
.

 x 4
 4 x 2
y  3 x 2
 y 2
 0


Đáp số:  x; y  0; 0 , 1; 2 , 2; 2
Câu 2.
2 2 2
1). Phương trình tương đương  x  3 y  2 x  3  2 y  1  10 .
Mà 10 chỉ có thể biểu diễn dưới dạng tổng 3 bình phương: 10  0 2  12  32.
x  3 y  0 
 x  3 y  0

Chú ý 2 x  3; 2 y  1 là số lẻ nên (*) 2 x  3  1 hoặc (**) 2 x  3  9 .
2 2

 
 2 y  1 2  9 2

  2 y  1  1
2
+ Xét hệ (*), từ phương trình đầu, suy ra x  3 y   2 3 y  3  1 (vô nghiệm).

2 x  3  3  x  y  0

+ Xét hệ (**) 2y  1  1  x  3 .
 
x  3 y  0  y  1


Đáp số:  x; y  0; 0 , 3;  1 .

Liên hệ tài liệu word toán sđt và zalo: 039.373.2038 TÀI LIỆU TOÁN HỌC
98
Website:tailieumontoan.com
Nhận xét. Bài toán nghiệm nguyên giải bằng phương pháp tách một số thành tổng các số
chính phương.
Nhắc lại kiến thức và phương pháp.
• Biến đổi tách thành tổng các số chính phương

5x 2  13 y 2  6 xy  4 3x  y

  x 2  6 xy  9 y 2   4 x 2  12 x  9  4 y 2  4 y  1  10
2 2 2
  x  3 y  2 x  3  2 y  1  10 .

• Tách số thành tổng các số chính phương

Vế Trái là tổng của 3 số chính phương nên tách vế phải thành tổng của 3 số chính

phương. Ta có 10  0 2  12  32.

• Đồng nhất hai vế theo các thứ tự khác nhau. Sử dụng các tính chất đặc biệt để giảm

bớt các trường hợp phải xét

• Giải các hệ phương trình ta nhận được nghiệm.

2 1 1 1 1 1 1 1
2). Ta có P         
 3  ab bc ca  3ab 3bc 3ca a  b2  c 2
2

2 9 16
 
3 ab  bc  ca 3ab  3bc  3ca  a 2  b2  c 2
1 2
Mà ab  bc  ca   a  b  c  3 .
3
2
3ab  3bc  3ca  a 2  b2  c 2  ab  bc  ca  a  b  c
1
 ( a  b  c )2  ( a  b  c )2  12 .
3
2 9 16 10
P .   .
3 3 12 3
Dấu “=” xảy ra khi a  b  c  1.
10
Vậy Pmin  .
3
Nhận xét: Bài toán sử dụng bất đẳng thức Cosi từ đó dựa vào điểm rơi của bài toán để
phát hiện ra bổ đề cần dùng.
Nhắc lại kiến thức và phương pháp:
2
• Hằng đẳng thức a  b  c  a 2  b2  c 2  2 ab  bc  ca .
• Bất đẳng thức Cosi cho hai số thực không âm a  b  2 ab .
• Từ bất đẳng thức Cosi, phát triển ra các đánh giá sau:
1 1 1 9 1 1 1 1 16
   ;     .
a b c abc a b c d abcd
• Đánh giá qua tổng các đại lượng không âm:
2 2 2
a  b  b  c  c  a  0  a 2  b2  c 2  ab  bc  ca .

Liên hệ tài liệu word toán sđt và zalo: 039.373.2038 TÀI LIỆU TOÁN HỌC
99
Website:tailieumontoan.com
Ý tưởng: Nhận thấy đây là một dạng bất đối xứng, vậy nên ta sẽ dự đoán dấu “=” xảy ra
tại a  b  c mà a  b  c  3 , nên suy ra a  b  c  1 . Việc biết trước điểm rơi sẽ giúp ta vận
dụng các đánh giá một cách linh hoạt hơn. Tiếp theo, biểu thức có xuất hiện a 2  b2  c 2
2
nên ta cần nghĩ ngay đến hằng đẳng thức a  b  c  a 2  b2  c 2  2 ab  bc  ca nên ta cần
một đánh giá ở mẫu số sao cho vừa xuất hiện hằng đẳng thức đó, vừa thỏa mãn điều kiện
1 1 1
điểm rơi của bài toán. Vì a  b  c  1 nên  do đó ta cần tạo ra các lượng
2 2
a b c 2
3 3
1
để cân bằng với biểu thức .
a  b2  c 2
2

1 1 1
Cụ thể là   , do đó biểu thức P trở thành:
3ab 3bc 3ca
2 1 1 1 1 1 1 1
P          2
 
3  ab bc ca  3ab 3bc 3ca a  b2  c 2
Vậy nên, theo bất đẳng thức Cosi, suy ra:
1 1 1 9 2 1 1 1 6
        
ab bc ca ab  bc  ca 3  ab bc ca  ab  bc  ca
1 1 1 1 16
   2  .
3ab 3bc 3ca a  b  c 2 2
3ab  3bc  3ca  a 2  b2  c 2
Mặt khác 3ab  3bc  3ca  a 2  b2  c 2  9  ab  bc  ca , nên ta được:
6 16
P  .
ab  bc  ca 9  ab  bc  ca
Từ đây, dựa vào tổng đánh giá các đại lượng không âm thì ta có:
2 2 2
a  b  b  c  c  a  0  a2  b2  c 2  ab  bc  ca
2
 a  b  c  3ab  bc  ca  9  3ab  bc  ca  ab  bc  ca  3 .
6 16 10
Cuối cùng, ta thu được P    .
3 93 3
Bài toán kết thúc.
Bài tập tương tự:
1. Cho a; b là hai số thực dương thỏa mãn a 2  b  2 . Tìm giá trị nhỏ nhất của biểu thức
1 1
P   2a b .
a2 b
2. Cho a; b; c là các số thực dương. Tìm giá trị nhỏ nhất của biểu thức:
a  b  cab  bc  ca 4bc
P  .
abc b  c 
2

Câu 3.

Liên hệ tài liệu word toán sđt và zalo: 039.373.2038 TÀI LIỆU TOÁN HỌC
100
Website:tailieumontoan.com

M
K
C
B

L
S
O
P
A D

TP TD TC
1). Gọi MN giao PQ tại T . Theo định lí Thales, ta có   .
TC TB TQ
Từ đó TC 2  TP.TQ .
Do TC là tiếp tuyến của (O) , nên TC 2  TM.TN .
Từ đó TM.TN  TC 2  TP.TQ , suy ra tứ giác MNPQ nội tiếp.
Nhận xét. Có nhiều cách để chứng minh bốn cùng điểm nằm trên một đường tròn, trong
đó đưa về chứng minh bốn điểm tạo thành một tứ giác nội tiếp là thông dụng nhất.
Nhắc lại kiến thức và phương pháp.
• Định lý Thales.
TP TD TC
PD  BC ta có    TC 2  TP.TQ
TC TB TQ
• Bình phương độ dài tiếp tuyến của đường tròn từ một điểm nằm ngoài đường tròn
bằng tích các đoạn chia ra bởi một cát tuyển từ điểm đó với đường tròn.
TC là tiếp tuyến của đường tròn ( O ) ta có TC 2  TM.TN
Kết hợp với trên, ta được TP.TQ  TM.TN
• Tứ giác có hai cạnh cắt nhau tại một điểm nằm ngoài tứ giác tạo thành các tích các
đoạn bằng nhau là tứ giác.
Tứ giác BCPD có cạnh BD và PC kéo dài cắt nhau tại điểm T nằm ngoài tứ giác và có
TP.TQ  TD.TB suy ra tứ giác BCPD là tứ giác nội tiếp hay bốn điểm B ; C ; P ; D cùng
nằm trên một đường tròn.

2). Gọi MP giao (O) tại điểm thứ hai S.


Ta có các biến đổi góc sau:
  CMS
KML   SCP
 (góc tạo bởi tiếp tuyến và dây cung)
  SPC
 MSC  (góc ngoài)
  MNQ
 MNC  (do các tứ giác MNPQ và MNSC nội tiếp).

Liên hệ tài liệu word toán sđt và zalo: 039.373.2038 TÀI LIỆU TOÁN HỌC
101
Website:tailieumontoan.com
.
 KNL
  KNM
Từ đó tứ giác MKLN nội tiếp, suy ra KLM   QPM , nên KL  PQ  OC . Vậy KL  OC.

Nhận xét. Chứng minh hai đường thẳng vuông góc ta sử dụng tính chất bắc cầu chứng
minh đường thẳng song song với đường thẳng khác vuông góc với đường thẳng kia.
Nhắc lại kiến thức và phương pháp.
• Góc tạo bởi tia tiếp tuyến - dây cung và góc nội tiếp cùng chắn một cung của một
đường tròn thì bằng nhau.
 là góc tạo bởi tai tiếp tuyến CP và dây cung CS ; Góc CMS
Góc SCP  là góc nội tiếp.

Hai góc này cùng chắn cung CS   CMS


 của đường tròn (O) suy ra KML   SCP

• Góc ngoài tại một đỉnh của tam giác bằng tổng hai góc trong không kề với nó.
Góc  là góc ngoài tại đỉnh
MSC S của tam giác CSP nên ta có
  CPS
MSC   SCP
  SCP
  MSC
  CPS

• Hai góc nội tiếp cùng chắn một cung của một đường tròn thì bằng nhau.
  CNM
+ CSM  (hai góc nội tiếp cùng chắn cung CM
 của đường tròn ngoại tiếp tứ giác
CSNM ).
  MNQ
+ CPS  (hai góc nội tiếp cùng chắn cung MQ
 của đường tròn ngoại tiếp tứ giác

MNPQ )
  CNM
Kết hợp trên, ta được KML   KNL
  MNQ .
• Tứ giác có hai đỉnh liên tiếp cùng nhìn một cạnh dưới hai góc bằng nhau thì tứ giác đó
là tứ giác nội tiếp.
Xét tứ giác MKLN có hai đỉnh M và N liên tiếp cùng nhìn cạnh KL dưới hai góc bằng
  KNL
nhau KML  suy ra MKLN là tứ giác nội tiếp.
  KNM
Theo nhắc lại kiến thức trên ta có MKLN là tứ giác nội tiếp nên KLM  (hai góc
 ).
nội tiếp cùng chắn cung KM
  QPM
KNM  (hai góc nội tiếp cùng chắn cung MC
 của đường tròn (O) ), suy ra
  KNM
KLM   QPM
.

• Một đường thẳng cắt hai đường thẳng đã cho tạo ra hai góc đồng vị bằng nhau thì hai
đường thẳng đã cho song song.
Hai đường thẳng KL và PQ có KLM   QPM  mà hai góc này ở vị trí đồng vị suy ra
KL  PQ .
• Đường thẳng a  b và b  c suy ra a  c .

KL  PQ
Ta có  , suy ra KL  OC .

PQ  OC

Câu 4.
Trả lời: Không tồn tại
Giả sử có chứa 36 số nguyên liên tiếp có tổng bằng
n  n  1    n  35  18 2n  35 không chia hết cho 4 vì 2n  35 là số lẻ. Mặt khác từ 9
9.8
số thì tổng ai  a j ( 1  i  j  9 ) nhận  36 giá trị trong đó mỗi số xuất hiện bốn lần; suy
2

Liên hệ tài liệu word toán sđt và zalo: 039.373.2038 TÀI LIỆU TOÁN HỌC
102
Website:tailieumontoan.com

ra  a  a   4 a
i j 1
 a2    a9  vì số các tổng bằng 36 nhận 36 giá trị là 36 số nguyên liên
tiếp nên ta suy ra:
18 2n  35  4 a1  a2    a9  .
(mâu thuẫn vì 18 2n  35 không chia hết cho 4).
Nhận xét. Đây là một bài toán tư duy, không có phương pháp chung
Nhắc lại kiến thức và phương pháp.
a a  1
• Tổng của các số nguyên từ 1 đến a bằng
2
35.36
n  n  1    n  35  36n  1  2    34  35  36n 
2
 36n  35.18  18 2n  35 .
• Số lẻ không chia hết cho 4.
Vì 2n  35 là tổng của một số chẵn và một số lẻ nên là số lẻ do đó 2n  35 không
chia hết cho 4.
• Tích của hai thừa số không chia hết cho 4 thì tích đó không chia hết cho 4.
Ta có 2n  35 không chia hết cho 4 (chứng minh trên) và 18 không chia hết cho 4 nên
18 2n  35 không chia hết cho 4.
• Từ 9 số bất kỳ, ta có thể tìm ra 36 cặp số m ; n .
Chứng minh: Với 1 cách chọn số m và 8 cách chọn số n nên có 1.8 cặp số m ; n .
Do đó, với 9 số m và 8 cách chọn số ta có 9.8 cách chọn cặp số m ; n .
Tuy nhiên, mỗi cặp số được lặp lại 2 lần thành m ; n và n ; m không được xét nên tất
cả có 36 cặp số m ; n được hình thành.
Áp dụng tính chất trên ta có từ 9 số bất kỳ ta lập được tổng ai  a j ( 1  i  j  9 ) nhận
9.8
2
 36 giá trị trong đó mỗi số xuất hiện 4 lần nên  (a  a )  4(a
i j 1
 a2    a9 ) vì số

các tổng bằng 36 nhận 36 giá trị là 36 số nguyên liên tiếp nên ta suy ra:
18 2n  35  4 a1  a2    a9  .
• Hai vế của một đẳng thức luôn có cùng số dư khi chia cho một số.
Ta có VT  18 2n  35 không chia hết cho 4 (chứng minh trên) và
VP  4 a1  a2    a9  chia hết cho 4 nên không thể tồn tại đẳng thức
18 2n  35  4 a1  a2    a9  .
Do đó không tồn tại 9 số nguyên a1 , a2 , , a9 sao cho tập các giá trị của tổng ai  a j (
1  i  j  9 ) có chứa 36 số nguyên liên tiếp.

ĐỀ SỐ 11.
Câu 1.
1). Sử dụng hằng đẳng thức
 
a 3  b3  c 3  3abc  a  b  c a 2  b2  c 2  ab  bc  ca .

Liên hệ tài liệu word toán sđt và zalo: 039.373.2038 TÀI LIỆU TOÁN HỌC
103
Website:tailieumontoan.com

Ta có x 3  y 3  8  6 xy  7
 
  x  y  2 x 2  y 2  4  xy  2 x  2 y  7 .

1 2 2 2
Mà x 2  y 2  4  xy  2 x  2 y   x  y   x  2   y  2   0 .
2  
Suy ra ta có 2 trường hợp:

x  y  2  1
+ TH1:  2 2
 y  1  x
x  y  4  xy  2 x  2 y  7

2  x  0  y  1
 x 2  1  x  4  x 1  x  2 x  2 1  x  7  3 x 2  3 x  0   .
 x  1  y  0

x  y  2  7
+ TH2:  2  y  5 x
x  y 2  4  xy  2 x  2 y  1

2
 x 2   5  x  4  x  5  x  2 x  2  5  x  1
x  2  y  3
 3 x 2  15 x  18  0   .
x  3  y  2

Đáp số  x; y  1; 0 , 0;  1 , 2; 3 , 3; 2 .
Nhận xét: Bài toán sử dụng phương pháp phân tích đẳng thức, kết hợp với kỹ thuật chọn
nghiệm nguyên để tìm x; y .
Nhắc lại kiến thức và phương pháp:


Hằng đẳng thức: a3  b3  c 3  3abc  a  b  c a2  b2  c 2  ab  bc  ca . 
Ý tưởng: Cái hay và khó ở bài toán này chính là bước phát hiện ra đẳng thức đã vốn rất
quen thuộc. Đầu tiên, quan sát phương trình nghiệm nguyên ta thấy:
x 3  y 3  1  6 xy  x 3  y 3  8  6 xy  7 và dễ dàng thấy rằng 8  2 3 ; 6 xy  3.x.y.2 vì thế nếu

đặt z  2 , ta sẽ có được một đẳng thức đối xứng rất đẹp là: x 3  y 3  z 3  3xyz  7 .
Khéo léo biến đổi, ta có:
3
 x 3  y 3  z 3  3 xyz  7   x  y  z 3  3 xy  x  y  z  7

  x  y  z x 2  2 xy  y 2  xz  yz  z 2   3 xy  x  y  z  7

  x  y  z x 2  y 2  z 2  xy  yz  xz  7 (*).

1
Mặt khác x 2  y 2  z 2  xy  yz  xz   x  y   y  z   z  x   0 . nên để phương trình
2 2 2

2 
(*) có nghiệm nguyên khi và chỉ khi:

x  y  2  1; x , y    x  0  y  1
• TH1. Với  2 

.

x  y  xy  2 x  2 y  3  x  1  y  0
2


x  y  2  7; x , y   x  2  y  3
• TH2. Với  2 

.

x  y  xy  2 x  2 y  3  x  3  y  2
2

Liên hệ tài liệu word toán sđt và zalo: 039.373.2038 TÀI LIỆU TOÁN HỌC
104
Website:tailieumontoan.com
Do đó, các cặp nghiệm nguyên cần tìm là
x; y  2; 3 , 3; 2 , 0;  1 , 1; 0 .
Bài toán kết thúc.
Bài tập tương tự:
1. Giải phương trình nghiệm nguyên x 3  y 3  1  3xy .
2. Giải phương trình nghiệm nguyên 8 x 3  y 3  6 xy  5 .
2
2). Với k là số nguyên dương và k 2  m   k  1  1 thì ta có k  m  k  1   m   k.
 
2 2
Cho m  k 2 , k 2  1, , k  1  1 (có tất cả k  1  k 2 giá trị) ta đều có
 2  2   2 
 k    k  1      ( k  1)  1   k .
     
Từ đó suy ra tổng cần tính bằng

  
1    2    3    4    5      8   
           
     
  (n  1)2    (n  1)2  1      n2  1 
       
 2
   
 2 2  1  1  32  2 2  2    n2  n  1  n  1
 
2
 2 2  1  2  32  2  2 2  3  4 2  3  32    n  1  n2  n  1  n  1
2
 12  2 2    n  1  n  1 n2
1 1
 n  1 n2  n  1 n2n  1  nn  14n  1 .
6 6
Ghi chú: Thí sinh có thể chứng minh bằng quy nạp toán học.
Nhận xét. Bài toán về phần nguyên
Nhắc lại kiến thức và phương pháp.
• Tính chất phần nguyên  x  a thì a  x  a  1
 m   k  k  m  k  1  k 2  m   k  12 hay k 2  m   k  12  1 với k là số nguyên
 
dương.
2
• Các số m nguyên dương thỏa mãn k 2  m   k  1  1 là

 2
m  k 2 ; k 2  1; k 2  2; ;  k  1  2;  k  1  1 .
2

• Số các số của dãy số tự nhiên liên tiếp tăng là “Số cuối - Số đầu + 1”
2 2
Có tất cả  k  1  1  k 2  1   k  1  k 2 số nguyên m thỏa mãn điều kiện
 2  2   2 
 k    k  1     
   
k  1  1  k .


• Nhóm các số hạng của tổng cần tính thành các nhóm có tính chất như số m.

Liên hệ tài liệu word toán sđt và zalo: 039.373.2038 TÀI LIỆU TOÁN HỌC
105
Website:tailieumontoan.com

   
Q   1    2    3    4    5      8   
           
 2   
  n  1    n  1  1      n2  1   
2

      

 
 2 2  1  1  32  2 2   2     n2  n  1  n  1
2

 
2
 2 2  1  2  32  2  2 2  3  4 2  3  32    n  1.n2  n  1.n  1
 2
 12  2 2    n  1   n  1 n2 .
 
1
• Áp dụng biểu thức 12  2 2    k 2  k  k  12 k  1
6
 2 2   2
Q  1  2 2    n  1   n  1 n2  n  1 n2  12  2 2    n  1 
   
1 1 1
 n  1 n2  n  1 n2n  1  n  1 n6n  2n  1  n  1 n4n  1 .
6 6 6

Câu 2.
1). Phương trình đầu của hệ tương đương với
3
 x  1  3 x  1  y 3  3 y  0
 2 
  x  y  1  x  1   x  1 y  y 2  3  0
 
 x  y  1  y  1 x .
Thế vào phương trình thứ hai của hệ ta được
2
2 x 2  x 1  x  1  x  11
 x  2  y  1
2 
 4 x  3 x  10  0   .
 x  5  y  9
 4 4
 5 9 
Đáp số  x; y  2;  1 ,  ; .
  4 4 
Nhận xét: Bài toán sử dụng phương pháp tìm nhân tử từ một phương trình sau đó thế vào
phương trình còn lại tìm nghiệm của hệ phương trình.
Nhắc lại kiến thức và phương pháp:
• Cách giải phương trình bậc hai tổng quát: a.t 2  b.t  c  0 ( a  0 ).
• Hằng đẳng thức: a3  b3  a  ba2  ab  b2  và
3
ax  by  a 3 x 3  3a 2 bx 2 y  3ab2 xy 2  b3 y 3 .
Ý tưởng: Quan sát hệ phương trình, phương trình đầu là phương trình bậc ba (bậc giảm từ
3 đến 0), phương trình còn lại là phương trình bậc hai vì thế không thể đưa về dạng đẳng
cấp bậc ba. Do đó việc kết hợp cả hai phương trình đã thất bại, vì vậy ta sẽ đi xét từng
phương trình một của hệ. Ở phương trình hai có thể làm nháp là: xét đenta bậc hai đối với
x hoặc y thì rõ ràng đenta không chính phương. Ta chuyển sang phương trình một, rõ
ràng ta thấy được sự độc lập giữa hai biến x; y . Ở biến y xuất hiện hàm y 3  3 y đồng thời
biến x xuất hiện hàm x 3  3x 2  6 x  4 . Vậy nên để tìm được nhân tử chung giữa x; y thì

Liên hệ tài liệu word toán sđt và zalo: 039.373.2038 TÀI LIỆU TOÁN HỌC
106
Website:tailieumontoan.com
3
ta cần đưa hàm x về hàm đơn giản y tức là đưa về dạng  x  a  3 x  a  y 3  3 y  0 .
3
Bằng phương pháp đồng nhất hệ số ta có:  x  a  3 x  a  x 3  3x 2  6 x  4  a  1 . Do đó
3
phương trình đầu của hệ   x  1  3 x  1  y 3  3 y  0 . Bây giờ ta xét đến hằng đẳng
thức dạng a3  b3  a  ba2  ab  b2  thì sẽ xuất hiện nhân tử chung là x  1  y như sau:
3
 x  1  3 x  1  y 3  3 y  0
 2 
  x  1  y  x  1   x  1 y  y 2   3 x  1  y  0
 
 2 
  x  y  1  x  1   x  1 y  y 2  3  0
 
y  1 x
  2 .
 x  1   x  1 y  y  3  0 (*)
2

2
 1  3

Phương trình (*) vô nghiệm vì (*)   x  1  y  y 2  3  0 .
 2  4
Và thế y  1  x vào phương trình hai giải phương trình bậc hai.
Bài toán kết thúc.
Bài tập tương tự:
 3 3
x  4 y  y  4 x

1. Giải hệ phương trình  2 .

 y  4 y  5  2  3 x


8  2 13 8  2 13 
Đáp số:  x; y   ; 
 3 3 
x 3  x  2  y 3  3 y 2  4 y
2. Giải hệ phương trình  5
x  y 3  1  0

Đáp số:  x; y  0;  1 .
2). Đặt a  x  3  x 2  0
a2  3
 a  x  3  x 2  a 2  2 ax  x 2  3  x 2  x  .
2a
b2  3
Tương tự đặt b  y  3  y 2  0  x  .
2b
b 3 a 3
Khi đó x  y     .
2 2 a 2b
2
Mà theo giả thiết a  b  9
a 9 3 a a 3 a 3
xy       2   2.
2 2a 2a 6 3 a 3 a
3 2
Lại có x 2  xy  y 2   x  y  x 2  xy  y 2  3 .
4
Dấu “=” xảy ra  x  y  1 .

Vậy x 2  xy  z 2 mim  3.
Nhận xét: Bài toán sử dụng phương pháp đặt ẩn phụ, khai thác giả thiết cùng bất đẳng
thức Cosi để tìm giá trị nhỏ nhất.
Nhắc lại kiến thức và phương pháp:
Liên hệ tài liệu word toán sđt và zalo: 039.373.2038 TÀI LIỆU TOÁN HỌC
107
Website:tailieumontoan.com

• Cách giải phương trình có dạng x  x 2  n  m


m2  n
x 2  n  m  x  x 2  n  x 2  2 mx  m2  x 
2m
Hoặc có thể giải bằng cách liên hợp như sau
n
x  x2  n  m 
 x2  n  x
m
n m2  n

 x  x2  n  x2  n  x  m   x 
m 2m
.
• Bất đẳng thức Cosi cho hai số thực dương a  b  2 ab .
Ý tưởng: Bài toán xuất hiện giả thiết khá phức tạp, chưa biết khai thác như thế nào. Nhưng
ở cả biểu thức P hay giả thiết bài cho đều có sự đối xứng giữa x , y nên ta dự đoán điểm

 
2
rơi xảy ra tại x  y  k . Khi đó thay ngược lại giả thiết, ta có: k  k 2  3  9  k  1 . Và
Pmin  3 , đến đây công việc sẽ thuận lợi hơn cho ta rất nhiều khi biết trước được điểm rơi.
2
Với x  y ta sẽ có đánh giá rất quen thuộc  x  y  0  x 2  y 2  2 xy
 4  x 2  xy  y 2   3 x 2  2 xy  y 2 
3 2 3 2
 x  y  P   x  y .
 x 2  xy  y 2 
4 4
Vậy nên nếu bây giờ, đi từ giả thiết ta tìm được x  y  2 thì coi như bài toán sẽ được giải
quyết. Biến x xuất hiện trong x  x 2  3 đồng thời y xuất hiện ở y  y 2  3 . Do đó tư
tưởng của ta sẽ là rút x; y từ giả thiết bằng cách đặt ẩn phụ như sau: đặt
a  x  x2  3  x2  3  a  x
a2  3 b2  3
. Tương tự với b  y  y 2  3 ta sẽ thu được y 
 x 2  3  a 2  2 ax  x 2  x  .
2a 2b
a2  3 b2  3
Khi đó, cần chứng minh   2 với điều kiện ab  9 . Ta dễ dàng chứng minh
2a 2b
được như sau:
a2  3 b2  3 a b 3 3 a 3 a 3
       2 . 2
2a 2b 2 2 2 a 2b 3 a 3 a
Bài toán kết thúc.
Bài tập tương tự:
 
1. Cho x; y là hai số thực thỏa mãn x  x 2  2 y  y 2  2  4 . Tìm giá trị nhỏ nhất 
của biểu thức P  2  x 2  y 2   x  y .

 
2. Cho x; y là hai số thực thỏa mãn x  y 2  1 y  x 2  1  1 . Tìm giá trị nhỏ nhất 
của biểu thức P  2  x 2  y 2   x  y .

Câu 3.

Liên hệ tài liệu word toán sđt và zalo: 039.373.2038 TÀI LIỆU TOÁN HỌC
108
Website:tailieumontoan.com

A B

F C
E D M S
P
R O

A B

F C
E D MS
P
O
R

1). Gọi S điểm đối xứng với P qua M. Theo tính chất đối xứng của hình thang cân dễ
thấy tứ giác ABSP cũng là hình thang cân.
  QAB
Ta lại có QPS   QRB
.
  ERP
Từ đó có EPQ   ERP ∽ EPQ (g – g),
  EPR
nên EQP   BPS
  ASE
 , suy ra tứ giác AEQS nội tiếp.
PF
Do đó PA.PQ  PE.PS  .2 PM  PF.PM , suy ra tứ giác AMQF nội tiếp. Từ đó suy ra
2
đường tròn ngoại tiếp tam giác AQF luôn đi qua M .
Nhận xét. Ta chứng minh đường tròn ngoại tiếp tam giác AQF luôn đi qua M , khi đó
tâm ngoại tiếp tam giác AQF luôn thuộc đường trung trực của AM là một đường thẳng
cố định.
Nhắc lại kiến thức và phương pháp.
• Hình thang có hai đỉnh kề cạnh đáy đối xứng qua trung trực của mỗi đáy là hình
thang cân. Hình thang cân có trục đối xứng là trung trực chung của hai đáy.
ABCD là hình thang cân nên A và B đối xứng qua trung trực của AB .
P và S đối xứng với nhau qua M hay P và S đối xứng qua trung trực của PS .
Suy ra ABSP là hình thang có A , B đối xứng qua trung trực của AB và P , S đối xứng
qua trung trực PS do đó ABSP là hình thang cân.

Liên hệ tài liệu word toán sđt và zalo: 039.373.2038 TÀI LIỆU TOÁN HỌC
109
Website:tailieumontoan.com
• Hai góc nội tiếp cùng chắn một cung của một đường tròn thì bằng nhau.
  RQA
RBA  (hai góc nội tiếp cùng chắn cung RA
 của đường tròn (O) ).
  RBA
EPR  (hai góc đồng vị của hai đường thẳng EP  AB ),
  RQA
suy ra EPR .

• Hình thang cân cũng là tứ giác nội tiếp.


  ABP
ABSP là hình thang cân nên ABSP cũng là tứ giác nội tiếp, do đó ASP  (hai góc
 của đường tròn ngoại tiếp tứ giác ABSP ).
nội tiếp cùng chắn cung AP
  AQE
Ta có ASE  suy ra tứ giác AEQS nội tiếp.

• Giao điểm hai đường chéo của tứ giác nội tiếp tạo ra tích các đoạn thẳng tương ứng
bằng nhau (Chứng minh dựa vào tam giác đồng dạng trường hợp góc – góc.
AEQS là tứ giác nội tiếp có P là giao điểm của hai đường chéo nên ta có
PF
PA.PQ  PE.PS  .2 PM  PF.PM .
2
• Tứ giác có giao điểm hai đường chéo tạo thành tích các đoạn thẳng tương bằng nhau
là tứ giác tứ giác nội tiếp (Chứng minh dựa vào tam giác đồng dạng trường hợp
cạnh.góc.cạnh).
Tứ giác AMQF có PA.PQ  PF.PM do đó AMQF nội tiếp. Từ đó suy ra đường tròn
ngoại tiếp tam giác AQF luôn đi qua M cố định.

2). Vì EA là tiếp xúc (O) và từ kết quả câu 1). ta có EA 2  ER.EQ  EP 2 . Từ đó có EA  EP ,


  EAP
suy ra DAP   EAD
  APE
  ACD
  PAC
.
 , suy ra QC  QD  QM  CD .
Do đó AP là phân giác DAC
Nhận xét. Chứng minh hai đường thẳng vuông góc, ta ghép vào đường tròn để áp dụng
tính chất “Đường thẳng đi qua điểm chính giữa cung và trung điểm của dây cung thì
vuông góc với dây cung tại trung điểm của dây” vào giải bài toán.
Nhắc lại kiến thức và phương pháp.
Áp dụng chứng minh phần 1). ta có EA 2  ER.EQ  EP 2 suy ra EA  EP .
• Tam giác có hai cạnh bằng nhau là tam giác cân. Tam giác cân có hai góc kề cạnh đáy
bằng nhau.
  EPA
Tam giác AEP có EA  EP suy ra AEP cân tại E suy ra EAP 
• Góc tạo bởi tia tiếp tuyến - dây cung và góc nội tiếp cùng chắn một cung của một
đường tròn thì bằng nhau.
 là góc tạo với tia tiếp tuyến AE và dây cung AD ; Góc ACD
Góc EAD  là góc nội tiếp
  ACD
 của đường tròn (O) nên ta có EAD
cùng chắn cung AD .
  EAD
Trừ vế theo vế của hai đẳng thức ta được EAP   EPA  ACD

• Góc ngoài tại một đỉnh của tam giác bằng tổng hai góc trong không kề với nó.
Tam giác
APC có 
EPA là góc ngoài tại đỉnh P suy ra
  ACD
EPA   PAC
  PAC
  EPA
  ACD
 kết hợp với đẳng thức trên ta được
  EAD
EAP   EPA
  ACD
  DAP
  PAC
 , suy ra AQ là phân giác góc DAC
.

Liên hệ tài liệu word toán sđt và zalo: 039.373.2038 TÀI LIỆU TOÁN HỌC
110
Website:tailieumontoan.com
• Tia phân giác của góc nội tiếp một đường tròn cắt đường tròn tại điểm chính giữa
cung bị chắn.
 chắn cung DC
AQ là phân giác góc nội tiếp DAC  của đường tròn (O) cắt (O) tại Q
.
nên suy ra Q là đỉnh chính giữa cung DC
• Đường thẳng đi qua điểm chính giữa cung và trung điểm của dây cung thì vuông góc
với dây cung tại trung điểm của dây.
 ; M là trung điểm của dây DC suy ra QM  DC
Ta có Q là đỉnh chính giữa cung DC
tại M .

Câu 4.
Trước hết ta chứng minh với mọi cách chọn 1000 ô trên bảng đã cho luôn tồn tại một bảng
con 2  2 chứa đúng 1 trong 1000 ô này.
Thật vậy, vì số hàng lớn hơn số ô được chọn nên tồn tại hai hàng liền nhau R1 , R2 mà R1
không chứa ô nào và R2 có chứa ít nhất một ô đã chọn. Vì số cột cũng lớn hơn số ô được
chọn nên tồn tại hai ô A , B cạnh nhau trên R2 mà chỉ có đúng một ô đã chọn. Gọi C , D là
2 ô nằm trên R1 và cùng cột với A , B . Bảng con 2  2 gồm 4 ô A , B , C , D chỉ có đúng một ô
được chọn.
Giả sử ta có thể thu được bảng gồm đúng 1000 ô màu đỏ sau hữu hạn lần đổi màu. Khi đó
theo chứng minh trên tồn tại một bảng vuông con 2  2 chứa đúng một ô màu đỏ, ba ô còn
lại màu xanh.
Vì ở trạng thái ban đầu tất cả các bảng vuông con 2  2 đều gồm 4 ô màu xanh nên mỗi
lần đổi màu hàng hoặc cột thì số ô màu đỏ và số ô màu xanh trong bảng vuông con 2  2
luôn là số chẵn. Do đó không thể thu được một bảng vuông con 2  2 có 1 ô màu đỏ, 3
màu xanh. Ta có mâu thuẫn.
Vậy không thu được bảng chứa đúng 1000 ô màu đỏ sau hữu hạn lần đổi màu.

ĐỀ SỐ 12.

Câu 1.
1). Cộng 2 phương trình của hệ, ta được 3x 2  y 2  4 xy  4  4 x
2 2 x  y  2
  2 x  y    x  2   .
 3 x  y  2

Nhận xét: Bài toán kết hợp sự tinh tế giữa hai phương trình, để đưa ra được một phương
trình tìm được mối quan hệ giữa x; y ( hay còn gọi là phương pháp hệ số bất định giải hệ
phương trình hữu tỷ ). Và từ đó thế ngược lại một trong hai phương trình của hệ ban đầu
để tìm nghiệm.
Nhắc lại kiến thức và phương pháp:
• Cách giải phương trình bậc hai tổng quát: a.t 2  b.t  c  0 ( a  0 ).
 f  x , y  0
• Đưa hệ phương trình đã cho về dạng  .
  

 g x, y  0

Liên hệ tài liệu word toán sđt và zalo: 039.373.2038 TÀI LIỆU TOÁN HỌC
111
Website:tailieumontoan.com
• Giả sử tồn tại k   sao cho f  x; y  k.g  x; y  0 ( i ). Coi phương trình ( i ) là phương
trình bậc hai ẩn x hoặc ẩn y , sau đó xét đenta sao cho đenta là một số chính phương,
từ đó sẽ tìm được k   cũng như mối quan hệ giữa x; y .
Ý tưởng: Nhận thấy hai phương trình của hệ không thể tìm được nhân tử chung trong
chính nó, đồng thời ở phương trình hai có sự độc lập của biến y nên có thể rút y và thế
vào phương trình một sẽ được phương trình bậc bốn, nhưng ta chưa biết nó có nghiệm
đẹp không, nhỡ chẳng may nó có nghiệm hữu tỷ hoặc nghiệm không tường minh. Vì vậy,
ta nghĩ đến hướng kết hợp cả hai phương trình của hệ, bằng cách tìm số k   thỏa mãn
phương trình f  x; y  k.g  x; y  0 ( i ). Sẽ có rất nhiều giá trị của k   đạt yêu cầu bài toán
nhưng ta chỉ tìm k duy nhất sao cho i là một số chính phương thì mới có thể tìm được
nhân tử từ ( i ), ta làm như sau:
x 2  y 2  xy  3  0
• Hệ phương trình đã cho tương đương với  2 .
2 x  3 xy  4 x  1  0

• Giả sử tồn tại k   thỏa mãn:
x 2  y 2  xy  3  k 2 x 2  3 xy  4 x  1  0
 2 k  1 x 2  3 ky  y  4 k x  y 2  k  3  0 (*).
• Xét phương trình (*), coi là phương trình bậc hai ẩn x , ta có:
x  3 ky  y  4 k   4 2 k  1 y 2  k  3
2

 9 k 2  2 k  3 y 2  8 k 3 k  1 y  16 k 2  4 2 k  1 k  3 .
Để x là một số chính phương thì hệ số của y 2 trong biệt thức x cũng phải là một số
chính phương, tức là ta cần phải đi giải phương trình nghiệm nguyên:

k  1
9 k 2  2 k  3  m2 . Dễ thấy phương trình này có nghiệm  2 , thay ngược lại x , ta

m  4

có:
 4  4y  2y  8
 3 x 2   4 y  4 x  y 2  4  0  x 
 x  y  2

  6
 2
 
 3x  2 y  2  0
.

   2 y  8   4  4y  2y  8 

 x x 
 6
• Với việc k  1 , hay nói cách khác là cộng vế với vế của hai phương trình trong hệ, ta có
thể biến đổi như sau:
x 2  y 2  xy  3  2 x 2  3 xy  4 x  1  0
2
 2 x  y 2  x  2   x  y  23 x  y  2  0 .
Bài toán kết thúc.
Bài tập tương tự:

 1

 x2  y 2 
 5
1. Giải hệ phương trình  .

 57


4 x 2
 4 x   y  3 x  1  0

 25
 2 1   11 2 
Đáp số:  x; y   ;  ,  ; .
 5 5   25 5 

Liên hệ tài liệu word toán sđt và zalo: 039.373.2038 TÀI LIỆU TOÁN HỌC
112
Website:tailieumontoan.com
x 2  y 2  1
2. Giải hệ phương trình  2 .
96 x  21x  3 y  28 xy  117

 24 7 
Đáp số:  x; y  1; 0 ,  ;  .
 25 5 
2). Phương trình tương đương với 2 x  3  2 3x  1  2 x  2 .

u  x  3
Đặt   v 2  u2  2 x  2 với u; v  0 , ta được

v  3 x  1


2 u  v  v 2  u2  v  uv  u  2  0
 v  u  2  3x  1  x  3  2  3x  1  7  x  4 x  3

x  3
 2x  6  4 x  3  2 x  3  x  3   
4 x  3  x 2  6 x  9
 



x  3 x  5  28
  2    x  5  28 .
x  10 x  3  0 x  3
 
Nhận xét: Bài toán sử dụng phương pháp đặt ẩn phụ để đưa phương trình từ phức tạp về
đơn giản hóa, sau đó dùng phương pháp nâng lũy thừa để tìm nghiệm của phương trình.
Nhắc lại kiến thức cũ và phương pháp:
• Hằng đẳng thức cơ bản a2  b2  a  ba  b .
• Cách giải phương trình vô tỷ dạng f  x  g  x  m
 f  x ; g  x  0  f  x ; g  x  0

 2 

 
 f  x  g  x  m
  f  x  g  x  m2  2 m g  x


 f  x ; g  x  0  x  x1


   .
  2
2 x  x



f  x   g  x   m

 4 m 2
g  x   2

Ý tưởng: Một bài toán đẹp, quan sát VT của phương trình có xuất hiện hai căn bậc hai
riêng biệt đồng thời trong căn chứa các biểu thức bậc nhất, cũng như VP của phương trình
cũng là một biểu thức bậc nhất, nên vậy ta có thể nâng lũy thừa để đưa phương trình ban
đầu về phương trình bậc bốn. Nhưng nếu tinh ý một chút, ta có


u  x  3
3x  1  x  3  2 x  2  2x  1 , do vậy nếu đặt 

với u; v  0 suy ra
v  3 x  1


v 2  u2  2 x  2  2  x  1 . Khi đó phương trình đã cho tương đương với
v 2  u2  2 v  u  v  uv  u  2  0 ( i ).
u  v  0
• Vì u; v  0 nên phương trình ( i )   , với u  v  0 phương trình vô nghiệm nên
v  u  2
ta chỉ cần giải phương trình v  u  2 .
• Với v  u  2 , ta có 3x  1  x  3  2  3x  1  7  x  4 x  3

x  3
 2x  6  4 x  3  2 x  3  x  3  

4 x  3  x 2  6 x  9
 


Liên hệ tài liệu word toán sđt và zalo: 039.373.2038 TÀI LIỆU TOÁN HỌC
113
Website:tailieumontoan.com
x  3 x  5  28
  2    x  5  28 .
x  10 x  3  0 x  3
 
Bài toán kết thúc.
Bài tập tương tự:
1. Giải phương trình: 4 x  1  2 x  2  2 x  1 . Đáp số: x  2  5
5  87
2. Giải phương trình: x  3  5 x  2  1  4 x . Đáp số: x 
8
Câu 2.
1). Ta có n3  0,  1mod 9 .
Vì 2001  3mod 9  x 3  9 m  1, y 3  9t  1, z 3  9u  1
Suy ra ta tìm x , y , z trong tập hợp 1, 4, 7, 10, 13
Không giảm tính tổng quát giả sử: x  y  z
• Giả sử z  13  z 3  2197  2001 (loại).
• Giả sử z  7  2001  x 3  y 3  z 3  3z 3  3.7 3 (loại).
Suy ra z  10  x 3  y 3  1001
Giả sử y  10  x  y  7  1001  x 3  y 3  2.7 3  2.343 (loại).
Suy ra y  10, x  1.
Đáp số: 1; 10; 10 , 10; 1; 10 , 10; 10; 1 .
Nhận xét. Giải phương trình nghiệm nguyên bằng phương pháp xét số dư kết hợp đánh
giá
Nhắc lại kiến thức và phương pháp.
• Lập phương của một số nguyên dương khi chia cho 9 chỉ dư 0, 1 hoặc 8 (hay 0, 1 hoặc
1 ). Viết dưới dạng tổng quát, ký hiệu toán học
n3  0 mod 9

 3
n  1mod 9 .
 3
n  1mod 9

Ta có x  y  z  2001 mà 2001  3 mod 9 nên mỗi x 3 ; y 3 ; z 3 chia 9 đều dư 1.
3 3 3

• Lập phương của một số nguyên dương khi chia cho 9 chỉ dư 1 thì số nguyên dương đó
chia 9 dư 1, 2 hoặc 5
Ta có x; y; z trong tập hợp 1; 4; 7; 10; 13
• Khi vai trò của x; y; z như nhau ta có thể giả sử chúng được sắp xếp theo một thứ tự
nào đó rồi từ đó dồn về một biến để tìm ra giá trị hoặc khoảng giá trị.
 2 2  2 2 2
 b   7 b    3   7    32 a  b 7 b 
2). Ta có a                
 2   2    2   2    4 4 
   
1  3 a 5b 
 a 2  2b2  ab     .
2  2 2 
1  3b 5c  1  3c 5a 
Tương tự b2  2c 2  bc     ; c 2  2 a2  ca    
2 2 2 2  2 2

Liên hệ tài liệu word toán sđt và zalo: 039.373.2038 TÀI LIỆU TOÁN HỌC
114
Website:tailieumontoan.com
Cộng 3 bất đẳng thức suy ra
a 2  2b2  ab  b2  2c 2  bc  c 2  2 a 2  ca  2 a  b  c .
Điều phải chứng minh.
Nhận xét: Bài toán sử dụng bất đẳng thức Bunhiacopxki dạng đơn giản hoặc lợi dụng
đánh giá của các đại lượng không âm.
Nhắc lại kiến thức và phương pháp:
• Bất đẳng thức Bunhiacopxki phát biểu dưới dạng: cho a; b; x; y là các số thực dương,
ta luôn có ax  by  a2  b2  x 2  y 2  .
2

2 2
• Các đại lượng không âm a  b  0; am  bn  0 .
Ý tưởng: Bất đẳng thức bài cho là một biểu thức đối xứng đẹp, với sự đối xứng này ta dự
đoán điểm rơi tại a  b  c . Bây giờ quan sát bên VT của biểu thức là các biểu thức đồng
nhất, trong căn bậc hai chứa hai biến số với vai trò như nhau a; b; c , mặt khác bên VP chứa
hàm số bậc nhất. Vậy nên ta tư duy nếu các biểu thức bậc hai trong căn được biểu diễn
hay được đánh giá qua một biểu thức chính phương thì nó sẽ xuất hiện biểu thức như ở
VP. Do a , b , c có vai trò như nhau, nên ta chỉ cần chỉ ra rằng với m , n dương sao cho
a 2  2b2  ab  ma  nb , a; b  0 , và hai căn còn lại chỉ là đánh giá tương tự. Và như dự
đoán ban đầu, điều ta mong muốn đó là:
2
a 2  2b2  ab  ma  nb  k a  b  0 với a , b , k  0 .
Vì vậy bằng cách đồng nhất hệ số biểu thức trên, ta có:
2
a 2  2b2  ab  ma  nb  a 2  2b2  ab  ma  nb

 1  m2  a 2  1  2 mn ab  2  n2  b2  k a  b  0
2


 k  1  m 2  2  n2 3 5 7

  m ; n ; k .
 4 4 16
2 k  2 mn  1

3 5
Với điều này, suy ra a2  2b2  ab  a  b; a , b  0 , từ đó suy luận ngược trở lại theo
4 4
Bunhiacopxki như sau:
 2  2 2
b   7 b    3   7    32 a  b 7 b 
2 2

a                
 2   2    2   2    4 4 
   
1  3 a 5b 
 a 2  2b2  ab     .
2  2 2
Tương tự cho hai căn thức còn lại, ta có:
1  3b 5c  1  3c 5a 
b2  2c 2  bc     ; c 2  2 a2  ca     .
2  2 2 2  2 2
Cộng các đánh giá trên lại với nhau, ta suy ra điều phải chứng minh.
Bài toán kết thúc.
Bài tập tương tự:
1. Cho a , b là hai số thực dương. Chứng minh bất đẳng thức:
4 a 2  2 ab  19b2  19 a 2  2 ab  4b2  5a  b .
2. Cho a , b , c là các số thực dương. Chứng minh bất đẳng thức:

Liên hệ tài liệu word toán sđt và zalo: 039.373.2038 TÀI LIỆU TOÁN HỌC
115
Website:tailieumontoan.com
a4 b4 c4 abc
   .
3
a b 3 3
b c 3 3
c a 3
2

Câu 3.
A

F O
E

C
D
B
P M
N
1). Gọi AD cắt (O) tại P khác A.
  PAC
Ta có PCM  (góc tạo bởi tiếp tuyến và dây cung)
 (góc đồng vị do EM  AC );
 PEM
  MEC
Suy ra tứ giác ECMP nội tiếp. Từ đó suy ra MPC   ECA
  CAP
  PM tiếp xúc (O)

Tương tự PN tiếp xúc (O) , suy ra MN tiếp xúc (O) tại P .


Nhận xét. Đối với bài toán này, ta dự đoán được MN tiếp xúc với (O) tại P . Sử dụng tính
chất từ một điểm nằm trên đường tròn ta có một và chỉ một tiếp tuyến với đường tròn.
Nhắc lại kiến thức và phương pháp.
• Một đường thẳng cắt hai đường thẳng song song tạo ra các góc đồng vị bằng nhau.
  PEM
EM  AC suy ra PAC  (hai góc đồng vị)

• Góc nội tiếp và góc tạo bởi tia tiếp tuyến và dây cung của một đường tròn cùng chắn
một cung thì bằng nhau.
  PCM
+ PAC  (góc nội tiếp và góc tạo bởi tiếp tia tiếp tuyến CM cùng chắn cung PC
của đường tròn (O) ).
  MEC
+ CAP  (góc nội tiếp và góc tạo bởi tia tiếp tuyến PM cùng chắn cung PC của
đường tròn (O) ).
• Tứ giác có hai đỉnh liên tiếp cùng nhìn một cạnh dưới một góc bằng nhau là tứ giác
nội tiếp.
Tứ giác ECMP có PEM  PCM  ) (chứng minh trên) suy ra ECMP là tứ giác nội
 (  PAC
tiếp.
• Hai góc nội tiếp cùng chắn một cung của một đường tròn thì bằng nhau.
  MEC
MPC  (hai góc nội tiếp cùng chắn cung MC của đường tròn ngoại tiếp tứ giác
ECMP )
• Tam giác có một đỉnh nằm trên trung trực của cạnh đối diện thì cân tại đỉnh đó.

Liên hệ tài liệu word toán sđt và zalo: 039.373.2038 TÀI LIỆU TOÁN HỌC
116
Website:tailieumontoan.com
Tam giác AEC có EO là trung trực của AC suy ra tam giác AEC cân tại E suy ra
  ECA
EAC 
• Góc tạo bởi một dây cung và một tia về phía ngoài đường tròn bằng góc nội tiếp chắn
cùng một cung thì tia đó là tiếp tuyến của đường tròn.
Kết hợp các ta có lý thuyết trên, suy ra PM là tiếp tuyến của đường tròn (O) tại P .
Hoàn toàn tương tự ta có PN là tiếp tuyến của đường tròn (O) tại P .
Vậy MN là tiếp tuyến của (O) tại P .

BN FN
2). Theo 1). dễ thấy BFA ∽ BNP  BNF ∽ BPA   (1).
BP AP
CM EM
Tương tự CME ∽ CPA   (2).
CP AP
BN CP FN FN BN
Từ (1) và (2), ta có   và theo giả thiết  , suy ra CP  BP  AD là
CM BP EM EM CM
.
phân giác góc BAC
Nhận xét. Cách thông thường để chứng minh một tia là phân giác của một góc ta chứng
minh tia đó chia góc thành hai góc bằng nhau.
Nhắc lại kiến thức và phương pháp.
• Các tam giác cân có một góc ở đáy bằng nhau thì đồng dạng với nhau.
Ta có FAB cân tại F và NBP cân tại N có   BPN
FAB  nên
  BPN
BFA ∽ BNP  BAF .
• Các góc nội tiếp cùng chắn một cung của một đường tròn thì bằng nhau.
  BPN
+ BFN  (hai góc nội tiếp cùng chắn cung BN của đường tròn ngoại tiếp tứ giác
FBNP ).
  BPF
+ BNF  (hai góc nội tiếp cùng chắn cung BF của đường tròn ngoại tiếp tứ giác
FBNP ).
• Hai tam giác các hai cặp góc tương ứng bằng nhau thì đồng dạng.
  BFN
Tam giác ABP và FBN có BAP  (  BPN
 ) (chứng minh trên) và BNF
  BPF

BN FN
(chứng minh trên) suy ra ABP ∽ FBN   hoàn toàn chứng minh tương tự
BP AP
CM EM
ta có CME ∽ CPA   .
CP AP
BN CP FN
Từ đây nhân vế theo về của hai đẳng thức ta được   .
CM BP EM
FN BN CP
Từ giả thiết  kết hợp với chứng minh trên ta suy ra  1 hay CP  BP .
EM CM BP
• Hai cung có hai dây cung bằng nhau thì bằng nhau.
  BP
CP  BP suy ra CP .
• Hai góc nội tiếp chắn hai cung bằng nhau thì bằng nhau.
  PAC
BAP  (hai góc nội tiếp lần lượt chắn hai cung CP  BP
 ),
.
suy ra AP hay AD là phân giác của BAC

Câu 4.

Liên hệ tài liệu word toán sđt và zalo: 039.373.2038 TÀI LIỆU TOÁN HỌC
117
Website:tailieumontoan.com
Không mất tính tổng quát, giả sử số nhỏ nhất bảng là 1. Giả sử a ở ô P , 1 ở ô Q . Ta có thể
đi từ P  Q theo hang cột qua các ô liền kề
P  P1  P2  P3    Pk  Q
a  a1  a2  a3    ak  1
Ta có k  199 (đường đi dài nhất từ ô a11  a100100 ).
Mặt khác ta có
a  1  ak  a1  ak  ak1  ak1  ak2    a2  a1  1  1  1    1  k  1. Suy ra
a  1  k  1  198  a  199.
Như vậy các số nguyên dương được viết vào bảng chỉ có thể là 1, 2, 3,…, 199. Nếu mỗi số
được viết vào nhiều nhất 50 ô thì suy ra tấ cả các số được viết vào nhiều nhất
199  50  9950. Suy ra một số ô không được viết vì bảng có 10000 ô. Do đó tồn tại một số
nguyên dương k được viết vào ít nhất 51 ô.

ĐỀ SỐ 13.

Câu 1. Khi x  5 , suy ra A  5  2 6  5  2 6


 3  2 3. 2  2  3  2 3. 2  2
2 2
  3 2    3 2 
 3 2 3 2 2 3.
Nhận xét: Bài toán khá cơ bản, khi ta chỉ cần thay giá trị biến vào biểu thức đã cho, từ đó
trục căn thức và tính giá trị của biểu thức.
Nhắc lại kiến thức và phương pháp:
2 2
• Hằng đẳng thức a  b  a2  2ab  b2 ; a  b  a2  2ab  b2 .
• Trị tuyệt đối f 2  x  f  x  f  x nếu f  x  0 .

Hoặc f 2  x  f  x   f  x nếu f  x  0 .
Ý tưởng: Bài toán này thuộc dạng cho gì làm lấy, tức là với giá trị x  5 thay vào biểu thức
ta được A  5  2 6  5  2 6 , bây giờ các biểu thức nằm trong căn do đó để rút gọn A
ta sẽ nghĩ đến chuyện khử căn thức, hay nói cách khác ta muốn có
2

52 6  m 3 n 2   3m2  2n2  2 6 mn , đồng nhất hệ số của biểu thức này

 5  3m 2  2n2

  m  n  1 , tương tự cho biểu thức còn lại, ta suy ra:

 mn  1

2 2
A  3 2    3 2   3 2  3 2 2 3

Bài toán kết thúc.


Bài tập tương tự:
1. Rút gọn biểu thức P  6  2 5  6  2 5 .
2. Rút gọn biểu thức P  11  4 6  11  4 6 .

Liên hệ tài liệu word toán sđt và zalo: 039.373.2038 TÀI LIỆU TOÁN HỌC
118
Website:tailieumontoan.com
Câu 2. Điều kiện: x  0; y  0 .

2 xy  1  5 y
Hệ phương trình tương đương với  2 .

2 xy  1  5 x
5
Từ hai phương trình, ta có y  5x  y  2 x .
2
Thay vào phương trình ban đầu của hệ, ta có
x  1
1 5 
2x  2
  4 x  5x  1  0   .
2x 2 x  1
 4
+ Với x  1  y  2 .
1 1
+ Với x   y  .
4 2
 1 1
Vậy hệ phương trình có nghiệm:  x; y  1; 2 ,  ;  .
 4 2 
Nhận xét: Bài toán kết hợp cả hai phương trình của hệ, từ đó tìm mối liên hệ giữa các biến
sau đó thế ngược lại vào một trong hai phương trình của hệ để tìm nghiệm.
Ý tưởng: Quan sát thấy ở vế trái của phương trình một và phương trình hai, vai trò của các
biến là như nhau, chính vì thế ta sẽ tạo ra được một đại lượng cân bằng giữa hai phương
1 2 xy  1 1 2 xy  1
trình đó. Thật vậy, quy đồng lên ta sẽ có được 2 x   và 2 y   , đại
y y x x
lượng cân bằng ở đây chính là biểu thức 2 xy  1 vì thế ta sẽ rút các giá trị còn lại ở từng
phương trình theo 2 xy  1 để rồi cho chúng xảy ra dấu  như sau:

2 x  1  5  5
 y 2 2 xy  1  y 5
  2  y  5x  y  2 x  0 .
 1  2
2 y   5 2 xy  1  5 x
 x
Thế y  2 x  0 vào phương trình một của hệ, ta được:
 1 1
1 5 y   x 
y    2 y  1 y  2  0   2 4.
y 2 
 y  2  x  1
Bài toán kết thúc.
Bài tập tương tự:

 1

 x 4
 y
1. Giải hệ phương trình  .

 1

 y 2

 x
 1   1 
Đáp số:  x; y  2  2; 1   , 2  2; 1   .
 2   2 
 x 2
  y  4  1
y y
2. Giải hệ phương trình  2 .
 y 1
  x  2 
 x x

Liên hệ tài liệu word toán sđt và zalo: 039.373.2038 TÀI LIỆU TOÁN HỌC
119
Website:tailieumontoan.com

1 n1  n
Câu 3. Ta có 
n1 n  n1 n  n1  n 
 n1  n  n  n  2 n .
1
Từ biểu thức chứng minh trên ta có
n
 2 n1 n .  
Cho n từ 1 đến n , ta có
1
1
2 2 1  
1
2
2 3 2  
...
1
n
 2 n1 n  
Cộng các bất đẳng thức trên, ta được
1 1 1
1

2
 ... 
n
2  2 1 2   
3  2  ...  2  n1 n 
 2 n  1  2 2  2 n  2 (điều phải chứng minh).
Nhận xét: Bài toàn liến quan đến bất đẳng thức số.
Nhắc lại kiến thức và phương pháp:
1
• Chứng minh rằng với mọi số nguyên n  1 ta luôn có 2 n .
n1 n
+ Chứng minh bất đẳng thức bằng cách biến đổi vế trái để so sánh với vế phải
1 n1  n n1  n
VT   
n1 n  n1 n  n1  n  n  1 n

 n1  n ;
+ Cộng cùng một lượng vào hai vế của một bất đẳng thức thì bất đẳng thức không đổi
chiều
1 0  n 1 n  n 1  n  n 1  n  n  n  2 n ;
suy ra VT  n  1  n  2 n  VP (điều cần chứng minh).
1 1 1 1
• Chứng minh    ...   2 n  2 với mọi số nguyên n  1 .
1 2 3 n
+ Biến đổi để áp dụng từ ý trên.
1 1 1
Ta có  2 n  n1 n    2 n12 n .
n1 n 2 n n
+ Thay các giá trị số vào bất đẳng thức tổng quát.
1
- Với n  1 ta có  2 2 2 1 .
1
1
- Với n  2 ta có  2 3 2 2 .
2
- ….
1
- Với n ta có  2 n12 n .
n
Liên hệ tài liệu word toán sđt và zalo: 039.373.2038 TÀI LIỆU TOÁN HỌC
120
Website:tailieumontoan.com
1 1 1 1
Cộng vế theo    ... 
1 2 3 n
 2 2  2 1  2 3  2 2  2 n  1  2 n  2 n  1  2 1 .
+ Tính chất “bắc cầu”: a  b và b  c thì a  c .
Ta có 2  n  1 1  2  
n 1  2 n  1  2 1  2 n  2 .
 1
  1  1  ...  1  2 n  1  2 1
Ta có  1 2 3 n

2 n  1  2 1  2 n  2
1 1 1 1
thì    ...   2 n  2 (điều phải chứng minh).
1 2 3 n
Câu 4. Phương trình có hai nghiệm là x  1 và x  1 , thay vào phương trình ta được hệ

1  a  b  c  0
phương trình  .


1  a  b  c  0
Trừ hai phương trình trên, ta được: 2  2b  0  b  1 .
Cộng hai phương trình trên, ta được: a  c  0  c  a .
Phương trình trở thành x 3  ax 2  x  a  0
 x 2  x  a   x  a  0   x  a x 2  1  0 .
Theo giải thiết, phương trình có tập nghiệm là S  1; 1 , khi đó phương trình x  a  0
phải có nghiệm là 1 hoặc 1 , suy ra a  1 hoặc a  1 .
Vậy các số a; b; c cần tìm là a  1; b  1; c  1 hoặc a  1; b  1; c  1 .
Nhận xét: Bài toán sử dụng yêu cầu giả thiết của bài toán, từ các nghiệm bài cho, ta tính
được các giá trị a; b; c cần tìm.
Ý tưởng: Giả thiết bài toán cho x  1 hoặc x  1 là nghiệm của phương trình ban đầu, tuy
nhiên đây lại là phương trình bậc ba nên một trong hai nghiệm sẽ là nghiệm kép. Với hai
1  a  b  c  0
nghiệm đó, thế lại phương trình ban đầu ta có được:  . Trừ hai phương
1  a  b  c  0
trình này cho nhau ta sẽ tìm được giá trị của b là 2  2b  0  b  1 khi đó
a  c  0  a  c thế ngược lại phương trình bài cho suy ra x 3  ax 2  x  a  0
 x  a
 x  x 2  1  a x 2  1  0   x  a x 2  1  0  
 x  1

Vì phương trình chỉ nhận tập nghiệm S  1; 1 nên giá trị a phải thỏa mãn điều kiện
 a  1  c  1
a  1  
 a  1  c  1
.

Vậy giá trị cần tìm là a  1; b  1; c  1 hoặc a  1; b  1; c  1 .
Bài toán kết thúc.
Bài tập tương tự:
1. Tìm a; b; c biết rằng phương trình x 3  ax 2  bx  c  0 có tập nghiệm là S  2; 2 .
2. Tìm m; n biết rằng phương trình x 3  x 2  mx  n  0 có nghiệm bội hai là x  1 .
Câu 5.

Liên hệ tài liệu word toán sđt và zalo: 039.373.2038 TÀI LIỆU TOÁN HỌC
121
Website:tailieumontoan.com

1). Hai tam giác vuông ADB và AEC có chung góc A nên chúng đồng dạng, suy ra
AD AB
  AD. AC  AB. AE .
AE AC
Nhận xét: Bài toán chứng minh đẳng thức có hai vế là tích của hai đoạn thẳng ta dựa vào
tỷ lệ giữa các cạnh từ tam giác đồng dạng.
Nhắc lại kiến thức và phương pháp:
• Hai tam giác có hai cặp góc tương ứng bằng nhau thì đồng dạng với nhau theo trường
hợp “góc - góc” (g - g).
Xét ADB và AEC có:
 : chung;
+ BAC
  AEC
+ ADB  (  90 );
Suy ra ADB ∽AEC (g – g).
• Hai tam giác đồng dạng có các cặp cạnh tương ứng tỷ lệ.
AD AB
ADB ∽AEC    AD. AC  AB. AE (điều phải chưng minh).
AE AC
2). Xét hai tam giác ADE và ABC có
+ Góc A  chung, mà AD  AE , suy ra ADE ∽ABC .
AB AC
AD ED
Do đó  .
AB BC
Mặt khác, tam giác ABD vuông tại D , có A   AD  cos 60  1  AD
  60 , suy ra cos A
AB 2 AB
1 ED
   BC  2 ED .
2 BC
Nhận xét: Bài toán chứng minh đẳng thức dựa vào quan hệ giữa các đoạn thẳng từ các
đẳng thức suy ra từ tỷ lệ thức.
Nhắc lại kiến thức và phương pháp:
• Hai tam giác có hai góc tương ứng bằng nhau và có hai cặp cạnh kề góc tương ứng tỷ
lệ thì đồng dạng với nhau theo trường hợp “cạnh – góc - cạnh” (c – g - c).
Xét hai tam giác ADE và ABC có
 : chung;
+ A
AD AE
+  (chứng minh 1).);
AB AC
AD ED
Suy ra ADE ∽ABC (c – g – c),   .
AB BC

Liên hệ tài liệu word toán sđt và zalo: 039.373.2038 TÀI LIỆU TOÁN HỌC
122
Website:tailieumontoan.com
• Trong một tam giác vuông, cạnh góc vuông kề góc 60 (hay cạnh đối diện với góc 30 )
bằng nửa cạnh huyền (chứng minh bằng hệ thức lượng trong tam giác vuông hoặc sử
dụng tính chất đường trung tuyến ứng với cạnh huyền).
ABD vuông tại
D , có   60 , áp dụng hệ thức lượng
A ta được:
  AD  cos 60  AD  1  AD  AD  1 .
cos A
AB AB 2 AB AB 2
• Tính chất “bắc cầu”: a  b và b  c thì a  c .

 AD ED

 
 ED 1
Ta có  AB BC    2 ED  BC  BC  2.ED (điều phải chứng minh).

 AD 1 BC 2
 

 AB 2

3). Kéo dài BD cắt đường tròn ngoại tiếp tam giác ABC tại H  .
Xét hai tam giác vuông AHD và AH  D có
Cạnh AD chung;
  HAC
BHC  (góc có cạnh tương ứng vuông góc);

  CAH
HBC .

Mà HH  vuông góc với AC , nên tam giác AHH  cân tại A hay AC là đường trung trực
của HH  .
Với H  là điểm đối xứng của H qua AC .
Suy ra AC là trung trực của đoạn HH  .
Hai tam giác AH C và AHC bằng nhau
Suy ra bán kính hai đường tròn ngoại tiếp của hai tam giác AHC và bằng nhau mà
đường tròn ngoại tiếp tam giác AH C chính là đường tròn (O) .
Vậy hai đường tròn ngoại tiếp hai tam giác ABC và AHC có cùng bán kính.
Nhận xét: Chứng minh hai đường tròn có cùng bán kính ta chứng minh chúng ngoại tiếp
hai tam giác bằng nhau.
Nhắc lại kiến thức và phương pháp:
• Hai góc cùng phụ với một góc thì bằng nhau.
  DCB
+ BD  AC nên DBC   90  DBC   90 ACB
  90 DCB ;
  ACB
+ AH  BC nên HAC   90  HAC ;
  90 ACB
  HAC
Suy ra DBC .
• Trong một đường tròn, hai góc nội tiếp cùng chắn một cung thì bằng nhau.
  
H  AC  H  BC (hai góc nội tiếp cùng chắn cung H C của đường tròn ngoại tiếp ABC )
 
hay H  AC  DBC .
• Tính chất bắc cầu: a  b và b  c thì a  c .
  HAC  và H   H 
DBC  AC  DBC nên HAC  AC .

  H  AC .
Hoàn toàn tương tự ta có HAC
• Hai tam giác có hai cạnh tương ứng bằng nhau và hai cặp góc kề cạnh tương ứng bằng
nhau thì bằng nhau theo trường hợp “góc - cạnh - góc” (c – g - c).

Liên hệ tài liệu word toán sđt và zalo: 039.373.2038 TÀI LIỆU TOÁN HỌC
123
Website:tailieumontoan.com

Xét hai tam giác HAC và H  AC có:


H 
+ HAC  AC
+ AC : cạnh chung;
H 
+ HAC  AC ;
Suy ra HAC H  AC (c – g – c).
• Đường tròn ngoại tiếp hai tam giác bằng nhau thì có bán kính bằng nhau.
HAC H  AC nên đường tròn ngoại tiếp HAC và đường tròn ngoại tiếp H ' AC có bán
kính bằng nhau.
Mà đường tròn ngoại tiếp H  AC chính là đường tròn (O) ngoại tiếp ABC .
Vậy hai đường tròn ngoại tiếp hai tam giác ABC và AHC có cùng bán kính (điều phải
chứng minh).

Câu IV.

Gọi H là hình chiếu vuông góc của C lên trụ đỡ AB . Đặt BH  x .


Tam giác AHC vuông tại H , ta được
2 2
HC 2  AC 2   AB  x  258 2  120  x .
Tam giác BHC vuông tại H , ta được
HC 2  BC 2  HB2  218 2  x 2 .
2
Ta có phương trình 258 2  120  x  218 2  x 2
58
 258 2  120 2  218 2  240 x  x  .
3
 BH 58 29
Ta lại có sin BCH   .
BC 3.218 327
29
Vậy góc nghiêng BC so với mặt nằm ngang là   sin1  50 516 .
327
Nhận xét: Bài toán thực tế đưa vào hình học phẳng để giải quyết bằng phương pháp tính
toán, các công thức hình học đơn giản.
Nhắc lại kiến thức và phương pháp:
• Định lý Py-ta-go: “Trong tam giác vuông, bình phương cạnh huyền bằng tổng bình
phương hai cạnh góc vuông”.
+ Tam giác AHC vuông tại H nên ta có AC 2  AH 2  HC 2
2
 HC 2  AC 2  AH 2  HC 2  AC 2   AB  x

Liên hệ tài liệu word toán sđt và zalo: 039.373.2038 TÀI LIỆU TOÁN HỌC
124
Website:tailieumontoan.com
2
 HC 2  258 2  120  x .
+ Tam giác BHC vuông tại H nên ta có BC 2  BH 2  HC 2
 HC 2  BC 2  HB2  HC 2  218 2  x 2 .
• Kết hợp các dữ kiện đã có, biến đổi theo cùng một lượng suy ra phương trình theo
một ẩn và giải phương trình đó.
 2
HC  258  120  x
2 2
2
Ta có  nên ta được phương trình 258 2  120  x  218 2  x 2 giải
HC  218  x
2 2 2

58
phương trình này ta được x  .
3
• Hệ thức lượng trong tam giác vuông.
 BH 58 29
Tam giác BHC vuông tại H nên ta có sin BCH   .
BC 3.218 327
• Sử dụng máy tính hoặc bảng số để tìm ra góc khi biết giá trị lượng giác của góc đó.
 29   sin1 29  50 516 .
Ta có sin BCH nên BCH
327 327
Vậy góc nghiêng BC so với mặt nằm ngang là 50 516 .

ĐỀ SỐ 14.

Câu 1. Cho biểu thức với a; b  0 .


 a 2  ab  b2 .a  b  a 2  ab  b2 .a  b
2 2 2
a 2  b2  ab b  a
. 2 2
P  ab ab  a3 b3  a3 b3
1). a 2   2  a  a  b b  a  a  b b 
  a    b  b  .  1  .  1 .  1  .  1
 b 2 
b   a 2 
a  b b  a a  
 b b  a a 

a  ab  b2 .a  b a  ab  b2 .a  b
2 2 2 2

a3 b3 a3 b3 1
   .
a b a  b a2  ab  b2  ab
2
a  b. 2  2 
b a 
a2 b2
2). Với a; b  0 , áp dụng bất đẳng thức Côsi ta có:
4 a  b  ab  2 4 ab  ab  5 ab .
1 1
Suy ra 1  5 ab   5  P   25 .
ab ab
Dấu “=” xảy ra khi và chỉ khi 4a  b .
Vậy Pmin  25 tại b  4 a .
Nhận xét: Đây là một bài toán dễ, đơn giản chỉ là việc khai triển rút gọn biểu thức P bằng
một vài phép biến đổi tương đương, ý khó là ý sau khi tìm giá trị nhỏ nhất của P với điều
kiện bài cho, nhưng với giải pháp chọn điểm rơi và bất đẳng thức Cosi bài toán hoàn toàn
được giải quyết.
Nhắc lại kiến thức và phương pháp:
• Hằng đẳng thức: a 3  b3  a  ba 2  ab  b2 
• Bất đẳng thức Cosi cho hai số thực dương ma , nb :

Liên hệ tài liệu word toán sđt và zalo: 039.373.2038 TÀI LIỆU TOÁN HỌC
125
Website:tailieumontoan.com

m.a  n.b  2 ma.nb ( m; n; a; b  0 ).


Dấu “=” xảy ra khi m.a  n.b .
Ý tưởng: Với ý rút gọn biểu thức P , ta sẽ đi làm hai công việc, một là rút gọn tử số, sau đó
là mẫu số. Với tử số của bài toán, ta có:
a2  ab  b2 a  b a3  b3 a  b .
2 2
a b  
   1 1  1   
 b a  a b 
 a3 b3 a3 b3
Bây giờ, ta mong muốn mẫu số sẽ xuất hiện ít nhất hai trong ba biểu thức mà ta đã rút gọn
trên tử số, biến đổi mẫu số ta có:
a 2 b2  a b  a  a  b  b 
  1
        1   
b2 a 2  b a  b  b  a  a 

b  a  b a  b
3 3
a  a  b  b  b  a  a

      a  b 2  2   .
b  b  a  a   b a  a2 b2
1
Vì thế, suy ra P  . Với ý sau, bài cho giả thiết 4 a  b  ab  1 để tìm giá trị nhỏ nhất của
ab
P hay nói cách khác chính là đi tìm max của T  ab . Để xuất hiện tích ab ở giả thiết, ta đã
thấy xuất hiện ab , bây giờ chỉ còn tổng 4a  b và để đánh giá nó về tích, ta chỉ còn cách là
áp dụng bất đẳng thức Cosi cho hai số thực dương 4 a; b khi đó ta có:
1
4 a  b  ab  2 4 ab  ab  5 ab  1  5 ab  ab   P  25 .
25
Do đó, P đạt giá trị nhỏ nhất là 25 tại 4a  b .
Bài toán kết thúc.
Bài tập tương tự:
 1
 : 1  a 1
1. Tìm giá trị nhỏ nhất của P     với a thỏa mãn điều kiện
 a  a a  1 a  2 a  1
a4 .
2x x  1 3  11x
2. Cho biểu thức P    . Tìm x  3 thỏa mãn điều kiện P  2 .
x  3 3  x x2  9
x  my  2  4 m (1)
Câu 2. Cho hệ phương trình  .
mx  y  3m  1 (2)

 8

 x

 x  2 y   6  5 .
1). Với m  2 , ta có hệ phương trình  



 2 x  y  7 
 19
 y


 5
2). Từ (2), ta có y  3m  1  mx thay vào (1).
x  m 3m  1  mx  2  4 m  m2  1 x  3m2  3m  2 (*).
Vì m2  1  0 với mọi m nên (*) luôn có nghiệm duy nhất.
Suy ra hệ đã cho luôn có nghiệm duy nhất với mọi m .
3m 2  3m  2
Từ phương trình (*), ta có x0 
m2  1
3m 2  3m  2 4 m 2  m  1
 y0  3m  1  m.  .
m2  1 m2  1
Xét đẳng thức đã cho, ta có VT  x0 2  y0 2  5 x0  y0   10

Liên hệ tài liệu word toán sđt và zalo: 039.373.2038 TÀI LIỆU TOÁN HỌC
126
Website:tailieumontoan.com
2
  x0  y0   5 x0  y0   10  2 x0 y0
 2 
  x0  y0   5 x0  y0   6  2 x0 y0  4
 
  x0  y0  2 x0  y0  3  2 x0 y0  4
3m 2  3m  2 4 m2  m  1
Thay các giá trị x0  ; y 0
 ta được
m2  1 m2  1
 3m 2  3m  2 4 m 2  m  1  3m  3m  2  4 m  m  1  2
2 2
VT     3  
 m2  1 m2  1  m2  1 m2  1 
3m 2  3m  2 4 m 2  m  1
2. . 4
m2  1 m2  1
2
4 m2  2 m 5m2  2 m  1 6 m2  6 m  4 4 m2  m  1 4 m  1
2

 .  . 
m2  1 m2  1 m2  1 m2  1 m2  1
2

 4 3 2 3 2 
 20 m  8 m  4 m  10 m  4 m  2 m 
  2
 24 m  6 m  6 m  24 m  6 m  6 m  16 m  4 m  4 : m2  1  0 .
 4 3 2 3 2 2
 
 
4 m  2 m  1
4 2

 
Nhận xét: Đây là bài toán cơ bản, chỉ là việc đi giải hệ phương trình bậc nhất hai ẩn với
những kỹ năng thế cơ bản, chỉ có thể gặp khó khăn ở việc chứng minh đẳng thức, nhưng
chỉ cần biến đổi một chút sẽ suy ra điều phải chứng minh.
Nhắc lại kiến thức và phương pháp:

ax  by  c
• Cách giải phương trình bậc nhất hai ẩn tổng quát  .

mx  ny  p

c  ax
Từ phương trình một của hệ, ta có: y  , thế xuống phương trình thứ hai ta được:
b
nc  ax
mx   p  mb  na x  bp  nc .
b
Ý tưởng: Ý đầu tiên của bài toán, rất cơ bản và không có gì đáng nói, cái khó là ở ý sau.
Tuy nhiên với việc đưa ra cách giải tổng quát của hệ phương trình bậc nhất hai ẩn ta có
thể dễ dàng biểu diễn được x0 ; y0 theo m đồng thời thay vào biểu thức
x0 2  y0 2  5 x0  y0   10  0 ta sẽ thấy m bị triệt tiêu hết. Trước hết, đi tìm x0 ; y0 ta có:
• Từ phương trình hai của hệ, có y  3m  1  mx .
• Thế vào phương trình một trong hệ, ta được:
x  m 3m  1  mx  2  4 m  m2  1 x  3m2  3m  2 .
• Để hệ phương trình có nghiệm  m2  1  0 luôn đúng với m .
3m 2  3m  2 4 m2  m  1
• Khi đó, nghiệm của hệ là x0  ; y 0
 .
m2  1 m2  1
Tuy nhiên nếu cứ để x0 ; y0 thay vào biểu thức x0 2  y0 2  5 x0  y0   10 thì ta sẽ gặp khó
khăn trong việc khai triển bậc bốn, vì thế ta sẽ làm đơn giản hơn một chút đó là:
2
x0 2  y0 2  5 x0  y0   10   x0  y0   5 x0  y0   6  10  2 x0 y0
  x0  y0  2 x0  y0  3  2 x0 y0  10 (*).

Liên hệ tài liệu word toán sđt và zalo: 039.373.2038 TÀI LIỆU TOÁN HỌC
127
Website:tailieumontoan.com
Bây giờ việc thay x0 ; y0 vào (*) sẽ giúp ta biến đổi nhẹ nhàng hơn rất nhiều và ta sẽ chứng
minh được  x0  y0  2 x0  y0  3  2 x0 y0  10  0 .
Bài toán kết thúc.
Bài tập tương tự:
1. Xác định m để hệ phương trình có nghiệm  x; y và thỏa mãn x 2  y 2 nhỏ nhất, với
2 x  y  5
 .
2 y  x  10 m  5
mx  y  2 m
Cho x0 ; y0 thỏa mãn hệ phương trình  . Tìm biểu thức liên hệ giữa x0 ; y0 độc
x  my  m  1
lập với m .
Câu 3. Ta có
a  x  a  b  y  b  0  a  b x 2  2 a 2  b2  x  a 3  b3  0 .
2 2

Vì a; b  0 suy ra a  b  0 , nên phương trình (*) có nghiệm duy nhất


2
'  0  a 2  b2   a  ba 3  b3   0
 a 4  2 a 2 b2  b4  a 4  ab3  a 3 b  b4  0
 2 a 2 b2  ab3  a 3 b  0
2
 2 ab a  b  0  a  b  0  a  b  a  b .
Nhận xét: Đây chỉ đơn thuần là một bài toán xét nghiệm của phương trình bậc hai tổng
quát, dựa vào tính chất điều kiện nghiệm của tam thức bậc hai suy ra điều phải chứng
minh.
Nhắc lại kiến thức và phương pháp:
• Phương trình bậc hai tổng quát mx 2  nx  p  0 ( x   ) (*).
p
+ TH1: Nếu m  0 thì (*)  nx  p  0  x   ( n  0 ).
n
+ TH2: Nếu m  0 thì (*)  n2  4 mp .
- Để (*) có nghiệm duy nhất  (*)  0  n2  4 mp .
- Để (*) có hai nghiệm phân biệt  (*)  0  n2  4 mp .
- Để (*) vô nghiệm  (*)  0  n2  4 mp .
• Giá trị tuyệt đối a  b    a  b .
Ý tưởng: Bài cho một phương trình bậc hai ẩn x với hai tham số a; b . Khai thác giả thiết,
2 2
giả thiết cho a; b  0 đồng thời a  x  a  b  x  b  0 ( i ) là một phương trình bậc hai có
nghiệm duy nhất. Vì thế ta cần khai triển ( i ) để đưa được về phương trình bậc hai tổng
quát, ta có:
(i )  a  x  a  b  x  b  0  a  b x 2  2 a 2  b2  x  a 3  b3  0
2 2

2
Với a; b  0 suy ra a  b  0 do đó  ( i ) a 2  b2   a  ba 3  b3  .
Nên để phương trình ( i ) có nghiệm duy nhất khi và chỉ khi:
2
( i )  0  a 2  b2   a  ba 3  b3   0
 a 4  2 a 2 b2  b4  a 4  ab3  ba 3  b4

Liên hệ tài liệu word toán sđt và zalo: 039.373.2038 TÀI LIỆU TOÁN HỌC
128
Website:tailieumontoan.com
2
 ab a  b  0  a  b  a  b , suy ra điều phải chứng minh.
Bài toán kết thúc.
Bài tập tương tự:
1. Cho a; b là các số thực khác 0 và a  b . Tìm mối liên hệ giữa để phương trình
2 2
a  x  a  b  x  b  0 có nghiệm duy nhất.
2. Xác định hai số thực a; b sao cho phương trình x 2  ax  b  0 có nghiệm cũng là
a; b .

Câu 4.


1). Ta có B   120 0  B
IC1  BIC    120 0  60 0  180 0 .
IC1  BAC
1 1

Mà hai góc này đối nhau. Nên tứ giác AB1 IC1 nội tiếp (điều phải chứng minh).
Nhận xét: Chứng minh một tứ giác là tứ giác nội tiếp, ta chứng minh tổng hai góc đối diện
của tứ giác bằng 180 .
Nhắc lại kiến thức và phương pháp:
• Hai góc đối đỉnh thì bằng nhau.

B  là hai góc đối đỉnh nên B
IC1 và BIC  .
IC1  BIC
1 1

• Tia phân giác của một góc chia góc đó thành hai góc bằng nhau và bằng một nửa góc
đó.
+ BI là phân giác của ABC   1 ABC
 nên IBC ;
2
 nên ICB 1
  ACB ;
+ CI là phân giác của ACB
2
  1 ABC
  ICB
suy ra IBC
2
  ACB
 .
 
• Tổng ba góc của một tam giác bằng 180 .
+ Tam giác ABC có ABC  ACB   180
  BAC
  ACB
 ABC   180  BAC
  180  120  60 .
  ICB
+ Tam giác IBC có IBC   BIC
  180

  180  1 ABC
  180  IBC
 BIC   ICB
2
  ACB

 

Liên hệ tài liệu word toán sđt và zalo: 039.373.2038 TÀI LIỆU TOÁN HỌC
129
Website:tailieumontoan.com

  180  1 .120  180  60  120 .


 BIC
2
• Tứ giác có tổng hai góc đối diện bằng 180 là tứ giác nội tiếp.
Tứ giác AB1 IC1 có B  AC1 và B  IC1 là hai góc đối diện, thỏa mãn
1 1


B AC  B   BIC
IC  BAC   60  120  180 do đó AB IC là tứ giác nội tiếp.
1 1 1 1 1 1

  BKC
2). Vì tứ giác BC1 IK nội tiếp nên BIC   60 0 (góc nội tiếp cùng chắn BC ) và
1 1 1

BIK 
  BC K (góc nội tiếp cùng chắn BK ).
1

Xét tam giác ABC , ta có


  180 0  BAC
KCB   180 0  60 0  ABC
  ABC   120 0  ABC
.
1

Xét tam giác BC1K , ta có


  BC
BIK    ABC
K  180 0  BKC   180 0  60 0  ABC
  120 0  ABC
.
1 1

  BIK
Suy ra KCB  , suy ra tứ giác ACKC nội tiếp (điều phải chứng minh).
1 1

Nhận xét: Chứng minh một tứ giác là tứ giác nội tiếp, ta chứng minh góc trong tại một
đỉnh bằng góc ngoài tại đỉnh đối diện
Nhắc lại kiến thức và phương pháp:
• Chứng minh các tứ giác nội tiếp.
Hoàn toàn tương tự phần 1) đã chứng minh ta có tứ giác BC1 IK và tứ giác CB1 IK là các
tứ giác nội tiếp.
• Các góc nội tiếp cùng chắn một cung của một đường tròn thì bằng nhau.
 và BKC
+ BIC  là hai góc nội tiếp cùng chắn cung BC
 của đường tròn ngoại tiếp tứ
1 1 1

  BKC
giác BC1 IK nên BIC   60 .
1 1

 và BC
+ BIK   của đường tròn ngoại tiếp tứ
K là hai góc nội tiếp cùng chắn cung BK
1

  BC
giác BC1 IK nên BIK  K.
1

• Tổng ba góc của một tam giác bằng 180 .


  CAB
+ Trong ABC có ABC   BCA
  180
  CAB
 ABC  B  
CK  180  B   CAB
CK  180  ABC 
1 1

B   60  120  ABC


CK  180  ABC ;
1

 C
+ Trong BKC1 có BKC  
BK  BC K  180
1 1 1

  ABC
 BKC   BC  
K  180  BC   ABC
K  180  BKC 
1 1 1 1


 BC   120  ABC
K  180  60  ABC 
1

  120  ABC
 BIK ;
  BIK
suy ra KCB .
1

• Tứ giác có góc trong bằng góc ngoài tại đỉnh đối diện thì tứ giác đó là tứ giác nội tiếp.
 là góc ngoài tại đỉnh I đối diện
 là góc trong tại đỉnh C và BIK
Tứ giác CKIB1 có KCB1

  BIK
với đỉnh C thỏa mãn KCB  nên CKIB là tứ giác nội tiếp.
1 1

Liên hệ tài liệu word toán sđt và zalo: 039.373.2038 TÀI LIỆU TOÁN HỌC
130
Website:tailieumontoan.com
  BAC
3). Vì BIC   KCC
  60 0 , suy ra tứ giác ACKC nội tiếp, nên AKC  (cùng chắn cung
1 1 1 1

KC1 ).
  ACC
Và AKC  (cùng chắn cung AC ).
1 1 1

 
Mà ACC  KCC (cùng chắn cung KC ) (giả thiết).
1 1 1

  AKC
Suy ra KAC  , suy ra tam giác C AK cân tại C  C A  C K (1).
1 1 1 1 1 1

Chứng minh tương tự: B1 A  B1K (2).


Từ (1) và (2), suy ra B1C1 là đường trung trực của AK nên AK  B1C1 (điều phải chứng
minh).
Nhận xét: Chứng minh hai đường thẳng vuông góc ta chứng minh cho một đường thẳng
là đường trung trực của đường thẳng còn lại
Nhắc lại kiến thức và phương pháp:
• Tứ giác có góc trong bằng góc ngoài tại đỉnh đối diện thì tứ giác đó là tứ giác nội tiếp.
 là góc trong tại đỉnh A và BIC
Tứ giác ACKC1 có BAC  là góc ngoài tại đỉnh I đối
1

  BAC
diện với đỉnh A thỏa mãn BIC   60 0 nên ACKC là tứ giác nội tiếp.
1 1

• Các góc nội tiếp cùng chắn một cung của một đường tròn thì bằng nhau.
  ACC
+ AKC  (hai góc nội tiếp cùng chắn cung AC
 của đường tròn ngoại tiếp tứ
1 1 1

giác ACKC1 ).
  KCC
+ KAC  (hai góc nội tiếp cùng chắn cung KC
 của đường tròn ngoại tiếp tứ
1 1 1

giác ACKC1 ).
• Phân giác của một góc chia góc đó thành hai góc bằng nhau.
 nên ta có ACC
CC1 là phân giác của ACB   KCC
 , suy ra KAC
  AKC
.
1 1 1 1

• Tam giác có hai góc kề một cạnh bằng nhau là tam giác cân.
  AKC
Tam giác C1 AK có KAC  , suy ra C AK cân tại C
1 1 1 1

• Tam giác cân có hai cạnh bên bằng nhau.


Tam giác C1 AK cân tại C1 nên C1 A  C1K .
Chứng minh tương tự: B1 A  B1K .
• Từ một điểm cách đều hai đầu mút của một đoạn thẳng thì nằm trên đường trung trực
của đoạn thẳng đó.
+ C1 A  C1K nên C1 thuộc đường trung trực của AK ;
+ B1 A  B1K nên B1 thuộc đường trung trực của AK ;
suy ra C1 và B1 cùng thuộc đường trung trực của AK hay B1C1 là trung trực của AK
hay B1C1  AK .

Câu 5. Áp dụng bất đẳng thức Côsi cho hai số a; b không âm, ta có:
3  1 1 1 1 3 1
a 2  b   a 2    b   2 a 2 .  b   a 2  b   a  b  .
4  4 2 4 2 4 2
3 1
Tương tự ta có b2  a   b  a  .
4 2

Liên hệ tài liệu word toán sđt và zalo: 039.373.2038 TÀI LIỆU TOÁN HỌC
131
Website:tailieumontoan.com
2 2
 3  3  1  
1  1 
Do đó a 2  b  b2  a    a  b    a    b   .
 4  4  2  4  4 
2
Áp dụng bđt phụ  x  y  4 xy; x , y , ta có:
2
         
a  1   b  1   4 a  1 b  1   2 a  1 2b  1  .
 
4   
4   
4  
4   
2  2 

 3  3  1  1
Suy ra a 2  b  b2  a    2 a  2b   .
 4  4   2  2
 1

 a2 

 4

 1 1
Dấu “=” xảy ra  b 
2
 a  b  (thỏa mãn).

 4 2

 1 1

 a b


 4 4
1
Vậy a  b  .
2
Nhận xét: Thực chất bài toán là đi chứng minh bất đẳng thức, bởi nhẽ dấu đẳng thức xảy
ra tại chính điểm rơi hay tại nghiệm cần tìm của phương trình bài toán đã cho.
Nhắc lại kiến thức và phương pháp:
• Bất đẳng thức Cosi cho hai số thực dương: a  b  2 ab .
• Hệ quả của bất đẳng thức Cosi cho hai số thực không âm:
2
x  y  2 xy   x  y  4 xy , x; y  0 .
Ý tưởng: Bài toán cho một phương trình đối xứng mặt khác lại chứa hai biến nên nhiều
khả năng sẽ dùng đến bất đẳng thức. Trước hết do vai trò của hai biến a; b như nhau nên
dễ thấy a  b là nghiệm của phương trình. Thế ngược lại phương trình đã cho, ta có:
2 2
 2   
a  a  3   2 a  1  với a; b  0
 
4   2 
1 1
 a b .
2 2
1
Với điểm rơi này, sẽ thấy a 2  b2  , vậy nên áp dụng bất đẳng thức Cosi chúng ta có:
4
3  1 1 1 1 1
a 2  b   a 2    b   2 a 2 .  b   a  b 
4  4 2 4 2 2
3  1 1 1 1 1
b2  a   b2    a   2 b2 .  a   a  b  .
4  
4 2 4 2 2
Do đó, từ phương trình ban đầu, ta được:
2 2
         
2 a  1 2b  1  a  b  1   4 a  1 b  1  a  b  1  .
 
2   
2   2   
4   
4   2 
1 1 1 1 1 1
Ta thấy   nên suy ra a  b   a   b  do vậy:
2 4 4 2 4 4
2
 1  1  1 1
4 a  b    a   b    a  b  0  a  b  0 .
2

 4   4   4 4

Liên hệ tài liệu word toán sđt và zalo: 039.373.2038 TÀI LIỆU TOÁN HỌC
132
Website:tailieumontoan.com
Và khi đã chứng minh được a  b  0 việc còn lại chỉ là thế ngược lại phương trình đã cho
để tìm nghiệm của bài toán.
Bài toán kết thúc:
Bài tập tương tự:
1. Tìm hai số thực dương a; b thỏa mãn điều kiện
 1 1 
 a  b  
 a  3b
   2 .
b  3a 
2. Cho a; b là hai số thực dương, giải phương trình sau:
1 1 2
 
1  2 a 1  2b 2  ab

ĐỀ SỐ 15.
Câu 1.
2
3 3  a 1 
1). Ta có S  2  2 2   3 2 2  3  a 1 :  2  a 1 
2

 22 2  2 2    a 1 . 
 a 1
 4  2  2  4 (với a  0; a  1 ).
Nhận xét: Vẫn là các bài toán nằm trong mô tuýt khai triển rút gọn biểu thức, bài toán này
cũng vậy điểm nhấn chính là phát hiện ra các hằng đẳng thức bậc hai, bậc ba nằm trong
căn thức.
Nhắc lại kiến thức và phương pháp:
2 3
• Hằng đẳng thức bậc hai, bậc ba  x  y  x 2  2 xy  y 2 và  x  y  x 3  3x 2 y  3xy 2  y 3 .
• Khai căn bậc hai, bậc ba 3
x 3  3 x 2 y  3 xy 2  y 3  x  y
+ x 2  2 xy  y 2  x  y  x  y với x  y  0 .
+ x 2  2 xy  y 2  x  y  x  y với x  y  0 .
Ý tưởng: Bài toán hơi đặc biệt ở chỗ, vừa xuất hiện hằng số, vừa xuất hiện biểu thức chứa
biến vì thế ta sẽ đi rút gọn từng biểu thức một.
• Với S1  6  4 2 . 3 20  14 2 , có xuất hiện căn bậc hai, căn bậc ba vì thế ta sẽ nghĩ đến
chuyện khai căn thức. Xét với căn bậc hai, ta sẽ đưa nó về dạng
2

64 2  ab 2   a 2  2 2 ab  2b2
a 2  2 b 2  6
 
a  2 2


4 2  2 2 ab 



 64 2  2  2   2 2

 b  1
3
Tư duy tương tự ta sẽ có được 3
20  14 2  3 2  2   2 2 .

 
Do đó, suy ra S1  2  2 2  2  4  2  2 . 

Liên hệ tài liệu word toán sđt và zalo: 039.373.2038 TÀI LIỆU TOÁN HỌC
133
Website:tailieumontoan.com
  a 1
• Với S2  3 a  3 a  3a  1 :   1 , biểu thức này cũng có chứa căn bậc ba nhưng
 2( a  1) 
nó đã khó hơn vì chứa biến, vẫn kiểu tư duy ở S1 , dễ thấy
3
3
a  3 a  3a  1  3  
a  1  a  1 đồng thời ở đẳng thức sau

a 1 a2 a 1 a 1
1   suy ra S2  2 .
2  a 1  2  a 1  2

Do đó kết luận được rằng S  S1  S2  4 .


Bài toán kết thúc.

2
2). Ta có P  x  y  x 2  xy  y 2  x  y   x  y  3 xy .
x y
Thay   1  2 x  2 y  1  3 xy vào biểu thức P ta được
1 x 1 y
2 2
P xy  x  y  2  x  y  1  x  y   x  y  1  x  y  x  y  1  1 (vì x  y  1 ).
Giải thích x  y  1 .
x y x y
Từ giả thiết ta có ; là các số dương mà   1 , nên ta có
1 x 1 y 1 x 1 y
 x
  2x  1
  1

 1   0  
 x

 1  x  1  x  2
 
 
  x  y 1.

 y 
 2 y  1 
 1
  1   0  y

1  y 
 1 y 

 2
 
Vậy P  1 .
Nhận xét: Bài toán sử dụng giả thiết, sau đó thể biến vào biểu thức cần tính giá trị và dùng
điều kiện trị tuyệt đối suy ra giá trị của biểu thức.
Nhắc lại kiến thức và phương pháp:
2
• Hằng đẳng thức: a  b  a 2  2 ab  b2
• Trị tuyệt đối f  x   f  x nếu f  x  0 .
Ý tưởng: Bài toán với yêu cầu là tính giá trị của biểu thức với giả thiết cho sẵn, thì điều
đầu tiên cần làm đó chính là khai thác giả thiết. Thường thì họ sẽ cho một giả thiết phức
tạp và bắt ta đơn giản hóa nó, bài toán này cũng vậy, giả thiết chứa hai phân thức do đó ta
sẽ nghĩ đến chuyện quy đồng như sau có giả thiết 
x 1  y  y 1  x
 1  x  y  2 xy  1  x  y  xy
1  x1  y
 2 x  2 y  1  3 xy . Xét đến biểu thức P  x  y  x 2  xy  y 2 , việc xuất hiện tổng x  y
độc lập sẽ hướng ta tư duy đến việc trong căn thức cũng xuất hiện tổng đó, vậy ta có biến
đổi:
2 2
P xy  x  y  3 xy  x  y   x  y  2  x  y  1
2
 xy  x  y  1  x  y  x  y 1

Liên hệ tài liệu word toán sđt và zalo: 039.373.2038 TÀI LIỆU TOÁN HỌC
134
Website:tailieumontoan.com
Đề bài yêu cầu tính giá trị của P nên nhiều khả năng, P sẽ là một hằng số thì khi đó ta
cần có x  y  1  1  x  y hay nếu chứng minh được bất đẳng thức x  y  1 thì P  1 và ta
x y
sẽ chứng minh điều này dựa vào giả thiết, đó từ giả thiết ta có ; là các số dương
1 x 1 y
 x
  2x  1
  1

 1 
 0 

 x
x y  1  x  1  x  2  x  y 1.
mà   1 nên ta có  
 

1 x 1 y 
 y  2 y  1  1
 1 
 0 
 y

 1  y 
 1  y 

 2
 
Bài toán kết thúc.

Câu 2.
1). Đỉnh cổng là đỉnh của Parabol y  ax 2 ( a  0 ) trùng với gốc tọa độ O 0; 0 .

Gọi điểm biểu thị hai chân cổng trên đồ thị hàm số là A , B ta có A , B đối xứng qua trục

tung và cách nhau 4 đơn vị. H là giao điểm của AB với Oy ( A; B; H nằm phía dưới trục
hoành).
2
2 4
Ta có OH  OA  AH  2 2
 
2 5     16  4 , suy ra
 2 
H 0;  4 từ đây suy ra

A 2;  4 và B2;  4 .
2
Parabol y  ax 2 ( a  0 ) đi qua điểm A nên ta có 4  a 2  4  4a  a  1 (thỏa
mãn), điều phải chứng minh.
Nhận xét: Bài toán này, thực chất là sử dụng các tính chất của hàm số, đặc biệt là hàm số
bậc hai dạng y  ax 2 với a  0 , ngoài ra còn sử dụng các công thức liên quan đến khoảng
cách, độ dài,…
Nhắc lại kiến thức và phương pháp:
• Đồ thị hàm số y  ax 2 với a  0 là một Parabol nhận gốc tọa độ làm đỉnh và trục trung
làm trục đối xứng, phần lõm hướng xuống dưới,
ta được hình vẽ

Liên hệ tài liệu word toán sđt và zalo: 039.373.2038 TÀI LIỆU TOÁN HỌC
135
Website:tailieumontoan.com

Các điểm khác gốc tọa độ nằm trên parabol này đều ở phía dưới trục hoành.
• Định lý Py-ta-go: “Trong một tam giác vuông có tổng bình phương hai cạnh góc
vuông bằng bình phương cạnh huyền).
Tam giác OAH vuông tại H , áp dụng định lý Py-ta-go ta có
AH 2  HO 2  OA 2  OH 2  OA 2  AH 2  OH  OA 2  AH 2
2
2 4
 OH   
2 5     16  4 .
 2 

• Một điểm nằm trên trục tung thì khoảng cách của điểm đó với gốc tọa độ bằng giá trị
tuyệt đối tung điểm của điểm đó.
Điểm H nằm trên trục tung và nằm phía dưới trục hoành nên điểm H có tung độ âm
và hoành độ bằng 0.
Ta có OH  yH  yH do H nằm dưới trục hoành nên yH  0
 y H  4  y H  4  H  0 ;  4 
suy ra A 2 ;  4 và B2 ; 4 .
• Một điểm thuộc đồ thị hàm số thì khi thay giá trị hoành độ và tung độ của điểm đó
vào hàm số ta được một phương trình đúng.
Điểm A 2 ;  4 thuộc đồ thị hàm số y  ax 2 nên ta có thay x  2 và y  4 vào hàm số
ta được phương trình đúng 4  a.2 2  4 a  4  a  1 (điều phải chứng minh).

2). Gọi giao điểm cảu đường biểu diễn chiều cao của xe tải với Parabol  P tại C và D (
C ; D nằm về hai phía của trục tung).
Ta có CD song song với AB và Ox .
K là giao điểm của CD và Oy .
Khoảng cách giữa DC và AB là 2,5 đơn vị nên suy ra OK  4  2,5  1,5 khi đó ta có
3
phương trình đường thẳng CD là y  .
2

Liên hệ tài liệu word toán sđt và zalo: 039.373.2038 TÀI LIỆU TOÁN HỌC
136
Website:tailieumontoan.com

x   6
3  c 2 .
Phương trình hoành độ giao điểm của CD và Parabol  P là x 2  
2 
 6
xD  2

Khi đó CD  6 . Tại độ cao 2,5 m thì chiều rộng của cổng là 6m lớn hơn 2,4m là chiều
rộng của xe tải nên xe tải có thể đi qua cổng.
Nhận xét: Vì chiều cao của xe tải thấp hơn chiều cao của cổng, để biết xe tải có thể đi qua
cổng hay không ta phải xét xem, tại độ cao của cổng bằng với chiều cao của xe thì chiều
rộng của cổng bằng bao nhiêu, nếu chiều rộng đó lớn hơn chiều rộng của xe tải thì xe tải
sẽ đi qua, nếu nhỏ hơn chiều rộng xe tải thì không đi qua, nếu bằng nhau thì vì bài toán
được xử lý trên lý thuyết nên ta chấp nhận là có đi qua.
Nhắc lại kiến thức và phương pháp:
• Khoảng cách giữa hai điểm nằm trên một đường vuông góc với hai đường thẳng song
song bằng khoảng cách giữa hai đường thẳng song song đó.
Ta có khoảng cách giữa CD và AB chính là chiều cao của xe tải nên bằng 2,5 khi đó
HK  2,5  OH  OK  2,5  4  OK  2,5
 OK  4  2,5  1,5 suy ra K 0 ;  1,5 .
• Đường thẳng song song với trục hoành và đi qua một điểm nào đó thì chính tập hợp
các điểm có tung độ là tung độ của điểm đó.
Ta có CD  Ox và CD đi qua điểm K 0 ;  1,5 nên ta có hàm số biểu thị đường thẳng
3
CD là y  1,5   .
2
• Phương trình hoành độ giao điểm/ Hệ phương trình tọa độ giao điểm.
Phương trình hoành độ giao điểm của đường thẳng CD với parabol là

 3 
 6
 x 

   
 xC  
3 3 C
2 2 .
x 2    x 2   
 

2 2  3 
 6

 xD  
 xD 

 2 
 2
• Khoảng cách giữa hai điểm cùng thuộc một đường thẳng song song với trục hoành
bằng giá trị tuyệt đối hiệu hai hoành độ của hai điểm.
6 6
Ta có C và D cùng thuộc CD  Ox nên CD  xC  xD   
2 2

 6  6.

• Đối với bài toán trên, nếu tại độ cao của cổng bằng với chiều cao của xe thì chiều rộng
của cổng bằng bao nhiêu, nếu chiều rộng đó lớn hơn chiều rộng của xe tải thì xe tải sẽ
đi qua, nếu nhỏ hơn chiều rộng xe tải thì không đi qua, nếu bằng nhau thì vì bài toán
được xử lý trên lý thuyết nên ta chấp nhận là có đi qua.
Ta có 6  2,4 tức là chiều rộng của cổng tại độ cao 2,5 lớn hơn chiều rộng của xe tải
nên xe tải đi qua được cổng.
2
Câu 3. Từ đẳng thức đã cho ta có a 2  2 b  1 a  b  1  0 là phương trình bậc hai ẩn a, ta
2 2
có   b  1  b  1  4b .

Liên hệ tài liệu word toán sđt và zalo: 039.373.2038 TÀI LIỆU TOÁN HỌC
137
Website:tailieumontoan.com

Vì phương trình trên có nghiệm nguyên nên điền kiện cần ta có   là một số chính
phương. Khi đó ta có b là một số chính phương.
- Với b  0 , ta có a  1 . Ta thấy 0 và 1 là hai số chính phương liên tiếp (đúng với dpcm).
 2
 a  b  1  4b 
- Với b  0 , ta có 
 b 1 .
2
 a  b  1  4b 
  b  1
2 2
Ta thấy b và  b 1  là hai số chính phương liên tiếp;  b 1 và b là hai số chính
phương liên tiếp (điều phải chứng minh).
Vậy a và b là các số chính phương liên tiếp.
Nhận xét: Bài toán này, thực chất là sử dụng các tính chất của hàm số, đặc biệt là hàm số
bậc hai dạng y  ax 2 với a  0 , ngoài ra còn sử dụng các công thức liên quan đến khoảng
cách, độ dài,…
Nhắc lại kiến thức và phương pháp:
• Biến đổi đẳng thức đã cho thành phương trình bậc hai đối với một ẩn
a 2  b2  1  2 ab  a  b  a 2  2 a b  1  b2  1  2b  0
2
 a 2  2 a b  1  b  1  0 coi là phương trình bậc hai ẩn a tham số b .

• Phương trình bậc hai Ax 2  2 Bx  C  0 với A  0 có biệt thức  được tính bởi công
thức   B2  AC .
2
Phương trình a 2  2 a b  1  b  1  0 là phương trình bậc hai, ta có
2 2
  b  1  b  1  b2  2b  1  b2  2b  1  4b .
• Điều kiện để một phương trình bậc hai có nghiệm khi   0 .
2
Phương trình a 2  2 a b  1  b  1  0 là phương trình bậc hai có nghiệm khi   0
hay 4b  0  b  0 .
• Điều kiện cần để một phương trình bậc hai có nghiệm nguyên khi  là số chính
phương.
2
Với a; b là các số nguyên thì phương trình a 2  2 a b  1  b  1  0 có  là số chính
phương suy ra 4b là số chính phương.
• Tích của một số chính phương với một số nguyên là một số chính phương khi số
nguyên đó là số chính phương.
Ta có 4b là số chính phương và 4  2 2 là số chính phương nên b cũng là số chính
phương.
• Số chính phương luôn lớn hơn hoặc bằng 0.
+ Với b  0 là số chính phương ta suy ra a  1 thỏa mãn là hai số chính phương liên
tiếp.
+ Với b  0 là số chính phương thì   0
• Phương trình bậc hai Ax 2  2 Bx  C  0 với A  0 có   0 thì có hai nghiệm phân biệt
B   B  
x hoặc x  .
A A

Liên hệ tài liệu word toán sđt và zalo: 039.373.2038 TÀI LIỆU TOÁN HỌC
138
Website:tailieumontoan.com
2
Khi b  0 thì a 2  2 a b  1  b  1  0 có   0 nên có hai nghiệm phân biệt
 2
 a  b  1  4b 
  b 1  .
 2
 a  b  1  4b 
  b  1

• Hai số chính phương liên tiếp là hai số bình phương của hai số nguyên liên tiếp.
2
+ Với a   b  1 và b là số chính phương ta có b và  b  1 là hai số nguyên liên
2 2
tiếp nên  b  và  b  1 là hai số chính phương liên tiếp hay b và a là hai số chính
phương liên tiếp.
2
+ Với a   b  1 và b là số chính phương ta có b  1 và b là hai số nguyên liên
2 2
tiếp nên  b  1 và  b  là hai số chính phương liên tiếp hay a và b là hai số chính
phương liên tiếp.
Câu 4.

  AEF
1). Nối EM . Tứ giác EFBC là tứ giác nội tiếp nên ABC  (1).
  ACB
XBA  (góc nội tiếp và góc tạo bởi tia tiếp tuyến-dây cung cùng chắn cung AB của
(O) (2).
BC   MEC
.
MEC có ME  MC  nên MEC cân tại M , suy ra BCA
2
  MEC
Kết hợp với (2), ta có được XBA .
  XBC
Cộng vế theo vế với (1), ta được ABC   AEF
  MEC

  XEM
 XBM   AEF  MEC
  XEM   XBM   XEM
  180 0 .
  XEB
Suy ra EXBM là tứ giác nội tiếp, suy ra XMB  (3).

Liên hệ tài liệu word toán sđt và zalo: 039.373.2038 TÀI LIỆU TOÁN HỌC
139
Website:tailieumontoan.com
  FAD
Tứ giác AFHE là tứ giác nội tiếp nên FEB .
  FDA
Kết hợp với (3), suy ra XMB  mà FAD   FCB nên XMB  FCB hai góc này ở vị trí
đồng vị của XM và FC suy ra XM  FC mà FC  AB , do đó XM  AB (điều phải chứng
minh).
Nhận xét: Bài toán chứng minh hai đường thẳng vuông góc ta sử dụng mối quan hệ của
hai đường thẳng với một đường thẳng thứ ba bằng tính chất từ quan hệ vuông góc và
quan hệ song song
Nhắc lại kiến thức và phương pháp:
• Tứ giác có hai đỉnh liên tiếp cùng nhìn một cạnh dưới hai góc bằng nhau là tứ giác nội
tiếp.

 
BE  AC  BEC  90
Trong ABC có đường cao BE và CF nên  Tứ giác EFBC có hai

CF  AB    90
CFB


đỉnh liên tiếp E và F cùng nhìn cạnh BC dưới hai góc cùng bằng 90 nên tứ giác
EFBC là tứ giác nội tiếp.
• Tứ giác nội tiếp có góc trong tại một đỉnh bằng góc ngoài tại đỉnh đối diện.
 là góc ngoài
 là góc trong tại đỉnh B và AEF
Tứ giác EFBC là tứ giác nội tiếp có ABC
  AEF
tại đỉnh E đối diện với B nên có ABC .
• Trong một đường tròn, góc tạo bởi tia tiếp tuyến - dây cung và góc nội tiếp cùng chắn
một chung thì bằng nhau.
 là góc tạo bởi tia tiếp tuyến BX và dây cung BA chắn cung BA
+ XBA  của đường tròn
(O) ;
 là góc nội tiếp chắn cung BA
+ ACB  của đường tròn (O) ;
  ACB
suy ra XBA .

• Tam giác có hai cạnh bằng nhau là tam giác cân.


BC
MEC có ME  MC  nên MEC cân tại M .
2
• Tam giác cân có hai góc kề cạnh đáy bằng nhau
  MEC
MEC cân tại M nên BCA .
  MEC
suy ra XBA  và ABC
  XBC
  AEF
  MEC

 XBM   AEF
  XEM   XEM
  MEC    XEM
 XBM   180 0 .
• Tứ giác có tổng hai góc trong đối diện bằng 180 là tứ giác nội tiếp.
  XEM
+ Tứ giác EXBM có XBM   180 0 nên tứ giác EXBM là tứ giác nội tiếp
  XEB
XMB  (hai góc nội tiếp cùng chắn cung XB
 của đường tròn ngoại tiếp tứ giác

EXBM );
  AEH
+ Tứ giác AFHE có AFH   90  90  180 nên tứ giác AFHE là tứ giác nội tiếp
  FAD
nên FEB ;
  FDA
suy ra XMB  mà FAD  nên XMB
  FCB   FCB
.

• Hai góc ở vị trí đồng vị của hai đường thẳng bằng nhau thì hai đưởng thẳng này song
song.

Liên hệ tài liệu word toán sđt và zalo: 039.373.2038 TÀI LIỆU TOÁN HỌC
140
Website:tailieumontoan.com
 và FCB
Hai góc XMB  ở vị trí đồng vị của hai đường thẳng XM và FC thỏa mãn
  FCB
XMB  nên suy ra XM  FC .

• Đường thẳng a  b và a  c thì b  c .


XM  FC
Ta có   XM  AB (điều phải chứng minh).
FC  AB

  BEM
2). Tứ giác ABME là tứ giác nội tiếp nên SXM  mà BEM
  EBM
 ( MBE cân tại M ).
  DFE
Tứ giác BFHD là tứ giác nội tiếp nên EBM .
  HFD
Kết hợp với trên suy ra SXM  (*).

  90 0  1 BC
Ta có S; M ; O thẳng hàng do OM và SO cùng vuông góc với BC , suy ra MSB .
2
Tứ giác AFDE là tứ giác nội tiếp nên ta có
  FCA
FDA   90 0  1 BC
  90 0  BAC 
2
  FDA
suy ra MSB  , kết hợp với (*) ta có MXS ∽HFD (g - g) (điều phải chứng minh)
Nhận xét: Chứng minh hai tam giác đồng dạng thông qua trường hợp đồng dạng góc.góc
có nghĩa là chứng minh hai tam giác có hai cặp góc bằng nhau thì hai tam giác đó đồng
dạng
Nhắc lại kiến thức và phương pháp:
• Tứ giác nội tiếp và các kiến thức liên quan đã nhắc lại ở trên.
  BEM
+ Tứ giác ABME là tứ giác nội tiếp nên SXM  (hai góc nội tiếp cùng chắn cùng
 của đường tròn ngoại tiếp tứ giác ABME ).
BM
  DFE
+ Tứ giác BFHD là tứ giác nội tiếp nên EBM  (hai góc nội tiếp cùng chắn cung
 của đường tròn ngoại tiếp tứ giác BFHD ).
HD
• Tam giác cân có hai góc kề cạnh đáy bằng nhau.
Tam giác BME cân tại M nên BEM  EBM
 , suy ra SXM
  HFD.
• Tiên đề Ơ-clit: “Từ một điểm nằm ngoài một đường thẳng đã cho kẻ được một và chỉ
một đường thẳng vuông góc với đường thẳng đó.
Ta có OM và SO cùng vuông góc với BC nên S , M , O thẳng hàng
• Tổng của hai góc nhọn trong một tam giác vuông bằng 90 .
  SBM
Tam giác SMB vuông tại M nên MSB   90 .
• Trong một đường tròn, số đo góc tạo bởi tia tiếp tuyến - dây cung bằng nửa số đó
cung mà góc đó chắn.
 là góc tạo bởi tia tiếp tuyến BS và dây cung BC chắn cung BC
Ta có SBM  nên

  1 BC
SBM   90 0  1 BC
 , suy ra MSB .
2 2
• Tứ giác nội tiếp.
Tứ giác AFDE có hai đỉnh liên tiếp E và F cùng nhìn cạnh AD dưới góc vuông nên
  FCA
AFDE là tứ giác nội tiếp, suy ra FDA  (hai góc nội tiếp cùng chắn cung FA
 của
  FDA
đường tròn ngoại tiếp tứ giác AFDE ), suy ra MSB .
• Hai tam giác có hai cặp góc bằng nhau thì đồng dạng với nhau.

Liên hệ tài liệu word toán sđt và zalo: 039.373.2038 TÀI LIỆU TOÁN HỌC
141
Website:tailieumontoan.com
  FDH
Xét MXS và HFD có: MSX  ; SXM 
  HFD
Suy ra MXS ∽ HFD (g – g) (điều phải chứng minh).


 AC BC

ABC ∽AEF ( g - g )  

 AF EF .
3). Ta có 

 AC CD
ACD ∽AFY (g - g)  


 AF FY
BC CD EF BC
    (điều phải chứng minh).
EF FY FY CD
Nhận xét: Chứng minh hai tỷ số bằng nhau dựa vào tam giác đồng dạng suy ra các cặp
cạnh tương ứng tỷ lệ
Nhắc lại kiến thức và phương pháp:
• Hai tam giác có hai cặp góc bằng nhau thì đồng dạng.
+ Xét AEF và ABC có:
 chung;
- BAC
  ABC
- AEF  (góc ngoài và góc trong tại đỉnh đối diện của tứ giác nội tiếp FECB );
suy ra AEF ∽ ABC (g – g).
+ Xét ACD và AFY có:
  90 ;
  AYF
- ADC  
  AFY
- ACD  (góc ngoài và góc trong tại đỉnh đối diện của tứ giác nội tiếp FECB );
suy ra ACD ∽ AFY (g – g).
• Hai tam giác đồng dạng có các cặp cạnh tương ứng tỷ lệ.
AC BC
+ AEF ∽ ABC  AF  EF ;
AC CD
+ ACD ∽ AFY  AF  FY ;
BC CD EF BC
suy ra EF  FY  FY  CD (điều phải chứng minh).


 x A  xB  a



y  yB  b
Câu 5. Đặt  A với a; b; c; d là các số nguyên (hiệu các số nguyên là số nguyên).

 xB  xC  c


 y B  yC  d


x  xC  a  c
Khi đó ta có  A .
 y A  yC  b  d

2 2
X  AB  a 2  b2 ; Y  BC  c 2  d 2 ; Z  CA   a  c   b  d
Theo công thức Hê-rông ta có:
X  Y  Z X  Y  Z Z  X Y Y  Z  X
S ABC  . . .
2 2 2 2
 2S ABC  X  Y  ZX  Y  ZZ  X  Y Y  Z  X .
Tính X  Y  ZX  Y  Z như sau:
2
X  Y  ZX  Y  Z  X  Y   Z 2  X 2  Y 2  2 XY  Z 2
Liên hệ tài liệu word toán sđt và zalo: 039.373.2038 TÀI LIỆU TOÁN HỌC
142
Website:tailieumontoan.com

2 a 2
 b2 c 2  d 2   2 ac  2bd .

Tính Z  X  Y Y  Z  X  như sau:


2
Z  X  Y Y  Z  X  Z 2  X  Y   Z 2  2 XY  X 2  Y 2

 2 ac  2bd  2 a 2
 b2 c 2  d 2  .
Kết hợp lại, ta có
  
2S ABC  4  a 2
 b2 c 2  d 2   ac  bd  ac  bd  a 2
 b2 c 2  d 2  
  

a  b2 c 2  d 2   ac  bd  4 a 2 d 2  2 abcd  b2 c 2


2 2
4
2
4 ad  bc  4 ad  bc .
Là một số nguyên với mọi a; b; c; d là số nguyên (điều phải chứng minh).
Nhận xét: Bài toán sử dụng tư duy của đại số và số học, áp dụng một số công thức tính
toán của hình học
Nhắc lại kiến thức và phương pháp:
• Hiệu của hai số nguyên là một số nguyên.

 x A  xB  a



y  yB  b  x A  xC  a  c
Đặt  A 
 với x A , xB , xC , y A , yB , yC , a , b , c , d là các số nguyên.

 xB  xC  c 
 y A  yC  b  d
 

 y B  yC  d


• Công thức tính khoảng cách giữa hai điểm M ; N trong mặt phẳng tọa độ
2 2
MN   x M  xN    yM  yN  .

 2 2



X  AB   x A  xB    y A  yB   a2  b2


Ta có Y  BC 
2 2


 xB  xC    y B  yC   c 2  d 2 .

 2 2 2 2
Z  CA   xA  xC    y A  yC    a  c   b  d



abc
• Công thức Hê-rông: Trong một tam giác có độ dài ba cạnh là a; b; c . Với p  là
2
nửa chu vi của tam giác. Ta có công thức tính diện tích tam giác đó
S  p  p  a p  b p  c .
Áp dụng vào tam giác ABC với ba cạnh AB; BC ; CA có độ dài lần lượt là X ; Y ; Z ta
được:
X  Y  Z  X  Y  Z  X Y  Z  X Y  Z 
S ABC  .  X .  Y .  Z
2  2   2   2 
X  Y  Z X  Y  Z Z  X Y Y  Z  X
 . . .
2 2 2 2
X  Y  ZX  Y  ZZ  X  Y Y  Z  X

4
 4S ABC  X  Y  ZX  Y  ZZ  X  Y Y  Z  X .

Liên hệ tài liệu word toán sđt và zalo: 039.373.2038 TÀI LIỆU TOÁN HỌC
143
Website:tailieumontoan.com
• Tính X ; Y ; Z theo a; b; c .
+ Tính X  Y  ZX  Y  Z như sau:
X  Y  ZX  Y  Z  X  Y   Z X  Y   Z
• Hằng đẳng thức A 2  B2   A  B A  B
2
X  Y  ZX  Y  Z  X  Y   Z 2
2
• Hằng đẳng thức  A  B  A 2  2 AB  B2
X  Y  ZX  Y  Z  X 2  2XY  Y 2  Z 2
2
 2
   
2 2
2
 a2  b2  2 a2  b2 c 2  d 2  c 2  d2 

 a  c   b  d  .

2
• Hằng đẳng thức  A A
2 2
X  Y  ZX  Y  Z  a2  b2  2 a 2  b 2 c 2  d 2  c 2  d 2   a  c    b  d
 a 2  b2  2 a 2  b2 c 2  d 2  c 2  d 2  a 2  2 ac  c 2  b2  2bd  d 2
 2 a 2  b2 c 2  d 2  2 ac  2bd .
- Hoàn toàn tương tự ta tính được Z  X  Y Y  Z  X  như sau:
2
Z  X  Y Y  Z  X  Z 2  X  Y   Z 2  2 XY  X 2  Y 2

 2 ac  2bd  2 a 2
 b2 c 2  d 2   2 ac  2bd  2 a 2  b2 c 2  d 2 .
Suy ra
4S ABC   2 a 2  b2 c 2  d 2  2 ac  2bd  2 ac  2bd  2 a 2  b2 c 2  d 2 
  
 2S ABC   a 2  b2 c 2  d 2  ac  bd  a 2  b2 c 2  d 2  ac  bd
  

a  b2 c 2  d 2   ac  bd  a 2 d 2  2 abcd  b2 c 2


2 2
 2S ABC 
2
 2S ABC  ad  bc  ad  bc .
• Tổng, Hiệu, Tích của các số nguyên là số nguyên.
Ta có a; b; c; d là các số nguyên nên ad  bc là số nguyên.

ĐỀ SỐ 16.

Câu 1.
2
1). Phương trình đã cho tương đương với phương trình x  3 x  2  0
 x 1
  x  1 x  2  0   .
 x  2
Phương trình có nghiệm x  2;  1; 1; 2
Nhận xét: Bài toán cơ bản là đi giải phương trình bậc hai, tuy nhiên để tìm nghiệm của
phương trình thì cần phải qua bước giải dấu trị tuyệt đối.
Ý tưởng: Đây là một phương trình chứa trị tuyệt đối cơ bản, như đã biết rằng x  x nếu
và x  x nếu x  0 . Vì thế ta có thể chia trường hợp và đưa về giải phương trình bậc hai.

Liên hệ tài liệu word toán sđt và zalo: 039.373.2038 TÀI LIỆU TOÁN HỌC
144
Website:tailieumontoan.com
2
Tuy nhiên ta có thể đưa về phương trình bậc hai trước vì x 2   x  do đó phương trình đã
2
cho  x  3 x  2  0
 x 1
  x  1 x  2  0   . Đến đây giờ thì rất dễ để suy ra nghiệm của phương trình là
 x  2
x  2;  1; 1; 2 .
Bài toán kết thúc.
Bài tập tương tự:
1. Giải phương trình x 2  5 x  4  x  1 .


Đáp số: x  1; 5;  3  6 . 
5 7
2. Giải phương trình x  2013  x  2014  1 .
Đáp số: x  2013 hoặc x  2014 .
2). Cộng vế với vế các phương trình đã cho ta được x  y  z  9 .
Phương trình đầu có dạng 2 x   x  y  z  1  x  4 .
Phương trình thứ hai có dạng 2 y   x  y  z  3  y  3 .
Phương trình thứ ba có dạng 2 z   x  y  z  5  z  2 .
Thử lại thỏa mãn.
Vậy x  4; y  3; z  2 .
Nhận xét: Bằng phương pháp cộng vế, cộng các phương trình của hệ với nhau, ta tìm
được biểu thức chứa mối liên hệ giữa x , y , z từ đó thế ngược lại tìm nghiệm của hệ phương
trình.
Ý tưởng: Quan sát được sự đối xứng giữa các phương trình trong hệ, nếu phương trình
trên chứa biến thì phương trình dưới lại có biểu thức triệt tiêu biến đó vì thế ta sẽ nghĩ đến
việc cộng các phương trình lại với nhau, khi đó ta có x  y  z  9 . Với biểu thức này, xét
riêng từng phương trình, cụ thể phương trình một ta thấy có xuất hiện tổng y  z do đó ta
sẽ rút được y  z  x  9 nên suy ra x  x  9  1  x  4 . Tương tự ta sẽ tìm được
nghiệm của hệ là x  4; y  3; z  2 .
Bài toán kết thúc.
Bài tập tương tự:
x  3 y  2 z  8

1. Giải hệ phương trình 2 x  2 y  z  6 .

3 x  y  z  6
Đáp số:  x; y; z  1; 1; 2 .
2 x  3 y  4 z  5

2. Giải hệ phương trình 4 x  5 y  z  6 .

3 x  4 y  3 z  7
 22 131 39 
Đáp số:  x; y; z   ; ; .
 101 101 101
Câu 2.
1). Theo định nghĩa phép toán T, ta có:
Liên hệ tài liệu word toán sđt và zalo: 039.373.2038 TÀI LIỆU TOÁN HỌC
145
Website:tailieumontoan.com
1 1 1 1 1 1
5T 6    ; 7T 8   
5 6 30 7 8 56
1 1
suy ra P  5 T 6T 7 T 8   T   .
 30   56 
1 1
Vậy P   T    30  56  26 .
 30   56 

Nhận xét: Bài toán tư duy từ ký hiệu, tính toán,… Nhìn qua, tưởng chừng đây là một bài
toán khó vì hình thức lạ. Tuy nhiên, bản chất của bài toán rất đơn giản, chỉ là các phép tính
phân số cơ bản đã học từ lớp 6.
Nhắc lại kiến thức và phương pháp:
1 1
• Phép toán T được định nghĩa như sau aTb   với a và b là các số thực khác 0 tùy
a b
ý.
1 1 65 1
+ 5T 6     .
5 6 5.6 30
1 1 8 7 1
+ 7T 8     .
7 8 7.8 56
• Thay các giá trị vào biểu thức lớn.
1 1 1 1
P  5 T 6T 7 T 8   T      30  56  26
 30   56  1 1
30 56
Vậy P  26 .

2). Ta ký hiệu các điều kiện như sau


6 a 2  20 a  15  0 (1).
15b2  20b  6  0 (2).
ab  1 (3).
Dễ thấy các phương trình (1) và (2) đều có hai nghiệm phân biệt.
Do (3) nên b khác 0. Chia hai vế của (2) cho b2 ta được
2
 1  1
6    20    15  0 (4).
 b   b 
1
Từ (1), (3) và (4), suy ra a và là hai nghiệm khác nhau của phương trình
b
6 x 2  20 x  15  0 (5).

 1 10
 a 
 b 3 .
Theo định lí Vi-ét 

 a 5
 

b 2

3
ab2  9 ab  1 a  1
3
5  10 
3
2015
Từ đó   9 a     9   
b3 b 
 
b 2 
 3  6
b3 6
suy ra  , điều phải chứng minh.
ab2  9 ab  1
3
2015

Liên hệ tài liệu word toán sđt và zalo: 039.373.2038 TÀI LIỆU TOÁN HỌC
146
Website:tailieumontoan.com
Nhận xét: Bài toán sử dụng phép ẩn phụ hóa, đưa về các phương trình đối xứng, đồng
thời kết hợp với biểu thức đồng bậc ở giả thiết để suy ra điều phải chứng minh.
Ý tưởng: Quan sát giả thiết bài toán, giữa hai phương trình bậc hai có sự đối xứng giữa các
1
hệ số của chúng, tuy nhiên sự khác biệt nằm ở 6a2  15 và 15b2  6 vì thế ta sẽ đặt t  khi
b
6 a 2  20 a  15  0
đó  2 , vì thế ta có được a , t đều là nghiệm của phương trình
6t  20t  15  0


a  t   10
6 x 2  20 x  15  0 . Theo định lý Viet ta có  3  a  1   10 và a  5 . Khi có được
 5 b 3 b 2
at 
 2
3
b
điều này, ta sẽ khai thác đến biểu thức 3
, đây là một biểu thức đồng bậc ba
ab2  9 ab  1
nên ta sẽ “ chia để trị “ nên chia cả tử và mẫu cho b3 ta được:
b3 1 1 6
   suy ra đpcm.
ab2  9 ab  1
3
a  1
3
5  10 
3
2015
 9 a    9  
b  b  2  3 
Bài toán kết thúc.
Bài tập tương tự:
1. Cho a và b là các số thực thỏa mãn điều kiện 5a2  6a  1  0 ; b2  6b  5  0 ; ab  1 .
a  ab  1
Tính giá trị của P  .
b
2. Cho a và b là các số thực thỏa mãn điều kiện 5a2  6a  1  0 ; b2  6b  5  0 ; ab  1 .
b3 a
Tính giá trị của Q   .
ab2  9 ab  1 b 3

Câu 3.
1). Giả sử a và b là các số tự nhiên sao cho n  2015  a 2 ; n  2199  b2 .
suy ra b  ab  a  184 hay b  ab  a  2 3.23 .
Vì b  a và b  a là các số có cùng tính chẵn lẻ và b  a  b  a nên chỉ xảy ra hai trường hợp
b  a  2 b  a  4
 (I) và  (II).
b  a  92 b  a  46
Trường hợp thứ nhất
a  45
 I     n  10 (thỏa mãn).
b  47
Trường hợp thứ hai
a  21
II     n  1574  0 (không thỏa mãn).
b  25
Vậy n  10.
Nhận xét. Bài toán số học sử dụng tính chất số chính phương, phương trình ước số
Nhắc lại kiến thức và phương pháp.
• Số chính phương được viết dưới dạng x 2 với x là số nguyên.

Liên hệ tài liệu word toán sđt và zalo: 039.373.2038 TÀI LIỆU TOÁN HỌC
147
Website:tailieumontoan.com

+ n + 2015 là số chính phương nên có dạng a 2 với a là số nguyên.


+ n + 2199 là số chính phương nên có dạng b2 với b là số nguyên
• Hằng đẳng thức A 2 − B2 = ( A + B )( A − B ) .

n  2015  a 2
Trừ vế theo vế của  cho nhau ta được
 2
n  2199  b

b2  a 2  n  2199  n  2015  b  ab  a  184 .
• Phương trình ước số: b  ab  a  184  2 3.23 .
• Đánh giá loại bỏ bớt trường hợp: “Vì b  a và b  a là các số có cùng tính chẵn lẻ và
b  a  b  a ”. Vì tổng, hiệu của hai số lẻ là số chẵn; tổng, hiệu của hai số chẵn là số
chẵn; tổng, hiệu của một số chẵn và một số lẻ là một số lẻ. Nên hiệu và tổng luôn cùng
chẵn hoặc cùng lẻ.
• Các hệ phương trình suy ra từ phương trình ước số.

 
b  a  2  a  45
b  a  92  b  47 n  10
     kết hợp với điều kiện n là số tự

b  a  4  a  21 n  1574
 
b  a  46  b  25
nhiên nên suy ra n  10 .
Vậy n  10.
2). Trong dãy số nói trên, 9 số đầu tiên: 1,2,3,...,9 là các số có 01 chữ số.
90 số tiếp theo: 10,11,12,...,99 là các số có 02 chữ số.
900 số tiếp theo: 100,101,102,...,999 là các số có 03 chữ số.
Như vậy, bằng cách viết nói trên ta thu được một số có:
9  2  90  3  900  4  2893 chữ số.
Vì 9  2  90  2016  2893 nên chữ số thứ 2016 của dãy số là một chữ số của số có 03 chữ số.
Ta có 2016  9  2  90  3  609, số có 03 chữ số đầu tiên là 100, số có 03 chữ số thứ 609 là
609  100  1  708 do đó chữ số thứ 2016 trong dãy đã cho là chữ số 8.
Nhận xét. Bài toán số học sử dụng tính chất số chính phương, phương trình ước số
Nhắc lại kiến thức và phương pháp.
cd
• Tính số các số viết theo quy luật của một dãy số ta tính bằng công thức s  1
h
trong đó s là số các số của dãy, c là số cuối của dãy, d là số đầu của dãy, h là khoảng
cách giữa các số.
9 1
+ Từ 1 đến 9 có  1  9 số có một chữ số.
1
99  10
+ Từ 10 đến 99 có  1  90 số có hai chữ số.
1
999  100
+ Từ 100 đến 999 có  1  900 số có ba chữ số.
1
• Phân tích các chữ số.

Liên hệ tài liệu word toán sđt và zalo: 039.373.2038 TÀI LIỆU TOÁN HỌC
148
Website:tailieumontoan.com
+ Ký hiệu A  123456789101112 98999100101
  989991000 có nghĩa là 12...89 có a chữ

a b c

số, 101199 có b chữ số, 100101998999 có c chữ số.



 a  9.1  9


- 
Ta có b  90.2  180 suy ra số A có


c  900.3  2700


a  b  c  4  9  180  2700  4  2893 chữ số.
- Ta có 2016 thỏa mãn a  b  2016  2893  189  2016  2893 nên suy ra số thứ 2016
là số có ba chữ số.
+ Phân tích xem chữ số thứ 2016 của số A là chữ số thứ mấy từ chữ số 1 của c .
Ta có 2016  1.9  2.90  3.609 số có 03 chữ số đầu tiên là 100, số có 03 chữ số thứ 609 là
609  100  1  708 do đó chữ số thứ 2016 trong dãy đã cho là chữ số 8.

Câu 4.
I

A M B

H
O
E

D C

1). Do ABCD là hình vuông nên hai đường chéo vuông góc, hai đường chéo tạo với các
cạnh của hình vuông góc 45 .
  MAO
Tam giác AME vuông cân đỉnh A , suy ra AM  AE ; EAO   45
  EOA
suy ra AMO  AEO (c – g – c), suy ra MOA .
.
Vậy OA là phân giác trong của góc MOE
.
Chứng minh tương tự, ta có OB là phân giác trong của góc MOF
  MOB
Mặt khác, MOA   AOB
  90 o  MOE
  MOF
  2 AOB
  180 hay E; O; F thẳng hàng;
điều phải chứng minh.
Nhận xét. Bài toán chứng minh ba điểm thẳng hàng ta chứng minh góc tạo bởi ba điểm là
góc bẹt.
Nhắc lại kiến thức và phương pháp.

Liên hệ tài liệu word toán sđt và zalo: 039.373.2038 TÀI LIỆU TOÁN HỌC
149
Website:tailieumontoan.com
• Hình vuông có hai đường chéo tạo với nhau góc vuông, đường chéo tạo với cạnh góc
45 .
  CAD
ABCD là hình vuông nên BAC   45 hay MAO .
  OAE

• Hai tam giác có hai cặp cạnh tương ứng bằng nhau và góc xen giữa hai cạnh bằng
nhau thì hai tam giác bằng nhau theo trường hợp “cạnh - góc - cạnh” (c - g - c).
Xét AMO và AEO có:
+ AM  AE (giả thiết);
  OAE
+ MAO ;

+ AO : cạnh chung;
Suy ra AMO  AEO (c – g - c).
• Hai tam giác bằng nhau có hai góc tương ứng bằng nhau.
  AOE
AMO  AEO  AOM  , suy ra OA là phân giác của MOE
 nên MOE
  2 MOA
.
 nên MOF
Chứng minh tương tự ta có OB là phân giác trong của góc MOF   2 MOB
.

• Ba điểm tạo thành góc có số đo 180 thì ba điểm đó thẳng hàng.


  EOM
EOF   MOF
  2 MOA
  MOB
  2.AOB
 
  2.90  180 , suy ra ba điểm E; O; F

thẳng hàng (điều phải chứng minh).


  MEA
2). Tứ giác AEHM nội tiếp đường tròn đường kính ME nên MHA   45 .
  MFB
Tứ giác BFHM nội tiếp đường tròn đường kính MF nên MHB   45 , suy ra
  AHM
AHB   MHB
  90 .
Ta thấy O và H cùng nhìn AB dưới một góc vuông nên bốn điểm A; B; H ; O cùng nằm
trên đường tròn đường kính AB .
Nhận xét. Bài toán chứng minh ba điểm thẳng hàng ta chứng minh góc tạo bởi ba điểm là
góc bẹt.
Nhắc lại kiến thức và phương pháp.
• Trong đường tròn, hai góc nội tiếp chắn cùng một cung thì bằng nhau.
  MEA
+ Tứ giác AEHM nội tiếp đường tròn đường kính ME nên MHA  (hai góc nội
  45 nên MHA
 ). Mà MEA
tiếp cùng chắn cung MA   45 .
  MFB
+ Tứ giác BFHM nội tiếp đường tròn đường kính MF nên MHB  (hai góc nội
  45 nên MHB
 ). Mà MFB
tiếp cùng chắn cung MB   45 .

• Cộng vế theo vế của hai đẳng thức ta được một đẳng thức đúng.
  MHB
MHA   45  45  AHB
  90 .

• Một điểm nhìn một đoạn dưới một góc vuông thì thuộc đường tròn đường kính là
đoạn thẳng đó.
  90 nên điểm H thuộc đường tròn đường kính AB ;
+ Có AHB
  90 nên điểm H thuộc đường tròn đường kính AB ;
+ Có AOB

Liên hệ tài liệu word toán sđt và zalo: 039.373.2038 TÀI LIỆU TOÁN HỌC
150
Website:tailieumontoan.com
Suy ra điểm H ; O thuộc đường tròn đường kính AB hay bốn điểm A; B; H ; O cùng
nằm trên đường tròn đường kính AB (điều phải chứng minh).
3). Đường thẳng MH cắt đường tròn đường kính AB tại điểm thứ hai I ( I khác H ).
  BHI
Ta có AHI   45 nên I là điểm chính giữa cung AB (không chứa O ) của đường tròn
đường kính AB .
Do A; B; O là các điểm cố định nên I là điểm cố định ( I đối xứng với O qua đường thẳng
AB ).
Vậy, khi M di động trên cạnh AB , đường thẳng MH luôn đi qua điểm cố định I ( I đối
xứng với O qua đường thẳng AB ).
Nhận xét. Bài toán chứng minh đường thẳng cố định sử dụng tính chất một yếu tố có mối
liên hệ cố định với các yếu tố cố định thì cố định.
Nhắc lại kiến thức và phương pháp.
• Trong một đường tròn, hai góc nội tiếp bằng nhau chắn hai cung thì hai cung đó bằng
nhau.
  BHI
+ Ta có AHI  (  45 );
 là góc nội tiếp chắn cung AI
+ AHI  của đường tròn đường kính AB );
 là góc nội tiếp chắn cung BI
+ BHI  của đường tròn đường kính AB );

Suy ra AI  mà I  AB
  BI .
 nên I điểm chính giữa cung AB

• Một yếu tố có mối liên hệ cố định với các yếu tố cố định thì cố định.
Ta có O; A; B cố định nên I cố định. Do đó đường thẳng MH luôn đi qua điểm cố
định I với I là điểm đối xứng với O qua đường thẳng AB .
Câu 5. Xét đồ thị của hàm số y  f  x.
Trên mỗi miền x  1 ; 1  x  2 ; 2  x  3 ; 3  x  4 ; x  4 (gồm 5 miền), y  f  x là các hàm
số bậc nhất.
Đồ thị hàm số y  f  x là đường gấp khúc gồm 02 tia và 03 đoạn thẳng liên tiếp nhau. Mặt
khác f  x  0, x   nên tồn tại giá trị nhỏ nhất của f  x trên  và giá trị nhỏ nhất này sẽ
đạt được tại đầu mút nào đó của các tia hoặc các đoạn thẳng.
Nói cách khác min f  x  min  f 1 ; f 2 ; f 3 ; f 4  f 3  8 .
Giá trị nhỏ nhất của hàm số y  f  x bằng 8, đạt được khi x  3 .
ĐỀ SỐ 17.
Câu 1.
1). Điều kiện 0  x  1 .
Phương trình tương đương với
3
x3  x
 
1 x  1  1  3  
1 x  1  x  x  3 .

 
Nếu 0  x  1  3 1  x  1  3 , đồng thời x  x 3  1  4  3,
suy ra VT  VP (loại).
Thử lại ta thấy x  1 là nghiệm.

Liên hệ tài liệu word toán sđt và zalo: 039.373.2038 TÀI LIỆU TOÁN HỌC
151
Website:tailieumontoan.com
Nhận xét: bài toán kết hợp giữa phương pháp nhân liên hợp và phương pháp đáng giá để
tìm nghiệm của phương trình.
Nhắc lại kiến thức và phương pháp:
• Biểu thức liên hợp x  m  x   xm x  xm  x 
m
 xm x  với x  0; x  m  0 .
xm  x

m  f  x  0
• Đánh giá: m  f  x  n  g  x  h  x với  .

n  2 m


 m  f x  n  n
  
Ta có: 0  f  x  m   , suy ra phương trình vô nghiệm. Vậy x  m
 g  x  h  x  2 m

là nghiệm của phương trình đã cho.
Ý tưởng: Bài toán xuất hiện ba căn thức nằm trong một tích, sẽ rất khó để định hình ra
hướng giải, ẩn phụ sẽ rất phức tạp. Nhưng nếu xét hai căn thức đầu tiên ta thấy
2 2
   x
x3   3 . Vì thế ta sẽ nghĩ ngay đến chuyện dùng hằng đẳng thức dạng

a 2  b2  a  ba  b . Khi đó phương trình đã cho tương đương với:


3
x3  x
 
1  x  1  1  3 1  x  3  x  3  x ( i ).

Với phương trình ( i ), ta sẽ đi nhẩm một vài giá trị nghiệm đẹp thỏa mãn các yêu cầu là
1  x  0 và các biểu thức trong căn thức là số chính phương vì thế ta khẳng định nó có
nghiệm duy nhất x  1 , đồng thời 1 lại là miền chặn của biến do đó ta sẽ đi đánh giá
phương trình ( i ). Tức là với 0  x  1 ta sẽ đi chứng minh ( i ) vô nghiệm như sau:


3 1  x  3  3
0  x 1  VT  VP  ( i ) vô nghiệm.

 x  3  x  3


Vậy ta kết luận x  1 là nghiệm của phương trình đã cho.
Bài toán kết thúc.
Bài tập tương tự:
3. Giải phương trình  x  2  x 1  
2  x 1  1.
Đáp số: x  2 .
4. Giải phương trình  x 1 x2  
3 x 1  1.
Đáp số: x  3 .

2).
+ Xét x  y  0 là nghiệm.
+ Xét x  0; y  0 hệ phương trình tương đương với
1 1
  1 2

   1 2

 (1)

x
2
y 2 
x
2
y2

  
 .

 1 1 
  1  

 
 1  1 2  2   8
  
 1  
  4    (2)
  xy     xy 
 x y 

  x y 

Liên hệ tài liệu word toán sđt và zalo: 039.373.2038 TÀI LIỆU TOÁN HỌC
152
Website:tailieumontoan.com

 1 1
3

  2
1 1  x y
Thay (1) vào (2) ta thu được    
8  x y1.
 x y  
 1
  1

 xy

Nhận xét: bài toán sử dụng phép chia các biến, sau đó kết hợp hai phương trình tìm mối
liên hệ giữa hai biến để tìm nghiệm của hệ phương trình.
Ý tưởng: quan sát thấy hệ phương trình có dáng dấp của hệ phương trình đối xứng loại
hai, tức là sự xuất hiện của tổng x  y và tích xy sẽ làm ta nghĩ đến phép đặt Viet là
S  x  y ; P  xy để từ đó giải hệ hai ẩn S , P . Nhưng nếu làm như thế ta thu được hệ
 2 2
S  2 P  2 P


S 1  P  4 P 2 , hệ này giải bằng phương pháp thế sẽ thu được phương trình lũy thừa bậc
 


4 phức tạp, vì thế ta sẽ nghĩ đến phương án khác đó chính phương pháp “ chia để trị “.
Trước hết là xét phương trình hai, vế trái của nó xuất hiện tích đồng thời vế phải cũng
xuất hiện tích số dạng xy.xy . Do đó ta sẽ nghĩ đến việc chia một biểu thức bên vế trái cho
 1 1  1 1 1
xy và ta được như sau:   1    4 . Phương trình này chủ đạo là hai biến ;

 x y  xy  x y vì
1 1
vậy ta cũng chia phương trình một để xuất hiện hai biến này, đó là x 2  y 2  2 . Và nếu đặt
ax  1 a 2  b2  2 a 2  b2  2
   
by  1 hệ phương trình đã cho trở thành a  b1  ab  4 a  b2  2 ab  4
 
a  b  2
2 2
a  b  2
2 2

  
a  ba 2  2 ab  b2   4 a  b3  8
 

 2
a b  2 2
1 1
   ab1  1 x y 1
 x y
a  b  2

Bài toán kết thúc.
Bài tập tương tự:

 2 2

 x  1  y  1  27 xy
1. Giải hệ phương trình  2 .

 x  1 y 2  1  10 xy


1   1 
 2    
Đáp số:  x; y   ; 2  3  , 2; 2  3 , 2  3; .
2 
 
3
 3 1 
 x  y 1    16
3 

  xy 
2. Giải hệ phương trình   2 .
 2  1 
 x  y 1    8
2 

  xy 
Câu 2.
 1 1
1). Ký hiệu K   3 n    , do n  1  K  1 .
 27 3 
3 3
1 1  1  1  2 
Ta có K  3 n  
  K  1  K    n   K  
27 3  3  27  3 

Liên hệ tài liệu word toán sđt và zalo: 039.373.2038 TÀI LIỆU TOÁN HỌC
153
Website:tailieumontoan.com
K 1 1 4 8
 K3  K2   n   K 3  2K 2  K 
3 27 27 3 27
K 4 1 3
 K 3   n  K 2  K 3  3K 2  K   K 3  n  K 2  K  1
3 3 3
2
 1 1
suy ra n  K 2  n   3 n    không biểu diễn được dưới dạng lập phương của một số
 27 3 
nguyên dương.
Nhận xét. Áp dụng kiến thức về phần nguyên, quan hệ giữa phần nguyên của một số với
số đó.
Nhắc lại kiến thức và phương pháp.
• Áp dụng giả thiết của bài toán đề tìm ra điều kiện.
1 1
Ta có n là số nguyên dương nên n  1 khi đó 3 n   1.
27 3
• Giá trị phần nguyên của một số không vượt qua số đó.
1 1  1 1
3 n   1 nên  3 n     1 hay K  1 .
27 3  27 3 
• Một số thực bất kỳ có giá trị lớn hơn hoặc bằng giá trị của phần nguyên của nó và nhỏ
hơn số nguyên liền sau số nguyên biểu diễn phần nguyên của số thực đó.
Ta có
   
 3 n 1  1  3 n 1  1   3 n 1  1  1
 27 3  27 3  27 3 

1 1
 K  3 n   K  1.
27 3
3
Tiếp tục biến đổi như trên bài giải ta được K 3  n  K 2  K  1 .
• Không tồn tại một lập phương nào giữa hai lập phương liên tiếp.
3 3
Ta có K 3 và K  1 là hai lập phương liên tiếp. Mà K 3  n  K 2  K  1 nên n  K 2
không thể viết được dưới dạng lập phương của một số nguyên dương.
2
 1 1
Vậy với mọi số nguyên dương n , biểu thức n   3 n    không biểu diễn được
 27 3 
dưới dạng lập phương của một số nguyên dương.

2). Ta có 6  x 2  5  6  y 2  5  z 2  5

 6  x  y x  z  6  y  z y  x  z  xz  y


3  x  y   2  x  z  3  x  y   2  y  z   z  x   z  y 
  
2 2 2
9x  9 y  6z 3
   3 x  3 y  2 z ,
2 2
3x  3 y  2 z 2
suy ra P  
6  x 2  5  6  y 2  5  6  z 2  5 3
Đẳng thức xảy ra khi x  y  1; z  2

Liên hệ tài liệu word toán sđt và zalo: 039.373.2038 TÀI LIỆU TOÁN HỌC
154
Website:tailieumontoan.com
2
Vậy Pmin  .
3
Nhận xét: Bài toán sử dụng phép thế từ giả thiết và bất đẳng thức Cosi cho hai số thực
dương để tìm giá trị nhỏ nhất của biểu thức.
Ý tưởng: Quan sát thấy, bài toán có sự đối xứng giữa hai kiến x; y nên điểm rơi sẽ xảy ra
tại x  y  kz . Thế lại giả thiết ta sẽ tìm được x  y  1; z  2 . Giả thiết cho xy  yz  zx  5
đồng thời số 5 này cũng xuất hiện ở biểu thức P nên ta sẽ nghĩ đến chuyện thế giả thiết
vào P . Khi đó ta có:
3x  3 y  2 z
P .
6  x  y x  z  6  y  z y  z  z  xz  y
Và điều ta cần là sử dụng đánh giá nào đó để triệt tiêu tử số và mẫu số, tức là tìm số m
thỏa mãn:
6  x  y x  z  6  y  z y  z  z  xz  y  m3x  3 y  2 z .
1
Thì lúc đó P  . Câu chuyện tiếp theo là tìm m , quan sát thấy các biểu thức trong căn
m
đều là tích của hai thừa số dương, ta sẽ sử dụng bất đẳng thức Cosi cho hai số thực dương
nhưng phải thỏa mãn điều kiện điểm rơi. Với căn thức cuối, với điểm rơi
x  y  z  x  z  y thì ta có ngay rằng
x  y  2z
 z  x z  y  .
2
Cũng với tư duy đó, ta sẽ thấy:
3  x  y   2  x  z
6  x  y x  z 
2
3  y  z   2  y  x
6  y  z y  x 
2
3
Nên 6  x  y x  z  6  y  z y  z   z  x z  y  3x  3 y  2 z .
2
2 2
Tức là P   Pmin  tại x  y  1; z  2 .
3 3
Bài toán kết thúc.
Bài tập tương tự:

1. Cho a; b; c  0 và thỏa mãn a  b  c  1 . Chứng minh rằng:


bc ca ab 1
P    .
a  bc b  ca c  ab 2
2. Cho a; b; c  0 và thỏa mãn a  b  c  1 . Chứng minh rằng:
bc ca ab 3
P    .
a  bc b  ca c  ab 2

Câu 3.

Liên hệ tài liệu word toán sđt và zalo: 039.373.2038 TÀI LIỆU TOÁN HỌC
155
Website:tailieumontoan.com

B P C

M
I N

A D

Q
  BPM
1). Tứ giác BPIM nội tiếp và AD  BC , suy ra MAD   BIM
 , nên tứ giác AMID nội

tiếp.
Tương tự tứ giác DNIA nội tiếp.
Vậy các điểm A; M ; I ; N ; D thuộc một đường tròn ( K ) .
Nhận xét. Chứng minh năm điểm cùng thuộc một đường tròn ta chứng minh cho bốn
điểm trong đó tạo thành một tứ giác nội tiếp.
Nhắc lại kiến thức và phương pháp.
• Hai góc nội tiếp cùng chắn một cung của một đường tròn thì bằng nhau.
  BIM
BPM  (hai góc nội tiếp cùng chắn cung BM
 của đường tròn ngoại tiếp tứ giác
BPIM ).
• Một đường thẳng cắt hai đường thẳng song song tạo ra các cặp góc so le trong bằng
nhau.
  MAD
BPM  (hai góc so le trong của BP  AD ), suy ra BIM
  MAD
.

• Tứ giác có góc ngoài tại một đỉnh đối diện bằng góc trong không kề với nó thì tứ giác
đó là tứ giác nội tiếp.
 là góc ngoài tại đỉnh I và góc MAD
Tứ giác MIDA có góc BIM  là góc trong tại đỉnh

A không kề với I thỏa mãn BIM   MAD


 nên suy ra MIDA là tứ giác nội tiếp.
Hoàn toàn tương tự ta chứng minh được DNIA là tứ giác nội tiếp.
Do đó 5 điểm A; M ; I ; N ; D cùng thuộc một đường tròn.

2). Do các tứ giác BPIM và CPIN nội tiếp nên ta có QMI   BPI  CNI  , suy ra tứ giác MINQ

nội tiếp.
Mà M ; I ; N  ( K ) , suy ra tứ giác MINQ nội tiếp đường tròn ( K ) .
Vậy Q thuộc đường tròn ( K ) .
Nhận xét. Chứng minh một điểm nằm trên một đường tròn ta chứng minh điểm đó cùng
ba điểm nằm trên đường tròn tạo thành một tứ giác nội tiếp.
Nhắc lại kiến thức và phương pháp.
• Tứ giác nội tiếp có góc ngoài tại một đỉnh bằng góc trong tại đỉnh không kề với nó.

Liên hệ tài liệu word toán sđt và zalo: 039.373.2038 TÀI LIỆU TOÁN HỌC
156
Website:tailieumontoan.com
+ Tứ giác BPIM là tứ giác nội tiếp có góc ngoài tại đỉnh M và góc trong tại đỉnh I
  BPI
không kề với nó nên QMI .

+ Tứ giác CPIN là tứ giác nội tiếp có góc ngoài tại đỉnh P và góc trong tại đỉnh N
  CNI
không kề với nó nên BPI   CNI
 , suy ra QMI  .

• Áp dụng nhắc lại kiến thức.


 là góc trong tại đỉnh
 là góc ngoài tại đỉnh N và góc QMI
Tứ giác MINQ có góc INC
  QMI
M không kề với N thỏa mãn INC  nên suy ra MINQ là tứ giác nội tiếp.

• Qua ba điểm không thẳng hàng xác định được một và chỉ một đường tròn.
Bốn điểm M ; I ; N ; Q cùng thuộc một đường tròn mà ba điểm M ; I ; N cùng thuộc
đường tròn ( K ) nên suy ra Q cũng thuốc đường tròn ( K ) .

  PIC
3). Khi P; I ; Q thẳng hàng, kết hợp với Q thuộc đường tròn ( K ) ta có AIQ  (đối đỉnh);
  PNC
PIC  (do tứ giác NIPC nội tiếp).
  QND
PNC  (đối đỉnh); QND
  QID (do tứ giác INDQ nội tiếp).

 AIQ  nên IP là phân giác góc BIC


 , suy ra IQ là phân giác DIA
  QID .
PB IB ID IB  ID BD PB BD
Do đó       .
PC IC IA IC  IA AC PC CA
Nhận xét. Chứng minh đẳng thức là tỷ số của các đoạn thẳng ta thường sử dụng mối liên
hệ giữa các đoạn thẳng thông qua định lý Ta-lét, tính chất đường phân giác trong/ngoài
tam giác, tính chất của tỷ lệ thức.
Nhắc lại kiến thức và phương pháp.
• Hai góc đối đỉnh thì bằng nhau.
 và góc PIC
+ Góc AIQ   PIC
 là hai góc đối đỉnh nên AIQ .
 và góc QND
+ Góc PNC   QND
 là hai góc đối đỉnh nên PNC .
• Áp dụng nhắc lại kiến thức.
+ PIC  (hai góc nội tiếp cùng chắn cung PC
  PNC  của đường tròn ngoại tiếp tứ giác
PINC ).
  QID
+ QND  (hai góc nội tiếp cùng chắn cung QD
 của đường tròn ngoại tiếp tứ giác
  QID
QNID ), suy ra AIQ  .
• Một tia nằm giữa hai tia và chia góc tạo bởi hai tia đó thành hai góc bằng nhau thì tia
này là tia phân giác.
Ta có IQ nằm giữa IA và ID mà AIQ   QID
 suy ra IQ là phân giác của góc AID.
• Hai góc đối đỉnh có chung đường phân giác hay tia phân giác của góc này là tia đối
của tia phân giác góc kia.
+ IQ là phân giác của góc AID.
+ IQ là tia đối của tia IP .
 và góc BIC
Góc AID 
 là hai góc đối đỉnh, suy ra IP là phân giác của góc BIC
• Đường phân giác trong của một tam giác chia cạnh dối diện thành hai đoạn thẳng có
tỷ số bằng với tỷ số hai cạnh tương ứng của tam giác.
 của tam giác CBI nên PB  IB
Ta có IP là phân giác của góc trong BIC
PC IC
• Định lý Ta-lét.
Liên hệ tài liệu word toán sđt và zalo: 039.373.2038 TÀI LIỆU TOÁN HỌC
157
Website:tailieumontoan.com
IB IC IB ID PB IB ID
Ta có BC  AD nên
   , suy ra   .
ID IA IC IA PC IC IA
PB IB ID IB  ID BD PB BD
• Tỷ lệ thức       (điều phải chứng minh).
PC IC IA IC  IA AC PC AC

Câu 4. Giả sử A có n số, chúng ta xếp chúng theo thứ tự 1  x1  x2  x2    xn  100


(1).
Suy ra với mỗi k  1,2,3, , n  1 ta có xk1  xi  x j  xk  xk  2 xk (2), với 1  i , j  k.
Áp dụng kết quả 2 ta thu được x2  1  1  2 ; x3  2  2  4 ; x4  8 ; x5  16 ; x6  32 ;
x7  64 , suy ra tập A phải có ít nhất 8 phần tử.
+) Giả sứ n  8  x8  100 .
Vì x6  x7  32  64  96  x8  2 x7  x7  50.
Vì x5  x6  16  32  48  x7  2 x6  x6  25.
25
Vì x4  x5  8  16  24  25  x6  2 x5  x5  (mâu thuẫn).
2
+) n  9 ta có tập 1; 2; 3; 5; 10; 20; 25; 50; 100 thỏa mãn yêu cầu bài toán .
Đáp số: n  9 .
Nhận xét. Các bài toán dạng này chủ yếu đánh giá tư duy và không có cách giải tổng quát.
Nhắc lại kiến thức và phương pháp.
• Từ 1 đến 100 có 100 số tự nhiên. Tập hợp A là tập hợp con của tập  có phần tử nhỏ
nhất bằng 1 và lớn nhất bằng 100 nên tập hợp A không vượt quá 100 phần tử.
Tổng quát, tập hợp A có 2  n  100 phần tử, sắp xếp các phần tử này theo thứ tự
1  x1  x2  x2    xn  100

x  a
• Theo đề bài có x  a  b với x, a, b đều là thuộc tập hợp A nên ta có  do đó mỗi

x  b

k  1,2,3, , n  1 ta có xk1  xi  x j  xk  xk  2 xk với 1  i , j  k.
• Áp dụng kết quả xk1  xi  x j  xk  xk  2 xk ta được
x2  1  1  2 ; x3  2  2  4 ; x4  8 ; x5  16 ; x6  32 ; x7  64 ,
suy ra tập A phải có ít nhất 8 phần tử.
• Giả sử n  8 theo thứ tự giả sử ta suy ra được x8  100 .
• Áp dụng các kiến thức trên, cùng cách tính toán giải phương trình bậc nhất.
+ Vì x6  x7  32  64  96  x8  2 x7  x7  50.
+ Vì x5  x6  16  32  48  x7  2 x6  x6  25.
25
+ Vì x4  x5  8  16  24  25  x6  2 x5  x5  (mâu thuẫn).
2
• Vì A là tập hợp có ít nhất 8 phần tử mà xét trường hợp có 8 phần tử cho kết quả mâu
thuẫn nên tập hợp A có ít nhất 9 phần tử.
Với n  9 theo thứ tự giả sử ta suy ra được x9  100 từ đó ta tìm được một tập hợp
1,2,3,5,10,20,25,50,100 thỏa mãn yêu cầu bài toán.
Đáp số: n  9 .

Liên hệ tài liệu word toán sđt và zalo: 039.373.2038 TÀI LIỆU TOÁN HỌC
158
Website:tailieumontoan.com

ĐỀ SỐ 18.

Câu 1.
1). Hệ phương trình đã cho tương đương với

xy  x  y  2

 3
x  y  6  27 x  y  9 xy 3 x  y
 3 3 3


 xy  x  y  2
xy  x  y  2
   
x 3  y 3  3 xy  x  y  3 x  y3  x  y3  3 x  y3
 
xy  x  y  2 x  y
     x  y  1.
x  y  3 x  y xy  x  y  2
 
Vậy nghiệm của hệ là x  y  1 .
Nhận xét: Bài toán sử dụng phép thế hằng số từ một phương trình vào phương trình còn
lại sau đó sử dụng hằng đẳng thức tìm nhân tử.
Nhắc lại kiến thức và phương pháp:
3
• Hằng đẳng thức bậc ba ax  by  a 3 x 3  3a 2 bx 2 y  3ab2 xy 2  b3 y 3 .
• Phương trình dạng f 3  x; y  g 3  x; y  f  x; y  g  x; y .
Ý tưởng: Ở cả hai phương trình của hệ, các biến x; y đều nằm trong các biểu thức bậc 3.
Và đặc biệt là cả hai phương trình cũng đều chứa hằng số. Vì vậy nếu thế hằng số này vào
hằng số của phương trình kia thì rõ ràng ta sẽ thu được một phương trình bậc ba đẳng cấp
của hai biến x; y .
Cụ thể như sau: 9 xy 3x  y  3xy  x  y  26 x 3  2 y 3 (*)
 x 3  3 x 2 y  3 xy 2  y 3  27 x 3  27 x 2 y  9 xy 2  y 3
3 3
  x  y   3 x  y  x  y  3 x  y  x  y .
Tuy nhiên, phương trình (*) là một phương trình đẳng cấp bậc ba nếu ta chia phương
trình cho y 3 , sẽ thu được một phương trình bậc ba. Ta có thể sử dụng máy tính cầm tay để
giải quyết phương trình đó. Với x  y , thế ngược lại phương trình một ta sẽ tìm được
nghiệm của hệ là x  y  1 .
Bài toán kết thúc.
Bài tập tương tự:

 y  2 x  y  3
1. Giải hệ phương trình:  2 .
x  4 xy  3  4 x 2  y 2

 2 15 15   2 15 15 

Đáp số:  x; y  0;  3 , 
 5
 ;  , 
5   5
; .
5 
x 2  y 2  2

2. Giải hệ phương trình:  .
 x  y1  xy4  32

Đáp số: x  y  1 .

Liên hệ tài liệu word toán sđt và zalo: 039.373.2038 TÀI LIỆU TOÁN HỌC
159
Website:tailieumontoan.com
2). Điều kiện 4  x  4.
x
Phương trình đã cho tương đương với
x4 2
 
4  x  2  2x .

+Với x  0 là nghiệm.
+ Giải 4x 2  2  x4 2 
Đặt u  x  4; v  4  x ta thu được

 v  2u  2

 2
2
 u2  2u  2  8  5u2  8u  4  0
 2
u  v  8

 2 14
u  ; v  2 96
 5 5  x4   x  (thỏa mãn).
 5 25
u  2 (l)
96
Vậy phương trình có hai nghiệm x  0; x   .
25
Nhận xét: Sử dụng phương pháp nhân liên hợp, sau đó đặt ẩn phụ tìm nghiệm của bài
toán.
Nhắc lại kiến thức và phương pháp:
ab
• Hằng đẳng thức a  b   a b  
a b  a b
a b
.

• Giải phương trình tổng quát dạng f  x  g  x  m



 f  x  g  x  m

 f  x ; g  x  0
2

  .
 f  x  g  x  2 m g  x  m2  f  x  g  x  m2  2  4 mg  x
  
Ý tưởng: Không khó để nhận thấy, phương trình có một nghiệm là x  0 . Đồng thời vế trái
của phương trình có xuất hiện biểu thức x  4  2 , dễ thấy rằng
2
x  
x  4  22   x4 2  
x  4  2 . Vì thế, phương trình đã cho tương đương với:
 x4 2  0  x  0
 x4 2  4x 2  2   x4 2  
x  4  2  
 4  x  2 x  4  2
Phương trình còn

lại có thể giải bằng cách tổng quát nêu ở trên, hoặc có thể giải quyết bằng cách đặt ẩn phự

v  4  x
như sau:  ( u; v  0 ). Ta có hệ phương trình

u  x  4



 v 2  u2  8 2 14 2 96

 u ; v  x4   x  .
 5 5 5 25
 v  2u  2

Bài toán kết thúc.
Bài tập tương tự:
1. Giải phương trình  x  1 1  
1  x  1  2x .
24
Đáp số: x  0; x  
25
2. Giải phương trình  x  9 1  
9  x  1  3x .
Đáp số: x  0 .
Câu 2.
Liên hệ tài liệu word toán sđt và zalo: 039.373.2038 TÀI LIỆU TOÁN HỌC
160
Website:tailieumontoan.com
2
1). +) 412  40  1  40 2  80  1  81 (mod 100).
414  812 (mod 100)  80 2  160  1 (mod 100)  61 (mod 100).
 415  61.41 (mod 100)  60.40  100  1 (mod 100)  1 (mod 100).
21
 415   41105  1 (mod 100)  41106  41 (mod 100).
503
+) 57 4  1 (mod 100)  57 2012  57 4   1 (mod 100).
Suy ra A  41106  57 2012  41  1 (mod 100).
Vậy 2 chữ số cuối cùng của A là 42.
Nhận xét. Muốn tìm hai chữ số tận cùng của một số (thường gặp là một lũy thừa) ta xét số
dư của số đó khi chia cho 100
Nhắc lại kiến thức và phương pháp.
2
• Hằng đẳng thức bình phương của một tổng a  b  a 2  2 ab  b2 .
2
Ta có 412  40  1  40 2  80  1  4.100  81  0  81  81 (mod 100).
• Tính chất của đồng dư thức: a  b (mod n )  ac  bc (mod n ) với a ; b ; c và n đều là
các số nguyên dương.
Ta có 412  81mod100  414  812 mod100
2
812  80  1  8.100  160  1  0  60  1  61mod100
• Tính chất của đồng dư thức: a  b mod n  ac  bc mod n với a , b , c và n đều là các
số nguyên dương.
Ta có 414  61 mod100  415  61.41  2501  1 mod100
21
 415   121  1 mod100  41105  1 mod100  41106  41 mod100 Hoàn toàn áp dụng
503
các kiến thức như trên ta có 57 4  1(mod100)  57 2012  57 4   1(mod100) .
• Tính chất đồng dư thức: a  b mod n và c  d mod n khi đó a  c  b  d mod n .
Ta có 41106  41mod100 và 57 2012  1 mod100 .
Khi đó 41106  57 2012  41  1  42 mod100 .
Vậy hai chữ số cuối cùng của số A  41106  57 2012 là 42 .

1 5
2). Tập xác định x .
2 2
x 2  5  4x 2 5  3x 2
Ta có x 5  4x 2   .
2 2
2x  1  1 3( x 2  1)
3 2x  1  3( )  3x  .
2 2
Cộng hai bất đẳng thức trên ta thu được
y  3 2x  1  x 5  4x 2  4.
Vậy ymax  4 khi x  1.
Nhận xét: bài toán sử dụng việc kết hợp đánh giá điểm rơi cũng như bất đẳng thức Cosi
để tìm giá trị lớn nhất của biểu thức ban đầu.
Nhắc lại kiến thức và phương pháp:
Liên hệ tài liệu word toán sđt và zalo: 039.373.2038 TÀI LIỆU TOÁN HỌC
161
Website:tailieumontoan.com

• Bất đẳng thức Cosi cho hai số thực dương a  b  2 ab .


• Giả sử m là điểm rơi của một bài toán, ta sử vận dụng các đánh giá quen thuộc xung
2
quanh điểm rơi là x 2  m2  2 mx   x  m  0 .
Ý tưởng: Biểu thức bài cho chứa hai căn thức bận hai, đồng thời yêu cầu là tìm giá trị nhỏ
nhất vậy nên ta sẽ đánh giá qua hai bước, đó là: sử dụng đánh giá cosi để khử căn thức,
khéo léo biến đổi theo điểm rơi để khử dần biến số. Vậy nên việc quan trọng nhất đó là dự
đoán điểm rơi của bài toán, tuy nhiên như đã nói ta cần sử dụng Cosi cho từng căn thức
một, nên ta sẽ đánh giá cho mỗi căn như sau:
32 x  1  
2 x  1    2  2 x  1  3 2 x  1 
2 

2
m x  n 5  4 x
2 2 2 2
 2
m 2
 4n2  x 2  5n2
mx.n 5  4 x   x 5  4x 
2 2 mn
32 x  1    m 2  4 n 2  x 2  5n 2
Do đó, suy ra P   . Bây giờ, ta sẽ có hướng tư duy là sử
2  2 mn
dụng đánh giá nào đó để khử hết biến. Biểu thức cuối có sự xuất hiện của x; x 2 do đó ta sẽ
nghĩ là nên đánh giá x về x 2 hay ngược lại. Và ta sẽ chọn giải pháp đầu tiên
x2  k 2 3 x 2  k 2  k  k m2  4n2  x 2  5n2
2 2
x  k  2 kx  2 x  . Khi đó, ta có: P   . Và nếu
2k 2k  2 mn
muốn khử được hết biến thì đầu tiên ta quan sát được là hai mẫu số phải bằng nhau và
tổng hệ số của x 2 bằng 0. Chính vì thế, ta được:

k   mn; 4n  m  3
2 2
m  n  k    1
  
2 x  1   ; m2 x 2  n2 5  4 x 2  x  1

Và từ đó, ta có đánh giá sau:
2x  1  1 x2  5  4x2
P  3 2 x  1  x 5  4 x 2  3. 
2 2
2 2 2
5  3x 3x  3 5  3x
 3x     4  Pmax  4 .
2 2 2
Bài toán kết thúc.
Bài tập tương tự:
1. Cho x; y; z là các số thực dương thỏa mãn xy  yz  zx  5 . Chứng minh rằng:
3 x 2  3 y 2  z 2  10 .
2. Chứng minh rằng với mọi 0  x  1 ta có bất đẳng thức sau:
9 x 1  x 2  13 x 1  x 2  16 .

Câu 3.

Liên hệ tài liệu word toán sđt và zalo: 039.373.2038 TÀI LIỆU TOÁN HỌC
162
Website:tailieumontoan.com

A
R

Q S
P

B C

N M

  TQA
1). Do TPA   90 0 , nên tứ giác TAPQ nội tiếp.
  QTP
Do đó MTC   QAP
 (do tứ giác TAPQ nội tiếp)  BAN
  MAC  (do MN  BC ), suy ra

tứ giác MTAC nội tiếp, suy ra T  (O) .


Nhận xét. Có nhiều cách để chứng minh một điểm nằm trên một đường tròn, trong đó có
cách áp dụng tính chất “Có một và chỉ một đường tròn đi qua ba điểm không thẳng
hàng”.
Nhắc lại kiến thức và phương pháp.
• Tứ giác có hai đỉnh liên tiếp cùng nhìn cạnh đối dưới hai góc bằng nhau là tứ giác nội
tiếp.
Tứ giác TAPQ có đỉnh P và Q cùng nhìn cạnh AT dưới hai góc TPA   TQA
  90 0 suy
ra TAPQ là tứ giác nội tiếp.
• Hai góc nội tiếp cùng chắn một cung của một đường tròn thì bằng nhau.
  QAP
QTP  hay MTC
  BAN
 (hai góc nội tiếp cùng chắn cung QP
 của đường tròn

ngoại tiếp tứ giác TAPQ ).


• Hai dây cung song song của một đường tròn tạo ra hai dây bị chắn hai bên bằng nhau
(Chứng minh dựa vào tính chất hình thang cân).
Trong đường tròn (O) có hai dây cung MN và BC song song nên ta có BN   MC.

• Hai góc nội tiếp chắn hai cung bằng nhau của một đường tròn thì bằng nhau.
 chắn cung BN
Trong đường tròn (O) có góc nội tiếp BAN  chắn
 và góc nội tiếp MAC
  MAC
 . Kết hợp với trên ta suy ra BAN
cung MC .
Áp dụng kiến thức trên, tứ giác MTAC có hai đỉnh liên tiếp T và A cùng nhìn cạnh
  MAC
MC dưới hai góc bằng nhau MTC  (  BAN ), suy ra MTAC là tứ giác nội tiếp.
• Có một và chỉ một đường tròn đi qua ba điểm không thẳng hàng.
Liên hệ tài liệu word toán sđt và zalo: 039.373.2038 TÀI LIỆU TOÁN HỌC
163
Website:tailieumontoan.com
Theo chứng minh trên ta có bốn điểm M ; T ; A ; C cùng thuộc đường tròn mà ba
điểm M ; A ; C cùng nằm trên đường tròn (O) , nên suy ra T cũng là điểm nằm trên
đường tròn (O) .

  PTA
2). Từ tứ giác TAPQ nội tiếp ta có PQA   CTA
  ABC
  PQ  BC  MN .

Từ đó Q  (1).
SA  NMA
  AMN
Mà tứ giác AMNR nội tiếp, suy ra ARN   180 0 (2).
 Q
Từ (1) và (2), suy ra QRA SA  180 0 , suy ra tứ giác ARQS nội tiếp, ta có điều phải chứng
minh.
Nhận xét. Có nhiều cách để chứng minh bốn cùng điểm nằm trên một đường tròn, trong
đó đưa về chứng minh bốn điểm tạo thành một tứ giác nội tiếp là thông dụng nhất.
Nhắc lại kiến thức và phương pháp.
Kiến thức nhắc lại trên
  PTA
PQA  (hai góc nội tiếp cùng chắn cung  của đường tròn ngoại tiếp tứ giác TAPQ ).
PA
  ABC
CTA  (hai góc nội tiếp cùng chắn cung  của đường tròn (O) )
AC
  PTA
Suy ra PQA   CTA  ABC.

• Một đường thẳng cắt hai đường thẳng đã cho tạo ra hai góc ở vị trí đồng vị bằng nhau
thì hai đường thẳng đã cho song song.
  ABC
Theo chứng minh trên ta có PQA  mà hai góc này ở vị trí đồng vị của hai đường
thẳng QS và BC suy ra QS  BC kết hợp với giải thiết MN  BC ta có QS  BC  MN .
• Một đường thẳng cắt hai đường thẳng song song tạo ra cặp góc đồng vị bằng nhau.
  AMN
Theo chứng minh trên ta có QS  MN nên hai góc ở vị trí đồng vị ASQ .

• Tứ giác nội tiếp có tổng hai góc đối diện bằng 180 .
  AMN
Tứ giác AMNR nội tiếp đường tròn (O) nên ta có ARN   180 0 . Kết hợp với trên,
  ASQ
ta có ARN   180 0 .

• Tứ giác có tổng hai góc đối diện bằng 180 là tứ giác nội tiếp.
Tứ giác ARQS có ARN  ASQ
  180 0 suy ra ARQS là tứ giác nội tiếp hay bốn điểm A ;

R ; Q ; S cùng thuộc một đường tròn.

Câu 4. Giả sử các số của tập hợp X được sắp theo thứ tự (đánh số lại)
x1  x2    xn .
Ta có x1  x2  x1  x3    x1  xn  x2  xn  x3  xn    xn1  xn , suy ra đối với một tập n

số thực phân biệt bất kỳ ta luôn có ít nhất n  1  n  2  2n  3 giá trị phân biệt của các

tổng xi  x j .

Vậy C X   2n  3 .

Xét tập X1  1; 2; ; n , khi đó với mọi 1  i  j  n

Liên hệ tài liệu word toán sđt và zalo: 039.373.2038 TÀI LIỆU TOÁN HỌC
164
Website:tailieumontoan.com
xi  x j  i  j  3; 4; ; 2n  1  C X1   2n  3

Vậy C X   2n  3 .


min

nn  1 nn  1
Số các tổng xi  x j ( 1  i  j  n ) bằng , suy ra C X   .
2 2

Xét tập X2  2; 2 2 ;; 2n  , thì với mọi 1  i  j  n

xi  x j  2 i  2 j .

Giả sử tồn tại 1  r  s  n : xr  xs  xi  x j  2r  2 s  2i  2 j

2 r |2 i nn  1
 2 1  2
r sr
  2 1  2
i j i
  2i |2r  r  i  s  j  C X2   2 .


nn  1
Vậy C X   .
  max 2

ĐỀ SỐ 19.
Câu 1.
1). Cộng hai phương trình của hệ ta thu được
3
x 3  y 3  6 xy  8  x 3  y 3  2  3 xy 2  0


  x  y  2 x 2  y 2  4  xy  2 y  2 x  0 
2
Ta luôn có x 2  y 2  2  xy  2 y  2 x đẳng thức xảy ra khi và chỉ khi x  y  2 .
Vậy nếu x 2  y 2  4  xy  2 y  2 x  0 ta suy ra x  y  2 (loại) vì không thỏa mãn phương
trình 7 xy  y  x  7 .

x  y  2  y  2  x
Vậy thu được hệ  .

7 xy  y  x  7

x  1  y  1

2
Suy ra 7 x 2  x  2  2 x  7  7 x  12 x  5  0   .
x  5  y  9
 7 7
Nhận xét: Bài toán sử dụng phương pháp thế (hay cộng vế) để ra được phương trình có
mối liên hệ giữa các biến. Sau đó thế ngược lại tìm nghiệm của hệ phương trình.
Nhắc lại kiến thức và phương pháp:
• Tổng các đại lượng không âm:
2 2 2
a  b  b  c  a  c  0  a2  b2  c 2  ab  bc  ca
• Đẳng thức:

a3  b3  c 3  3abc  a  b  c a2  b2  c 2 ab  bc  ca 
Ý tưởng: Cả hai phương trình của hệ, đều xuất hiện nhân tử x  y vì thế ta sẽ nghĩ đến
chuyện thế x  y từ phương trình một vào phương trình hai (hoặc ngược lại), do đó ta có
được x 3  y 3  6 xy  8  0 ( i ). Đến đây ta mong muốn sẽ biểu diễn mối quan hệ giữa x , y ,

Liên hệ tài liệu word toán sđt và zalo: 039.373.2038 TÀI LIỆU TOÁN HỌC
165
Website:tailieumontoan.com
3
quan sát phương trình ( i ), ta thấy rằng 2  8 và 6 xy  3xy 2 do đó nếu đặt z  2
thì ( i )  x 3  y 3  z 3  3xyz  0 . Một biểu thức đối xứng rất đẹp, bằng cách nhóm nhân tử,
ta có:
3
x 3  y 3  z 3  3 xyz  0   x  y  z 3  3 xy  x  y  z  0

 
  x  y  z x 2  2 xy  y 2  xz  yz  z 2  3 xy  x  y  z  0


  x  y  z x 2  y 2  z 2  xy  yz  xz  0 (*). 
1 2 2 2
Dễ thấy x 2  y 2  z 2  xy  yz  xz   x  y   y  z   z  x  do đó, phương trình
2  
x  y  z  0 x  y  2
   . Công việc còn lại là thay z  2 suy ra  . Nhưng x  y  2
x  y  z  x  y  2
loại vì không thỏa mãn phương trình hai trong hệ. Với x  y  2 thay xuống phương trình
5 9 
hai, ta tìm được nghiệm của hệ phương trình là  x; y  1; 1 ,  ; .
  7 7 
Bài toán kết thúc.
Bài tập tương tự:
x 3  y 3  1  3 xy
1. Giải hệ phương trình  2 .
x  y 2  x  2 y  3  0

 1  33 5  33   1  33 5  33 
Đáp số:  x; y  1; 1 ,  ;  , 
  4
;  .

 4 4   4 

x 3  y 3  x  2 xy  3
2. Giải hệ phương trình  .

x  xy  4

Đáp số: vô nghiệm
2). Điều kiện 1  x  1 .
Phương trình tương đương với
2
2  x 1   1  x2  x  1  1  x  2 x  1

  
x  1  1 x  2 x  1  2  x  1  x  1  1 x    x  1  1 x  2  
x  1 1  0 .

+ Giải x  1  1  x  2  2  2 1  x2  4  1  x2  1  x  0 .
+ Giải x 1 1 x  0 .
Đáp số x  0 .
Nhận xét: bài toán sử dụng phương pháp nhóm nhân tử chung, sau đó nâng lũy thừa bậc
hai để tìm nghiệm của phương trình.
Nhắc lại kiến thức và phương pháp:
 f  x  0
• Giải phương trình f  x.g  x  0   .
 g  x  0

• Giải phương trình a  f  x  a  f  x  b

Liên hệ tài liệu word toán sđt và zalo: 039.373.2038 TÀI LIỆU TOÁN HỌC
166
Website:tailieumontoan.com
a  f  x  a a  f  x  a
 
   .
2 a  2 a  f  x  b
2 2 2 4  a 2  f 2  x  b2  2 a2
   
Ý tưởng: Bài toán xuất hiện ba căn thức, nhưng có điều đặc biệt ở đây là căn thức còn lại là
tích của hai căn thức kia. Mặt khác 1  x 2 , 1  x có sự đồng nhất hệ số, do đó ta sẽ nhóm

hai căn này lại nên ta được nhân tử chung như sau: 1  x2  1  x  1  x  
x  1  1 . Và

ta mong muốn biểu thức x  3  3 x  1 sẽ phân tích được biểu thức có chứa x  1  1.
Thật vậy, nếu coi h  x  x  3  3 x  1 là một phương trình bậc hai ẩn x  1 ta sẽ thấy:

h  x  x  1  3 x  1  2   x 1 1  
x  1  2 . Chính vì thế bài toán của ta được giải

quyết như sau:


x  3  1  x2  3 x  1  1  x
  x 1 1  
x  1  2  1 x  
x 1 1  0
 x11
  x 1 1  x  1  1  x  2  0   
 x  1  1  x  2
.

Phần còn lại chỉ là việc bình phương các phương trình và tìm nghiệm như ở trên đã nêu.
Ta được nghiệm của phương trình là x  0.
Bài toán kết thúc.
Bài tập tương tự:
1. Giải phương trình x  4  4  x 2  3 x  2  2  x .
Đáp số: x  1 .
2. Giải phương trình 2 x  3  1  4 x 2  3 2 x  1  1  2 x .
Đáp số: x  0 .
Câu 2.
1). Nhận xét: a; b là các số nguyên thỏa mãn a2  b2  3 thì a; b 3 thật vậy, vì
a 2  0,1mod 3 ; b2  0,1mod 3 .

 a 2  0 mod 3
2 2 

suy ra a  b  0 mod 3    a , b 3 .

b 2
 0  mod 3


  
Phương trình tương đương với 6 x 2  9 y 2  x 2  y 2  28  9 3 . 
 2
x  0 mod 3

suy ra x 2  y 2  0 mod 3    x  3 x1 ; y  3 y1 ( x1 ; y1   ).

 y 2
 0  mod 3


Thay vào phương trình ta thu được 5  9 x12  8  9  y12  28  9 3
 5  x12  8  y12  28  9 2 .
Lập luận tương tự ta thu được x1  3x2 ; y1  3 y2 ( x2 ; y2   ).
Và nhận được phương trình 5  9 x22  8  9  y22  28  9 2

Liên hệ tài liệu word toán sđt và zalo: 039.373.2038 TÀI LIỆU TOÁN HỌC
167
Website:tailieumontoan.com

 5  x22  8  y22  28  9 .
Tương tự ta có x2  3x3 ; y2  3 y3 ( x3 ; y3   ) và thu được 5  x32  8  y32  28 .
28
Từ phương trình suy ra y32   22 .
8
 2 28
 y3  0  x32 
Suy ra  5  x32  2 2 ; y32  1 .
 y 2  1  x2  22
 3 3

 x22  9  2 2 ; y22  9 .
 x12  9 2  2 2 ; y12  9 2  x 2  9 3  2 2 ; y 2  9 3 .
Đáp số: x  2  33 ; y  33 , x  2  33 ; y  33 , x  2  33 ; y  33 , x  2  33 ; y  33 .
Nhận xét. Bài toán nghiệm nguyên giải bằng phương pháp xét số dư hay đồng dư thức
Nhắc lại kiến thức và phương pháp.
• Một số chính phương chia hết cho 3 chỉ tồn tại dư 0 hoặc 1. Do đó tổng của hai số
chính phương chia cho 3 dư 0 hoặc dư 2. Nên a; b là các số nguyên thỏa mãn a2  b2  3
thì a; b  3 ,
a 2  0 mod 3

2 2 

suy ra a  b  0 mod 3    a , b 3 .

b 2
 0  mod 3


• Một số chia hết cho 3 có dạng x = 3 k .
  
Phương trình tương đương với 6 x 2  9 y 2  x 2  y 2  28  9 3 . 
Suy ra

 x 2  0 mod 3
2 2 

x  y  0 mod 3    x  3 x1 ; y  3 y1 ( x1 ; y1   ).

 y 2
 0  mod 3


Thay vào phương trình ta thu được 5  9 x12  8  9  y12  28  9 3
 5  x12  8  y12  28  9 2 .
Lập luận tương tự ta thu được
x1  3 x2 ; y1  3 y2 ( x2 ; y2   ).
Và nhận được phương trình 5  9 x22  8  9  y22  28  9 2
 5  x22  8  y22  28  9 .
Tương tự ta có x2  3x3 ; y2  3 y3 ( x3 ; y3   ) và thu được 5  x32  8  y32  28 .
• Hai số hạng không âm luôn nhỏ hơn hoặc bằng tổng.
28
Từ phương trình suy ra y32   22 ,
8
 2 28
 y3  0  x32 
suy ra 
 5  x32  2 2 , y32  1  x22  9  2 2 ; y22  9
 y 2  1  x2  22
 3 3

 x12  9 2  2 2 ; y12  9 2  x 2  9 3  2 2 ; y 2  9 3 .
Đáp số: x  2  33 ; y  33 , x  2  33 ; y  33 , x  2  33 ; y  33 , x  2  33 ; y  33 .

Liên hệ tài liệu word toán sđt và zalo: 039.373.2038 TÀI LIỆU TOÁN HỌC
168
Website:tailieumontoan.com

2 1
2). Ta có P  1  x 2 y 2  2 xy  .
xy xy

 x  y 2 1
Đặt t  xy     .
 2  4
P 1 1 15 17
Ta thu được  t   16t   15t  2 16    P  17 .
2 t t 4 2
Dấu “=” xảy ra khi
1
xy  Pmin  17 .
2
Nhận xét: Bài toán sử dụng bất đẳng thức Cosi kết hợp với giả thiết tìm giá trị nhỏ nhất
của thức bài cho.
Nhắc lại kiến thức và phương pháp:
• Bất đẳng thức Cosi cho hai số thực dương a  b  2 ab .
2
1 1 2  a  b 
• Các hệ quả từ bất đẳng thức Cosi   ; ab    .
a b ab  2 
Ý tưởng: Bài toán có sự đối xứng giữa hai biến x; y ( vai trò của chúng như nhau ) vì thế
1
điểm rơi ta khẳng định là x  y , kết hợp với giả thiết suy ra được tại x  y  thì Pmin . Mặt
2
khác, xét với biểu thức P , có xuất hiện đại lượng xy trong căn, vì vậy ra nghĩ đến bất
1 1
đẳng thức Cosi để đánh giá đại lượng  về f  xy .
x y
1 1 2 2 1  x2 y 2 1
Ta có   P 2  xy
x y xy xy xy
Và khai thác giả thiết, ta cũng sẽ đánh giá về xy , với điểm rơi x  y thì ta có đánh giá
2
2  x  y  1
 x  y  0  xy     . Và nếu đặt t  xy thì ta cần tìm giá trị nhỏ nhất của biểu

 2  4
1 1
thức P  2  t với t  . Bằng sự khéo léo trong chọn điểm rơi, ta đánh giá như sau:
t 4
1 1 15
2 16t   15t  2 2 16t.  15t  2 2 16   17
t t 4
 Pmin  17 .
1
Dấu “=” xảy ra khi x  y  .
2
Bài toán kết thúc.
Bài tập tương tự:
1. Cho x; y là các số thực dương thỏa mãn x  y  2 . Tìm giá trị nhỏ nhất của biểu thức
1 1
P     1  x 2 y 2 .
 x y 

Liên hệ tài liệu word toán sđt và zalo: 039.373.2038 TÀI LIỆU TOÁN HỌC
169
Website:tailieumontoan.com
2. Cho x; y là các số thực dương thỏa mãn x  y  2 . Tìm giá trị nhỏ nhất của biểu
4x 2 y 2 x2 y2
thức P  2
  2 .
x 2
 y2  y2 x

Câu 3.

M
Q
Q M
N N E
E F O
F
O H
H P
P
B C
B C K

1). Ta có   BHC
BPC   180  BAC   BEC
 , suy ra tứ giác AEPF nội tiếp, nên BFC   1800 .

Mặt khác từ các tứ giác AQFN ; AQEM nội tiếp ta có


  MQA
MQN   NAQ
  MEA
  NFA
  1800 .

Vậy M ; N ; Q thẳng hàng.


Nhận xét. Từ ba điểm bất kỳ tạo thành hai tia có chung gốc. Khi hai tia đó tạo với nhau
một góc tù thì ba điểm đã cho thẳng hàng.
Nhắc lại kiến thức và phương pháp.
• Hai góc nội tiếp cùng chắn một cung của một đường tròn thì bằng nhau.
+ BPC  (hai góc nội tiếp cùng chắn cung BC
  BHC  của đường tròn ngoại tiếp tam giác

HBC ).
  AFN
+ AQN  (hai góc nội tiếp cùng chắn cung AN
 của đường tròn ngoại tiếp tam

giác FAN ).
  AEM
+ AQM  (hai góc nội tiếp cùng chắn cung AM
 của đường tròn ngoại tiếp tam

giác AME ).
• Tứ giác có tổng hai góc đối diện bằng 180 là tứ giác nội tiếp. Tứ giác nội tiếp có tổng
hai góc đối diện bằng 180 .
Gọi chân đường cao từ đỉnh B và C của tam giác ABC là E và F  .
 
Tứ giác AE HF  có AE H  AF  H  90  90  180 , suy ra AE HF  là tứ giác nội tiếp, khi
   mà F  HE  BHC 
 (hai góc đối đỉnh) nên
đó F  AE  F  HE  180  F  HE  180  BAC

Liên hệ tài liệu word toán sđt và zalo: 039.373.2038 TÀI LIỆU TOÁN HỌC
170
Website:tailieumontoan.com
  180 BAC
BHC  kết hợp với trên ta được BPC
  180 BAC   BAC
  BPC   180 suy ra

tứ giác AEPF là tứ giác nội tiếp.


  AEP
Suy ra AFP   180  180 AFP

  180 AEP
  180
  
  BEC
 BFC   180  NFA
  AEM
  180 kết hợp với trên, ta có
  AQM
NAQ   180  NQM
  180 hay ba điểm N ; Q; M thẳng hàng.

  ANQ
2). Ta có các góc nội tiếp bằng nhau AFQ   ANM
  ABM
 suy ra FQ  BE . Tương tự

EQ  CF .
  QFP
Từ đó tứ giác EQFP là hình bình hành, suy ra QAN   QEP
  QAM
 hay AQ là phân

.
giác MAN
 thì A , P , Q thẳng hàng.
Nếu AP là phân giác MAN
  QAC
Từ đó nếu PQ giao BC tại K thì KAC   QME
  NMB
  PCK

Vậy AKC ∽ CKP , suy ra KC 2  KP.KA .


Tương tự KB2  KP.KA .
Từ đó KB  KC hay K là trung điểm.
Nhận xét. Chứng minh một đường thẳng đi qua trung điểm của một đoạn thẳng khác ta
chứng minh giao điểm của đường thẳng với đoạn thẳng là trung điểm của đoạn thẳng.
Nhắc lại kiến thức và phương pháp.
• Một đường thẳng cắt hai đường thẳng khác tạo ra cặp góc đồng vị bằng nhau thì hai
đường thẳng song song.
  ANQ
Từ kiến thức ở trên, ta có AFQ   ANM   ABM mà AFQ
 và ABM
 ở vị trí đồng

vị của hai đường thẳng FQ và BE nên suy ra FQ  BE . Hoàn toàn tương tự ta có


EQ  CF .
• Tứ giác có hai cặp cạnh song song là hình bình hành.
Tứ giác EQFP có FQ  BE và EQ  CF suy ra EQFP là hình bình hành khi đó
  QFP
QAN   QEP
  QAM
 suy ra AQ là phân giác MAN
 . Do đó A , Q , P thẳng hàng.
  QME
KAC  (hai góc nội tiếp cùng chắn cung QE
 của đường ngoại tiếp tam giác

AME ).
  PCK
QME  (hai góc nội tiếp cùng chắn cung NB
 của đường tròn O ),

  PCK
 KAC  mà lại có góc PKC
 chung của hai tam giác AKC và CKP nên suy ra
AKC ∽ CKP (g – g)
AK KC
   KC 2  AK.KP hoàn toàn tương tự ta có KB2  KP.KA . Từ đó suy ra
CK KP
KC 2  KB2  KC  KB hay K là trung điểm của BC .

Câu 4.
Giả sử k là chỉ số mà x1  x2    xk  0  xk1    x192 .
Ký hiệu S  x1  x2    xk ; S  xk1  xk2    x192

Liên hệ tài liệu word toán sđt và zalo: 039.373.2038 TÀI LIỆU TOÁN HỌC
171
Website:tailieumontoan.com
2013
 S  S  0  S  S  2013  S  S  .
2
Do x1  x2    x192 suy ra S  kx1 ; S  192  k x192
S S S S
 x1    x1  ; x192 
k k k 192  k
 
S S 2013 2013 2013  192
 x192  x1      .
192  k k 2 192  k  2k 2 k 192  k
 192  k  k  2 192 2
Ta có 2 k 192  k  2  
  2
 2 
2013  192 2013
 x192  x1   .
192 2 96
2
Dấu “=” xảy ra khi
2013 2013
x1  x2    x96   ; x97  x98    x192 
192 192

ĐỀ SỐ 20.
Câu 1.
1). Dễ thấy đẳng thức sau đúng với a  b .
b b 2 b 2 b b 2b 2
  , suy ra   .
a  b a  b a2  b2 a  b a  b a2  b2
Do đó đẳng thức đã cho tương đương với
y 2y2  y2 2 y 4   4
2 y 8  8 y8
  y
  2     4     4
x  y x2  y 2  x 2  y 2 x 4  y 4   x 4  y 4 x8  y 8  x8  y 8
y
  4  y  4 x  4 y  5 y  4 x , điều phải chứng minh.
xy
Nhận xét: Bài toán sử dụng đẳng thức (bổ đề luôn đúng) để ghép vào biểu thức đã cho để
chứng minh.
Ý tưởng: Trước hết, xét ở đẳng thức cần chứng minh ta có 5 y  4 x vì thế ta sẽ tách thành
y
y  4x  4 y   4 . Việc tách này là có cơ sở vì số 4 xuất hiện ở vế phải của giả thiết,
xy
y
cũng như các mối liên hệ giữa mẫu số các phân thức ở vế trái. Với 4  thế ngược lại
xy
y 2y2 4y4 8 y8 y
giả thiết của bài toán, tức là ta sẽ cần chứng minh  2   
xy x y 2 4
x y 4 8
x y 8
xy
(*).
Đẳng thức (*) hoàn toàn có thể chứng minh bằng phương pháp biến đổi tương đương đó
y y 2y2 4y4 8 y8
là:   2   0
x  y x  y x  y 2 x 4  y 4 x8  y 8
2y2 2y2 4y4 8 y8
    0
x2  y 2 x2  y 2 x4  y 4 x8  y 8

Liên hệ tài liệu word toán sđt và zalo: 039.373.2038 TÀI LIỆU TOÁN HỌC
172
Website:tailieumontoan.com
4y4 4y4 8 y8 8 y8 8 y8
    0    0.
x4  y 4 x4  y 4 x8  y 8 x8  y 8 x8  y 8
Với cách biến đổi trên, để làm xuôi ngược lại thì ta đã sử dụng một bổ đề đẳng thức rất
b b 2 b 2
đẹp đó là   2 .
a  b a  b a  b2
Bài toán kết thúc.


 x  y2 x  3 y  12
2). Hệ đã cho tương đương với  ,
 


6 x  y  xy  x  y  12
x  y  0
suy ra  x  y2 x  3 y   x  y6  xy  
 2 x  3 y  6  xy. (loại)
x  3
Ta có 2 x  3 y  6  xy   x  3 y  2  0   .
y  2
+ Với x  3 , thay vào phương trình đầu của hệ ta có
 y  1
18  3 y 2  3 y  12   .
y  2

+ Với y  2 , thay vào phương trình đầu của hệ ta có
x  3
2 x 2  2 x  12  12   .
 x  4

Vậy hệ có nghiệm  x; y  3;  1 , 3; 2 , 4; 2 .
Nhận xét: Bài toán sử dụng phương pháp thế hằng số ở cả hai phương trình, sau đó
phương trình thu được phân tích thành nhân tử và thế ngược lại một trong hai phương
trình của hệ tìm nghiệm của hệ phương trình.
Ý tưởng: Thoạt nhìn, ta sẽ nghĩ đến hướng xét delta ẩn x hoặc y ở phương trình thứ hai
của hệ và mong muốn đenta chính phương. Nhưng hướng đi này sẽ thất bại, vì dễ thấy
cũng từ phương trình hai, ta tách được rằng 6  x  y  xy  x  y  12   x  y xy  6  12 .
Mặt khác, xét vế trái của phương trình một nếu coi đây là một phương trình đẳng cấp bậc
hai, ta sẽ có được 2 x 2  3 y 2  xy   x  y2 x  3 y . Khi đó hệ phương trình đã cho tương

 x  y2 x  3 y  12
đương với  .



 x  y xy  6  12
Đây là một hệ rất đẹp vì nhân tử x  y; 12 đều xuất hiện ở cả hai phương trình, chính vì
thế suy ra:
x  y
 x  y2 x  3 y   x  y xy  6  
 2 x  3 y  xy  6 (*)
x  3
Với (*), dễ thấy nhân tử như sau: (*)   x  3 y  2  0   .
y  2
Việc còn lại là thế ngược lại tìm nghiệm của hệ phương trình.
Bài toán kết thúc.

Câu 2.

Liên hệ tài liệu word toán sđt và zalo: 039.373.2038 TÀI LIỆU TOÁN HỌC
173
Website:tailieumontoan.com
1). Do x; y là các số nguyên lớn hơn 1 nên x; y  2
 4 xy  1  7 x  7 y  4 xy  1
 4 x 2 y 2  4 xy  1  4 x 2 y 2  7 x  7 y  4 x 2 y 2  4 xy  1
2 2
 2 xy  1  4 x 2 y 2  7 x  7 y  2 xy  1 .
Mà 4 x 2 y 2  7 x  7 y là số chính phương và 1  2 xy  1  2 xy  1 ;
2
nên ta có 4 x 2 y 2  7 x  7 y  2 xy  x  y , điều phải chứng minh.
Nhận xét. Bài toán chứng minh đẳng thức từ những điều kiện đã cho.
Nhắc lại kiến thức và phương pháp.
• Xét x  y
x  y  0 7 x  7 y  0
+ Với x  y ta có     7 x  7 y  4 xy  1 .
xy  0 4 xy  1  0
+ Với x; y  2 ta có 4 xy  1  8 x  1  7 x  x  1  7 x  y  1  7 x  7 y
Suy ra 4 xy  1  7 x  7 y  4 xy  1
 4 x 2 y 2  4 xy  1  4 x 2 y 2  7 x  7 y  4 x 2 y 2  4 xy  1
2 2
 2 xy  1  4 x 2 y 2  7 x  7 y  2 xy  1
+ Bình phương của hai số nguyên có giá trị tuyệt đối là hai số tự nhiên liên tiếp được
2 2
gọi là hai số chính phương liên tiếp. Giữa n 1 và n  1 có duy nhất số chính
phương n2 với n là số nguyên.
2 2 2
+ Ta có giữa 2 xy  1 và 2 xy  1 có duy nhất số chính phương 2xy mà
2 2
2 xy  1  4 x 2 y 2  7 x  7 y  2 xy  1 suy ra
2
4 x 2 y 2  7 x  7 y  2 xy  4 x 2 y 2  7 x  7 y  4 x 2 y 2
 7 x  7 y  0  x  y (trái với điều kiện x  y )
• Xét y  x
Chứng minh hoàn toàn tương tự như trên, ta được x  y (trái với điều kiện y  x ).
Mối quan hệ giữa các số nguyên a  b hoặc a  b hoặc a  b .
Ta có x  y và y  x đều không tồn tại nên chỉ có x  y (điều phải chứng minh).

2). Ta có x 3  y 3  x 2  y 2  xy
  
  x  y x 2  y 2  xy  x 2  y 2  xy   x  y  1 x 2  y 2  xy  0 
 x 2  y 2  xy  0  x  y  0
   .

 x  y  1  0 x  y  1
5
+ Với x  y  0  P  .
2
+ Với x  y  1  0  x; y  1 ,
1 1 2 1
suy ra P    4 , Dấu “=” xảy ra khi và chỉ khi x  1; y  0 .
2 0 1 0

Liên hệ tài liệu word toán sđt và zalo: 039.373.2038 TÀI LIỆU TOÁN HỌC
174
Website:tailieumontoan.com

1 0 2 04
P   , Dấu “=” xảy ra khi và chỉ khi x  0; y  1 .
2  1 1 1 3
4
Vậy Pmax  4  x  1; y  0 và Pm in   x  0; y  1 .
3
Nhận xét: Khai thác giả thiết cũng như biểu thức bài cho, tìm điều kiện chặn của biến để
tìm giá trị nhỏ nhất, giá trị lớn nhất của biểu thức.
Nhắc lại kiến thức và phương pháp:
• Hằng đẳng thức a3  b3  a  ba2  ab  b2 
m  p f  x
• Xét biểu thức P  , với 0  a  f  x  b , ta có:
n  q f  x
mp a mp a mp b mp b
P  Pmin  ; P  Pmax  .
nq b nq b nq a nq a
Ý tưởng: Đi từ giả thiết của bài toán, sự xuất hiện của x 3  y 3 làm ta nghĩ đến hằng đẳng
thức x 3  y 3   x  y x 2  xy  y 2  , khi đó giả thiết trở thành:
x 3  y 3  x 2  xy  y 2   x  y x 2  xy  y 2   x 2  xy  y 2
x  y  0
  x  y  1 x 2  xy  y 2   0  
x  y  1

Ta chỉ cần xét với x  y  1 , mặt khác kết hợp với điều kiện ở biểu thức ta có được điều

0  x  1
kiện chặn của x , y là 0  x , y  1   .

0  y  1


Suy ra
1 
 
2 
x 1  x 2
 1 x 2 x 4
•  ; P   .
 y 3  y 2 2 y 1 y 3
2 

 1 


1 
 2 
x 2  x 3
  1 x 2 x
•  ; P  4.
 y 2  y 1 2 y 1 y
2 

 1 


4
Do đó Pmax  4  x  1; y  0 và Pm in   x  0; y  1 .
3
Bài toán kết thúc.

Câu 3.

Liên hệ tài liệu word toán sđt và zalo: 039.373.2038 TÀI LIỆU TOÁN HỌC
175
Website:tailieumontoan.com

F O
E
P

B D C

K
  AEB
1). Ta có AFC   ADC
  ADB
  180 suy ra tứ giác AEPF nội tiếp.
Nhận xét. Để chứng minh bốn điểm cùng thuộc một đường tròn ta đưa về chứng minh tứ
giác AEPF bằng cách chỉ ra tứ giác này có tổng hai góc đối diện bằng 180 .
Nhắc lại kiến thức và phương pháp.
• Các góc nội tiếp cùng chắn một cung của một đường tròn thì bằng nhau.
  ADC
+ AFC  (hai góc nội tiếp cùng chắn cung AC của đường tròn ngoại tiếp tam giác
ACD ).
  ADB
+ AEB  (hai góc nội tiếp cùng chắn cung AB của đường tròn ngoại tiếp tam giác
ABD ).
• Tứ giác có tổng hai góc đối diện bằng 180 là tứ giác nội tiếp.
  ADB
ADC   180 (hai góc kề bù) nên AFC   180 , suy ra tứ giác AEPF là tứ
  AEB
giác nội tiếp hay bốn điểm A ; E ; P ; F cùng thuộc một đường tròn.

 = LFC
2). Từ tứ giác AEPF nội tiếp, suy ra AEB  (1).
 = FCB
Ta lại có FCL   BCL   BAQ
 = PBC  = DAE
  BAQ = BAE
 (2).

Từ (1) và (2), suy ra FCL ∽ EAB .


Nhận xét. Nhớ lại các trường hợp đồng dạng của hai tam giác kết hợp với dữ kiện đề bài
cùng các ý đã chứng minh được từ ý trên để tìm ra hướng chứng minh của bài toán. Đối
với bài toán này, ta chứng minh hai tam giác đồng dạng theo trường hợp “góc - góc”
Nhắc lại kiến thức và phương pháp.
• Tứ giác nội tiếp có góc trong bằng góc ngoài tại đỉnh đối diện.
  LFC
Tứ giác AEPF là tứ giác nội tiếp (chứng minh trên) nên AEB 
• Các góc nội tiếp cùng chắn một cung của một đường tròn thì bằng nhau.

Liên hệ tài liệu word toán sđt và zalo: 039.373.2038 TÀI LIỆU TOÁN HỌC
176
Website:tailieumontoan.com
  BAQ
+ BCL  (hai góc nội tiếp cùng chắn cung BQ của đường tròn (O) ).
  EBC
+ EAD  (hai góc nội tiếp cùng chắn cung DE của đường tròn ngoại tiếp tam giác
ABD ).
• Tam giác cân có hai góc kề đáy bằng nhau.
  PCB
Tam giác PBC có PB  PC nên tam giác PBC cân tại P suy ra PBC .
  FCB
Kết hợp lại, ta được BCL   FCL
  QAE
  ABQ   BAE
.

• Hai tam giác có hai góc tương ứng bằng nhau thì đồng dạng.
Xét FCL và EAB có: AEB   LFC (chứng minh trên) và FCL
  BAE
 (chứng minh
trên), suy ra FCL ∽ EAB (g – g).
FL FC
3). Từ FCL ∽ EAB , suy ra = hay FL.EA = FC.EB (3).
BE AE
Chứng minh tương tự EK.FA = FC.EB (4).
FL EK
Từ (3) và (4), suy ra FL.EA = EK.FA hay = , suy ra EF  KL .
FA EA
 = ALK
Ta lại có QLK   ALQ
  AFE  ABE
  APE
  ABE
  PAB.
  PAC
Tương tự ta có QKL .
  PAB
Suy ra QKL   QLK
  PAC
.

Nhận xét. Với các bài toán chứng minh hai tổng bằng nhau, ta dựa vào mối quan hệ giữa
các góc để đưa các vế cùng bằng một lượng nào đó.
Nhắc lại kiến thức và phương pháp.
• Tính chất hai tam giác đồng dạng .
 FL FC
 =  FL. AE  FC.EB
FCL ∽ EAB   BE AE .
  
FLC  ABE
Chứng minh hoàn toàn tương tự ta có EK.FA = FC.EB
• Định lý Ta-lét đảo.
FL EK
Từ chứng minh trên, ta được FL.AE  EK.FA  = , suy ra EF  KL .
FA EA
• Đường thẳng cắt hai đường thẳng song song tạo ra các góc đồng vị bằng nhau.
  ALK
EF  KL (chứng minh trên), suy ra AFE .

• Các góc nội tiếp cùng chắn một cung của một đường tròn thì bằng nhau.
  APE
AFE  (hai góc nội tiếp cùng chắn cung AE của đường tròn ngoại tiếp tứ giác
AFPE ).
Kết hợp các ý chứng minh trên ta có được điều đã trình bày trong bài giải trên
 = ALK
QLK   ALQ
  AFE
  ABE
  APE  ABE  . Hoàn toàn chứng minh tương tự ta
  PAB
  PAC
có được QKL  . Từ đó ta có điều cần chứng minh QKL
  PAB
  QLK
  PAC
.

Câu 4.
Từ giả thiết dễ thấy m tập con thuộc dãy là phân biệt. Vì A có 31 phần tử nên số tập con
31  30
có đúng 2 phần tử của A là  Ký hiệu ak 2  k  31 là số các tập có đúng k phần
2

Liên hệ tài liệu word toán sđt và zalo: 039.373.2038 TÀI LIỆU TOÁN HỌC
177
Website:tailieumontoan.com
tử, nằm trong dãy đã cho, suy ra m  a2  a3    a31 . Xét một tập hợp có k phần tử suy
k  k  1 k  k  1
ra số các tập con có 2 phần tử của tập đó là  ak tập này sẽ có ak tập con 2
2 2
phần tử. Mà theo giả thiết với 2 phần tử bất kỳ của A thì chúng không thể đồng thời
thuộc 2 tập có k phần tử của dãy  các tập con 2 phần tử nói trên là phân biệt.
k  k  1 31  30 1
Suy ra ak   ak  31  30 
2 2 k  k  1
 1 1 1 
 a2  a3    a31  31.30    
 1.2 2.3 30.31
 1 1 1 1 1
 m  31.30 1        .
 2 2 3 30 31
Vậy m  900, điều phải chứng minh.
Nhận xét. Bài toán về phần nguyên
Nhắc lại kiến thức và phương pháp.
• Bài toán cách chọn số từ một tập hợp số: “Trong n số bất kỳ ta chọn n cách chọn số thứ
n n 1
nhất, n 1 cách chọn số thứ hai, khi đó có cách chọn hai số khác nhau từ n số
2
đã cho”.
Từ giả thiết dễ thấy m tập con thuộc dãy là phân biệt. Vì A có 31 phần tử nên số tập
31  30
con có đúng 2 phần tử của A là  Ký hiệu ak 2  k  31 là số các tập có đúng k
2
phần tử, nằm trong dãy đã cho, suy ra m  a2  a3    a31 . Xét một tập hợp có k phần
k  k  1 k  k  1
tử suy ra số các tập con có 2 phần tử của tập đó là  ak tập này sẽ có ak
2 2
tập con 2 phần tử. Mà theo giả thiết với 2 phần tử bất kỳ của A thì chúng không thể
đồng thời thuộc 2 tập có k phần tử của dãy  các tập con 2 phần tử nói trên là
phân biệt.
• Phân số có mẫu là tích hai số tự nhiên liên tiếp: “Với x là số tự nhiên thỏa mãn điều
1 1 1
kiện xác định ta có   ”
x  x  1 x x  1
k  k  1 31  30 1
ak   ak  31  30 
2 2 k  k  1
 1 1 1 
 a2  a3    a31  31.30    
 1.2 2.3 30.31
 1 1 1 1 1  1
 m  31.30 1          31.30.1    900 .
 2 2 3 30 31 
 31

ĐỀ SỐ 21.

Câu 1.
1). Ta có với mọi số nguyên m thì m2 chia cho 5 dư 0 , 1 hoặc 4.
+ Nếu n2 chia cho 5 dư 1 thì n2  5k  1  n2  4  5k  5  5 ( k   * ).

Liên hệ tài liệu word toán sđt và zalo: 039.373.2038 TÀI LIỆU TOÁN HỌC
178
Website:tailieumontoan.com

nên n2  4 không là số nguyên tố.


+ Nếu n2 chia cho 5 dư 4 thì n2  5k  4  n2  16  5k  20 5 ( k   * ).
nên n2  16 không là số nguyên tố.
Vậy n2  5 hay n chia hết cho 5.
Nhận xét. Bài toán áp dụng tính chất chia hết, chia có dư của một số chính phương khi
chia cho 5; tính chất số nguyên tố, hợp số,…
Nhắc lại kiến thức và phương pháp.
• Một số chính phương khi chia cho 5 chỉ tồn tại số dư 0 hoặc 1 hoặc 4. Chứng minh:
+ m  5k  m2  25k 2 chia 5 dư 0 (đúng).
+ m  5k  1  m2  25k 2  10 k  1 chia 5 dư 1 (đúng).
+ m  5k  2  m2  25k 2  20 k  4 chia 5 dư 4 (đúng).
+ m  5k  3  m2  25k 2  30 k  9 chia 5 dư 4 (đúng).
+ m  5k  4  m2  25k 2  40 k  16 chia 5 dư 1 (đúng).
• Áp dụng tính chất chia hết, chia có dư vào bài toán; “Số nguyên tố” là số chỉ có hai
ước là 1 và chính nó.
+ n chia 5 dư 1 thì n2  4  5 nên n2  4 không phải là số nguyên tố (loại).

+ n chia 5 dư 4 thì n2  16  5 nên n2  16 không phải là số nguyên tố (loại).

+ Do đó nếu n2  4 và n2  16 là số nguyên tố thì chỉ còn tồn tại trường hợp n2 chia
hết cho 5. Khi đó n chia hết cho 5.
2). Ta có x 2  2 y  x  y  2  x  1  x 2  2  y  1 x  2  y 2  1  0 (1).
Để phương trình (1) có nghiệm nguyên x thì   theo y phải là số chính phương.
2
Ta có   y 2  2 y  1  2 y 2  2  y 2  2 y  3  4   y  1  4 .
  chính phương nên   0; 1; 4 .
2
+ Nếu   4   y  1  0  y  1 , thay vào phương trình (1), ta có
x  0
x 2  4 x  0  x  x  4  0   .
x  4

2
+ Nếu   1   y  1  3  y   .
2 y  3
+ Nếu   0   y  1  4   .
 y  1
+ Với y  3 , thay vào phương trình (1), ta có:
2
x 2  8 x  16  0   x  4  0  x  4 .
+ Với y  1 , thay vào phương trình (1), ta có x 2  0  x  0 .
Vậy phương trình có 4 nghiệm nguyên:  x; y  0; 1 , 4; 1 , 4; 3 , 0;  1 .
Nhận xét. Bài toán giải phương trình nghiệm nguyên bằng phương pháp biến đổi đưa về
phương trình bậc hai theo một ẩn, ẩn còn lại là tham số. Từ đó xét các điều kiện cần và đủ
của  hoặc  ' .

Liên hệ tài liệu word toán sđt và zalo: 039.373.2038 TÀI LIỆU TOÁN HỌC
179
Website:tailieumontoan.com
Nhắc lại kiến thức và phương pháp.
• Biến đổi phương trình về phương trình bậc hai theo một ẩn, ẩn còn lại là tham số.
x 2  2 y  x  y  2  x  1  x 2  2 y.x  2 y 2  2 x  2  0

 x 2  2  y  1 x  2  y 2  1  0

là phương trình bậc hai ẩn x , tham số y .


2
 
• Phương trình bậc hai ax 2  2b x  c  0 với a  0 có   b  ac , phương trình này có

nghiệm khi và chỉ khi   0 .


Phương trình x 2  2  y  1 x  2  y 2  1  0 có    y  1  2  y 2  1
2

 y 2  2 y  1  2 y 2  2  y 2  2 y  3   y  1 y  3  0

  y  1 y  3  0  1  y  3 .

• Phương trình bậc hai có nghiệm, để nghiệm nguyên thì có điều kiện cần là  là số
chính phương; điều kiện đủ là sau khi có nghiệm, thay ngược lại thỏa mãn ta có thể
nhận kết quả.
+ Điều kiện cần   y 2  2 y  3 là số chính phương.
2
Mà   y 2  2 y  3  4   y  1  0    4 nên ta có   0; 1; 4 .
 y  1
- Với   0   y  1 y  3  0   .
y  3
- Với   1  y 2  2 y  3  1  y 2  2 y  2  0

 
2
 y 2  2 y  1  3  0   y  1  3  y  1   3

y  3  1
  .
 y   3  1

- Với   4  y 2  2 y  3  4  y 2  2 y  1  0
2
  y  1  0  y  1  0  y  1 .

+ Điều kiện đủ: Thử lại nghiệm


 y  1  x  0
- Với   0   (thỏa mãn điều kiện).
y  3  x  4
y  3  1
- Với   1   (không thỏa mãn y   ) .
 y   3  1
x  0
- Với   4  y  1   (thỏa mãn điều kiện).
x  4
Vậy phương trình đã cho có nghiệm  x; y  0; 1 , 4; 1 , 4; 3 , 0;  1
Câu 2.

Liên hệ tài liệu word toán sđt và zalo: 039.373.2038 TÀI LIỆU TOÁN HỌC
180
Website:tailieumontoan.com
 
   3  5 3  5 
 3 5 3 5  
1). Ta có A  2     2   
 2 2   5  5 5  5 
4 

5 1 
4 
5 1 

 
 2
  
 3 5 5 5  3 5 5 5 
    
 15  3 5  5 5  5  15  3 5  5 5  5 
  2    2. 20  2 .

 
5 5 5 5  
 
 25  5  20

Vậy A  2 .
Nhận xét: Đây là một bài toán rút gọn biểu thức đơn giản, điểm đáng chú ý chính là việc
phát hiện ra hằng đẳng thức.
Ý tưởng: Biểu thức A khá phức tạp khi chứa các biểu thức căn, thậm chí còn có căn trong
căn, vì thế ta sẽ khử dần căn thức. Đầu tiên quan sát hai căn thức 3  5 và 3  5 , ta
muốn trục căn thức vậy bằng cách nào đó ta cần biểu diễn đại lượng trong căn phải là một
số chính phương. Dựa vào hằng đẳng thức bậc hai ta sẽ đồng nhất hệ số như:
2
3 5  ab 5 
2

 a 2  5b2  3 1  1  5 
2 2 
 a  5b  2 ab 5   ab . Suy ra 3  5   


 2 ab  1 2 
 2

2
 1  5 
Tương tự ta cũng có 3  5    , khi đó biểu thức A trở thành:
 2 

 
   3  5 3  5 
 3 5 3 5  
A 2    2   
 2 2   5  5 5  5 
4

5 1 4 
5 1   

 
 3 5 5 5  3 5 5

 2
   
5 


 5 5 5 5   

 15  3 5  5 5  5  15  3 5  5 5  5 
 2    2. 20  2
 25  5  20
Bài toán kết thúc.
Bài tập tương tự:
x  2  x2  4 x  2  x2  4
1. Rút gọn biểu thức P   .
x  2  x2  4 x  2  x2  4
 1 1  a 1
2. Rút gọn biểu thức Q     : .
a  a a  1 a  2 a  1
2). Phương trình
x  2x  3x  4 x  5  m
  x 2  2 x  8 x 2  2 x  15  m (1).
2
Đặt x 2  2 x  1   x  1  y ( y  0 ), phương trình (1) trở thành:
 y  9 y  16  m  y 2  25 y  144  m  0 (2).

Liên hệ tài liệu word toán sđt và zalo: 039.373.2038 TÀI LIỆU TOÁN HỌC
181
Website:tailieumontoan.com
2
Với mỗi giá trị y  0 thì phương trình  x  1  y có 2 nghiệm phân biệt, do đó phương
trình (1) có 4 nghiệm phân biệt, thì phương trình (2) phải có 2 nghiệm dương phân biệt.
  0   4 m  49  0
  49
 S  0  25  0   m  144 .
  4
P  0 144  m  0
 
49
Vậy với  m
 144 thì phương trình (1) có 4 nghiệm phân biệt.
4
Nhận xét: Bài toán sử dụng phép đặt ẩn phụ tương ứng từ đó đưa về phương trình bậc hai
chứa tham số và giải quyết yêu cầu bài toán theo điều kiện của ẩn phụ.
Nhắc lại kiến thức và phương pháp:
• Đặt t  f 2  x suy ra t  0 .
• Xét phương trình bậc hai tổng quát ax 2  bx  c  0 với a  0 . Để phương trình có
hai nghiệm dương phân biệt khi và chỉ khi

  b2  4 ac  0 
b2  4 ac

 
 
x1  x2  0  b c

 
  0;  0

 x x 0 
 a
 a
 1 2
Ý tưởng: Đây là một phương trình đa thức bậc bốn chứa tham số, với yêu cầu của bài toán
là tìm điều kiện để phương trình có bốn nghiệm phân biệt. Tuy nhiên chưa có một cách
tổng quát nào để giải quyết dạng toán đó, vì thế ta cần đưa cái lạ về cái quen thuộc. Cụ thể
là ta thấy tích các đa thức sẽ bằng tích của hai phương trình bậc hai, nếu ẩn phụ hóa một
đa thức bậc hai thì ta sẽ đưa phương trình bậc bốn ban đầu về phương trình bậc hai. Và
chú ý  x  2 x  4  x 2  2 x  8 và  x  3 x  5  x 2  2 x  15 nên ta sẽ thấy điểm chung là
2
x2  2x  1 , do đó đặt y  x 2  2 x  1   x  1  0 thì phương trình đã cho
  y  9 y  16  m  y 2  25 y  144  m  0  . Khi đó yêu cầu bài toán sẽ trở thành: tìm
m để phương trình  có hai nghiệm dương phân biệt. Đến đây, công việc ta phải làm đã
đơn giản hơn rất nhiều, để  có hai nghiệm dương phân biệt khi và chỉ khi:
  0   4 m  49  0
  49
S  0  25  0   m  144
  4
P  0 144  m  0
 
49
Vậy   m  144 chính là giá trị cần tìm.
4
Bài toán kết thúc.
Bài tập tương tự:
1. Xác định a sao cho x 4  ax 3  2a  1 x 2  ax  1  0 có hai nghiệm khác nhau và lớn
hơn 1 .
1
Đáp số: 4  2 5  a  .
2
2. Giải phương trình  x  3 x  12 x  4 x  16  20 x 2  0 .
7  241
Đáp số: x  6  2 21 hoặc x  .
2

Liên hệ tài liệu word toán sđt và zalo: 039.373.2038 TÀI LIỆU TOÁN HỌC
182
Website:tailieumontoan.com
Câu 3.
1). Điều kiện: x  1 (*).
Ta có x 2  x  4  2 x  1 1  x


 x2  2x x  1  x  1  2 x  x  1  3  0 . 
Đặt x  x  1  y ( y  1 (**)), phương trình trở thành y 2  2 y  3  0.
 y  1
y 2  2 y  3  0   y  1 y  3  0   .
y  3

+Với y  1 , không thỏa mãn điều kiện (**).
+ Với y  3 , ta có phương trình:
x  x 1  3  x 1  3  x
x  3
 x  3

 
 2
 2

x  1  9  6 x  x
 x  7 x  10  0


 x3


 x  2  x  2 , thỏa mãn điều kiện (*).

 
 x  5


Vậy phương trình có nghiệm x  2 .
Nhận xét: Bài toán sử dụng phương pháp ẩn phụ đưa về phương trình bậc hai, từ ẩn phụ
đó sử dụng phương pháp nâng lũy thừa để tìm nghiệm của phương trình ban đầu.
Nhắc lại kiến thức và phương pháp:
• Cách giải phương trình bậc hai tổng quát at 2  bt  c  0 .

 f  x , g  x  0

• Giải phương trình f  x  g  x   .


 f  x  g 2  x

Ý tưởng: Tư duy ẩn phụ hóa, sau khi chuyển vế phải sang vế trái của phương trình ta
được x 2  x  4  2 x x  1  2 x  1  0 . Quan sát hệ số hai trước các căn thức, đồng thời có
sự xuất hiện của x 2 nên ta nghĩ đến hằng đẳng thức x 2  2 x x  1  x  1 , khi đó phương
trình đã cho được viết lại thành: x 2  2 x x  1  x  1  2 x  x  1  3  0 
2
   
 x  x  1  2 x  x  1  3  0  y2  2y  3  0 .

Với y  x  x  1 , tuy nhiên với điều kiện x  1 suy ra y  1 do đó chỉ nhận được nghiệm
y  3 nên x  x  1  3  x  1  3  x

3  x  1
  2  x  2 là nghiệm duy nhất của phương trình.
x  7 x  10  0


Bài toán kết thúc.
Bài tập tương tự:
1. Giải phương trình 3 x  6x  3  x6  x  3 .
Đáp số: x  6 hoặc x  3 .
 
2. Giải phương trình x 2  3  x 2  2 x  1  2 x 2  2 .

Đáp số: x  7 hoặc x   7 .

Liên hệ tài liệu word toán sđt và zalo: 039.373.2038 TÀI LIỆU TOÁN HỌC
183
Website:tailieumontoan.com

x 3  xy 2  10 y  0 x 3  xy 2   x 2  6 y 2  y  0 (1)

2). Ta có  2   2
 2 
x  6 y  10
2
  x  6 y  10

 (2)
Từ phương trình (1), ta có x 3  xy 2   x 2  6 y 2  y  0
 x 3  xy 2  x 2 y  6 y 3  0
 x 3  2 x 2 y  x 2 y  2 xy 2  3 xy 2  6 y 3  0
  x  2 y x 2  xy  3 y 2   0
x  2y
  2 2
.
 x  xy  3 y  0
2
 y 11y 2
+ Trường hợp 1: x  xy  3 y  0   x   
2 2
0xy0.
 2  4
Với x  y  0 , không thỏa mãn phương trình (2).
+ Trường hợp 2: x  2 y , thay vào phương trình (2), ta có:
y  1  x  2
4 y 2  8 y 2  12  y 2  1  
 y  1  x  2
.

Vậy hệ phương trình có 2 nghiệm  x ; y  2; 1 , 2;  1 .
Nhận xét: Bài toán sử dụng phương pháp thế để đưa hệ phương trình về phương trình
đẳng cấp bậc ba biểu diễn mối quan hệ của hai biến x , y từ đó thế ngược lại tìm nghiệm
của hệ phương trình.
Nhắc lại kiến thức và phương pháp:
• Phương trình đẳng cấp bậc ba có dạng x 3  ax 2 y  bxy 2  cy 3  0 .
Nháp: chia phương trình cho y 3 ( vì là làm nháp nên ta đã mặc định y  0 ), khi đó
phương trình đã cho trở thành:
3 2
 x     
   a x   b x   c  0 .

 y   
 y   y 
x
Coi đó là phương trình bậc ba ẩn t  , tìm được nghiệm t ta sẽ biểu diễn được mối
y
liên hệ giữa hai biến x; y .
Ý tưởng: Hệ phương trình chứa hai phương trình, xét với phương trình một là một
phương trình bậc ba bởi sự xuất hiện của x 3  xy 2 , phương trình hai là một phương trình
bâc hai chứa x 2  6 y 2 , điểm tương quan giữa hai phương trình này chính là hằng số 10 ,
việc thế hằng số sẽ giúp ta cân bằng được bậc của phương trình. Cụ thể như sau: thế
10  x 2  6 y 2 vào phương trình thứ nhất trong hệ, ta được:
x 3  xy 2   x 2  6 y 2  y  0  x 3  xy 2  x 2 y  6 y 3  0

 x 3  2 x 2 y  x 2 y  2 xy 2  3 xy 2  6 y 3  0   x  2 y x 2  xy  3 y 2   0
x  2y
x  2y  x  2y
  2   
2 2  
2  y   11y x  y  0
 x  xy  3 y  0   x    0 
 2  4
Từ hai trường hợp đó, thế ngược lại một trong hai phương trình của hệ dễ dàng tìm được
nghiệm của hệ phương trình đã cho.
Liên hệ tài liệu word toán sđt và zalo: 039.373.2038 TÀI LIỆU TOÁN HỌC
184
Website:tailieumontoan.com
Bài toán kết thúc.
Bài tập tương tự:

 x 2  y 2  3 xy  12


1. Giải hệ phương trình  2 .

 2  x  y   y 2
 14


 
Đáp số:  x; y   2;  2 , 1; 2 , 1;  2 .
2 y  x 2  y 2   3 x

2. Giải hệ phương trình  2 .
x  x  y 2   10 y

Câu 4.
E
A
F

P
Q
H O

I
B C
M N

  AEB
1). Ta có AKB  (vì cùng chắn cung AB
 của đường tròn ngoại tiếp tam giác AEB )
  AEB
Mà ABE  (tính chất đối xứng) suy ra AKB
  ABE
 (1).
  AFC
AKC  (vì cùng chắn cung AC
 của đường tròn ngoại tiếp tam giác AFC)
  AFC
ACF  (tính chất đối xứng), suy ra AKC
  ACF
 (2).

Mặt khác ABE  (cùng phụ với BAC


  ACF  ) (3).
  AKC
Từ (1), (2) và (3), suy ra AKB .
 hay KA là phân giác trong của góc BKC
Gọi P; Q lần lượt là các giao điểm của BE với AC và CF với AB .

Ta có BC  R 3 , nên BOC   1 BOC


  120 0 ; BAC   60 .
2
   60 0  ABP
Trong tam giác vuông ABP có APB  900 ; BAC   30 0 hay ABE
  ACF
  30 0 .
  APH
Tứ giác APHQ có AQH   180 0
  PHQ
 PAQ   180 0  PHQ
  120 0  BHC
  120 0 (đối đỉnh).
  ABE
Ta có AKC   30 0 ; AKB
  ACF
  ABE   30 0 (theo chứng minh trên).
  AKC
Mà BKC   AKB
  AFC
  AEB
  ACF
  ABE   BKC
  60 0 , suy ra BHC   180 0 , nên tứ giác
BHCK nội tiếp.
Nhận xét: Bài toán chứng minh một tia là phân giác của một góc ta chứng minh tia đó chia
góc thành hai phần bằng nhau.
Nhắc lại kiến thức và phương pháp:

Liên hệ tài liệu word toán sđt và zalo: 039.373.2038 TÀI LIỆU TOÁN HỌC
185
Website:tailieumontoan.com
• Trong một đường tròn, các góc nội tiếp cùng chắn một cung thì bằng nhau.
  AEB
+ Đường tròn ngoại tiếp tam giác AEB có AKB  hai góc nội tiếp cùng chắn
.
cung AB
  AFC
+ Đường tròn ngoại tiếp tam giác AFC có AKC  hai góc nội tiếp cùng chắn
.
cung AC
• Hai điểm đổi xứng nhau qua một đường thẳng thì thì đường thẳng đó là trung trực
của đoạn nối hai điểm đã cho.
+ Điểm E và B đối xứng nhau qua AB nên suy ra AB là trung trực của EB .
+ Điểm F và C đối xứng nhau qua AC nên suy ra AC là trung trực của FC .
• Một điểm thuộc trung trực của một đường thẳng thì cách đều hai đầu của đoạn thẳng.
+ A  AB và AB là trung trực của EB nên ta có AE  AB .
+ A  AC và AC là trung trực của FC nên ta có AF  AC .
• Tam giác có hai cạnh bằng nhau là tam giác cân. Tam giác cân có hai góc kề đáy bằng
nhau.
  AEB
+ Tam giác AEB có AE  AB nên AEB cân tại A suy ra ABE .

+ Tam giác AFC có AF  AC nên AFC cân tại A suy ra ACF   AFC
.
• Hai góc cùng cộng với một góc được hai góc bằng nhau thì bằng nhau.





APB APB 
  90  ABP
  BAC

  90  ABE
  90 BAC

  ACF
, suy ra ABE .







  
  
AQC AQC  90  ACQ  CAQ  90  ACF  90 BAC

• Một tia nằm giữa hai tia là hai cạnh của một góc và chia góc thành hai góc bằng nhau
là tia phân giác của góc đó.
 AKB   AEB


  
 AKC  AFC
   mà tia AK nằm giữa hai tia AB và AC nên suy ra
  AKC
Ta có  ABE  AEB  AKB

 ACF  AFC


  ACF
 ABE 

AK là tia phân giác của BAC  (điều phải chứng minh).

Ta có BC  R 3 , nên BOC   1 BOC


  120 0 ; BAC   60 .
2
  90 0 ; BAC
Trong tam giác vuông ABP có APB   60 0  ABP
  30 0 hay ABE
  ACF
  30 0 .
  APH
Tứ giác APHQ có AQH   180 0

  PHQ
 PAQ   120 0  BHC
  180 0  PHQ   120 0 (đối đỉnh).

  ABE
Ta có AKC   30 0 ; AKB
  ACF
  ABE
  30 0 (theo chứng minh trên).
  AKC
Mà BKC   AKB
  AFC
  AEB
  ACF
  ABE
  60 0 , suy ra BHC
  BKC
  180 0 , nên tứ giác
BHCK nội tiếp.
Nhận xét. Bài toán chứng minh một tứ giác là tứ giác nội tiếp ta chứng minh tổng hai góc
trong đối diện bằng 180 là tứ giác nội tiếp.
Nhắc lại kiến thức và phương pháp.
Liên hệ tài liệu word toán sđt và zalo: 039.373.2038 TÀI LIỆU TOÁN HỌC
186
Website:tailieumontoan.com
• Định lý Cos: Trong tam giác có các cạnh a; b; c góc 90    180 đối diện với cạnh a .
Ta có công thức: a 2  b2  c 2  2bc cos 180 
Tam giác OBC có BC 2  OB2  OC 2  2.OB.OC.cos 180 
2

 R 3   R2  R2  2.R.R.cos 180 

 2 R2 cos 180   3 R2  2 R2  R2
1
 cos 180    180   60    120 .
2
• Trong một đường tròn, góc nội tiếp và góc ở tâm cùng chắn một cung thì số đo góc nội
tiếp bẳng nửa số đo góc ở tâm đó.
 là góc nội tiếp và BOC
Đường tròn ngoại tiếp ABC có BAC  là góc ở tâm cùng chắn

  BOC  120  60 .
 nên BAC
cung BC
2 2
• Trong tam giác vuông, tổng hai góc nhọn bằng 90 .
  90 0 ; BAC
ABP có APB   ABP
  90 0  ABP
  90 0  BAC
  ABP
  90 60  30 0 hay
  ACF
ABE   30 0 .

• Tổng bốn góc trong một tứ giác bằng 360 .


  APH
Tứ giác APHQ có AQH   PAQ
  PHQ
  360 0

  PHQ
 90  90  PAQ   360 0

  PHQ
 PAQ   180 0  60  PHQ   120 0
  180 0  PHQ

• Hai góc đổi đỉnh thì bằng nhau.


  BHC
PHQ  (hai góc đối đỉnh). Suy ra BHC
  120 0

• Tứ giác là tứ giác nội tiếp ta chứng minh tổng hai góc trong đối diện bằng 180 là tứ
giác nội tiếp.
  BKC
Tứ giác BHCK có BHC   120  AKC
  AKB

  AEB
 120  AFC   120  ACF
  ABE
  120  30  30  180 0

Suy ra BHCK là tứ giác nội tiếp.


2). Gọi (O  ) là đường tròn đi qua bốn điểm B; H ; C ; K . Ta có dây cung BC  R 3 ,
  60 0  BAC
BKC  nên bán kính đường tròn (O  ) bằng bán kính R của đường tròn (O) .

Gọi M là giao điểm của AH và BC thì MH vuông góc với BC , kẻ KN vuông góc với BC
( N thuộc BC ), gọi I là giao điểm của HK và BC .
1 1 1
Ta có SBHCK  SBHC  SBCK  BC.HM  BC.KN  BC  HM  KN  .
2 2 2
1 1
SBHCK  BC  HI  KI   BC.KH (do HM  HI ; KN  KI ).
2 2
Ta có KH là dây cung của đường tròn ( O  ; R ) suy ra KH  2 R (không đổi), nên SBHCK lớn
nhất khi KH  2 R và HM  KN  HK  2 R .
1
Giá trị lớn nhất SBHCK  R 3.2 R  3R2 .
2

Liên hệ tài liệu word toán sđt và zalo: 039.373.2038 TÀI LIỆU TOÁN HỌC
187
Website:tailieumontoan.com

Khi HK là đường kính của đường tròn (O  ) thì M ; I ; N trùng nhau suy ra I là trung
.
điểm của BC nên ABC cân tại A . Khi đó A là điểm chính giữa cung lớn BC
Nhận xét. Bài toán tìm giá trị lớn nhất cho diện tích của một hình ta biểu diễn diện tích đó,
đánh giá - so sánh với các yếu tố cố định.
Nhắc lại kiến thức và phương pháp.
• Hai đường tròn cắt nhau tại hai điểm tạo thành một dây cung ta được các cung chứa
góc tương ứng bằng nhau thì hai đường tròn có cùng bán kính.
  60 0  BAC
Ta có dây cung BC  R 3 , BKC  nên bán kính đường tròn ( O  ) bằng bán

kính R của đường tròn ( O ).


• Diện tích của một hình được chia thành tổng diện tích các hình.
1 1 1
SBHCK  S BHC  S BCK  BC.HM  BC.KN  BC  HM  KN  .
2 2 2
• Quan hệ đường xiên - đường vuông góc: Trong các đường kẻ từ một điểm đến một
đường thẳng thì đường vuông góc là đường ngắn nhất.
 HM là đường vuông góc từ H đến BC nên HM là đường ngắn nhất kẻ từ H
đến BC . Do đó ta có HM  HI .
 KN là đường vuông góc từ K đến BC nên KN là đường ngắn nhất kẻ từ K
đến BC . Do đó ta có KN  KI .
1 1
Suy ra SBHCK  BC  HI  KI   BC.KH
2 2
• Quan hệ đường kính - dây cung trong một đường tròn: Trong một đường tròn, đường
kính là dây cung lớn nhất.
KH là dây cung của đường tròn ( O  ; R ) nên KH  2 R nên SBHCK lớn nhất khi KH  2 R
và HM  KN  HK  2 R .
• Khi tìm được điều kiện để dấu bằng xảy ra trong một bài toán tìm Giá trị lớn nhất,
nhỏ nhất ta chỉ cần thay giá trị đó vào biểu thức ban đầu ta tìm được giá trị lớn nhất,
nhỏ nhất đó.
1
Giá trị lớn nhất SBHCK  R 3.2 R  3R2 .
2
• Khi HK là đường kính của đường tròn ( O  ) thì M ; I ; N trùng nhau suy ra I là
trung điểm của BC nên ABC cân tại A . Khi đó A là điểm chính giữa cung lớn
.
BC
  120 0 ; BKC
3). Ta có BOC   BKC
  60 0 , suy ra BOC   180 0 , nên tứ giác BOCK nội tiếp
đường tròn.
  OC
Ta có OB  OC  R  , suy ra OB   BKO
  CKO .
 hay KO là phân giác góc BKC
 nên K ; O; A thẳng hàng hay AK đi qua O cố định.
Do KA là phân giác góc BKC
Nhận xét. Bài toán chứng minh các điểm thẳng hàng, ta chứng minh các điểm đó cùng
thuộc một đường thẳng nào đó.
Nhắc lại kiến thức và phương pháp.
Liên hệ tài liệu word toán sđt và zalo: 039.373.2038 TÀI LIỆU TOÁN HỌC
188
Website:tailieumontoan.com
• Tứ giác có tổng hai góc trong đối diện bằng 180 là tứ giác nội tiếp.
  BKC
Tứ giác BOCK có BOC   120 0  60 0  180 0 . Suy ra tứ giác BOCK là tứ giác nội
tiếp.
• Trong một đường tròn, hai dây cung bằng nhau thì hai cung bằng nhau.
  OC
Đường tròn ngoại tiếp tứ giác BOCK có OB  OC  R nên OB .

• Trong một đường tròn, hai góc nội tiếp cùng chắn hai cung bằng nhau thì bằng nhau.
  OC
Đường tròn ngoại tiếp tứ giác BOCK có OB   CKO
 nên BKO  suy ra KO là phân
.
giác của BKC
• Các điểm cùng thuộc một đường thẳng cố định thì thẳng hàng.
 nên K ; O nằm trên đường phân giác của BKC
 KO là phân giác của BKC .
 nên K ; A nằm trên đường phân giác của BKC
 KA là phân giác góc BKC .

Suy ra ba điểm O; A; K thẳng hàng.


1 1 1
Câu 5. Ta có P   
1 1   1 1   1 1
x  2  2  y  2  2  z  2  2 
 z y  z x   x y 
1 1 1
Đặt  a;  b;  c thì a , b , c  0 và a 2  b2  c 2  1 .
x y z
a b c a2 b2 c2
Ta có P      
b2  c 2 c 2  a 2 a 2  b2 a 1  a 2  b 1  b2  c 1  c 2 
Áp dụng bất đẳng thức Côsi cho 3 số dương ta có
3
2 1 1  2 a 2  1  a 2  1  a 2  4
a 1  a
2 2
  .2 a 2 1  a 2 1  a 2   
2 2  3
 
 27
2 a2 3 3 2
 a 1  a 2     a (1).
3 3 a 1  a 2
 2

b2 3 3 2
Tương tự  b (2).
b 1  b 2
 2

c2 3 3 2
 c (3).
c 1  c 2
 2

3 3 2 3 3
Từ (1), (2) và (3), ta có P   a  b2  c 2   .
2 2
1
Dấu “=” xảy ra khi a  b  c  hay x  y  z  3 .
3
3 3
Vậy giá trị nhỏ nhất của P là .
2
thiết, áp dụng bất đẳng thức Cosi để tìm giá trị nhỏ nhất của biểu thức.
Ý tưởng: Bài toán là một bất đẳng thức đối xứng, do vai trò các biến x , y , z là như nhau nên
3 3
dấu đẳng thức xảy ra tại x  y  z  3 và giá trị nhỏ nhất của P là . Ta sẽ đi từ giả
2

Liên hệ tài liệu word toán sđt và zalo: 039.373.2038 TÀI LIỆU TOÁN HỌC
189
Website:tailieumontoan.com
1 1 1
thiết trước, vì giả thiết chứa các biểu thức phân số nên ta đặt  a;  b;  c , khi đó
x y z
a 2  b2  c 2  1 và
a b c a2 b2 c2
P      .
b2  c 2 c 2  a 2 a 2  b2 a 1  a 2  b 1  b2  c 1  c 2 
a2
Do tính đối xứng nên ta chỉ cần xét một biểu thức đồng thời cần đánh giá nó qua
a1  a 2 

biểu thức trung gian để ta có thể sử dụng giả thiết bài cho. Với điểm rơi x  y  z  3 ta
1
có a  b  c  , với điểm rơi này ta thấy:
3
1 2 2
a  2a  và 1  a2  nên theo bất đẳng thức Cosi suy ra:
3 3 3
3
2 a 2
 1  a2  1  a2  8
2 a 2 1  a 2 1  a 2   
27 27
2 4 2 a2 3 3 2
 a2 1  a2    a1  a2     a .
27 3 3 a1  a 
2
2

Tương tự đánh giá cho b , c ta được:


a2 b2 c2 3 3 2 3 3
P     a  b2  c 2   .
a1  a 2
 b1  b 2
 c 1  c 2
 2 2

3 3
Vậy giá trị nhỏ nhất của P là .
2
Bài toán kết thúc.
Bài tập tương tự:
1. Cho a; b; c là các số thực dương thỏa mãn a2  b2  c 2  3 . Chứng minh bất đẳng
1 1 1
thức 2      3a  b  c  3 3 .
 a b c 
2. Cho x; y; z là các số thực dương thỏa mãn x; y; z  0; 1 và xy  yz  zx  1 . Tìm giá
trị nhỏ nhất của biểu thức
x y z
P 2
 2
 .
1 x 1 y 1  z2

ĐỀ SỐ 22.
Câu 1.
1). Giả sử a; b là hai số thực phân biệt thỏa mãn
a 2  3b  2
a).  2  a 2  b2  3a  b  0 .
b  3a  2

 a  ba  b  3a  b  0  a  ba  b  3  0
 a  b  0 ( l)

 a  b  3
.

3
b). Với a  b  3  a  b  27
Liên hệ tài liệu word toán sđt và zalo: 039.373.2038 TÀI LIỆU TOÁN HỌC
190
Website:tailieumontoan.com
 a 3  b3  3ab a  b  27  a 3  b3  9 ab  27
2
 a 2  3a  b2  3b  4  a  b  2 ab  3a  b  4  ab  2 .
Vậy a 3  b3  45 .
Nhận xét: Đây là bài toán hết sức cơ bản, đề yêu cầu gì ta làm đó, với một vài phép biến
đổi tương đương cùng kết hợp hằng đẳng thức quen thuộc ta sẽ suy ra được điều phải
chứng minh.
Nhắc lại kiến thức và phương pháp:
2
• Hằng đẳng thức a  b  a2  2ab  b2
3
và a  b  a3  3a2 b  3ab2  b3 .
ma  nb  p  2
(1)
• Hệ phương trình đối xứng dạng  2 .
mb  na  p (2)

Lấy (1) + (2), ta có: ma  b   na  b  2 p .
2 2

a  b

Lấy (1) – (2), ta có: ma  b   na  b  
2 2
.
a  b  n
 m
Từ đó suy ra:
   n2  n
2 p  m a  b  2 ab  na  b  m 2  2 ab  n.  ab  ...
2

   m  m
Bài tập kết thúc.
Bài tập tương tự:
3. Cho a; b thỏa mãn a2  4b  3; b2  4a  3 . Tính a5  b5 .
4. Cho a; b thỏa mãn a2  2b2  4b  7; b2  2a2  4a  7 . Tính giá trị của biểu thức
a3  b3  ab .

2). Ta thấy x  y  0 là nghiệm của phương trình.


Nếu y  0 , nhân hai vế của phương trình với y , ta được
2 xy  3 y 2  5 xy 2 2 x  3 y  5 xy
 2  2
4 x  y 2  5 xy 2 4 x  y 2  5 xy 2
 

2 x  3 y  5 xy 2 x  3 y  5 xy

  2  
 2

 2 x  xy  y 2
 0  2
4 x  y  5 xy
2
 
2 x  3 y  5 xy 2 x  3 y  5 xy
   
 x  y2 x  y  0  x  y2 x  y  0
 
2 x  3 y  5 xy
  x y1
 x  y  0
 
 .
2 x  3 y  5 xy 2  4

2 x  y  0  x  5 , y  5

Nhận xét: Bài toán sử dụng phương pháp thế tích từ phương trình một xuống phương
trình hai của hệ, và đưa về phương trình đẳng cấp bậc hai để tìm mối liên hệ giữa hai biến
rồi thế ngược lại phương trình một, tìm nghiệm của hệ phương trình.

Liên hệ tài liệu word toán sđt và zalo: 039.373.2038 TÀI LIỆU TOÁN HỌC
191
Website:tailieumontoan.com
Nhắc lại kiến thức và phương pháp:
• Phương trình đẳng cấp bậc hai ẩn x , y là ax 2  bxy  cy 2  0 (*).
Làm nháp: Chia phương trình (*) cho y 2 ta có:
2
 x  x x x
a    b    c  0 . Coi đây là một phương trình bậc hai ẩn , giả sử tìm được  m
 y   y  y y
x
hoặc  n , do đó ta phân tích được
y
 x  my
(*)  ax 2  bxy  cy 2  0   x  my x  ny  0   .
 x  ny

Ý tưởng: Dạng hệ sử dụng phương pháp thế khá là đặc trưng nhưng cái khó là tìm các đại
lượng biểu diễn nhân tử chung giữa hai biến x; y . Nhìn vào vế trái của từng phương
trình, với phương trình một là bậc nhất, còn phương trình hai là bậc hai, tương tự ở vế
phải, bậc của phương trình hai cũng lớn hơn một bậc so với phương trình một. Điều nó sẽ
làm ta nghĩ đến phép nhân để đồng bậc hóa chúng, nhân ở đây sẽ là nhân chéo từng
phương trình. Tuy nhiên nếu quan sát kỹ hơn một chút, cả hai phương trình đều xuất hiện
5xy nên ta sẽ thế 5 xy  2 x  3 y xuống phương trình hai (thực chất đây cũng là việc nhân
chéo nhưng ta đã giảm biến y , khi đó hệ phương trình đã cho trở thành:
2 x  3 y  5 xy
 2 x  3 y  5 xy 
2 x  3 y  5 xy 

 
  x  y .
 2
4 x  y 2  2 x  3 y y 2 x 2  xy  y 2  0 
  
 2 x  y  0
Việc còn lại chỉ là tìm nghiệm của hệ phương trình đã cho.
Bài toán kết thúc.
Bài tập tương tự:

x  3 y  5 xy
1. Giải hệ phương trình  2 .
2
 2
x  y  5 xy

2 1  1
Đáp số:  x; y  0; 0 ,  ;   , 1;  .
 5 5   2 
3 x  3 y  6 xy

2. Giải hệ phương trình  2 .
 2 2
9 x  y  6 xy

1   5 5
Đáp số:  x; y  0; 0 ,  ;  1 ,  ;  .
 3   6 4 
Câu 2.
1). Ta có xy  1   x  1 y  1 , suy ra xy  1  xy  1  x  y .
Mà xy  1  x  y  xy  1  x  y   x  1   y  1   x  1 y  1 , suy ra x  1  y  1 và y  1  x  1 ,
nên x  y
2
x 2  1   x  1 ta có x  1  x  1 , suy ra 2  x  1 , nên x  2 hoặc x  3 .
Nhận xét: Bài toán tìm số nguyên thỏa mãn một điều kiện, sử dụng tính chất chia hết của
một tổng, hiệu, tích,…
Nhắc lại kiến thức và phương pháp:
• Biến đổi đại số.

Liên hệ tài liệu word toán sđt và zalo: 039.373.2038 TÀI LIỆU TOÁN HỌC
192
Website:tailieumontoan.com
Ta có ( x − 1)( y − 1) = xy − x − y + 1 nên:
( xy − 1)  ( x − 1)( y − 1) ⇔ ( xy − 1)  ( xy − x − y + 1) .
• Một số luôn chia hết cho chính nó.
Ta có ( xy − x − y + 1)  ( xy − x − y + 1) .
• Hai số chia hết cho một số thì hiệu của hai số đó.
( xy − 1)  ( xy − x − y + 1)
 ⇒ ( x + y − 2 )  ( xy − x − y + 1)
( xy − x − y + 1)  ( xy − x − y + 1)
⇔ ( x − 1) + ( y − 1)  ( xy − x − y + 1) ⇔ ( x − 1) + ( y − 1)  ( x − 1)( y − 1) .
• Tính chất chia hết a  b thì ka  b với a; b; k là các số nguyên.
( x − 1) + ( y − 1)  ( x − 1)( y − 1) ⇒ ( x − 1) + ( x − 1)( y − 1)  ( x − 1)( y − 1) .
2

• Một tổng hai số hạng chia hết cho một số trong đó có một số hạng chia hết cho số đó
thì số hạng còn lại cũng chia hết.
( x − 1)2 + ( x − 1)( y − 1)  ( x − 1)( y − 1) 2
   x  1   x  1 y  1   x  1   y  1 ;
( x − 1)( y − 1)  ( x − 1)( y − 1)
hoàn toàn tương tự ta có ( y − 1)  ( x − 1) .
• Có hai số thỏa mãn: Số thứ nhất chia số thứ hai và số thứ hai chia hết cho số thứ nhất
thì hai số bằng nhau.

 x  1   y  1
Ta có   x 1  y 1  x  y .



 y  1   x  1

Thay lại vào đề bài ta có ( x 2 − 1)  ( x − 1) ⇔ ( x − 1)( x + 1)  ( x − 1)


2 2

⇔ ( x + 1)  ( x − 1) ⇔ ( x − 1) + 2   ( x − 1) ⇒ 2  ( x − 1) .
• Số thứ nhất chia hết cho số thứ hai thì số thứ hai là ước của số thứ nhất.
2  ( x − 1) ⇒ ( x − 1) ∈ U( 2 ) ={±1; ± 2} .

vì x ≥ 2 ⇔ x − 1 ≥ 1 suy ra ( x − 1) ∈ {1; 2}
x − 1 = 1 x = 2 ⇒ y = 2
⇔ ⇔ .
x − 1 = 2 x = 3 ⇒ y = 3
Vậy x= y= 2 hoặc x= y= 3 .
x 2 y 2  1
2). Ta có x 2 y 2  2 y  1  0  2 y  x 2 y 2  1  y 
2
xy xy
P 
3x y  1  2 3 x 2 y 2  1
2 2

 3 Px 2 y 2  2 xy  P  0 .
Ta có  4  12P 2 .
Phương trình có nghiệm khi  0
1 1 1
 4  12 P 2  0  1  3 P  1  P P .
3 3 3
3 12 3 7
Vậy MaxP = ⇔ x =− và y = − .
3 21 6

Liên hệ tài liệu word toán sđt và zalo: 039.373.2038 TÀI LIỆU TOÁN HỌC
193
Website:tailieumontoan.com
Nhận xét: Bài toán tìm giá trị của một biểu thức sau khi biến đổi biểu thức trở thành tham
số của một phương trình bậc hai theo một ẩn
Nhắc lại kiến thức và phương pháp:
• Biến đổi giả thiết.
x 2 y 2  1
Ta có x 2 y 2  2 y  1  0  2 y  x 2 y 2  1  y  .
2

• Đưa biểu thức đã cho về một ẩn hoặc một biểu thức cố định.
−x2 y 2 − 1 xy
Thay y = vào biểu thức P ta có P =
2 2 2
−x y − 1
3 +1
2
2 xy 2 xy
= = .
( 2 2
3 − x y − 1 + 2 −3 x y − 1
2 2
)
2a
Đặt xy = a ta được P = .
−3a 2 − 1
• Tìm giá trị lớn nhất, nhỏ nhất của một biểu thức bằng cách đưa biểu thức đó trở thành
tham số của một phương trình bậc hai
2a
=P 2
+ P 0 Khi đó ta có ∆′ = 12 − 3P 2 .
⇒ 3 Pa 2 − 2 a=
−3a − 1
• Phương trình bậc hai có nghiệm khi và chỉ khi ∆ hoặc ∆ ' không âm.
Ta có Vì phương trình có nghiệm nên ∆′ ≥ 0 ⇔ 1 − 3P 2 ≥ 0
( )(
⇔ 1 − 3P 1 + 3P ≥ 0 ⇔ ) ( 3P − 1 )( )
3P + 1 ≥ 0

−1 1 1
⇔ −1 ≤ 3 P ≤ 1 ⇔ ≤P≤ ⇒P≤ .
3 3 3
• Khi xét đến giá trị nhỏ nhất, giá trị lớn nhất của biểu thức cần phải xét xem “dấu bằng
xảy ra khi nào?”.
1
Ta có dấu bằng xảy ra khi vào chỉ khi P = ⇔ ∆ ' = 0 ⇒ Phương trình bậc hai có
3
2
nghiệm kép a = , thay vào biểu thức ban đầu ta có:
3
2
2
 2 
   2 y  1  0  2 y   7  y   7  x  xy  3   12 .
 3  3 6 y 7 7 3

6
3 12 3 7
Vậy MaxP = ⇔ x =− và y = − .
3 21 6
Câu 3.

Liên hệ tài liệu word toán sđt và zalo: 039.373.2038 TÀI LIỆU TOÁN HỌC
194
Website:tailieumontoan.com

BD AB
1). Ta có AD là phân giác   mà BED; CDF là tam giác cân,
DC AC
BE AB
   BC  FE .
CF AC
Nhận xét. Bài toán chứng minh hai đường thẳng song song sử dụng định lý Ta-lét đảo
Nhắc lại kiến thức và phương pháp.
• Tâm đường tròn nội tiếp tam giác là giao điểm của ba đường phân giác.
Trong ∆ABC có I là tâm đường tròn nội tiếp nên suy ra AI hay AD là phân giác của
.
BAC
• Tính chất đường phân giác trong tam giác.
DB AB
Trong ∆ABC có AD là phân giác nên = .
DC AC
• Hai điểm đối xứng nhau qua một đường thẳng thì đường thẳng đó là trung trực của
đoạn thẳng nối hai điểm.
+ E là điểm đối xứng của D qua BI nên BI là trung trực của DE ;
+ F là điểm đối xứng của D qua CI nên CI là trung trực của DF .
• Một điểm thuộc trung trực của một đoạn thẳng thì điểm đó cách đều hai đầu mút của
đoạn thẳng đó.
+ B ∈ BI mà BI là trung trực của DE nên BD = BE ;
+ C ∈ CI mà CI là trung trực của DF nên CD = CF ;
BD BE BE AB BE CF
suy ra = do đó = ⇔ =
CD CF CF AC BA CA
• Định lý Ta-lét đảo: Hai điểm thuộc hai đoạn thẳng, chia hai đoạn thành các đoạn
thẳng tương ứng tỷ lệ thì hai đường thẳng chứa hai đoạn thẳng đó song song.
BE CF
Theo định lý Ta-lét đảo ta có
= thì EF  BC .
BA CA
  EDB
2). Ta có BC  EF  EFD   BED
.
  180 0  AEM
Mà APM   BED
  APM
  DEF
.
  APN
Tương tự: DFE   APN
  APM
  DFE   MPN
  FED .
  MDN
Mà MJN   EDF
  MJN
  MPN
  180 0  MPNJ nội tiếp.

Nhận xét: Bài toán chứng minh tứ giác nội tiếp bằng cách chứng minh tổng hai góc trong
bằng 180° .
Liên hệ tài liệu word toán sđt và zalo: 039.373.2038 TÀI LIỆU TOÁN HỌC
195
Website:tailieumontoan.com
Nhắc lại kiến thức và phương pháp:
• Hai đường thẳng song song tạo ra các góc ở vị trí so le trong bằng nhau.
 = EDB
Ta có EF  BC , suy ra FED .
• Tam giác có hai cạnh bằng nhau là tam giác cân, tam giác cân có hai góc kề cạnh đáy
bằng nhau.
 = BED
Tam giác ∆BDE có BD = BE nên ∆BDE cân tại B suy ra BDE  , suy ra

FED 
= EDB .
= BED
• Tứ giác nội tiếp có tổng hai góc đối diện bằng 180° .
Tứ giác APME giác là tứ nội tiếp nên  + AEM
APM = 180 0
    
⇔ APM =180 − AEM =BED ⇒ APM =DEF .
0

Hoàn toàn tương tự có


  APN
DFE   APN
  APM
  DFE
  FED
  MPN
.
• Trung điểm; Đường trung bình của tam giác.
 MJ  DF

+ MJ là đường trung bình của ∆DFE nên  1 ;
 MJ = DF
 2
DF
+ N là trung điểm của DF nên DN = ;
2
 MJ  DN ( DF )
suy ra  .
=MJ DN =( DF )
• Tứ giác có một cặp cạnh song song và bằng nhau là hình bình hành.
Tứ giác MJND có MJ  DN và MJ = DN nên MJND là hình bình hành.
• Hình bình hành có hai góc đối diện bằng nhau.
  MDN
+ MJND là hình bình hành nên MJN ;
  EDF
+ MDN  (hai góc so le trong của MN  FE suy ra tính chất đường trung bình);
  MPN
suy ra MJN   180 0 .

• Tứ giác có tổng hai góc trong đối diện bằng 180° là tứ giác nội tiếp.
  MPN
Tứ giác MPNJ có MJN   180 0 nên tứ giác MPNJ là tứ giác nội tiếp.

  DEF
3). Ta có APM   JNM
 và JPM   JEM
  JPM
  APM
 , suy ra 3 điểm A; P ; J thẳng

hàng.
Nhận xét: Bài toán chứng minh ba điểm thẳng hàng bằng cách chứng minh ba điểm cùng
thuộc một đường thẳng đặc biệt
Nhắc lại kiến thức và phương pháp:
• Tứ giác nội tiếp có góc ngoài và góc trong không kề với nó bằng nhau.
 = BED
Tứ giác MPAE là tứ giác nội tiếp nên MPA  = DEF
 mà BED  ( = BDE
 ), suy ra
 = DEF
MPA .
• Trung điểm, Đường trung bình, Hình bình hành,…(đã nhắc lại phần trên).
 MN  FE

+ MN là đường trung bình của ∆DEF nên  1 ;
 MN = FE
 2

Liên hệ tài liệu word toán sđt và zalo: 039.373.2038 TÀI LIỆU TOÁN HỌC
196
Website:tailieumontoan.com
1
+ J là trung điểm FE nên EJ = EF ; suy ra MN  JE và JE = MN nên MNJE là hình
2
    (hai góc nội tiếp cùng chắn cung MJ
bình hành nên MEJ = MNI mà MEJ = MNP  của
 , suy ra 
  DEF
đường tròn ngoại tiếp tứ giác MNPJ ) suy ra JPM  nên
JPM = APM
PJ ≡ PA hay ba điểm A; P ; J thẳng hàng.
Câu 4.
1). Theo đề bài, các số nguyên dương được sắp xếp theo từng hàng chéo của bảng: Hàng
chéo thứ nhất có 1 số, hàng chéo thứ hai có 2 số,……
Giả sử số x nằm ở hàng chéo thứ k thì ta có:
k  k  1 k  k  1 1  1  8 x 1  1  8x
x  k
2 2 2 2
1  1  8 x
k .
2

1  1  8.2015
Áp dụng x  2015 ta có k   63 .
2
k  k  1
Số đầu tiên ở hàng chéo thứ k  63 là  1  1954 .
2
Như vậy số 2015 nằm ở vị trí thứ 2015  1954  1  62 của hàng chéo thứ 63 (vị trí áp chót)
Tọa độ của nó là 2,62 .
2). Theo Côsi cho 4 số ta có
4  abc  ab  bc  ac  4 4 a 3 b3 c 3  1  abc  a  b  c  3 3 abc  3 3 a 2 b2 c 2 .
BĐT tương đương a 2  b2  c 2  3 3 a 2 b2 c 2  2 ab  bc  ac (1).
Đặt 3
a 2  x; 3 b2  y ; 3 c 2  z ( x; y ; z  0 ).
 x 3  y 3  z 3  3 xyz  2 x 3 y 3  2 z 3 x 3  2 z 3 y 3 .
Áp dụng bất đẳng thức Schur bậc 3:
x 3  y 3  z 3  3 xyz  xy  x  y  yz  y  z  xz  x  z
 x  x  y x  z  y  y  x y  z  z  z  x z  y  0
với mọi số thực không âm x; y; z .
Chứng minh bất đẳng thức
Do vai trò x; y; z như nhau, giả sử x  y  z  z  z  x z  y  0 .
Ta xét x  x  z  y  y  z  x 2  xz  yz  y 2   x  y x  y  z  0
 x  x  z x  y  y  y  z x  y  0  x  x  z x  y  y  y  z y  x  0
 x  x  y x  z  y  y  x y  z  z  z  x z  y  0 .
(điều phải chứng minh).
Ta có x  y  z  3xyz  xy  x  y  yz  y  z  xz  x  z
3 3 3

 2 x3 y 3  2 z3 x3  2 z3 y 3 .
xyz 
Dấu “=” xảy ra khi   a b c 1.
 x  y; z  0

Liên hệ tài liệu word toán sđt và zalo: 039.373.2038 TÀI LIỆU TOÁN HỌC
197
Website:tailieumontoan.com
Nhận xét: Bài toán sử dụng bất đẳng thức Cosi, phép đặt ẩn phụ đồng thời chứng minh
qua bất đẳng thức trung gian hay là bất đẳng thức bổ đề (bất đẳng thức Schur bậc ba) để
suy ra điều phải chứng minh.
Nhắc lại kiến thức và phương pháp:
• Bất đẳng thức Cosi cho ba số thực dương a  b  c  3 3 abc .
• Bất đẳng thức Schur bậc ba cho các số thực dương
x 3  y 3  z 3  3 xyz  xy  x  y  yz  y  z  xz  x  z .
Bài toán kết thúc.
Bài tập tương tự:
1. Chứng minh rằng với mọi a; b; c  0 , ta luôn có:
2 a 2  b2  c 2   abc  8  5a  b  c .
2. Cho a; b; c  0 là các số thực dương thỏa mãn a  b  c  3 . Chứng minh rằng
 1 1 1 12
5      3.
 a b c  abc

ĐỀ SỐ 23.

Câu 1.
3a  b  c  x

1). Đặt 3b  c  a  y .

3c  a  b  z
3 3 3 3
Ta có 3a  3b  3c  24  3a  b  c  3b  c  a  3c  a  b
3
  x  y  z  24  x 3  y 3  z 3
3 3
  x  y  z  24   x  y  z  3 x  y y  z x  z
 24  3 x  y y  z x  z  0  24  32 a  4b2b  ac2c  4 a  0
 24  24 a  2bb  2cc  2 a  0  a  2bb  2cc  2 a  1 .
Nhận xét: Bài toán sử dụng phương pháp đặt ẩn phụ, khai thác biến đổi từ giả thiết, điểm
nhấn ở đây chính là hằng đẳng thức bậc ba để suy ra điều phải chứng minh.
Nhắc lại kiến thức và phương pháp:
3
• Hằng đẳng thức  x  y  x 3  3x 2 y  3xy 2  y 3
• Hằng đẳng thức
3 3 2
 x  y  z   x  y  3 x  y z  3 x  y z 2  z 3
2
 x 3  3 x 2 y  3 xy 2  y 3  3 x  y z  3 x  y z  z 3
2
 x 3  y 3  z 3  3 xy  x  y  3 x  y z  3 x  y z
 x 3  y 3  z 3  3 x  y xy  xz  yz  z 2 
 x 3  y 3  z 3  3 x  y y  z z  x .
Ý tưởng: Bài toán cho giả thiết khá là cồng kềnh, phức tạp thậm chí bậc to, nhưng quan sát
các đại lượng bên vế phải, ta thấy các biểu thức 3a  b  c ,
3b  c  a và 3c  a  b độc lập so với nhau mặt khác:

Liên hệ tài liệu word toán sđt và zalo: 039.373.2038 TÀI LIỆU TOÁN HỌC
198
Website:tailieumontoan.com
3a  b  c  3b  c  a  3c  a  b  3a  3b  3c
Vì thế nếu đặt x  3a  b  c; y  3b  c  a; z  3c  a  b thì giả thiết bài cho tương đương
3
với  x  y  z  24  x 3  y 3  z 3 . Khoan nói đến chuyện biến đổi nói, bây giờ ta sẽ quan
sát đến biểu thức cần chứng minh, với phép ẩn phụ trên ta có thể hoàn toàn rút a; b; c theo
x; y ; z . Xét biểu thức a  2b biểu thức này không chứa c vậy nên ta cần tìm mối liên hệ
giữa x; y; z sao cho c triệt tiêu, dễ thấy ở x chứa c còn y chứa c nên suy ra
x  y  2 a  4b  2 a  2b , tương tự y  z  2 b  2c ; z  x  2 c  2 a . Do đó ta cần chứng
minh  x  y y  z z  x  8 . Bây giờ ta quay ngược lại với giả thiết, xuất hiện hằng đẳng
3
thức bậc ba, nên ta sẽ khai triển nó, ta được:  x  y  z  x 3  y 3  z 3  3 x  y y  z z  x
nên suy ra:
x 3  y 3  z 3  3 x  y y  z z  x  24  x 3  y 3  z 3
  x  y y  z z  x  8  điều phải chứng minh.
Bài toán kết thúc.
Bài tập tương tự:
1. Cho a; b; c là ba số thực dương thỏa mãn điều kiện
3 3 3 3
 a  b  c  12  a  b  c  b  c  a  a  c  b . Chứng minh rằng 2 abc  1 .


2 x  2 y  xy  5
2). Ta có 
27  x  y  y  7  26 x 3  27 x 2  9 x
3


 x  2 y  2  9


 


 27 x  y  y 3  7  26 x 3  27 x 2  9 x
 y 3  x 3  7  3 x  y x  2 y  2  27 x 3  27 x 2  9 x
2 3
 y 3  x 3  8  3 xy  x  y  12  x  y  6  x  y  3 x  1
3 3
  x  y  2  3 x  1  x  y  2  3 x  1
x  1  y  1

 y  1  2 x   x  22 x  1  9   .
 x   7  y  8
 2
 7 
Vậy  x; y  1; 1 ,  ;  8 .
  2 
Nhận xét: Bài hệ phương trình rất hay ở chỗ kết hợp cả hai phương trình, sau đó nhóm lại
hằng đẳng thức đưa được về phương trình biểu diễn mối liên hệ giữa x; y và thế ngược
lại tìm nghiệm của hệ phương trình.
Nhắc lại kiến thức và phương pháp:
3
• Hằng đẳng thức  x  y  x 3  3x 2 y  3xy 2  y 3 .
3 3 2
• Hằng đẳng thức  x  y  z   x  y  3 x  y z  3 x  y z 2  z 3

Liên hệ tài liệu word toán sđt và zalo: 039.373.2038 TÀI LIỆU TOÁN HỌC
199
Website:tailieumontoan.com
2
 x 3  3 x 2 y  3 xy 2  y 3  3 x  y z  3 x  y z  z 3
2
 x 3  y 3  z 3  3 xy  x  y  3 x  y z  3 x  y z
 x 3  y 3  z 3  3 x  y xy  xz  yz  z 2 
 x 3  y 3  z 3  3 x  y y  z z  x
3 3
• Giải phương trình tổng quát  f  x   g  x
  f  x  g  x  f 2  x  f  x g  x  g 2  x  0  f  x  g  x .
  
Ý tưởng: Phương trình thứ hai trong hệ khá phức tạp, ta sẽ đi khai thác nó trước, để ý bên
vế phải của phương trình hai, xuất hiện các tổng 27 x 2  9 x mà ta thấy
2 3
27 x 2  9 x  3.3 x  3.3 x có bóng dáng của hằng đẳng thức nếu thêm đại lượng 3 x  13
3
khi đó : 26 x 3  27 x 2  9 x  3x  1  x 3  1
Chuyển đại lượng x 3  1 sang vế trái của phương trình hai, ta được:
3
x 3  y 3  8  27  x  y  3 x  1 ( i ).
Điều còn lại là ta chưa khai thác tới phương trình một, nhưng quan sát ( i ) một chút, ta
thấy vế phải là một lũy thừa bậc ba, ta sẽ biến đổi vế trái ( i ) cũng về dạng đó. Bởi lẽ
3
x 3  y 3  8  x 3  y 3  2 3 ta sẽ nghĩ đến chuyện đưa vế trái ( i ) về dạng  x  y  2 . Hay nói
cách khác, kết hợp phương trình một của hệ, ta cần chứng minh:
3
x  y  8  27  x  y   x  y  2 .
3 3

Sẽ có hai cách để chứng minh, một là đi xuôi, hai là chứng minh ngược:
• Từ phương trình một của hệ, có  x  2 y  2  9 thế vào vế trái của ( i ), ta được:
3
x 3  y 3  2 3  3 x  y x  2 y  2   x  y  2 .
3
• Ta có  x  y  2  x 3  y 3  2 3  3 x  y x  2 y  2
 3 x  y x  2 y  2  27  x  y  2 x  2 y  xy  5
3 3
Khi đó ta có  x  y  2  3x  1  x  y  2  3x  1  y  2 x  1 , thế ngược lại phương
trình một của hệ ta sẽ tìm được nghiệm của hệ phương trình ban đầu.
Bài toán kết thúc.
Bài tập tương tự:

2 x  2 y  xy  5
1. Giải hệ phương trình  .
27  x  y  y  7  7 x 3  12 x 2  6 x
3

Câu 2.

n  5  x 2
1). Đặt  ( x , y  ?; x , y  0 ).
 2
n  30  y

 y 2  x 2  25   y  x y  x  1.25 vì ( x , y  ?; x , y  0 ).
y  x  1
  y  13

Lại có y  x  y  x nên  
 .

 y  x  25
 
x  12

Thay vào ta tính được n  139 thảo mãn.
Nhận xét: Bài toán số học sử dụng tính chất số chính phương, phương trình ước số

Liên hệ tài liệu word toán sđt và zalo: 039.373.2038 TÀI LIỆU TOÁN HỌC
200
Website:tailieumontoan.com
Nhắc lại kiến thức và phương pháp:
• Số chính phương được viết dưới dạng a 2 với a là số nguyên.
+ n + 5 là số chính phương nên có dạng x 2 với x là số nguyên.
+ n + 30 là số chính phương nên có dạng y 2 với y là số nguyên
• Hằng đẳng thức A 2 − B2 = ( A + B )( A − B ) .

n  5  x 2
Trừ vế theo vế của  cho nhau ta được
 2
n  30  y

y 2 − x 2 = ( n + 30 ) − ( n + 5 ) ⇔ ( y − x )( y + x ) = 25 .
• Phương trình ước số.
( y − x )( y + x ) =25 =1.25 =−
( 1) . ( −25 ) =5.5 =−
( 5 ) . ( −5 ) ;
= y − x 1 =  2 y 26 =   y 13  =n + 30 169
   
  y + x = 25 x = y − 1 =   x 12  n + 5 144
=
=   y − x 25 =  2 y 26 =   y 13  =n + 30 169
   
y + x = 1   x = y − 25   x =−12  n + 5 =144
   
= y − x 5 =  2 y 10 = y 5  =n + 30 25

 y+x=5 
 x= y−5= 
 x 0 
 n+5 0
=
suy ra   ⇔

⇔

⇔

  y − x =−5  2 y =−10 y = −5  n + 30 =25
   
  y + x =−5   x =y + 5 = x 0  n + 5 0
=
  y − x =−1  2 y =−26 y = −13  n + 30 =169
   
  y + x =−25   x =y + 1   x =−12  n + 5 =144
   
  y − x =−25  2 y =−26 y = −13  n + 30 =169

 y + x =−1   
 x 12 
 n + 5 144
   x =y + 25=  =

 =n + 30 169 =  n 139
 
= n + 5 144= n 139 n = 139
⇔ ⇔  ⇒ 139 (do n là số tự nhiên).
⇒n=
n + 30 = 25 n =−5  n = 139
 
 n + 5 =0  n =−5
2). Ta thấy 1  x  y  3  x  y và x , y  ?  x , y là số chính phương.
 x  y  3; x ; y  ?
Đặt x  a; y  b; x  y  3  c a , b , c  ?

 a b  c 1

 
a  b  c  1
 x  y  a 2  b 2  
  2
  2 2

 x  y  3  c 2 c  a  b  3



2
 a  b  1  a 2  b2  3  2 a  2b 2 ab  3
 
a  2  x  4
b  3  y  9
 a  1b  1  2   
 
a  3 x  9
 
 
b  2  y  4

Nhận xét: Bài toán số học sử dụng tính chất số chính phương, phương trình ước số.

Liên hệ tài liệu word toán sđt và zalo: 039.373.2038 TÀI LIỆU TOÁN HỌC
201
Website:tailieumontoan.com
Nhắc lại kiến thức và phương pháp:
• Khi a là số nguyên ta có được a là số chính phương.
Ta có x; y; x + y + 3 là các số nguyên nên ta có thể biển diễn như sau
 x = a ⇒ x = a2

 y =b ⇒ y =b
2
với a , b , c là các số nguyên dương.
 2
 x + y + 3 = c ⇒ x + y + 3 = c
Từ đây ta có

 a  b  c 1
 

 2 2
x  y  a  b   a  b  c  1  a  b  12  a 2  b2  3 .


 
c 2  a2  b2  3
2 
x  y  3  c


• Hằng đẳng thức ( x + y − z ) = x 2 + y 2 + z 2 + 2 xy − 2 xz − 2 yz
2

( a + b − 1)
2
− a 2 − b2 =3 ⇔ a 2 + b2 + 1 + 2 ab − 2 a − 2b − a 2 − b2 =3
⇔ 2 ab − 2 a − 2b =2 ⇔ ab − a − b =1 ⇔ a ( b − 1) − ( b − 1) =2
⇔ ( a − 1)( b − 1) =
2.
• Phương trình ước số ( a − 1)( b − 1) =2 =2.1 =1.2 =( −2 ) . ( −1) =( −1) . ( −2 ) .
Vì a; b ≥ 0 nên ( a − 1) ; ( b − 1) ≥ −1 .
=a − 1  a 3 =
2 = x 9
  
= b −1 1 = b 2 = y 4
Ta có   
⇔ ⇒  .
a − 1
= 1 = a 2 = x 4
  
b − 1
=  b 3 =
2 =   y 9
Vậy ( x; y ) ∈ ( 9; 4 ) , ( 4; 9 ) .
x y z
3). Ta có P   
y  z4 z x4 x y4
4x 4y 4z
P  
4 y  z4 4 z x4 4 x y4
4x 4y 4z
  
y  z44 x z44 x y44
 x y z 
 4     6 .
 y  z x  z x  y 
x  4
Dấu = xảy ra khi x  y  z  4   .
 y  9
Nhận xét: Bài toán sử dụng kỹ thuật chọn điểm rơi, kết hợp với bất đẳng thức Cosi đưa về
được một bất đẳng thức quen thuộc gọi là bất đẳng thức Nesbitt, bất đẳng thức có tới 45
cách chứng minh từ đó tìm ra giá trị nhỏ nhất của biểu thức đã cho.
Nhắc lại kiến thức và phương pháp:
• Bất đẳng thức Cosi cho hai số thực dương a  b  2 ab .
• Bất đẳng thức Cosi cho ba số thực dương a  b  c  3 3 abc .
• Bất đẳng thức Nesbitt cho ba số thực dương a; b; c là

Liên hệ tài liệu word toán sđt và zalo: 039.373.2038 TÀI LIỆU TOÁN HỌC
202
Website:tailieumontoan.com
a b c 3
   .
bc ca ab 2
Chứng minh: Bất đẳng thức đã cho tương đương với:
 a   b   c  9
  1    1    1 
 b  c   c  a   a  b  2
abc bca cab 9
   
bc ca ab 2
 1 1 1  9
 a  b  c   
 a  b b  c c  a  2
 1 1 1 
 a  b  b  c  c  a     9 .
  a  b b  c c  a 
Áp dụng bất đẳng thức Cosi cho ba số thực dương, ta có:
a  b  b  c  c  a  3 3 a  bb  cc  a
1 1 1 3
  
ab bc ca 3
a  bb  cc  a
Nhân hai bất đẳng thức trên với nhau suy ra điều phải chứng minh.
Ý tưởng: Bài toán là một bất đẳng thức đối xứng, vai trò các biến x; y; z là như nhau,
không khó để thấy nếu P  m , dấu đẳng thức xảy ra khi và chỉ khi x  y  z  k . Với giá trị
x  y  z  k thay ngược lại P , ta có:
3k
P  m  9 k 2  2 m2 k  4 m2  0 (*).
2k  4
Để bất phương trình (*) có nghiệm với mọi m , k thì (*)  0 khi đó ta được
m4  36 m2  0  m2 m2  36  0  m2  36  m  6 . Do đó ta tìm được P  6 và dấu đẳng
thức xảy ra khi k  4  x  y  z  4 . Bây giờ, quan sát biểu thức P , chứa ba phân thức
đồng thời căn thức xuất hiện ở mẫu số mỗi phân thức, bằng cách nào đó ta sẽ đánh giá
khử căn bậc hai. Với điểm rơi đã tìm được là x  y  z  4 , ta thấy y  z  4  4 , vì vậy đã
khử căn bậc hai ta áp dụng bất đẳng thức Cosi như sau:
yz
 y  z  4  4  2 4. y  z  4  4 y  z  4  y  z  4  .
4
Tương tự cho các biểu thức còn lại, ta suy ra:
 x y z  x y z 3
P  4      6     (*).
 y  z z  x x  y  yz zx xy 2
Và nếu chứng minh được bất đẳng thức (*) bài toán sẽ được hoàn thành. Nhận thấy nếu
lấy tử cộng mẫu trong từng phân thức của (*) ta sẽ được đại lượng x  y  z chung, chính
vì thế, mỗi phân thức ta sẽ cộng thêm 1 khi đó, ta có (*)
xyz xyz xyz 9
   
yz zx xy 2
 1 1 1  9
  x  y  z   
 x  y y  z z  x  2
 1 1 1 
  x  y   y  z   z  x     9 .
   x  y y  z z  x 

Liên hệ tài liệu word toán sđt và zalo: 039.373.2038 TÀI LIỆU TOÁN HỌC
203
Website:tailieumontoan.com
Đặt a  x  y; b  y  z; c  z  x ta có:
1 1 1 1
a  b  c    3 3 abc .3 3  9  (*) luôn đúng (điều phải chứng minh).
a b c abc
Bài toán kết thúc.
Bài tập tương tự:
1. Cho x; y; z là các số thực dương lớn hơn 1 . Tìm giá trị nhỏ nhất của biểu thức
x y z
P   .
yz2 z x2 x y2
2. Cho x; y; z là các số thực dương lớn hơn 3 . Tìm giá trị nhỏ nhất của biểu thức
2x y z
P   .
y  z6 z  2x  6 2x  y  6

Câu 3.

1). Gọi P là điểm đối xứng của A qua M


 HP  HM  MB  2 HM  AH  HN
 H là trung điểm của NP .
Mà BH  NP , suy ra tam giác PNB cân tại B  BN  BP .
Mặc khác lại có M là trung điểm của BC ; AP . Do đó tứ giác ACPB là hình bình hành, suy
ra AC  BP  AC  BN .
Nhận xét: Chứng minh hai đoạn thẳng bằng nhau ta chứng minh cho chúng cùng bằng
một đoạn thẳng thứ ba
Nhắc lại kiến thức và phương pháp:
• Hai điểm đối xứng nhau qua một điểm thì điểm đó là trung điểm của đoạn thẳng nối
hai điểm đã cho.
P là điểm đối xứng với A qua M nên M là trung điểm của AP do đó ta có
PM = AM ⇔ PM + HM = AM + HM
Liên hệ tài liệu word toán sđt và zalo: 039.373.2038 TÀI LIỆU TOÁN HỌC
204
Website:tailieumontoan.com
⇔ PH = AH + HM + HM = AH + 2 HM = AH + AN = HN
⇒ H là trung điểm của PN .
• Tam giác có đường trung tuyến cũng là đường cao thì tam giác đó là tam giác cân.
Tam giác ∆PNB có BH vừa là đường trung tuyến H là trung điểm của PN ) vừa là
đường cao ( BH ⊥ NP ) nên ∆PNB cân tại B
• Tam giác cân có hai cạnh bên bằng nhau.
Tam giác ∆PNB cân tại B nên BP = BN .
• Tứ giác có hai đường chéo cắt nhau tại trung điểm mỗi đường là hình bình hành.
Tứ giác ACPB có M vừa là trung điểm của BC vừa là trung điểm của AP nên ACPB
là hình bình hành.
• Hình bình hành có các cạnh đối diện bằng nhau.
 AC = PB
ACPB là hình bình hành nên AC = PB , suy ra  BN (điều phải chứng
⇒ AC =
 BP = BN
minh).
  APB
2). Do tứ giác ACPB là hình bình hành, suy ra PAC .
  ANB
Mà tam giác PBN cân tại B  APB   ANB
  PAC

  BNQ
 CAN 
Ta có AC  NB; NQ  AN
  NCD
BNQ CAN  NBD   N ; B; C ; D cùng thuộc một đường tròn C ; G là giao điểm

     BQG

DQG với DBC , suy ra CAG .

 GBQ ∽GCA  GA  GQ  GA  GQ .
  GCA
Mà GBQ
AC QB NB NC
  BDC
Mà BNC   AGQ
 NBC ∽GAQ
  NCB
 GQA   NCB
  GDC
  GC  NB  NG  BC .
Nhận xét: Chứng minh hai đoạn thẳng bằng nhau ta chứng minh cho chúng cùng bằng
một đoạn thẳng thứ ba.
Nhắc lại kiến thức và phương pháp:
• Hình bình hành có hai cạnh đối diện song song.
  APB
ACPB là hình bình hành nên AC  PB suy ra PAC  (hai góc so le trong).
• Tam giác cân có hai góc ở đáy bằng nhau.
Tam giác PBN cân tại B nên APB = ANB
 , suy ra PAC
 = ANB
 hay CAN  = BNQ.

Có AC  NB (chứng minh phần 1).


NQ  AN ( Q đối xứng với A qua N ).
• Hai tam giác có một góc bằng nhau và hai cạnh kề góc tương tứng bằng nhau thì bằng
nhau theo trường hợp “cạnh - góc - cạnh” (c – g – c).
Xét ∆CAN và ∆BNQ có:
+ CA = NB ;
 = BNQ
+ CAN ;
  BQG
+ AN = NQ ; CAG 

Suy ra ∆CAN =  = NBQ


∆BNQ (c – g – c), suy ra ACN  hay DCN
 = NBD
.

Liên hệ tài liệu word toán sđt và zalo: 039.373.2038 TÀI LIỆU TOÁN HỌC
205
Website:tailieumontoan.com
• Tứ giác có hai đỉnh liên tiếp cùng nhìn một cạnh dưới hai góc bằng nhau là tứ giác nội
tiếp.
 = NBD
Tứ giác NBCD có hai đỉnh B; C liên tiếp cùng nhìn cạnh DN với hai góc DCN 

nên NBCD là tứ giác nội tiếp hay N ; B; C ; D cùng thuộc một đường tròn.
  BQG
3). Ta có CAG .

 GBQ ∽GCA  GA  GQ  GA  GQ ;
  GCA
Mà GBQ
AC QB NB NC
  
và BNC  BDC  AGQ NBC ∽GAQ ;
  NCB
 GQA   NCB   GC  NB  NG  BC .
  GDC
Nhận xét: Chứng minh hai đoạn thẳng bằng nhau ta chứng minh cho chúng cùng bằng
một đoạn thẳng thứ ba
Nhắc lại kiến thức và phương pháp:
• Tứ giác nội tiếp có góc trong bằng góc ngoài tại đỉnh đối diện.
  BQG
DNGB là tứ giác nội tiếp nên CAG .

• Hai góc nội tiếp cùng chắn một cung thì bằng nhau.
  GCA
GBQ  (hai góc nội tiếp cùng chắn cung DG
 của đường tròn ngoại tiếp tứ giác

BCGD ).
• Hai tam giác có hai cặp góc bằng nhau thì đồng dạng theo trường hợp “góc - góc” (g –
g); Tam giác đồng dạng có các cặp cặp tương ứng tỷ lệ và các góc tương ứng bằng
nhau.
Xét GBQ và GCA có:
  BQG
+ CAG ;
  GCA
+ GBQ ;
GA GQ GA GQ
Suy ra GBQ ∽GCA (g – g), suy ra    .
AC QB NB NC
• Hai tam giác có hai góc bằng nhau và hai cặp cạnh kề góc tương ứng tỷ lệ thì đồng
dạng theo trường hợp “cạnh - góc - cạnh” (c – g – c); Tam giác đồng dạng có các cặp
cặp tương ứng tỷ lệ và các góc tương ứng bằng nhau.
Xét NBC và GAQ có:
GA GQ
+  ;
NB NC
  AGQ
+ BNC   BDC
 ;
 
  NCB
Suy ra NBC ∽GAQ (c – g – c), suy ra GQA ,
  GDC
 NCB   GC  NB  NG  BC .
Câu 4. Giả sử trên mặt phẳng có n điểm thẳng hàng thì tồn tại một đường thẳng.Theo bài
ra các điểm đã cho không cùng nằm trên một đường thẳng nên tồn tại ít nhất một điểm
không cùng nằm trên đường thẳng đó nối điểm đó với n  1 điểm đã cho ta được n  1
đường thẳng với đường thẳng đi qua n  1 điểm ta được n đường thẳng, thay n  2015 thì
tồn tại ít nhất 2015 đường thẳng.
Nhận xét: Bài toàn tư duy từ việc lập đường thẳng đi qua các điểm kết hợp với tính toán
số đường thẳng được tạo thành.
Liên hệ tài liệu word toán sđt và zalo: 039.373.2038 TÀI LIỆU TOÁN HỌC
206
Website:tailieumontoan.com
Nhắc lại kiến thức và phương pháp:
• Với n điểm thẳng hàng luôn tồn tại một đường thẳng duy nhất đi qua n điểm đó.
• Với n 1 điểm thẳng hàng và 1 điểm nằm ngoài đường thẳng đi qua n 1 điểm đó.
Khi đó từ 1 điểm đó ta kẻ được n 1 đường thẳng tới n 1 điểm. Trường hợp này ta
có n đường thẳng được tạo thành. Đây là trường hợp có ít đường thẳng được tạo ra
nhất.
Thay n  2015 ta có 2015 đường thẳng phân biệt được tạo thành.

ĐỀ SỐ 24.

Câu 1.
1). x  1  x  3 xác định  x  1 và x  3 đồng thời xác định.
x  1 xác định  x  1  0  x  1 ; x  3 xác định  x  3  0  x  3 .
Vậy điều kiện xác định của biểu thức x  1  x  3 là x  3 .
Nhận xét: Bài toán tìm điều kiện xác định theo quy tắc biểu thức dưới dấu căn không âm.
Nhắc lại kiến thức và phương pháp:
• Một biểu thức xác định khi mỗi phân thức, đa thức, đơn thức trong đó xác định.
x  1  x  3 xác định khi và chỉ khi x  1 và x  3 xác định.
• Biểu thức dưới dấu căn không âm.
x  1  0 
  x  1
Ta có x  1 và x  3 xác định khi và chỉ khi 
 


x  3  0 
 x  3

x3.
2 2
2). Với x  2 2 , ta có A  2 2  3  3  2 2   2 1    
2 1

 2  1  2 1    
2 1  
2 1  2 .

Nhận xét: Bài toán tìm giá trị của biểu thức khi biết giá trị của ẩn số.
Nhắc lại kiến thức và phương pháp:
• Đối chiếu giá trị của biến với điều kiện xác định đã tìm được.
Với x  2 2 thỏa mãn x  3
• Thay giá trị của biến vào biểu thức.
Thay x  2 2 vào biểu thức A ta được A  2 2  3  3  2 2
2
• Hằng đẳng thức bình phương của một tổng a 2  2 ab  b2  a  b và hằng đẳng thức
2
bình phương của một hiệu a 2  2 ab  b2  a  b .

A  2 2  3  32 2
2 2
 1  2.1. 2   2  1  2.1. 2   2
2 2
 1  2   1  2 
• Hằng đẳng thức a2  a .

Liên hệ tài liệu word toán sđt và zalo: 039.373.2038 TÀI LIỆU TOÁN HỌC
207
Website:tailieumontoan.com
2 2
A 1  2   1  2   1  2  1 2

• So sánh hai căn bậc hai với a , b không âm ta có a  b thì a b.


2  1 0  2  1  2  1  2 1 0

a khi a  0
• Quy tắc phá (bỏ) dấu giá trị tuyệt đối a   .

a khi a  0

+ 1  2  0 nên 1  2  1  2 ;

+ 2  1  0  1  2  0 nên 1  2  2  1 ;

Suy ra A  1  2  1  2  1  2     
2 1  1 2  2  1  2 .

3). Hoành độ của điểm cần tìm là nghiệm phương trình 2 x 2  8


 x  2 .

Vậy có hai điểm thỏa mãn là: 2; 8 và 2; 8 .

Nhận xét: Bài toán tìm một điểm thuộc một đồ thị hàm số biết tung độ của điểm đó.
Nhắc lại kiến thức và phương pháp:
• Một điểm nằm trên đồ thị của một hàm số có tọa độ thỏa mãn hàm số đó.

Điểm này nằm trên đồ thị của hàm số y  2 x 2 có tọa độ  x; 2 x 2  .

• Thay giá trị số của điểm đó vào giá trị theo ẩn để tìm giá trị của ẩn số.

Điểm này có tung độ bằng 8 nên ta có 2 x 2  8  x 2  4  x  2

Vậy có hai điểm thỏa mãn yêu cầu bài toán là 2; 8 và 2; 8 .

4). Vì tam giác ABC vuông tại A nên AC  BC 2  AB2  52  32  4 .

 AC 4
Do đó cos ACB  .
BC 5

Nhận xét: Bài toán tìm giá trị cos của một góc áp dụng hệ thức giữa cạnh và góc trong tam
giác vuông.
Nhắc lại kiến thức và phương pháp:
• Định lý Py-ta-go trong tam giác vuông: “Bình phương cạnh huyền bằng tổng bình

phương hai cạnh góc vuông”.

ABC vuông tại A nên BC 2  AC 2  AB2  52  AC 2  32

 AC 2  52  32  25  9  16  AC  16  4 .

ck
• Hệ thức giữa cạnh và góc trong tam giác vuông cos   với  là góc nhọn tam giác
ch

vuông; ck là cạnh góc vuông kề góc  và ch là cạnh huyền.

Liên hệ tài liệu word toán sđt và zalo: 039.373.2038 TÀI LIỆU TOÁN HỌC
208
Website:tailieumontoan.com

 CA 4
ABC vuông tại A có cos ACB  .
CB 5

Câu 2.

1). Với điều kiện x  0 và x  1 , ta có:

Q

 x 1

 
2   x x 1
 . 
 1 x 




  
x  1 x  1 x  1    
x 1 x 1  x 

 
 x 1 
2    
1   x  1  x  1 x 1
   . x      .   .
 x 1 x  1  
x   x  1   x  x
Nhận xét: Bài toán rút gọn chứa biến ta áp dụng các ,…
Nhắc lại kiến thức và phương pháp:
• Điều kiện xác định: Biểu thức dưới mẫu khác 0 và biểu thức dước dấu căn bậc chẵn
không âm.
x  0 
 x  0
 
 x  1  0  x  1  0
x  1  0  x  1  0 x  0
    .
  x  1

 x  1  0  x  1  0
 
 x  x  0  x 1  x  0
   
• Phân tích các tử thức và mẫu thức thành nhân tử.
 1 2   x  x 1  x 
Q    .  
 x  1 x  1  x  1 x  x 

 1
 
2
  x x 1
 
. 
1 x  

.
 x 1
  x 1  
x  1  
 
x 1 x 1 x   

• Giản ước các phân thức.
 1 2   x  x 1  x 
Q    .  
 x  1 x  1  x  1 x  x 
 
 1 2   1 
   . x   .
 x 1
  x 1  x  1  


 x 

• Quy đồng mẫu thức các phân thức.


 1 2   x  x 1  x 
Q    .  
 x  1 x  1  x  1 x  x 
 
 x 1 2   x 1 
   .   .

  x 1  x 1   x 1   
x  1   x

x 

• Cộng, trừ các phân thức cùng mẫu.


 1 2   x  x 1  x 
Q    .  
 x  1 x  1  x  1 x  x 

Liên hệ tài liệu word toán sđt và zalo: 039.373.2038 TÀI LIỆU TOÁN HỌC
209
Website:tailieumontoan.com

x 1 x 1 x 1
 .  .
 x 1  x 1 x x

x 1
2). Với x  0 và x  1 ta có Q  .
x
x 1
Do đó Q  1   1  x  1   x .
x
1
 2 x 1 x (thỏa mãn điều kiện).
4
1
Vậy với x  thì Q  1 .
4
Nhận xét: Bài toán rút gọn chứa biến ta áp dụng các ,…
Nhắc lại kiến thức và phương pháp:
• Nếu lại điều kiện xác định bài toán.
ĐKXĐ: x  0; x  1
• Giá trị của biểu thức bằng một số.
x 1
Với Q  1 ta có  1
x
• Giải phương trình chứa ẩn ở mẫu.
x 1 1 1
 1  x  1   x  2 x  1  x   x
x 2 4
• Đối chiếu giá trị tìm được của biến với điều kiện xác định ban đầu.
1
Với x  thỏa mãn x  0; x  1 nên nhận giá trị này.
4
1
Vậy: Q  1 thì x  .
4
Câu 3.
1).
a). Với m  3 , ta có phương trình (1) trở thành x 2  4 x  3  0
Ta có a  b  c  1  4  3  0 nên phương trình có 2 nghiệm phân biệt x1  1; x2  3
Vậy với m  3 , phương trình đã cho có 2 nghiệm phân biệt x1  1; x2  3 .
b). Ta có x 2  2 m  1 x  m2  6  0 (1).
Phương trình (1) là phương trình bậc 2 ẩn x có
  m  1  m2  6  7  2 m .
2

7
Phương trình (1) có các nghiệm x1 ; x2 khi   0  7  2 m  0  m  (*).
2

x  x2  2 m  1
Khí đó theo định lý Vi-et ta có  1 .
2 
x1 .x2  m  6


Do đó x12  x2 2   x1  x2   2 x1 x2  4 m  1  2 m2  6  2 m2  8 m  16 .
2 2

m  0
Vậy x12  x2 2  16  2 m2  8 m  16  16   .
m  4
Kết hợp điều kiện (*), ta có m  0 là giá trị thỏa mãn.
Liên hệ tài liệu word toán sđt và zalo: 039.373.2038 TÀI LIỆU TOÁN HỌC
210
Website:tailieumontoan.com
 x  2  x  y  3  y (1)

2). Ta có  2 .
x   x  32 x  y  5  x  16 (2)
x  2  0 x  2
Điều kiện:   .
 y  0  y  0
Với x  2; y  0 , phương trình (1)  x  2  x  y  2  x  2  y  0
 2 2 
 x2 

   y    
x2  x2 y 0 
 x2  y 0  y x2
(do x2  
x  2  y  1  0, x  2; y  0 ).
Thay y  x  2 vào phương trình (2) ta được phương trình
x 2   x  32 x   x  2  5  x  16  x 2   x  3  x  16
2

x  1

2
 2 x  5x  7  0   .
 x   7 ( l)
 2
Với x  1  y  3 .
Vậy hệ phương trình đã cho có nghiệm  x; y  1; 3 .
Câu 4.

1).
  ANH
a). Xét đường tròn ( I ) có AMH   90 0 (góc nội tiếp chắn nửa đường tròn) nên
HM ; HN tương ứng là đương cao của tam giác vuông ABH ; ACH .
+) ABH vuông tại H , có đường cao HM nên AM.AB  AH 2 .
+) ACH vuông tại H , có đường cao HN nên AN .AB  AN .AC .
Nhận xét: Chứng minh đẳng thức có hai vế là tích hai đoạn thẳng sử dụng hệ thức lượng
trong giác giác vuông.
Nhắc lại kiến thức và phương pháp:
• Góc nội tiếp chắn nửa đường tròn là góc vuông
Đường tròn ( I ) có đường kính AH và các điểm M ; N nằm trên đường tròn ( I ) nên ta
 ; ANH
có AMH  là các góc vuông hay AM  MH và AN  NH

Liên hệ tài liệu word toán sđt và zalo: 039.373.2038 TÀI LIỆU TOÁN HỌC
211
Website:tailieumontoan.com
• Trong tam giác vuông, bình phương một cạnh góc vuông bằng tích cạnh huyền với
hình chiếu của cạnh góc vuông đó trên cạnh huyền
 ABH vuông tại H , đường cao HM nên AH 2  AM.AB
 ACH vuông tại H , đường cao HN nên AH 2  AN .AC
Suy ra AM.AB  AN .AC (đpcm)

AM AN
b). Theo câu a) ta có AM.AB  AN .AC  .
AC AB
Xét AMN và ACB có A chung, AM  AN , nên AMN ∽ACB (c – g - c).
AC AB
       AMN
Do đó AMN  ACB  BCN  BMN  ACB  BMN   BMN
  180 0 .
 ; BMN
Mà các góc BCN  ở vị trí đối diện nên tứ giác BMNC nội tiếp.

Nhận xét: Chứng minh một tứ giác nội tiếp bằng cách chứng minh tứ giác đó có góc trong
bằng góc ngoài tại đỉnh đối diện.
Nhắc lại kiến thức và phương pháp:
d a
• Tính chất tỷ lệ thức a.b  c.d   với a , b , c , d là các số khác 0
b c
AM AN
Ta có AM.AB  AN .AC   .
AC AB
• Hai tam giác có hai góc bằng nhau và cặp cạnh kề góc đó tương ứng tỷ lệ thì hai tam
giác này đồng dạng theo trường hợp “cạnh – góc - cạnh” (c – g – c).
Xét AMN và ACB có
 chung;
+ A
AM AN
+  ;
AC AB
Suy ra AMN ∽ACB (c – g – c).
• Hai tam giác đồng dạng có các góc tương ứng bằng nhau
  ACB
AMN ∽ACB  AMN  hay AMN 
  NCB

• Tứ giác có góc trong bằng góc ngoài tại đỉnh đối diện là tứ giác nội tiếp
Tứ giác BMNC có góc trong NCB và góc ngoài AMN  thỏa mãn AMN
  NCB
 nên tứ giác
BMNC là tứ giác nội tiếp (điều phải chứng minh).
2).
a). Ta có tam giác ABC vuông tại A và O là trung điểm của cạnh BC nên
  OCA
OA  OB  OC OAC cân tại O  OAC   OAC
  BCN
.
  ACB
Mà AMN   BCN
 nên AMN  OAC  AMN
  DAN
.
Vì AMN vuông tại A nên
  ANM
AMN   90 0  DAN
  ANM
  90 0  ADN
  90 0 .
  90 0  MN là đường kính của đường tròn ( I )  I là trung điểm của MN nên
Mà MAN
  90 0 .
ADI
  AHO
Xét AID và AOH có ADI  chung do đó AID ∽AOH (g - g).
  90 0 và A

Nhận xét: Chứng minh hai tam giác đồng dạng theo trường hợp “góc - góc”
Nhắc lại kiến thức và phương pháp:
Liên hệ tài liệu word toán sđt và zalo: 039.373.2038 TÀI LIỆU TOÁN HỌC
212
Website:tailieumontoan.com
• Trong tam giác vuông, đường trung tuyến ứng với cạnh huyền bằng nửa cạnh huyền.
BC
ABC vuông tại A và O là trung điểm của cạnh BC nên OA  OB  OC  .
2
• Tam giác có hai cạnh bằng nhau là tam giác cân. Tam giác cân có hai góc kề cạn đáy
bằng nhau.
  OCA
OAC có OA  OC nên OAC cân tại O suy ra OAC  hay OAC
  BCN
 mà
  AMN
BCN  nên OAC  AMN
.
• Tam giác vuông có tổng hai góc nhọn bằng 90 . Trong tam giác có tổng hai góc bằng
90 là tam giác vuông.
  ANM
 AMN vuông tại A nên AMN   90
  ANM
 OAC   90  DAN
  AND
  90 .
  AND
 ADN có DAN   90 nên ADN vuông tại D hay AO  MN tại D .
• Hai tam giác có hai cặp góc tương ứng bằng nhau thì đồng dạng theo trường hợp “góc
- góc” (g – g).
Xét AID và AOH có:
  AHO
+ ADI  (  90 0 );

 : chung;
+ A

Suy ra AID ∽AOH (g – g) (điều phải chứng minh).


AD AI 1 AO
b). Vì ADI ∽AHO     .
AH AO AD AH . AI
1 1 1 BC
Mà AO  BC ; AI  AH   .
2 2 AD AH 2

Mặt khác, vì tam giác ABC vuông tại A và AH là đường cao nên AH 2  HB.HC , suy ra
1 HB  HC 1 1
   .
AD HB.HC HB HC
Nhận xét: Chứng minh một đẳng thức dựa vào các mối quan hệ hình học
Nhắc lại kiến thức và phương pháp:
• Hai tam giác đồng dạng có các cặp cạnh tương ứng tỷ lệ
AD AI AH . AI 1 AO
AID ∽AOH    AD   
AH AO AO AD AH . AI

 1 1 1

 AO  BC BC BC
 2 1 2 1 BC
Mà     2  

 1 AD 1 1 AD AH 2
 AI  AH AH . . AH AH 2


 2 2 2

• Trong tam giác vuông, bình phương đường cao bằng tích hai hình chiếu của hai cạnh

góc vuông trên cạnh huyền.

ABC vuông tại A và AH là đường cao nên AH 2  HB.HC ; suy ra

1 BC BH  HC BH HC
   
AD HB.HC BH .HC BH .HC BH .HC

Liên hệ tài liệu word toán sđt và zalo: 039.373.2038 TÀI LIỆU TOÁN HỌC
213
Website:tailieumontoan.com
1 1 1
   (điều phải chứng minh).
AD HC BH

3). Vì tứ giác BMNC nôi tiếp


  MNC
 PBM   ANM
  PBM   MNC   ANM  180 0 (1).
  ANM
Vì tứ giác ANMK nôi tiếp  PKM  (2).
  PKM
Từ (1) và (2), suy ra PBM   180 0 , do đó tức giác PKMB nội tiếp

  PMB
 PKB   AMN  ACB
  AKB
  ACB
  AKB
  PKB
  180 0 .
  BAC
Do đó tứ giác BKAC nội tiếp  BKC   90 0 .

Nhận xét: Chứng minh một góc có số đo bằng 90 ta đi chứng minh góc đó bằng với một
góc khác có số đo bằng 90 .
Nhắc lại kiến thức và phương pháp:
• Tứ giác nội tiếp có góc trong bằng góc ngoài tại đỉnh đối diện.
 và góc ngoài tại đỉnh đối diện
+ Tứ giác BMNC là tứ giác nội tiếp có góc trong MNC
 nên PBM
PBM   MNC

  ANM
 PBM   MNC
  ANM
  PBM
  ANM
  180 0 ;

 và góc ngoài tại đỉnh đối diện


+ Tứ giác ANMK là tứ giác nội tiếp có góc trong ANM
 nên ANM
PKM   PKM
;

  PKM
suy ra PBM   180 0 .

• Tứ giác có tổng hai góc đối diện bằng 180 là tứ giác nội tiếp.
  PKM
Tứ giác PKMB có PBM   180 0 nên PKMB là tứ giác nội tiếp.

• Trong một đường tròn, hai góc nội tiếp cùng chắn một cung thì bằng nhau.
  PMB
PKB  của đường tròn ngoại tiếp tứ giác
 (hai góc nội tiếp cùng chắn cung PB

PKMB ).

• Hai góc đối đỉnh thì bằng nhau


 và AMN
PMB  là hai góc đối đỉnh nên PMB
  AMN
;

  ACB
Mà AMN  (chứng minh trên);

suy ra ACB   AKB


  PKB   ACB   PKB
  AKB   180 0 , suy ra tứ giác BKAC là tứ giác nội

  BAC
tiếp, suy ra BKC  (hai góc nội tiếp cùng chắn cung BC
 ).

  90 0 nên BKC
Mà BAC   90 (điều phải chứng minh).

Câu 5.

Liên hệ tài liệu word toán sđt và zalo: 039.373.2038 TÀI LIỆU TOÁN HỌC
214
Website:tailieumontoan.com

3 x 2  6 x  6  0
1). Điều kiện xác định   x  1 3 .

2  x  0

Với x  1  3 , phương trình đã cho tương đương với:
2
3 x 2  6 x  6  3  2  x 2  x  7 x  19 2  x

 3 x 2  6 x  6  2  x 3 x 2  5 x  7 

 3 x 2  6 x  6  2  x  2  x  3 x 2  5 x  8
3x2  5x  8
  2  x  3 x 2  5 x  8
2
3x  6 x  6  2  x
 3x2  5x  8  0

 .

1  2  x 3 x 2  6 x  6  2  x


(do
3 x 2  6 x  6  2  x  0, x  1  3 ).
8
+) 3x 2  5x  8  0  x  1 (thỏa mãn điều kiện) hoặc x  (không thỏa mãn điều kiện).
3
+) 1  2  x  3x2  6 x  6  2  x 
 1  2  x  3 x 2  6 x  6. 2  x
 x  1  3 x 2  6 x  6. 2  x (*).
Vì x  1  3 nên x  1  0  3x 2  6 x  6. 2  x do đó (*) vô nghiệm.
Vậy phương trình đã cho có nghiệm duy nhất x  1 .

2). Ta có: a 4  b4  ab a 2  b2  , a; b  


Thật vậy a 4  b4  ab a 2  b2   a 4  b4  a 3 b  ab3
 a  ba 3  b3   0  a  b a 2  ab  b2   0 (luôn đúng a; b   )
2

Do đó a 4  b4  c  ab a 2  b2   c  a 4  b4  c  ab a 2  b2   abc 2  0 (vì a; b; c  0 và abc  1 ).


c c c c
  (vì c  0 )  4 
a  b  c ab a  b   abc
4 4 2 2 2
a  b  c ab a  b2  c 2 
4 2

c c2 c c2
    (1).
a 4  b4  c abc a 2  b2  c 2  a4  b4  c a2  b2  c 2

b b2
Tương tự  (2).
b4  c 4  a a2  b2  c 2
a a2
và 4  (3).
b  c 4  a a2  b2  c 2
Cộng theo vế các bất đẳng thức (1), (2) và (3), ta có:
a b c a2 b2 c2
     1
b4  c 4  a a4  c 4  b a4  b4  c a2  b2  c 2 a2  b2  c 2 a2  b2  c 2
 T  1, a; b; c  0 thỏa mãn abc  1 .
Với a  b  c  1 thì T  1 . Vậy maxT  1 .

Liên hệ tài liệu word toán sđt và zalo: 039.373.2038 TÀI LIỆU TOÁN HỌC
215
Website:tailieumontoan.com
ĐỀ SỐ 25.

Câu 1.
1). Ta có P  2  3  6  2  42 3   3 1 
2
 32 3 1    3  1 
3 1  3 1 
  3  1 3  1  3  1  2 .

Nhắc lại kiến thức và phương pháp:


• Quy tắc khai phương một tích: Với a , b không âm ta có a.b  a . b .

P  2 3  
6  2  2 3  2.3  2  2  3  2. 3  2 
 2 3. 2. 3  1 .

• Quy tắc nhân hai căn bậc hai: Với a , b không âm ta có a . b  a.b

P  2  3 . 2.  
3 1  2 2 3 .   
3 1  42 3.  3 1 .
2
• Hằng đẳng thức bình phương của một hiệu a 2  2 ab  b2  a  b

P  42 3.  3 1 
2 2
  3  3 2  1.  3 1    3 1 .  3 1 . 

a khi a  0
• Hằng đẳng thức a2  a  
 .

a khi a  0

2
P  3 1 .  3 1   3 1  3 1    3 1  3 1 
(vì 3  1  3  1  1  3  1  0 ).
• Hằng đẳng thức a  ba  b  a 2  b2 .
2
P  3 1  3 1    3 1 2
 3 1  2 .
2 x  y  3 x  6  y  x  3
2). Ta có     .
 
x  y  6 2 6  y  y  3  y  3

Nhận xét: Bài toán giải hệ phương trình bằng phương pháp thế.
Nhắc lại kiến thức và phương pháp:
• Rút một ẩn từ một phương trình.
2 x  y  3 x  y  6 x  6  y
    .
x  y  6 2 x  y  3 2 x  y  3
• Thế ẩn đã rút vào phương trình còn lại rồi giải phương trình đó.

2 x  y  3  x  6  y
 x  6  y x  6  y
   
x  y  6 2 6  y  y  3 12  2 y  y  3 3 y  9

x  6  y x  6  y
   .
3 y  9  y  3
• Thay giá trị của ẩn vừa giải ra vào ẩn đã rút để tìm giá trị của ẩn còn lại.
Liên hệ tài liệu word toán sđt và zalo: 039.373.2038 TÀI LIỆU TOÁN HỌC
216
Website:tailieumontoan.com

2 x  y  3   y  3   y  3
   y  3
  .
x  y  6 x  6  y x  6  3 x  3
Vậy hệ phương trình đã cho có nghiệm  x ; y  3 ;  3 .
Câu 2.
1).
1
+ Với m  0 , phương trình (1) trở thành 2 x  1  0  x  .
2
2
 1 15
+ Với m  0 , ta có   4 m2  m  1  2 m     0 thì phương trình có 2 nghiệm phân
  4  16
biệt.
Vậy phương trình (1) luôn có nghiệm với mọi m .
Nhận xét: Bài toán trên là một phần của bài toán biện luận phương trình dạng
ax 2  bx  c  0 .
Nhắc lại kiến thức và phương pháp:
• Phương trình ax 2  bx  c  0 với a  0 trở thành phương trình dạng bx  c . Phương
trình này có nghiệm duy nhất khi b  0 hoặc có vô số nghiệm khi b  0 và c  0 .
1
Với m  0 phương trình (1) có dạng 2 x  1  0  x  . Do đó phương trình (1) có
2
nghiệm.
• Phương trình ax 2  bx  c  0 với a  0 trở thành phương trình bậc hai của ẩn x .
Phương trình này có nghiệm khi và chỉ khi   0 với   b2  4 ac hay  '  0 với
2 b
 '  b '  ac trong đó b '  .
2
Với m  0 phương trình (1) là phương trình bậc hai ẩn x tham số m .
2
 '  m  1  m 1  3m  m2  2 m  1  m  3m2  4 m2  2 m  1

• Chứng minh một biểu thức không âm.


2
 1 1  15  1  15
  4 m2  m  1  4 m2  2.2 m.     2 m     0 với mọi giá trị của m  0 .
 
4 16  16   4  16
Suy ra phương trình (1) luôn có nghiệm với mọi giá trị của m  0 .
Với m  0 phương trình có nghiệm duy nhất và với m  0 phương trình luôn có nghiệm.
Do đó phương trình luôn có nghiệm với mọi giá trị của m.
2
2). Ta có A  x12  x2 2   x1  x2   2 x1 x2 .

 2 m  1

 x1  x2 
 m
Theo định lý Vi-et, ta có  .

 1  3 m

 x1 .x2 

 m
4 m2  8 m  4 2  6 m 4 m2  8 m  4  2 m  6 m2
Nên A   
m2 m m2
10 m2  6 m  4

m2

Liên hệ tài liệu word toán sđt và zalo: 039.373.2038 TÀI LIỆU TOÁN HỌC
217
Website:tailieumontoan.com
Suy ra Am2  10 m2  6 m  4   A  10 m2  6 m  4  0 .
31
Ta có   4 A  31  0  A  .
4
31 4
Nên min A  khi m  
4 3
Nhận xét: Bài toán áp dụng định lý Vi-ét trong phương trình bậc hai và kiến thức tìm giá
trị lớn nhất, nhỏ nhất.
Nhắc lại kiến thức và phương pháp:

 b

 x1  x2  
 a.
• Định lý Vi-ét trong phương trình bậc hai 

 c
 x1 x2 


 a
Với m  0 phương trình (1) là phương trình bậc hai có hai nghiệm phân biệt x1 và x2 ,

 2 m  1

 x  x 
 1 2
m
ta có 

 1  3 m

 x1 x2 

 m
• Hằng đẳng thức bình phương của một tổng
2 2
a  b  a 2  2 ab  b2  a 2  b2  a  b  2 ab .
2
A  x12  x22   x1  x2   2 x1 x2 kết hợp với các giá trị đã tính được từ Định lý Vi-ét ta
được:
 2 m  1  2
2
A   x1  x2   2 x1 x2     2. 1  3m
 m  m

4 m2  8 m  4 2  6 m 4 m2  8 m  4  2 m  6 m2 10 m2  6 m  4
   
m2 m m2 m2
 Am2  10 m2  6 m  4   A  10 m2  6 m  4  0 (*) với mọi m  0 .
• Tìm giá trị lớn nhất nhỏ nhất của một biểu thức bằng cách sử dụng điều kiện có
nghiệm của phương trình bậc hai.
2
+ Với A  10 ta có m   .
3
+ Với A  10 ta có  '  9  4  A  10  4 A  31 .
31
Để phương trình (*) có nghiệm thì  '  0  4 A  31  0  A  . Dấu “=” xảy ra khi và
4
4
chỉ khi phương trình có nghiệm duy nhất m   .
3
31 4
Vậy, giá trị nhỏ nhất của biểu thức A bằng khi và chỉ khi m   .
4 3
Câu 3. Gọi x (dãy ghế) là số dãy ghế ban đầu  x  N , x  2 .
*

Theo đề bài ta có phương trình


80 80 80 x 80  x  2 2 x  x  2
 2  
x2 x x  x  2 x  x  2 x  x  2

Liên hệ tài liệu word toán sđt và zalo: 039.373.2038 TÀI LIỆU TOÁN HỌC
218
Website:tailieumontoan.com
 x  10
 80 x  80  x  2  2 x  x  2  x 2  2 x  80  0   .
 x  8(l)

Vậy lúc đầu có 10 dãy ghế, mỗi dãy ghế có 8 người ngồi.
Nhận xét: Giải bài toán bằng cách lập phương trình.
Nhắc lại kiến thức và phương pháp:
• Gọi ẩn và đặt điều kiện cho ẩn.
Gọi x (dãy ghế) là số dãy ghế ban đầu.
Vì x số dãy ghế nên x phải là số tự nhiên khác 0 (hoặc số nguyên dương). Từ đó có
điều kiện x  N * .
Vì đề bài cho “Nếu bớt đi 2 dãy ghế” thì vẫn phải còn ghế nên suy ra phải có nhiều
hơn 2 dãy ghế. Từ đó có điều kiện x  2 .
• Dựa vào các dữ kiện cho ở đề bài để lập được phương trình.
80
Ban đầu, có 80 người và x dãy ghế nên mỗi dãy ghế có người
x
80
Sau khi bỏ đi 2 dãy ghế, có 80 người và x  2 dãy ghế nên mỗi dãy ghế có người.
x2
Sau khi bỏ đi 2 dãy ghế, mỗi dãy ghế phải ngồi thêm 2 người thì mới đủ chỗ nên ta có
80 80
phương trình  2 .
x2 x
• Giải phương trình và so sánh với điều kiện rồi đưa ra kết luận.
80 80
  2  40 x  40  x  2  x  x  2  40 x  40 x  80  x 2  2 x
x2 x
 x  8
 x 2  2 x  80  0   x  8 x  10  0   kết hợp với điều kiện ban đầu, ta có
 x  10

x  10 .
80
Do đó ban đầu có 10 dãy ghế, mỗi dãy ghế có  8 người ngồi.
10
Câu 4.

B
1). Ta có A   90 0 , cộng lại bằng 2V .
Nhận xét: Chứng minh một tứ giác là tứ giác nội tiếp bằng cách chứng minh tứ giác đó có
tổng hai góc trong đối diện bằng 180 .
Nhắc lại kiến thức và phương pháp:
• Tiếp tuyến của một đường tròn thì vuông góc với bán kính tại tiếp điểm.
Liên hệ tài liệu word toán sđt và zalo: 039.373.2038 TÀI LIỆU TOÁN HỌC
219
Website:tailieumontoan.com
  90 .
+ MA là tiếp tuyến của (O) nên MA  AO hay MAO
+ MB là tiếp tuyến của (O) nên MB  BO hay MBO  90 .

• Tứ giác có tổng hai góc trong đối diện bằng 180 là tứ giác nội tiếp.
Tứ giác MAOB có MAO  MBO   90  90  180 nên MAOB là tứ giác nội tiếp một đường
tròn (đpcm).
2). Ta có MCA ∽MAD (g - g), suy ra tỉ số đồng dạng, suy ra điều phải chứng minh.
Nhận xét: Chứng minh đẳng thức có hai vế là tích của hai đoạn thẳng bằng cách sử dụng
tỷ số thông qua tam giác đồng dạng.
Nhắc lại kiến thức và phương pháp:
• Hai tam giác có hai cặp góc bằng nhau thì đồng dạng
Xét MCA và MAD có:
 : chung;
+ CAD
  MDA
+ MAC  (góc tạo bởi tia tiếp tuyến - dây cung và góc nội tiếp cùng chắn cung
 của (O) );
AC
suy ra MCA ∽MAD (g – g).
• Hai tam giác đồng dạng có các cặp cạnh tương ứng tỷ lệ.
MC MA
MCA ∽MAD   .
MA MD
• Tỷ lệ thức
MC MA
  MC.MD  MA 2 (điều phải chứng minh).
MA MD

3). Ta có OH .OM  OA 2 ; MC.MD  MA 2 ; MA 2  OA 2  MO 2 .


Nhận xét: Chứng minh đẳng thức dựa vào mối quan hệ giữa các đoạn thẳng.
Nhắc lại kiến thức và phương pháp:
• Trong tam giác vuông, bình phương cạnh góc vuông bằng tích cạnh huyền và hình
chiếu của cạnh góc vuông đó trên cạnh huyền.
Tam giác OAM vuông tại A , đường cao AH nên ta có OA 2  OM.OH .
• Định lý Py-ta-go: “Trong tam giác vuông, bình phương cạnh huyền bằng tổng bình
phương hai cạnh góc vuông”.
Tam giác OAM vuông tại A nên MA 2  OA 2  MO 2 .
Lại có MC.MD  MA 2 (chứng minh phần 2).
Suy ra VT  OH .OM  MC.MD  OA 2  MA 2  MO 2  VP (điều phải chứng minh).
3x2
Câu 5. Ta có  y 2  z 2  yz  1  3 x 2  2 y 2  2 z 2  yz  2
2
2 2 2
  x  y  z   x  z   x  y   2 .
2 2
Mà  x  z  0;  x  y  0 ,
2
Nên  x  y  z  2  x  y  z  2   2  B  2 .
2
Dấu “=” xảy ra khi x  y  z   , lúc đó min B   2 và max B  2 .
3
Nhận xét: Bài toán tìm giá trị lớn nhất, nhỏ nhất của một biểu thức có giả thiết từ một biểu
thức liên hệ giữa các biến.
Liên hệ tài liệu word toán sđt và zalo: 039.373.2038 TÀI LIỆU TOÁN HỌC
220
Website:tailieumontoan.com
Nhắc lại kiến thức và phương pháp:
• Biến đổi tương đương một đẳng thức.
3x2
 y 2  z 2  yz  1  3 x 2  2 y 2  2 z 2  yz  2
2
  x  y  y 2  2 xy  2 xz  2 yz   x 2  2 xy  y 2   x 2  2 xz  z 2   2 .
2 2

• Hẳng đẳng thức.


+ Bình phương của một hiệu.
2
x 2  2 xy  y 2   x  y
2
x 2  2 xz  z 2   x  z .
+ Bình phương của tổng ba số.
2
x 2  y 2  y 2  2 xy  2 xz  2 yz   x  y  z ,
3x2 2 2 2
suy ra  y 2  z 2  yz  1   x  y  z   x  y   x  z  2 .
2
• Tổng của các số hạng không âm luôn lớn hơn hoặc bằng một số hạng.

 2

 x  y  z  0


Ta có  2
 x  y  0 và
2
 x  y  z
2 2
  x  y    x  z  2 , nên ta được


 2




 x  z  0
2
 x  y  z 2  xyz  2  2 xyz 2
 2 B 2 .
• Khi đánh giá được giá trị biểu thức với giá trị số nào đó, ta phải tìm được điều kiện
xảy ra dấu “=” thì mới được kết luận giá trị lớn nhất, giá trị nhỏ nhất.
2
+ Xảy ra B   2 khi và chỉ khi x  y  z   .
3
2
+ Xảy ra B  2 khi và chỉ khi x  y  z  .
3
2
Vậy: Giá trị lớn nhất của B bằng 2 khi x  y  z  .
3
2
Giá trị nhỏ nhất của B bằng  2 khi x  y  z   .
3

ĐỀ SỐ 26.

Câu 1.
1). Ta có a  b  29  12 5  2 5
2
 3  2 5  2 5  32 5 2 5  3 .

Theo đề ra A  a 3  b3  a 2  b2  11ab  2015


 a  ba 2  b2  ab  a 2  b2  11ab  2015

 3a 2  b2  ab  a 2  b2  11ab  2015

Liên hệ tài liệu word toán sđt và zalo: 039.373.2038 TÀI LIỆU TOÁN HỌC
221
Website:tailieumontoan.com

 4 a 2  2 ab  b2   2015  4 a  b  2015  2051 .


2

Nhận xét: Bài toán kết hợp với việc rút gọn giả thiết đồng thời đưa biểu thức cần tính về
ẩn của giả thiết từ đó tính được giá trị của biểu thức.
Nhắc lại kiến thức và phương pháp:
2 2
• Hằng đẳng thức: a  b  a 2  2 ab  b2 ; a  b  a 2  2 ab  b2 và
a 3  b3  a  ba 2  ab  b2  .

• Cho a 2  a  a nếu a  0 hoặc a 2  a  a nếu a  0 .

Và 3
b 3  b; b   .
Ý tưởng: Đây là một bài toán cơ bản nhưng đòi hỏi tư duy cao, trước hết ta thấy rằng giả
thiết chứa các căn thức khá rắc rối vì thế ta cần đi rút gọn giả thiết, dễ thấy 29  12 5 nằm
trong căn bậc hai, do đó ta sẽ biến đổi nó về dạng chính phương như sau:
x 2  y 2  29 x  3
29  12 5   x  y  x 2  2 xy  y 2    
2

2 xy  12 5  y  2 5
 
2 2

 29  12 5  3  2 5   29  12 5  2 5  3  2 5  2 5  3.

Hay nói cách khác, từ giả thiết ta có: a  b  3 . Có cái mẹo rất hay ở đây là, ta sẽ chọn hai
giá trị a; b bất kỳ thỏa mãn a  b  3 như a  4; b  1 . Khi đó sẽ tính được A  2051 . Vậy nên
ta sẽ chọn cách biến đổi để tìm ra giá trị này, biến đổi biểu thức sao cho xuất hiện hiệu
a  b , do đó ta nhóm như sau:
A  a 2 a  1  b2 b  1  11ab  2015  a 3  b3  a 2  b2  11ab  2015
 a  ba 2  ab  b2   a 2  b2  11ab  2015

 3a 2  ab  b2   a 2  b2  11ab  2015

 4 a 2  2 ab  b2   2015  4 a  b  2015  4.32  2015  2051 .


2

Thật ra, khi tìm được a  b  3 ta sẽ rút a  b  3 và thế vào biểu thức A , sau đó khai triển
các tích ra ta cũng sẽ thấy biến b tự triệt tiêu cho nhau và giá trị của A  hằng số.
Bài toán kết thúc.
Bài tập tương tự:
1. Cho a  b  9  4 5  5 . Tính giá trị của biểu thức
P  a 3  b3  6 ab  2016 .
2. Cho a  b  14  6 5  5 . Tính giá trị của biểu thức
A  a 2 ( a  1)  b2 (b  1)  11ab  2016 .

2). Ta có xy  1  x 1  y   1  1  x 1  y   1  xy


2 2 2 2

 1  x 1  y   1  xy
2 2 2

 1  x 2  y 2  x 2 y 2  1  2 xy  x 2 y 2
2
 x 2  y 2  2 xy  0   x  y  0  y  x

 x 1  y 2  y 1  x2  x 1  x2  x 1  x2  0 .

Liên hệ tài liệu word toán sđt và zalo: 039.373.2038 TÀI LIỆU TOÁN HỌC
222
Website:tailieumontoan.com
Nhận xét: Bài toán hay ở chỗ khai thác triệt để giả thiết, vì giả thiết là manh mối quyết
định bài toán, khi tìm được x  y thì việc chứng minh trở nên rất đơn giản.
Nhắc lại kiến thức và phương pháp:
• Cách giải phương trình vô tỷ dạng cơ bản: f  x  g  x  h  x

h  x  f  x

 g  x  h  x  f  x   .




g  x   h 2
 x   2 h  x  f  x   f 2
 x 
Ý tưởng: Đi từ giả thiết cho đến biểu thức cần chứng minh, các đại lượng hoàn toàn đối
xứng nhau, ta thử suy đoán nếu x; y đối xứng tức là x  y khi đó biểu thức cần chứng
minh trở thành 2 x 1  x 2  0  x  y  0 thay ngược lại giả thiết, ta thấy đúng. Tiếp tục thử
với x  y , thay vào biểu thức cần chứng minh lẫn giả thiết, ta thấy luôn đúng. Như vậy
trong hai trường hợp ta xét, trường hợp tổng quát nhất là x  y . Do đó, ta cần biến đổi
giả thiết sao cho x  y . Với tư duy trong cách giải phương trình vô tỷ, sự xuất hiện của
căn thức sẽ làm ta nghĩ đến việc nâng lũy thừa, cụ thể hơn ở đây là bậc hai, hơn thế nữa
khi chuyến vế bình phương đại lượng x 2 y 2 sẽ bị triệt tiêu, nên ta biến đổi giả thiết như
sau:
xy  1  x 1  y   1  1  x 1  y   1  xy
2 2 2 2

1  xy

 
1  x 2 1  y 2   1  xy
2


 1  x 2  y 2  x 2 y 2  1  2 xy  x 2 y 2
2
 x 2  2 xy  y 2  0   x  y  0  x  y .
Với x  y , ta chỉ cần việc thế vào biểu thức cần chứng minh, ta được:
2
x 1  x  x 1  x 2  x 1  x 2  x 1  x 2  0 (điều phải chứng minh).
Ta có thể hoàn toàn tư duy theo kiểu đó là khai thác từ điều phải chứng minh, từ đó truy
ngược giả thiết bài cho.
Bài toán kết thúc.
Bài tập tương tự:
 
1. Cho x; y là hai số thực thỏa mãn x  x 2  1 y  y 2  1  0 , chứng minh rằng 
x 1  y 2  y 1  x2  0 .

 
2. Cho x; y là hai số thực thỏa mãn x  y 2  1 y  x 2  1  0 . Chứng minh rằng 
xy0.

Câu 2.
1
1). Điều kiện: x   .
4
Phương trình đã cho tương đương với
2x  3  x  24 x  1  2 x  2  4x  1 .
Đặt t  2 x  2  4 x  1 ( t  0 ).

Liên hệ tài liệu word toán sđt và zalo: 039.373.2038 TÀI LIỆU TOÁN HỌC
223
Website:tailieumontoan.com
t2  9
 t 2  8x  4 x  24 x  1  9  2 x  x  24 x  1  .
4
t  1 ( l )
Phương trình trở thành: t 2  4t  3  0   .
t  3
1
+ Với t  1  2 x  2  4x  1  1  4x  1  1  2 x  2 (vô nghiệm vì x  thì
4
1  2 x  2  0 ).
+ Với t  3  2 x  2  4 x  1  3
3  2 x  2  0

 4 x  1  3  2 x  2  
4 x  1  9  4  x  2  12 x  2


2  x  1 2
 4  x  .
 9
3 x  2  4
2
Vậy phương trình có nghiệm duy nhất x   .
9
Nhận xét: Bài toán sử dụng phương pháp đặt ẩn phụ đưa về phương trình bậc hai, từ ẩn
phụ suy ngược lại nghiệm của phương trình ban đầu.
Nhắc lại kiến thức và phương pháp:
2 2
• Hằng đẳng thức a  b  a 2  2 ab  b2 ; a  b  a 2  2 ab  b2 .
• Cách giải phương trình bậc hai tổng quát a.t 2  b.t  c  0 , a  0 .
• Giải phương trình chứa căn thức dạng f  x  g  x  m  0
m2  f  x  g  x  0

 2 f  x g  x  m  f  x  g  x  
2
.
4 f  x g  x   m2  f  x  g  x 2
 
 

Ý tưởng: Bài toán khá phức tạp vì chứa ba căn thức, tuy nhiên hướng tư duy đầu tiên là
phương trình chứa nhiều căn, kể cả căn bậc hai hay bậc ba ta sẽ nghĩ đến việc sử dụng ẩn
phụ. Điều đặc biệt trong bài toán này là căn bậc hai bên vế trái chính bằng tích của hai căn
bên vế phải vậy nên ta sẽ đặt a  2 x  2; b  4 x  1; a , b  0 nên 2 4 x 2  9 x  2  2 ab
Tiếp theo, sẽ biểu diễn đại lượng không chứa căn theo a , b , ta thấy rằng
a2  b2  9
a 2  b 2  4  x  2  4 x  1  8 x  9  2 x  . Khi đó thay vào phương trình ban đầu,
4
ta được:
a2  b2  9 ab
a 2  b 2  4  x  2  4 x  1  8 x  9  2 x  3  ab
4 2
2 a  b  1
 a 2  2 ab  b2  9  4 a  b  a  b  4 a  b  3  0  
a  b  3

Với hướng tư duy này, ta có làm gọn như sau: t  2 x  2  4 x  1 , khi đó
t2  9
t 2  8x  9  4 4x2  9x  2  2x  4x2  9x  2  nên phương trình đã cho trở thành:
4
t2  9 t  1 1
 3  t  t 2  4t  3  0  

. Tuy nhiên với điều kiện x    t  2 x  2  7  1
4 t  3 4
do đó t  1 loại, xét với t  3 , ta có: 2 x  2  4 x  1  3  2 x  4 x 2  9 x  2  0
Liên hệ tài liệu word toán sđt và zalo: 039.373.2038 TÀI LIỆU TOÁN HỌC
224
Website:tailieumontoan.com

x  0 2
 4 x 2  9 x  2  2 x  
 2  x  .
 2
9
4 x  4 x  9 x  2

Bài toán kết thúc.
Bài tập tương tự:
1. Giải phương trình x  2  x  2  4 x  15  4 x 2  4 .
97
Đáp số: x  .
36
2. Giải phương trình 5x  4 x 2  1  4 x  1  2 x  1 .
10 4
Đáp số: x   .
3 3
2). Điều kiện: y  2 x  1  0; 4 x  y  5  0; x  2 y  2  0; x  1 .
 y  2 x  1  0 x  1 0  0 
+ TH1: Ta có    (không thỏa mãn).
3  3 x  0  y  1 1  10  1

+ TH 2: Với x  1; y  1 .
Đưa phương trình thứ nhất về dạng tích ta được
x y2
 x  y  22 x  y  1 
y  2 x  1  3  3x
 
 1
  x  y  2  y  2 x  1  0 .
 y  2 x  1  3  3 x 

1
Do y  2 x  1  0 nên  y  2x  1  0  x  y  2  0 .
y  2 x  1  3  3x
Thay y  2  x , vào phương trình thứ 2, ta được
x2  x  3  3x  7  2  x
 x2  x  2  3x  7  1  2  2  x
3x  6 2x  3 1 
  x  2 x  1   b   x  2   1  x  0 .
3x  7  1 2  2  x  3 x  7  1 2  2  x 
3 1
Do x  1 nên   1 x  0 .
3x  7  1 2 2x
Vậy x  2  0  x  2  y  4 .
Nhận xét: Bài toán hay và khó, đòi hỏi nhiều kỹ năng tư duy từ việc đi khai thác phương
trình một ( tách thành tích rồi nhân liên hợp ) tìm mối liên hệ giữa hai biến, sau đó thế vào
phương trình hai nhẩm nghiệm và liên hợp một lần nữa mới tìm được nghiệm của hệ
phương trình.
Nhắc lại kiến thức và phương pháp:
• Hằng đẳng thức: a  b   a b  
a  b , a; b  0 .
• Phương trình ax 2  by 2  cxy  mx  ny  h  0 . Coi đó là phương trình bậc hai ẩn x ta
có: ax 2  cy  m x  by 2  ny  h  0 . Giả sử:
x  cy  m  4 a by 2  ny  h  f 2  y .
2

cy  m  f  y cy  m  f  y
Do đó x  hoặc x  .
a a
Liên hệ tài liệu word toán sđt và zalo: 039.373.2038 TÀI LIỆU TOÁN HỌC
225
Website:tailieumontoan.com
Ý tưởng: Bài toán khá là dài và rắc rối, cả hai phương trình đều chứa cân thức và các đa
thức bậc hai, ta chưa định hướng được sẽ đi theo con đường nào. Tuy nhiên, nếu quan sát
kỹ, phương trình một phức tạp nhưng đáng để ta khai thác vào nó. Trước hết, xét vế trái
của phương trình một, thực chất đây là một phương trình bậc hai, vậy nghĩ đến phương
trình bậc hai là ta nghĩ đến việc xét biệt thức  , còn với bài này ta cũng chưa khẳng định
được  có chính phương hay không nhưng hãy mạnh dạn xét nó. Coi đa thức
f  x; y  2 x 2  y 2  xy  5 x  y  2 là một phương trình bậc hai ẩn x , có
f  x; y  2 x 2  y 2  xy  5 x  y  2  2 x 2   y  5 x  y 2  y  2 . Xét  ta được:
 f x ; y   y  5  8 y 2  y  2  9  y  1 . Và ta đã may mắn khi  là một số chính
2 2


 x  5  y  3 y  1  y  1
 4 2 . Hay nói cách khác
phương, khi đó 
x  5  y  3  
y  1
  2y
 4
2 x 2  y 2  xy  5 x  y  2   x  y  22 x  y  1 .
Bây giờ ta sẽ xét đến vế phải của phương trình một, điều ngạc nhiên ở đây là
 y  2 x  1  3  3x  x  y  2 . Và biểu thức x  y  2 đồng thời xuất hiện ở vế phải vậy
nên ta tìm được nhân tử chung là x y2 . Biến đổi vế phải
x y2   y  2 x  1  3  3x  
y  2 x  1  3  3 x . Từ đó phương trình một được viết
lại thành:
x y2
 x  y  22 x  y  1 
y  2 x  1  3  3x
x  y  2

  1 .
 y  2 x  1  0 (*)
 y  2 x  1  3  3x

Mặt khác, từ điều kiện có y  2 x  1  0 nên phương trình (*) vô nghiệm.
Với x  y  2  y  2  x thế vào phương trình thứ hai trong hệ, ta có:
x 2  x  3  3 x  7  2  x ( i ).
Phương trình này cũng chứa hai căn thức ( i ), nhưng ta khó có thể dùng được phương
pháp đặt ẩn phụ vì vế trái ( i ) là một đa thức bậc bốn so với vế phải. Do đó, ta sẽ nghĩ đến
hướng liên hợp khi đoán trước được nghiệm. Trước hết, sẽ ưu tiên hướng nghiệm nguyên
với mục đích là các biểu thức trong căn là số chính phương nên ta sẽ đi giải các phương


3 x  7  m
2
x  2
 
trình nghiệm nguyên 2  x  n 2
 m  1 . Khi đó phương trình ( i ) được viết lại
 
 7 n  2
2  x   ; x   
 3
thành: x 2  x  2  3x  7  1  2  2  x
3x  6 2x
  x  2 x  1  
3x  7  1 2 2x

Liên hệ tài liệu word toán sđt và zalo: 039.373.2038 TÀI LIỆU TOÁN HỌC
226
Website:tailieumontoan.com
 x  2

  3 1 .
  1 x  0
 3x  7  1 2  2  x

3 1
Do điều kiện x  1 nên   1  x  0 , suy ra phương trình ( i ) có
3x  7  1 2  2  x
nghiệm  x; y  2; 4 .
Bài toán kết thúc.
Bài tập tương tự:

 2 2
2 x  3 x  y  y  4 x  x  1
1. Giải hệ phương trình  .

4 2 x  1  x  2  2 y



Đáp số:  x; y  18  4 21; 17  4 21 . 
 4 x 2  4 x  9 x  y  xy  3 y
   
2. Giải hệ phương trình  .
4 x  2 y  2 x  3 x  3
     
Đáp số:  x; y  1; 1 .
Câu 3.
1). Phương trình tương đương với x 4  x 2  20  y 2  y .
Ta thấy x 4  x 2  x 4  x 2  20  x 4  x 2  20  8 x 2
 x 2  x 2  1  y  y  1   x 2  4 x 2  5 .
Vì x; y   nên ta xét các trường hợp sau
+ TH1: y  y  1   x 2  1 x 2  2  x 4  x 2  20  x 4  3x 2  2
 2 x 2  18  x 2  9  x  3 .
Với x 2  9 , ta có y 2  y  9 2  9  20  y 2  y  110  0
 y  10 ; y  11 .
+ TH2: y  y  1   x 2  2 x 2  3  x 4  x 2  20  x 4  5x 2  6
7
 4 x 2  14  x 2  (loại).
2
4
+ TH3: y  y  1   x 2  3 x 2  4  6 x 2  8  x 2  (loại).
3
+ TH4: y  y  1   x 2  4 x 2  5
 8x2  0  x2  0  x  0 .
Với x 2  0 , ta có y 2  y  20  y 2  y  20  0  y  5 ; y  4 .
Vậy phương trình đã cho có nghiệm nguyên là:
x; y  3; 10 , 3;  11 , 3; 10 , 3;  11 , 0;  5 , 0; 4 .
Nhận xét. Bài toán giải phương trình nghiệm nguyên bằng cách biến đổi, đánh giá, xét giá
trị/ khoảng giá trị của các nghiệm.
Nhắc lại kiến thức và phương pháp.
• Biến đổi phương trình đã cho.
Ta có x 4  x 2  y 2  y  20  0  x 4  x 2  20  y 2  y .
• Sử dụng tính chất bất đẳng thức.
Liên hệ tài liệu word toán sđt và zalo: 039.373.2038 TÀI LIỆU TOÁN HỌC
227
Website:tailieumontoan.com
o Bất đẳng thức số 0  20 .
o Tích của một bình phương với một số dương thì không âm 8 x 2  0 .
o Cộng vào hai vế của một bất đẳng thức cùng một lượng thì bất đẳng thức không
0  20  x 4  x 2  x 4  x 2  20
thay đổi  .
0  8 x 2  x 4  x 2  20  x 4  x 2  20  8 x 2

o Tính chất của bất đẳng thức a  b ; b  c suy ra a  c
Ta có x 4  x 2  x 4  x 2  20 và x 4  x 2  20  x 4  x 2  20  8 x 2 suy ra
4 2 4 2 4 2 2
x  x  x  x  20  x  x  20  8 x
 x 2  x 2  1  y  y  1   x 2  4 x 2  5 .
• Bài toán trở thành tìm tích hai số tự nhiên liên y  y  1 thỏa mãn điều kiện trên.
Từ đây ta có:
 
y  y  1   x 2  1 x 2  2 ;  x 2  2 x 2  3 ;  x 2  3 x 2  4 ;  x 2  4 x 2  5 .
• Xét các trường hợp trên
o TH1: y  y  1   x 2  1 x 2  2  x 4  x 2  20  x 4  3x 2  2
 2 x 2  18  x 2  9  x  3 .
Với x 2  9 , ta có y 2  y  9 2  9  20  y 2  y  110  0
 y  10; y  11 (thỏa mãn).
o TH2: y  y  1   x 2  2 x 2  3
7
 x 4  x 2  20  x 4  5 x 2  6  4 x 2  14  x 2  (loại).
2
4
o TH3: y  y  1   x 2  3 x 2  4  6 x 2  8  x 2  (loại).
3
o TH4: y  y  1   x 2  4 x 2  5  8 x 2  0  x 2  0  x  0 .
Với x 2  0 , ta có y 2  y  20  y 2  y  20  0
 y  5; y  4 .
Vậy phương trình đã cho có nghiệm nguyên là:
x; y  3; 10 , 3;  11 , 3; 10 , 3;  11 , 0;  5 , 0; 4 .
2). Đặt M  k 4  8 k 3  23k 2  26 k  10 .
Ta có M   k 4  2 k 2  1  8 k  k 2  2 k  1  9 k 2  18 k  9
2 2  
  k 2  1  8 k  k  1  9  k  1   k  1 .  k  3  1 .
2 2 2

 
2 2
Để M là số chính phương khi và chỉ khi  k  1  0 hoặc  k  3  1 là số chính phương.
2
+ TH1:  k  1  0  k  1 .
2 2
+ TH2:  k  3  1 là số chính phương, đặt  k  3  1  m2 ( m   )
2
 m2   k  3  1  m  k  3m  k  3  1 .
m  k  3  1 
 m  k  3  1
Vì m; k    m  k  3   ; m  k  3   nên  hoặc 

m  k  3  1 
 m  k  3  1

 m  1; k  3
 k3.
 m  1; k  3

Liên hệ tài liệu word toán sđt và zalo: 039.373.2038 TÀI LIỆU TOÁN HỌC
228
Website:tailieumontoan.com

Vậy k  1 hoặc k  3 thì k 4  8 k 3  23k 2  26 k  10 là số chính phương.


Nhận xét: Bài toán tìm điều kiện để một số, biểu thức chứa tham số đã cho là một số chính
phương ta thường phân tích biểu thức đó thành nhân tử rồi áp dụng cách tính chất của số
chính phương.
Nhắc lại kiến thức và phương pháp:
• Phân tích đa thức thành nhân tử.
M  k 4  8 k 3  23 k 2  26 k  10   k 4  2 k 2  1  8 k  k 2  2 k  1  9 k 2  18 k  9
2 2  
  k 2  1  8 k  k  1  9  k  1   k  1 .  k  3  1 .
2 2 2

 
• Tích của một bình phương với một số dương là một số chính phương khi và chỉ khi
bình phương đó bằng 0 hoặc số dương đó là một số chính phương. Thật vậy, khi bình
phương đó bằng 0 thì tích đó bằng 0 là một số chính phương; khi số dương đó là một
số chính phương thì tích đó viết được dưới dạng bình phương của một tích
2
A 2 B2   AB là một số chính phương.
2 2
M là số chính phương khi và chỉ khi  k  1  0 hoặc  k  3  1 là số chính phương.
2
+ TH 1.  k  1  0  k  1 .
2 2
+ TH 2.  k  3  1 là số chính phương, đặt  k  3  1  m2 m  
2
 m2   k  3  1  m  k  3m  k  3  1 .
 Phương trình ước số

m  k  3  1

Vì m , k    m  k  3   , m  k  3   nên m  k  3  1 .


 m  k  3  1

 m  k  3  1

 Giải hệ phương trình bậc nhất bằng phương pháp cộng đại số hoặc phương pháp thế
m  k  3  1 
m  1
 
 m k31 


Ta có  
 k  3  k  3 .
 
m  k  3  1  m  1
 


m  k  3  1 
 k  3

Vậy k  1 hoặc k  3 thì k 4  8 k 3  23k 2  26 k  10 là số chính phương.
Câu 4.

Liên hệ tài liệu word toán sđt và zalo: 039.373.2038 TÀI LIỆU TOÁN HỌC
229
Website:tailieumontoan.com

E
P

H
O

I
B K C
A

  ANO
1). Theo giả thiết AMO   AIO
  90 0  5 điểm A; O; M ; N ; I thuộc đường tròn
  AMN
đường kính AO , suy ra AIN  ; AIM
  ANM
 (góc nội tiếp cùng chắn một cung).
  ANM
Ta có AM  AN , nên AMN cân tại A  AMN 
  AIM
 AIN  , suy ra điều phải chứng minh.
Nhận xét. Chứng minh các điểm cùng thuộc một đường tròn bằng cách chứng minh
chúng cùng thuộc quỹ tích của một cung chứa góc nào đó.
Chứng minh một tia là tia phân giác của một góc ta chứng minh tia đó chia góc thành hai
góc bằng nhau.
Nhắc lại kiến thức và phương pháp.
• Tiếp tuyến của đường tròn tại một điểm thì vuông góc với bán kính tại điểm đó
  90 .
+ AM là tiếp tuyến của đường tròn (O) tại M nên AM  MO tại M hay AMO
  90 .
+ AN là tiếp tuyến của đường tròn (O) tại N nên AN  NO tại N hay ANO
• Đường kính/Bán kính đi qua trung điểm của một dây cung thì vuông góc với dây
cung đó tại trung điểm hay là trung trực của dây cung đó.
  90 hay
I là trung điểm của dây cung BC nên OI là trung trực của BC hay IOB
  90 .
IOA
• Một điểm mình một đoạn thẳng dưới một góc vuông thì điểm đó thuộc đường tròn
đường kính là đoạn thẳng này.
  90 nên M thuộc đường tròn đường kính AO .
+ AMO
  90 nên N thuộc đường tròn đường kính AO .
+ ANO
  90 nên I thuộc đường tròn đường kính AO .
+ AIO
Suy ra 3 điểm M ; N ; I thuộc đường tròn đường kính AO hay năm điểm M ; N ; I ; O; A
cùng thuộc một đường tròn.
• Trong một đường tròn, các góc nội tiếp cùng chắn một cung thì bằng nhau.
  AMN
+ AIN  (hai góc nội tiếp cùng chắn cung AN
 của đường tròn đường kính AO ).
  ANM
+ AIM  (hai góc nội tiếp cùng chắn cung AM
 của đường tròn đường kính AO ).

Liên hệ tài liệu word toán sđt và zalo: 039.373.2038 TÀI LIỆU TOÁN HỌC
230
Website:tailieumontoan.com
• Hai tiếp tuyến cắt nhau tạo ra hai đoạn thẳng từ giao điểm đến hai tiếp điểm bằng
nhau.
Ta có AM và AN là hai tiếp tuyến của đường tròn (O) tại M và N nên AM  AN .
• Tam giác có hai cạnh bằng nhau thì cân tại đỉnh chung của hai cạnh đó.
Tam giác AMN có AM  AN nên AMN cân tại A .
• Tam giác cân có hai góc kề cạnh đáy bằng nhau.
  ANM
Tam giác AMN cân tại A nên AMN .
• Tính chất “bắc cầu”.
 AIN   AMN


    AIM (điều phải chứng minh).
Ta có  AIM  ANM suy ra AIN

  ANM
 AMN 

2 1 1
2). Ta có    2 AB. AC  AK  AB  AC   AB. AC  AK. AI
AK AB AC
(do AB  AC  2 AI ).
+ ABN ∽ ANC  AB.AC  AN 2 .
+ AHK ∽ AIO  AK.AI  AH .AO .
Tam giác AMO vuông tại M có đường cao MH , suy ra AH .AO  AM 2 , nên

AK. AI  AM 2 . Do AN  AM  AB. AC  AK. AI .

Nhận xét: Chứng minh một đẳng thức hình học bằng nhau, ta áp dụng các phương pháp
chứng minh đại số cùng các dữ liệu hình học đã có và suy ra được từ bài hình
Nhắc lại kiến thức và phương pháp:
• Một đường tròn có góc tạo bởi tia tiếp tuyến và dây cung bằng góc nội tiếp cùng chắn
một cung.
 là góc tạo bởi tia tiếp tuyến NA và dây cung NB chắn cung BN
+ ANB  của đường tròn
(O) .
 là góc nội tiếp chắn cung BN
+ ACN  của đường tròn (O) ;
  ACN
suy ra ANB .
• Hai tiếp tuyến cắt nhau ta có đường thẳng đi qua tâm đường tròn và giao điểm hai
tiếp tuyến vuông góc với đường thẳng đi qua hai tiếp điểm của hai tiếp tuyến với
đường tròn.
Ta có AM và AN là hai tiếp tuyến cắt nhau. Có OA cắt MN tại H nên OA  MN tại H
  90 .
hay AHK
• Hai tam giác có hai cặp góc tương ứng bằng nhau thì đồng dạng.
+ ABN và ANC có: NAC   ACN
 chung; ANB  nên ABN ∽ ANC (g – g).

+ AHK và AIO có: IAO chung; AHK  AIO


  90 nên AHK ∽ AIO (g – g).
• Hai tam giác có các cặp cạnh tương ứng tỉ lệ.
AB AN
+ ABN ∽ ANC    AB. AC  AN 2 .
AN AC
AK AO
+ AHK ∽ AIO    AK. AI  AH . AO .
AH AI

Liên hệ tài liệu word toán sđt và zalo: 039.373.2038 TÀI LIỆU TOÁN HỌC
231
Website:tailieumontoan.com
• Hệ thức lượng trong tam giác vuông: Bình phương của một cạnh góc vuông bằng tích
hình chiếu của nó trên cạnh huyền và cạnh huyền.
Tam giác AMO vuông tại M có đường cao MH hay AO là cạnh huyền, MA là cạnh góc
vuông và AH là hình chiếu của AM trên AO nên ta có AH .AO  AM 2 .
• Tính chất “bắc cầu”.

 AB. AC  AN 2



 AK. AI  AH . AO

  AK. AI  AH . AO  AM 2  AN 2  AB. AC (điều phải chứng minh).

 AH . AO  AM 2


 AM  AN


3). Ta có AN  NO; MP  NO; M  AN  AN  MP .
Do đó AMPN là hình bình hành  AN  MP  2 x .
AN NO 2x2
Tam giác ANO ∽ NEM    NE  .
NE EM R
2x2
+ TH1: Ta có NE  NO  OE   R  R2  x 2  2 x 2  R2  R R2  x 2
R

Đặt R2  x 2  t ( t  0 )  x 2  R2  t 2 .

 2t  R
Phương trình trở thành 2  R2  t 2   R2  Rt  2t 2  Rt  R2  0   .
t  R

Do t  0  t  R  R2  x 2  R  x  0  A  B (loại).
+ TH 2: Ta có

2x2
NE  NO  OE   R  R2  x 2  2 x 2  R2  R R2  x 2 .
R

Đặt R2  x 2  t ( t  0 )  x 2  R2  t 2 .

 2t  R
Phương trình trở thành 2  R2  t 2   R2  Rt  2t 2  Rt  R2  0   .
t  R
R 3
Do t  0  2t  R  2 R2  x 2  R  x   AO  2 R .
2

Vậy A thuộc BC , cách O một đoạn bằng 2R thì AMPN là hình bình hành.
Nhận xét: Bài toán tìm điều kiện của một yếu tố để thỏa mãn điều kiện nào đó, ta nắm
chắc các yếu tố cố định để biến đổi, biểu diễn yếu tố cần tìm theo các yếu tố cố định đó.
Nhắc lại kiến thức và phương pháp:
• Hai đường thẳng phân biệt cùng vuông góc với một đường thẳng thứ ba thì song song
với nhau.
 AN  NO

 MP  NO  AN / / MP .

 M  AN
• Tứ giác có một cặp cạnh đối song song và bằng nhau là hình bình hành.
Tứ giác AMPN có AN / / MP nên để AMPN là hình bình hành thì cần có điều kiện
AN  MP  2 x .
Liên hệ tài liệu word toán sđt và zalo: 039.373.2038 TÀI LIỆU TOÁN HỌC
232
Website:tailieumontoan.com
• Hai tiếp tuyến cắt nhau có đường nối giao điểm hai tiếp tuyến và tâm đường tròn là
tia phân giác của góc tạo bởi hai tiếp tuyến.
 nên MAO
AM và AN là hai tiếp tuyến cắt nhau nên AO là phân giác của MAN   OAN
.
• Tứ giác có tổng hai góc đối diện bằng 180 là tứ giác nội tiếp.
  ONA
Tứ giác AMON có AMO   90  90  180 nên AMON là tứ giác nội tiếp suy ra
  MNO
MAO  (hai góc nội tiếp cùng chắn cung MO
 của đường tròn ngoại tiếp tứ giác
AMON ).
ANO và NEM có ANO   NEM
  90
   ENM
 và NAO   MAO  suy ra  
AN NO 2x2
ANO  NEM  g.g    NE  .
NE EM R
• Vị trí tương đối của ba điểm thẳng hàng.
3
3
Câu 5. Ta có 12  a  b  4 ab  2 ab  4 ab .  
Đặt t  ab ( t  0 ) thì
12  8t 3  4t 2  2t 3  t 2  3  0  t  12t 2  3t  3  0 .
Do 2t 2  3t  3  0, t nên t  1  0  t  1 . Vậy 0  ab  1 .
1 1 2
Chứng minh được   , a; b  0 thỏa mãn ab  1 .
1  a 1  b 1  ab

1 1 1 1
Thật vậy, bất đẳng thức    0
1  a 1  ab 1  b 1  ab

ab  a ab  b  b  a  a b 
  0   
   0
1  a1  ab  1  b 1  ab    1  a 1  b 
 1  ab 


 b a  
ab  1
0.
 
1  ab 1  a1  b

Do 0  ab  1 nên bất đẳng thức này đúng.


2
Tiếp theo ta sẽ chứng minh  2015ab  2016, a; b  0 thỏa mãn ab  1 .
1  ab

2
Đặt t  ab ( 0  t  1 ), ta được  2015t 2  2016
1 t

 2015t 3  2015t 2  2016t  2014  0


 t  12015t 2  4030t  2014  0 .

Bất đẳng thức này đúng với 0  t  1 .

1 1
Vậy   2015ab  2016 .
1 a 1 b
Đẳng thức xảy ra a  b  1 .

Liên hệ tài liệu word toán sđt và zalo: 039.373.2038 TÀI LIỆU TOÁN HỌC
233
Website:tailieumontoan.com
Nhận xét: Đây là một bài toán hay và khó, khó ở chỗ phát hiện ra bất đẳng thức bổ đề để
suy ra được điều phải chứng minh.
Nhắc lại kiến thức là phương pháp:
• Bất đẳng thức Cosi cho hai số thực dương a  b  2 ab .
• Bất đẳng thụ phụ (bất đẳng thức bổ đề)
1 1 2
  , a; b  0, ab  1
1  a 1  b 1  ab
1 1 1 1
    0
1  a 1  ab 1  b 1  ab
ab  a ab  a
  0
1  a1  ab  1  b1  ab 
2


 a b  ab  1  0;  0  ab  1 .
1  a1  b1  ab 
Ý tưởng: Bài toán hoàn toàn đối xứng, vai trò của hai biến số là như nhau, nên dấu đẳng
thức sẽ xảy ra tại a  b  k , khi đó thay ngược lại giả thiết và biểu thức cần chứng minh ta
sẽ có:
8 k 3  4 k 2  12
 a  b  2
 2  k  1  a  b  1   .
 ab  1
2
 2015 k  2016
 1  k
Với điểm rơi này, ta thấy ab  1 thì 2015ab  2015 do đó ta chỉ cần chứng minh
1 1
  1 với ab  1 thì bài toán sẽ hoàn tất. Nhưng trước hết, ta cần khai thác giả
1 a 1 b
thiết để chứng minh ab  1 . Giả thiết đã chứa tích ab còn tổng a  b để đưa về tích ab , khi
đó ta sẽ sử dụng bất đẳng thức Cosi như sau:
3 3
a  b  2 ab  a  b  8 ab ab  a  b  4 ab  8 ab ab  4 ab
3 2
8  ab   4 ab   12  0    
ab  1 2 ab  3 ab  3  0  ab  1  0
1 1
 ab  1  2015ab  2015 . Vì thế nếu chứng minh được   1 , thì ta sẽ suy ra
1 a 1 b
1 1
  2015ab  2016 .
1 a 1 b

a  ab  
Điểm rơi a  b ta có  1  a  1  ab 1 1 2
. Đây chính là dấu
   

b  ab 
1  b  1  ab 1  a 1  b 1  ab

 

đẳng thức xảy ra, do đó ta cần chứng minh:
1 1 2
  ; a , b  0; ab  1 .
1  a 1  b 1  ab
Và điều này đã được chứng minh ở trên, chính vì thế ta được:
1 1 2
  2015ab   2015ab  2016
1 a 1 b 1  ab
   
ab  1 2015ab  4030 ab  2014  0  ab  1  0  ab  1 , điều phải chứng minh.
Bài toán kết thúc.
Liên hệ tài liệu word toán sđt và zalo: 039.373.2038 TÀI LIỆU TOÁN HỌC
234
Website:tailieumontoan.com
Bài tập tương tự:
1. Cho a; b là hai số thực dương thỏa mãn ab  1 . Chứng minh rằng:
1 1 2
  .
1  a 1  b 1  ab
2. Cho x; y; z là các số thực dương thuộc 1; 4 và x  y; x  z . Chứng minh rằng
x y z 34
   .
2 x  3 y y  z z  x 33

ĐỀ SỐ 27.

Câu 1.
1). Điều kiện: x  4 .
Phương trình tương đương với
x  1 x  4  2 x5 2 x4   x  1 x  5  0
 x 1  x4  x5 2   x4  x5  0 
  x 1  2  x4  x5  0 
 x  4  x  5 x  4  x  5
    x  5 (thỏa mãn).
 x  1  2 x  1  4

Vậy phương trình có nghiệm: x  5 .
Nhận xét: Bài toán sử dụng phương pháp nhóm nhân tử chung đưa về hai phương trình
tích chứa căn thức và cuối cùng sử dụng phương pháp nâng lũy thừa để tìm nghiệm của
phương trình đã cho.
Nhắc lại kiến thức và phương pháp:
• Cho phương trình tổng quát bậc hai ax 2  bx  c  0 , nếu tổng cách hệ số a  b  c  0
thì phương trình có một nghiệm là x  1 .

 f  x ,g  x  0 .

• Phương trình vô tỷ dạng f  x  g  x  
  


 f x  g  x
Ý tưởng: Phương trình bài cho chứa rất nhiều căn thức, tuy nhiên dễ dàng để ý thấy rằng
ta sẽ nhóm được hai căn thức có hệ số 2 ở trước nó, cụ thể là 2  
x  5  x  4 . Và xét các
đa thức bậc hai trong hai căn còn lại, ta đều thấy a  b  c  0 tức là các đa thức đó có dạng
 x 2  5 x  4 
 x  1x  4
x  1 f  x  0 , đi xét riêng từng căn thức suy ra  2 vậy nên ta
 x  4 x  5 

x  1x  5
nhóm được nhân tử chung là x  1 và đại lượng còn lại là x  4  x  5 . Chính vì thế
phương trình đã cho tương đương với:
x  1 x  4  2 x5 2 x4   x  1 x  5  0
 x 1  x4  x5 2   x4  x5  0 

Liên hệ tài liệu word toán sđt và zalo: 039.373.2038 TÀI LIỆU TOÁN HỌC
235
Website:tailieumontoan.com
 x4  x5
  x 1  2  
x  4  x  5  0  
 x  1  2
.

Hai phương trình còn lại là hai phương trình căn thức cơ bản, ta chỉ cần bình phương hai
vế là sẽ tìm được nghiệm của phương trình ban đầu và x  5 chính là nghiệm duy nhất.
Bài toán kết thúc.
Bài tập tương tự:
3. Giải phương trình x2  1  2 x  3  2 x  1  x2  2x  3 .
Đáp số: x  3 .
4. Giải phương trình x  1  x 2  2 x  3  x  1  3 x  3 .
Đáp số: x  3 hoặc x  4 .
2). Điều kiện: x  0; y  0 .

  1  1

 x   y    2 (1)

 y  x .


 2 2
2 x y  xy  4 xy  2 x  y (2)

2 1
Phương trình (2) tương đương 2 x  y  4  
y x
 1  1
 2  x     y    4 (3).
 y   x
 1
a  x 
y
Đặt  .
 1
b  y 
 x
ab  2
Kết hợp với (1) và (3), ta có hệ phương trình 
2 a  b  4
a 4  2 a  2 a 2  2 a  1  0 a  1
   .
b  4  2 a b  4  2 a b  2
 

 1

 x 1 

 a  1  y xy  1  y
Với  ta có   

b  2
 
y  1  2  
xy  1  2 x



 x
 y  2 x  2  y  2 x  2
    2
x 2 x  2  1  2 x 2 x  4 x  1  0
 
 
x  2  2 x  2  2
  hoặc  (thỏa mãn).
2 2
 
 y  2  y   2
2  2   2  2 
Vậy hệ phương trình có nghiệm:  x; y   ; 2  , 
  2
;  2  .

 2   
Nhận xét: Bài toán sử dụng phương pháp đặt hai ẩn phụ để đưa hệ phương trình đã cho
về hệ đơn giản, từ đó thế ngược lại tìm nghiệm của hệ phương trình ban đầu.
Ý tưởng: Quan sát cả hai phương trình của hệ, ta thấy phương trình một chứa các biểu
thức phân số, khá phức tạp nên ta sẽ làm động tác quy đồng để đơn giản hóa đó là
Liên hệ tài liệu word toán sđt và zalo: 039.373.2038 TÀI LIỆU TOÁN HỌC
236
Website:tailieumontoan.com

 x  1  y  1   2   xy  1 xy  1  2 . Đây rõ ràng là một phương trình bậc hai ẩn xy ,
 
 
y  x  xy
2
chính vì thế ta có  xy  1  2 xy
 xy  1  2
  . Và ta sẽ nhóm nhân tử xy ở phương trình thứ hai của hệ như sau:
 xy  1  2
xy 2 x  y  4 xy  2 x  y  xy 2 x  y  4  2 x  y , từ đây sẽ đi tìm được mối liên hệ giữa xy ,
sau đó thế ngược lại tìm nghiệm của hệ. Tuy nhiên ta cũng có thể tư duy theo một hướng
khác đó là đi khai thác phương trình hai. Điều mà ta muốn là mối liên hệ giữa x , y để rồi
thế vào phương trình một tìm nghiệm. Đặt f  x; y  2 x 2 y  xy 2  4 xy  2 x  y , ta coi đây là
một phương trình bậc hai ẩn x , khi đó f  x; y  2 y.x 2   y 2  4 y  2 x  y , xét
2
   y 2  4 y  2  8 y 2 nhưng đenta ở đây không chính phương tức là sẽ không biểu diễn
được mối liên hệ giữa x , y . Do đó ta sẽ đi khai thác cùng với phương trình một, ở phương
trình một có xuất hiện các biểu thức xy  1 và xy  1 nên khi xét phương trình hai, nhóm
các biểu thức đồng hệ số ta có 2 x 2 y  2 x  2 x  xy  1 và xy 2  y  y  xy  1 . Khi đó phương
trình hai của hệ tương đương với:
2  xy  1 xy  1  1  1
2 x  xy  1  y  xy  1  4 xy    4  2  x     y    4 nên nếu đặt
y x  y   x 
1 1
a x và b  y  thì hệ phương trình đã cho trở thành hệ
y x

 ab  2 
a 2  2 a  1  0  a  1

  
 . Từ đó, theo ẩn phụ hóa ta sẽ tìm được nghiệm của hệ

2 a  b  4 
 b  4  2 a 
b  2


2  2   2  2 
phương trình là  x; y   ; 2  ,  ;  2  .
 2   2 

Bài toán kết thúc.
Bài tập tương tự:

x  y  xy  1
1. Giải hệ phương trình  2 .
x  y  xy  7

2


 2

 x  y  2 y 2  8 x  1
2. Giải hệ phương trình  .
 2 2


 x  2 y  x  y    y  1  2 x
Câu 2. Ký hiệu  x; y là ước chung lớn nhất của hai dãy số nguyên x và y .
Gọi d  a; b  a  da1 ; b  db1 với a1 ; b1   1
Suy ra a 2  b2  d 2 a12  b12  và ab  d 2 a1b1
 d 2 a12  b12   d 2 a1b1  a12  b12   a1b1
 a12  b1  a1 .a1  b1 mà a1 ; b1   1  a1  b1 .
Tương tự b1  a1 , suy ra a1  b1  1 .
d 2 a12  b12 
Suy ra A   1.
2d 2 a1b1
Liên hệ tài liệu word toán sđt và zalo: 039.373.2038 TÀI LIỆU TOÁN HỌC
237
Website:tailieumontoan.com
Nhận xét: Bài toán tính giá trị của biểu thức sử dụng các tính chất của toán số học đặc biệt
là tính chất chia hết.
Nhắc lại kiến thức và phương pháp:
• Với m là một ước của n ta có thể biểu diễn n  k.m với m , k , n là các số nguyên.
d  a; b  a  da1 ; b  db1 với a1 ; b1   1
• Cộng, nhân vế theo vế dương của hai đẳng thức ta được một đẳng thức mới.
a  da1  a 2  b2  d 2 a 2  b2 
2
 
 a 2
  da  
Từ  
1 1

1
ta có  .
b  db  2 ab  d 2 a b 

 1 


b 2
  db1   1 1

• Tính chất chia hết.


+ a 2  b2   ab  d 2 a12  b12   d 2 a1b1  a12  b12   a1b1  a12  b1 .

a ; b   1
+  21 1  a1  b1 .
a  b
 1 1
Hoàn toàn tương tự ta có b1  a1 .
• Số thứ nhất chia hết cho số thứ hai, số thứ hai chia hết cho số thứ nhất thì hai số bằng
nhau.
a  b
Ta có  1 1 suy ra a1  b1 mà a1 ; b1   1 nên a1  b1  1 .
b1  a1
• Thay các giá trị đã có vào biểu thức ban đầu để tính giá trị biểu thức đó.
a 2  b2 d a1  b1  a1  b1  12  12 2
2 2 2 2 2

A      1.
2 ab 2d 2 a1b1 2 a1b1 2.1.1 2
Câu 3. Đặt x  a 2; y  b 2; z  c 2 . Ta cần chứng minh
x  2 y 2  2 z 2  2  3 x  y  z .
2 2

Ta có
x 2
 2 y 2  2   x 2  1 y 2  1  x 2  y 2  3  x 2 y 2  1  2 x 2  2 y 2  3
2
 x  y
3 
  x  2 y  2  2 xy  x  y 
2
2 2 2 2
 3   x  y  2
2 2 
3  
  x 2  2 y 2  2 z 2  2   x  y z 2  4  2  x  y  2 z 2 
2 2

2 
3 2  2
  4  x  y  z  2  x  y   2 z 2   3  x  y  z
2 
2
2  a  b  c
 a  1b  1c  1 
2 2 2
.
4
1
Dấu “=” xảy ra khi và chỉ khi a  b  c  .
2
Nhận xét: Bài toán sử dụng phương pháp đổi biến số và dùng bất đẳng thức Cosi khi
đoán trước được điểm rơi để suy ra điều phải chứng minh.
Ý tưởng: Bất đẳng thức đề bài cho là một bất đẳng thức đối xứng, đối xứng ở đây có nghĩa
là vai trò của các biến a , b , c là như nhau vì thế dấu đẳng thứa sẽ xảy ra tại a  b  c  k , khi
đó thế ngược lại vào bất đẳng thức đã cho, ta có được

Liên hệ tài liệu word toán sđt và zalo: 039.373.2038 TÀI LIỆU TOÁN HỌC
238
Website:tailieumontoan.com
3 1 1
4  k 2  1  27 k 3  k  abc . Vậy để điểm rơi đẹp, hay ta sẽ biến đổi
2 2
1
abc  2 a  2b  2c  1 nên nếu đặt x  2 a; y  2b; z  2c thì bài toán đã cho
2
trở thành:
x  2 y 2  2 z 2  2  3 x  y  z
2
2
 với điểm rơi x  y  z 1.
Bây giờ, để chứng minh được  ta sẽ bám sát theo điểm rơi tìm được, quan sát vế phải
của  , ta thấy xuất hiện các biểu thức x  y và z , vậy nên cần đánh giá
 2
x 2
 2 y 2  2  f  x  y  để đồng nhất bậc trong bất đẳng thức  , từ đó lợi dụng tính
 
chất của đa thức bậc hai suy ra điều phải chứng minh. Cụ thể đó là
x2  2 y 2  2  x2 y 2  1  2 x2  2 y 2  3 . Áp dụng bất đẳng thức Cosi ta có: x2 y 2  1  2xy
2 2
 x  y  x  y 3 
nên suy ra  x  2 y  2  2 xy  x  y 
2
và x  y 2 2 2 2 2 2
 3   x  y  2 .
2 2 2 
Khi đó, bất đẳng thức  tương đương với:
3  2   2
 x  y  2  z  2  3 x  y  z   x  y  2  z  2  2  x  y  z
2 2 2 2

2   
Bất đẳng thức này luôn đúng vì nếu đặt m  x  y và n  z thì chúng ta có
m  2n2  2  2 m  n  m2 n2  2 m2  2n2  4  2 m2  4 mn  2n2
2 2

2
 mn  2  0 luôn đúng với mọi m , n . Từ đó suy ra đpcm.
Bài toán kết thúc.
Bài tập tương tự:
1. Cho x , y , z là các số thực dương thỏa mãn x  y  z  3 . Tìm giá trị nhỏ nhất của biểu
thức P   x 2  2 y 2  2 z 2  2 .
2. Cho a , b , c là độ dài ba cạnh của một tam giác. Chứng minh rằng:
2
a b  b c  c a
2 2 2
 abc a 2  b2  c 2 a  b  c .
Câu 4.

Liên hệ tài liệu word toán sđt và zalo: 039.373.2038 TÀI LIỆU TOÁN HỌC
239
Website:tailieumontoan.com

  ACE
1). Tứ giác ABEC nội tiếp, suy ra ABE   180 .
  ACE
Mà EDC  và ADE
  EDC   ADE
  180 , nên ABE .
  DAE
Kết hợp với BAE   ABE
  ADE
.
Mặt khác EB  EC  ED nên AE là trung trực của đoạn BD
  ADB
Suy ra AE  BF (1) và AB  AD  ABD .
  DCF
Kết hợp với ABD  (cùng chắn cung AF ) và ADB
  FDC
 (đối đỉnh)
  FCD
Suy ra FDC  , nên tam giác FDC cân tại F  FD  FC .
Kết hợp ED  EC , suy ra EF là trung trực của DC , nên DC  EF (2).
Từ (1) và (2), suy ra D là trực tâm của tam giác AEF .
Nhận xét: Bài toán chứng minh một điểm là trực tâm của một tam giác, ta chứng minh
điểm đó là giao điểm của hai đường cao của tam giác.
Nhắc lại kiến thức và phương pháp:
• Tứ giác nội tiếp có tổng hai góc trong đối diện bằng 180 .
  ACE
Tứ giác ABEC nội tiếp, suy ra ABE   180 .

• Tam giác có hai cạnh bằng nhau là tam giác cân. Tam giác cân có hai góc kề đáy bằng
nhau.
  ECD
Tam giác EDC có ED  EC nên EDC cân tại E . Suy ra EDC  hay EDC
  ACE

 ABE   ACE
  180

    ADE
.
Từ đây ta có  ADE  EDC  180  ABE

  ACE
EDC 

• Trong một đường tròn, điểm nằm chính giữa cung tạo ra hai cung bằng nhau. Hai góc
nội tiếp chắn hai cung bằng nhau thì bằng nhau.

Liên hệ tài liệu word toán sđt và zalo: 039.373.2038 TÀI LIỆU TOÁN HỌC
240
Website:tailieumontoan.com
 nên BE
Trong đường tròn (O) có E là điểm chính giữa cung nhỏ BC   EC
 suy ra
  EAC
BAE   EAD
 hay BAE 

• Tổng ba góc trong một tam giác bằng 180 .


+ ABE có ABE   BAE
  AEB   180 BAE
  180  AEB .
  ABE
  AED
+ ADE có ADE   DAE
  180  AED
  180 DAE
  ADE
.
 AEB
  180 BAE
  ABE


   
 AED  180 DAE  ADE  
Kết hợp từ đây ta có   AEB  AED .
 ABE  ADE


BAE 
  EAD

• Hai tam giác có một cặp cạnh bằng nhau và có hai cặp góc kề cạnh tương ứng bằng
nhau thì bằng nhau theo trường hợp “cạnh - góc - cạnh” (c - g - c).
Xét BAE và DAE có:
  AED
+ AEB ;

+ AE : cạnh chung;
  EAC
+ BAE ;

 AB  AD
Suy ra BAE  DAE (g – c – g), suy ra  .
EB  ED

• Tam giác có hai cạnh bằng nhau là tam giác cân. Tam giác cân có hai góc kề đáy bằng
nhau.
  ADB
Tam giác ABD có AB  AD suy ra ABD cân tại A . Suy ra ABD 

• Trong một đường tròn, hai góc nội tiếp cùng chắn một cung thì bằng nhau.
  ACF
Trong đường tròn (O) có FBA  ) hay
 (hai góc nội tiếp cùng chắn cung FA
  DCF
ABD 

• Hai góc đối đỉnh thì bằng nhau.


  FDC
ADB  (hai góc đối đỉnh)
 ABD   ADB


     FDC
  FCD cân tại F  FC  FD .
Suy ra  ABD  DCF  DCF

  FDC
 ADB 

• Một điểm cách đều hai đầu của một đoạn thẳng thì nằm trên đường trung trực của
đường thẳng đó.
+ AB  AD nên A thuộc trung trực của BD ;
EB  ED nên E thuộc trung trực của BD ;
Suy ra AE là đường trung trực của đoạn BD suy ra AE  BD .
+ FC  FD nên F thuộc trung trực của CD ;
ED  EC nên E thuộc trung trực của CD ;

Liên hệ tài liệu word toán sđt và zalo: 039.373.2038 TÀI LIỆU TOÁN HỌC
241
Website:tailieumontoan.com
Suy ra FE là đường trung trực của đoạn CD suy ra FE  CD
• Trong một tam giác, giao điểm của hai đường cao được gọi là trực tâm của tam giác.
Tam giác AEF có AE  BD và FE  CD hay BD và CD là hai đường cao cắt nhau tại
D nên D là trực tâm của AEF (điều phải chứng minh).
2). Kẻ đường kính EK của O; R . Khi đó điểm K cố định.
  BND
Tứ giác BDNM nội tiếp nên BMD 

Suy ra BMD   90  1 BAC


  90  BCE  (3).
2
 .
Tứ giác ABMK nội tiếp nên BMK  180  BAK
  90  1 BAC
Mà BMK  (4).
2
  BMK
Từ (3) và (4), suy ra BMD 
Suy ra ba điểm M ; D; K thẳng hàng.
Do đó MD luôn đi qua điểm K cố định.
Nhận xét. Bài toán chứng minh một điểm là trực tâm của một tam giác, ta chứng minh
điểm đó là giao điểm của hai đường cao của tam giác.
Nhắc lại kiến thức và phương pháp.
• Trong một đường tròn, các góc nội tiếp chắn cùng một cung thì bằng nhau.
  BND
BMD  (hai góc nội tiếp cùng chắn cung BD  của đường tròn ngoại tiếp tứ giác
BDNM )
  90  BCE
 BMD   90  1 BAC
  90  CAE 
2
• Tứ giác nội tiếp có tổng hai góc trong đối diện bằng 180 .
  BAK
Tứ giác ABMK nội tiếp, suy ra BMK   180
  180  BAK
 BMK 

  90  1 BAC
Mà BMK 
2
• Ba điểm cùng thuộc một đường thẳng thì thẳng hàng.

   90  1 BAC


 BMD

 2   BMK
 suy ra MD; MK cùng là cạnh còn lại của một
Ta có   BMD

 1 
  90  BAC
 BMK


 2
góc nên M ; D; K cùng thuộc cạnh đó nên M ; D; K thẳng hàng (điều phải chứng minh).
Câu 5. Giả sử A  a1 ; a2 ; a3 ;...; a21  với a1 ; a2 ; a3 ;...; a21   và a1  a2  a3  ...  a21 .
Theo giả thiết ta có a1  a2  a3  ...  a11  a12  a13  ...  a21
 a1  a12  a2  a13  a3  ...  a21  a11 (1).
Mặt khác, với x; y   và x  y thì y  x  1
 a12  a2  10 ; a13  a3  10 ; …; a21  a11  10 (2).
Nên từ (1), suy ra a1  10  10  ...  10  100  a1  101 (vì 101  A ).
 101  a12  a2  a13  a3  ...  a21  a11  100
 a12  a2  a13  a3  ...  a21  a11  100 .

Liên hệ tài liệu word toán sđt và zalo: 039.373.2038 TÀI LIỆU TOÁN HỌC
242
Website:tailieumontoan.com
Kết hợp với (2), suy ra a12  a2  a13  a3  ...  a21  a11  10 (3)
 10  a12  a2  a12  a11   a11  a10   ...  a3  a2   10
 a12  a11  a11  a10  ...  a3  a2  1 (4).
Ta có a1  101 mà 102  A  a2  102 .
Kết hợp với (3) và (4), suy ra A  101; 102; 103; ...; 121 .
ĐỀ SỐ 28.

Câu 1.
2x  2 x x 1 x x 1
1). Ta có P   
x x x x x


2x  2

 
x 1 x  x  1  
x 1 x x 1   2x  2  x  x 1  x  x 1
x x  x  1 x  x  1 x x x

2x  2 2x  2 x  2
 2
x x
2). Ta có x  3  2 2  x  2  1 .
2
Thay vào biểu thức P  2  
2 1  2 
2 1
.

Tính được kết quả P  4 2  2


7 7 x
3). Đưa được  .
P 2x  2  2 x
7 x 7
Đánh giá 2 x  2  2 x  6 x , suy ra 0   .
2x  2  2 x 6
7
Vậy , chỉ nhận một giá trị nguyên đó là 1 khi
P
 x2 x  4
 
7 x  2x  2  2 x  2x  5 x  2    .
 x1 x  1
 2  4

Nhận xét: Hai ý đầu là những ý quen thuộc, rút gọn biểu thức và tính giá trị biểu thức, tuy
nhiên có ý cuối là ý nâng cao nhưng với bài toán nghiệm nguyên thì nếu ta xét được miền
nghiệm của biểu thức sẽ tìm được các giá trị nghiệm nguyên thỏa mãn.
Ý tưởng: Với yêu cầu rút gọn biểu thức P , đây là một yêu cầu quen thuộc khi ta phải đưa
các biểu thức cồng kềnh về đơn giản hóa, tức là sẽ khử dần được các căn thức như sau:
x x 1 t3  1
• Với P1  , nếu đặt t  x thì ta có P1  , dựa vào hằng đẳng thức quen
x x t2  t
thuộc t  1  t  1t 2  t  1 đồng thời nhóm nhân tử chung t 2  t  t t  1 nên
3

t2  t  1
P1  .
t

Liên hệ tài liệu word toán sđt và zalo: 039.373.2038 TÀI LIỆU TOÁN HỌC
243
Website:tailieumontoan.com

x2  x t4  t
• Với P2  , nếu đặt t  x thì ta có P2  , dựa vào hằng đẳng thức dạng
x xx t3  t2
t  t  t t 3  1  t t  1t 2  t  1 đồng thời nhóm nhân tử chung t 3  t 2  t 2 t  1 nên
4

t2  t  1
P2  .
t
t2  t  1 t2  t  1 2x  2 x  2
• Do đó P1  P2    2 , suy ra P  .
t t x
Khi x  3  2 2 , ta chỉ còn việc tính x là sẽ tính được giá trị của biểu thức P , để ý thấy
2 2
32 2   2 1 do đó x  
2  1  2  1 . Vậy nên ta tính được P  4 2  2 .

7 2x  2 x  2 7 x
Xét biểu thức , với P  suy ra Q   . Áp dụng bất đẳng thức
P x P 2x  2 x  2
Cosi ta có x  1  2 x ( dấu  không xảy ra vì điều kiện bài cho là x  1 ) nên
7 7 x 7 x 7
2 x  2 x  2  4 x  2 x  6 x do đó suy ra được Q     , vậy ta
P 2x  2 x  2 6 x 6
7
có Q   đồng thời thỏa mãn 0  Q  nên Q chỉ có thể nhận một giá trị nguyên duy nhất,
6
7
và giá trị nguyên đó chính là Q  1 . Từ đó ta kết luận được , chỉ nhận một giá trị nguyên
P
đó là 1 khi và chỉ khi
 x2 x  4
 
7 x  2x  2  2 x  2x  5 x  2   
 x1 x  1
 2  4

Bài toán kết thúc.
Bài tập tương tự:
x2  x 2x  x 2  x  1
Cho biểu thức P    với x  0 và x  1 .
x x 1 x x 1
1. Rút gọn biểu thức P .
2 x
2. Tìm x để biểu thức Q  nhận giá trị nguyên.
P
73 5
Đáp số: x  .
2
Câu 2.
1). Khi m  1 , ta có phương trình x 2  2 x  8  0 .
Giải phương trình ta được hai nghiệm: x1  2; x2  4 .
3
2). Tính được   m2  m  1 .
3
Để phương trình có hai nghiệm phân biệt khi m2  m  1  0 (*).
Gọi x1 ; x2 là hai nghiệm của phương trình, theo Vi-et ta có

x1  x2  2 m (1)

 .
x x  m  13 (2)


 1 2
2
Giả sử x1  x22 , thay vào (2), ta được x2  m  1 ; x1  m  1 .

Liên hệ tài liệu word toán sđt và zalo: 039.373.2038 TÀI LIỆU TOÁN HỌC
244
Website:tailieumontoan.com
Thay hai nghiệm x1 ; x2 vào (1), ta được
2 m  0
m  1  m  1  2 m  m2  3m  0  
m  3
.

Khẳng định hai giá trị m vừa tìm được thỏa mãn điều kiện (*), kết luận
Nhận xét: Bài toán sử dụng các tính chất của định lý Vi-et để khai thác tính chất nghiệm từ
đó tìm điều kiện yêu cầu của bài toán.
Nhắc lại kiến thức và phương pháp:
• Phương trình bậc hai tổng quát ax 2  bx  c  0 với a  0 .
Để phương trình trên có nghiệm    b2  4ac  0 . Khi đó, giả sử phương trình có hai
nghiệm phân biệt x1 ; x2 nên theo định lý Viet ta có
b c
và x1 x2  .
x1  x2  
a a
Ý tưởng: Ý đầu tiên của bài toán, chỉ cần thế m  1 rồi đi giải phương trình bậc hai (xét 
hoặc dùng máy tính ), còn ý sau phức tạp hơn khi đề bài yêu cầu tìm m với điều kiện cho
trước. Đọc đề ta sẽ phân tích được rằng phải đi làm hai công việc như sau:
1. Tìm m để phương trình có hai nghiệm phân biệt. Xét với phương trình bậc hai
3 3
x 2 – 2 mx  m – 1  0 , ta có:   m2  m – 1 . Để phương trình có hai nghiệm phân
3
biệt khi m2  m  1  0 . Tuy nhiên đây là phương trình bậc ba ẩn m với nghiệm lẻ,
do đó ta sẽ không đi giải nó mà sau khi tìm được m ta sẽ thế vào điều kiện.
2. Khi phương trình bài cho có hai nghiệm phân biệt đồng thời thỏa mãn một nghiệm
bằng bình phương nghiệm còn lại. Ta giả sử nó có hai nghiệm phân biệt x1 ; x2 và
x1  x22 . Theo định lý Vi-et, ta có:
x  x 2 ; x  x  2 m 
  2

 1 2 1 2 x1  m  1 ; x2  m  1
 3 

 x x   m – 1 
x1  x2  2 m

 1 2 

2 m  0
 m  1  m  1  2 m  m2  3m  0  
m  3

3
Thỏa mãn điều kiện m2  m – 1  0 .
Vậy m  0 hoặc m  3 là các giá trị cần tìm.
Bài toán kết thúc.
Bài tập tương tự:
1. Tìm m để phương trình x 2  mx  m  1  0 có hai nghiệm phân biệt x1 ; x2 đồng thời
1 1 x  x2
hai nghiệm đó thỏa mãn   1 .
x1 x2 2015
2. Cho phương trình: x 2  2m  3 x  m  0 . Tìm m để phương trình đã cho có hai
nghiệm phân biệt sao cho x12  x22 đạt giá trị nhỏ nhất.
Câu 3. Điều kiện: x  0 .
2x2  9 x
Phương trình trở thành 2
2 3 0.
x 2x2  9
x
Đặt t.
2x2  9

Liên hệ tài liệu word toán sđt và zalo: 039.373.2038 TÀI LIỆU TOÁN HỌC
245
Website:tailieumontoan.com
t  1

Phương trình trở thành 2t  3t  1  0  t  12t  t  1  0   1 .
3 2 2
t 
 2
+ Với t  1 , ta có x  2 x 2  9 (vô nghiệm).
1 x  0 3 2
+ Với t   , ta có 2 x 2  9  2 x   2  x  .
2  2 x  9 2
3 2
Vậy phương trình có nghiệm: x   .
2
Nhận xét: Bài toán sử dụng phương pháp đặt ẩn phụ, đưa phương trình ban đầu về
phương trình bậc ba, từ đó truy ngược lại nghiệm của phương trình đã cho.
Nhắc lại kiến thức và phương pháp:
• Phương trình bậc ba tổng quát ax 3  bx 2  cx  d  0 . Nếu tổng các hệ số a  b  c  d  0
thì phương trình đã cho có một nghiệm x  1 .
x
• Đặt  t , suy ra
mx 2  n
x2 nt 2
mx 2  n
 t 2
 1  mt 2
 x 2
 nt 2
 x 2

1  mt 2
.

Ý tưởng: Phương trình khá phức tạp vì vừa chứa căn thức, vừa xuất hiện phân số. Tuy
nhiên, có sự đối xứng giữa biến x và hằng số 9 , đồng thời khi bình phương biểu thức
2
 x  2
9
   x , nếu nghịch đảo chúng nên ta sẽ thấy xuất hiện 2 vì thế lượng dư
 2 
 2
2x  9 x
 2x  9 
9 9 2x2  9
sẽ là 2 . Do đó ta sẽ tạo thêm 2 với lượng để xuất hiện biểu thức  2  . Khi
x2 x2 x2
2x2  9 2x
đó phương trình đã cho trở thành: 2
  3  0 (*).
x 2x2  9
x 1
Đặt t  nên (*)  2
 2t  3  0  2t 3  3t 2  1  0 .
2x  92 t
t  1  x  2x2  9
2   3
 t  1 2t  1  0     x .
t   1 
 2 x  2 x 2
 9  0 2
 2 
3
Vậy phương trình có nghiệm duy nhất là x   .
2
Bài toán kết thúc.
Bài tập tương tự:
1. Giải phương trình x  x2  1  x  x2  1  2 .
Đáp số: x  1 .
1
2. Giải phương trình x 2  2 x x   3x  1 .
x
1 5
Đáp số: x  .
2
Câu 4.

Liên hệ tài liệu word toán sđt và zalo: 039.373.2038 TÀI LIỆU TOÁN HỌC
246
Website:tailieumontoan.com

F
O
E K
C
B H M
 chung
1). Xét hai tam giác AEF và ACB có góc A
  AHF
Ta có AEF  ; AHF
  ACB
 , suy ra AEF
  ACB   AHE
 (hoặc AFF  ; AHE
  ABC
 , suy ra
  ABC
AFE )
nên AEF ∽ACB .
AE AF
Từ tỷ số đồng dạng  , ta có AE.AB  AC.AF .
AC AB
Nhận xét. Bài toán chứng minh đẳng thức hình học có hai vế là tích hai đoạn thẳng ta sử
dụng tính chất tỷ lệ thức suy là từ một tỷ số đồng dạng.
Nhắc lại kiến thức và phương pháp.
• Trong một đường tròn, các góc nội tiếp chắn cùng một cung thì bằng nhau.
  AHF
Đường tròn tâm O đường kính AH có AEF  (hai góc nội tiếp cùng chắn
)
cung FA
• Tổng hai góc nhọn trong tam giác vuông bằng 90 .





AHF AFH    FHA
  90  FAH

  90
  ACH
 FHA ,







  

ACH AHC  90  CAH  ACH  90
  ACB
Suy ra AEF .
• Hai tam giác có hai cặp góc bằng nhau thì đồng dạng với nhau theo trường hợp “góc -
góc” (g - g).
Xét AEF và ACB có:
 : chung;
+ BAC
  ACB
+ AEF ;
Suy ra AEF ∽ACB (g - g).
• Hai tam giác đồng dạng có các cặp cạnh tương ứng tỷ lệ.
AE AF
AEF ∽ACB    AE. AB  AC. AF (điều phải chứng minh).
AC AB
2). Xét hai tam giác OHM   và OFM   có OM chung, OF  OH .
Có MF  MH (vì tam giác HFC vuông tại F , trung tuyến FM )
Suy ra OHM  OFM (c – c - c).
  90 0 , MF là tiếp tuyến của đường tròn đường kính AH .
Từ đó MFO
đường tròn ta chứng minh đường thẳng đó vuông góc với một bán kính tại một điểm nằm
trên đường tròn của bán kính đó.
Nhắc lại kiến thức và phương pháp.
Liên hệ tài liệu word toán sđt và zalo: 039.373.2038 TÀI LIỆU TOÁN HỌC
247
Website:tailieumontoan.com
• Trong tam giác vuông, đường trung tuyến ứng với cạnh huyền bằng nửa cạnh huyền.
Tam giác HFC vuông tại F có M là trung điểm của cạnh huyền HC nên FM là
HC
đường trung tuyến ứng với cạnh huyền do đó FM   HM  MC .
2
• Hai điểm cùng thuộc một đường tròn thì cách đều tâm đường tròn một khoảng bằng
bán kính.
 AH  AH
Ta có F , H thuộc O;  nên OF  OH  .
  2  2
• Hai tam giác có ba cặp cạnh bằng nhau thì bằng nhau theo trường hợp “cạnh - cạnh -
cạnh” (c – c - c).
Xét OHM   và OFM  có:
+ OM : chung;
HC
+ HM  FM (  );
2
AH
+ OF  OH (  );
2
Suy ra OHM  OFM (c – c – c).
• Hai tam giác bằng nhau có các góc tương ứng bằng nhau.
  OFM
OHM  OFM  OHM   90 suy ra OFM
 mà OHM   90 .

• Một đường thẳng đi qua một điểm nằm trên đường tròn và vuông góc với bán kính
của đường tròn tại điểm đó là tiếp tuyến của đường tròn.
 AH 
+ Điểm F nằm trên O;  ;
  2 
  90 hay FM  OF tại F ;
+ OFM
Suy ra FM là tiếp tuyến của đường tròn đường kính AH .
  BHO
3). Xét hai tam giác AHM và BHO có AHM   90 0 .
Trong tam giác vuông ABC , đường cao AH có
AH HM
AH 2  HB.HC  AH .2OH  HB.2 HM   ,
HB HO
  HBO
suy ra HBO ∽ HAM  HAM .

Nhận xét. Bài toán chứng minh hai góc bằng nhau ta chứng minh hai tam giác chứa hai
góc đó đồng dạng.
Nhắc lại kiến thức và phương pháp.
• Hệ thức lượng: Trong tam giác vuông, bình phương đường cao bằng tích hai hình
chiếu hai cạnh góc vuông trên cạnh huyền.
Tam giác ABC vuông tại A , đường cao AH , áp dụng hệ thức lượng ta có:
AH 2  HB.HC  AH .2OH  HB.2 HM  AH .OH  HB.HM
AH HM
  .
HB HO
• Hai tam giác có một cặp góc bằng nhau và cặp cạnh kề góc tương ứng tỷ lệ thì đồng
dạng với nhau theo trường hợp “cạnh – góc - cạnh” (c – g - c).
Xét AHM và BHO có:

Liên hệ tài liệu word toán sđt và zalo: 039.373.2038 TÀI LIỆU TOÁN HỌC
248
Website:tailieumontoan.com
  BHO
+ AHM  (  90 0 );
AH HM
+  ;
HB HO
Suy ra AHM ∽BHO (c – g – c).
• Hai tam giác đồng dạng có hai góc tương ứng bằng nhau.
AHM ∽BHO  HAM   HBO
 (điều phải chứng minh).
4). Gọi K là giao điểm của AM với đường tròn.
  HAM
Ta có HBO   MHK , suy ra BO  HK .

Mà HK  AM , suy ra BO  AM , suy ra O là trực tâm của tam giác ABM .


Nhận xét. Bài toán chứng minh một điểm là trực tâm của một tam giác ta chứng minh
điểm đó là giao điểm của hai đường cao của tam giác đó.
Nhắc lại kiến thức và phương pháp.
• Góc nội tiếp chắn nửa đường tròn là góc vuông.
 AH    90  HKM
  90 khi đó KHM
  KMH
  90 mà
Điểm K thuộc O;  nên AKH
 2 
  KMH
HAM   90 nên KHM
  HAM

• Hai góc ở vị trí đồng vị của hai đường thẳng bằng nhau thì hai đường thẳng đó song
song.
KHM
  HAM

   HBO
 mà hai góc này ở vị trí đồng vị của BO và HK nên
  KHM
HAM  HBO


suy ra BO  HK .
• Quan hệ từ vuông góc đến song song.
BO  HK
  BO  AM
HK  AM
• Trong một tam giác, giao điểm của hai đường cao là trực tâm của tam giác.
Tam giác ABC có BO là đường cao, AH là đường cao mà O nằm trên AH nên suy
ra O là giao điểm của hai đường cao AH và OB do đó O là trực tâm của ABC
(đpcm).
Câu V. Giả sử a  b  c , từ giả thiết suy ra ab  1 . Ta có bất đẳng thức sau
2
1 1 2 a  b ab  1
    0 (luôn đúng).
1  a 2 1  b2 1  ab 1  a2 1  b2 1  ab
2 1 3
Vậy ta cần chứng minh  
1  ab 1  c 2
2
 c  3  ab  3abc  c 2  ca  bc  3abc 2  a  b  c  3abc .
2 2


 2
 a  b  c  3ab  bc  ca  9
Bất đẳng thức hiển nhiên đúng vì  hay a  b  c  3  3abc .
 2




ab  bc  ca  3 3
 abc 
Dấu “=” xảy ra khi a  b  c  1 .
Nhận xét: Dựa trên suy đoán của điểm rơi, sử dụng bất đẳng phụ để đưa về phép biến đổi
tương đương, từ đó áp dụng bất đẳng thức Cosi để suy ra điều phải chứng minh.
Nhắc lại kiến thức và phương pháp:

Liên hệ tài liệu word toán sđt và zalo: 039.373.2038 TÀI LIỆU TOÁN HỌC
249
Website:tailieumontoan.com

• Bất đẳng thức Cosi cho ba số thực dương: a  b  c  3 3 abc .


2
• Hệ quả tương ứng a  b  c  3ab  bc  ca .
a , b  0 1 1 2
• Bất đẳng thức bổ đề  ta có   .
ab  1 1 a 2
1 b 2
1  ab
Ý tưởng: Đây là một bài toán khó, với ý tưởng lạ lẫm. Đầu tiên xét qua điểm rơi, với sự đối
xứng của các biến ta dễ thấy dấu đẳng thức xảy ra tại tâm a  b  c  1 . Vì thế, xét với từng
1 1 a 2  a.a  a.b
biểu thức ; , ta có các đại lượng cân bằng  , do đó
 2
1  a 1  b2
2 b  b.b  a.b

1 1 2
  , mặt khác xét dấu của bất đẳng thức bài cho, ta mong muốn
1 a 2
1 b 2
1  ab
1 1 2
  . Biến đổi tương đương, ta có được:
1 a 2
1 b 2
1  ab
2
1 1 2 a  b ab  1
   (*).
1 a 2
1 b 2
1  ab 1  a 2 1  b2 1  ab
Để (*)  0  ab  1 , tức là ta cần đi khai thác giả thiết để có ab  1 . Điều này dễ thấy vì đây
là bất đẳng thức thuần nhất nên giả sử a  b  c , khi đó:

ca

  3  ab  bc  ca  ab  ba  ba  3ab  ab  1

c  b

Vậy nên, bất đẳng thức bài cho trở thành:
2 1 3 3  2c 2  ab 3
     6  4c 2  2 ab  31  ab1  c 2 
1  ab 1  c 2
2 1  ab1  c  2
2

 c 2  3  ab  3abc 2  c 2  ca  bc  3abc 2  a  b  c  3abc



 2

a  b  c  3ab  bc  ca  9  a  b  3  3
Mặt khác, ta thấy  , từ đó suy ra được
 2
ab  bc  ca  3 abc  3abc  3
 3


a  b  3  3  3abc , suy ra điều phải chứng minh.
Bài toán kết thúc.
Bài tập tương tự:
1. Cho a; b là hai số thực dương thỏa mãn ab  1 , chứng minh rằng
1 1 2
 
1 a 2
1 b 2
1  ab
2. Cho x; y; z là các số thực dương thuộc 1; 4 và x  max x; y; z . Tìm giá trị nhỏ
x y z
nhất của biểu thức P    .
2x  3y y  z z  x

ĐỀ SỐ 24.

Câu 1.
1). Ta có   m2  8 .
  0 với mọi m nên phương trình đã cho luôn có hai nghiệm phân biệt.

Liên hệ tài liệu word toán sđt và zalo: 039.373.2038 TÀI LIỆU TOÁN HỌC
250
Website:tailieumontoan.com

x  x  m
 1 2
2 và giả thiết cho x 2  4 x 2 .
Theo định lý Vi-et ta có  1 2
 1
 x1 .x2  
 2
 
x  x  m 
x1  x2 
m
 1 2
2  2

1  1
Nên ta có  x1 .x2   (1) hoặc  x1 .x2   (2).
 2  2
 x  2 x  x  2 x
 1 2  1 2
 

+ Giải (1):
1
Ta có 2 x2  x2   (vô nghiệm), nên hệ phương trình (1) vô nghiệm.
2
+ Giải (2):
 1
 x2    x1  1  m  1
1  2
Ta có 2 x2  x2     .
2  1
 x2   x1  1  m  1
 2
Nhận xét: Bài toán áp dụng định lý Vi-ét trong phương trình bậc hai và biến đổi biểu thức.
Nhắc lại kiến thức và phương pháp:
• Phương trình bậc hai có hai nghiệm phân biệt khi và chỉ khi   0 .
Phương trình bậc hai 2 x 2  mx  1  0 có hai nghiệm phân biệt x1 và x2 với mọi giá trị
2
của tham số m vì   m  4.2.1  m2  8  8  0 với mọi giá trị của m .

 b

 x1  x2  
 a.
• Định lý Vi-ét trong phương trình bậc hai 

 c
 x1 x2 


 a

x  x  m
Ta có 
 1 2
2 kết hợp với đề bài cho x 2  4 x 2   x1  2 x2 .
 1 1 2  x  2 x
 x1 .x2    1 2

 2
• Giải các hệ phương trình.


 
 


 m 
 x  2 x  x1  2 x2 x1  2 x2

 x  x   1 2 
 


1 2
2 
  
 m  m  m
+ 
1  2 x 2  x 2    x 2  
  x2 

 x1 .x2   
 2 
 6 
 6
 2   
  1  2  2

x1  2 x2



 2 x2 .x2     
2. m    1  m  9


 2 
   
 6
 2
(vô nghiệm vì m2  0 nên không tồn tại m2  9 ).

Liên hệ tài liệu word toán sđt và zalo: 039.373.2038 TÀI LIỆU TOÁN HỌC
251
Website:tailieumontoan.com


 
 


 m 
 x   2 x  x  2 x 
x1  2 x2
 x  x   1 2 
 1 2 


1 2
2 
  
 m  m  m
+  1  2 x2  x2     x2   
 x2  

 x1 .x2   
 2 
 2 
 2
 2   

 
 1 
  m 
2
1  2
m  1
x1  2 x2

 2 x2 .x2   2.   


 2 
  
  2 
 2
 m  1
 m  1
 m  1  m  1
  
 
 1  1
 x1  2 x2  x2    x2  (thỏa mãn, nhận).
  2   2
 m x1  1 x1  1
x2    
 2

Vậy m  1 .
2). Ta có   m2  8 .
  0 với mọi m nên phương trình đã cho luôn có hai nghiệm phân biệt x1 ; x2 .
1 1  x 1
Theo định lý Vi-et ta có x1 .x2   suy ra x1 . x2   1   1 , m .
2 2 x 1
 2
Nhận xét: Bài toán áp dụng định lý Vi-ét trong phương trình bậc hai và kiến thức bất
phương trình,…
Nhắc lại kiến thức và phương pháp:
• Phương trình bậc hai có hai nghiệm phân biệt.
Phương trình bậc hai đã cho có hai nghiệm phân biệt với mọi giá trị của m (đã chứng
minh ở ý trên)
• Phương trình có nghiệm thỏa mãn điều kiện nào đó.
Ta có x  1 khi và chỉ khi x1 x2  1 vì x1 ; x2 dương.

 b

 x1  x2  
 a.
• Định lý Vi-ét trong phương trình bậc hai 

 c
 x1 x2 


 a
1 1 1 1
Ta có x1 .x2    x1 .x2     x1 . x2   1 .
2 2 2 2
Do đó x  1 .
Câu 2.
1
1). Điều kiện: x  .
3
 8 1 1  
Phương trình đã cho tương đương với 18 x 2  2 x    9  x     0
  33  3 
1 1
  x 
1 3 9 0
 18 x  8 x    9
 3  1 1
x 
3 3

Liên hệ tài liệu word toán sđt và zalo: 039.373.2038 TÀI LIỆU TOÁN HỌC
252
Website:tailieumontoan.com
 
 
 4    1  1 
  x   18  x    9 0 x 4.
 
9   3 1 1  9
 x   
 3 3
1  1 1
Chứng minh: Với x   18  x    9 0.
3  3 1 1
x 
3 3
4
Phương trình đã cho có nghiệm: x  .
9
Nhận xét: Bài toán giải phương trình đưa về phương trình tích.
Nhắc lại kiến thức và phương pháp:
1 1
• Tìm điều kiện xác định: Điều kiện: x   0  x  .
3 3
• Tách, thêm bớt phân tích nhân tử.
17 1  8  1 
18 x 2  2 x   9 x   0  18 x 2  2 x    9 x   3  0
3 3  3   3 

 1  1 1 
 18 x  8 x    9  x     0
 3   3 3 
1 1
  x 
1 3 9 0
 18 x  8 x    9
 3 1 1
x 
3 3
4
   x
4 1 9
 8  x   x    9 0
 
9   3 1 1
x 
3 3
 
 

 4  8 9 

  x  8 x     0 .
 
9  3 1 1 
 x   
 3 3
• Phương trình tích gồm hai thừa số, chứng minh có một thừa số khác 0 thì thừa số còn
lại luôn bằng 0.
8 9
+ Chứng minh 8 x   khác 0
3 1 1
x 
3 3
 8
8 x 
1 3 8 9
Với x  ta có   8x   0 .
3  1 3 1 1
 x   0 x 
 3 3 3
 
 

 4  8 9  4 4
+ Suy ra  x  8 x  
   0  x   0  x  .
 
9  3 1 1 9 9
 x   
 3 3

Liên hệ tài liệu word toán sđt và zalo: 039.373.2038 TÀI LIỆU TOÁN HỌC
253
Website:tailieumontoan.com
4
• Đối chiếu với điều kiện xác định để đi đến kết luận nghiệm x  thỏa mãn điều kiện
9
1 4
x  , nên phương trình nhận x  là nghiệm.
3 9
2). Phương trình tương đương với  x  3 y  1 x  y  3  15 (1).
Do x; y là số nguyên không âm nên từ (1), ta có
x  3 y  1  5 x  3 y  1  3 x  3 y  1  15  x  3 y  1  1
       .
 x  y  3  3  x  y  3  5  x  y  3  1 x  y  3  15
Vậy  x; y  2; 0 .
Nhận xét: Bài toán giải phương trình nghiệm nguyên bằng cách đưa về phương trình ước
số
Nhắc lại kiến thức và phương pháp:
• Phân tích một vế thành nhân tử, vế còn lại là một số.
x 2  3 y 2  4 xy  4 x  10 y  12  0

   
 x 2  3 xy  x  xy  3 y 2  y  3 x  9 y  3  15  0

 x  x  3 y  1  y  x  3 y  1  3  x  3 y  1  15
  x  3 y  1 x  y  3  15
• Tìm các ước của số ở một vế.
15  1.15  3.5  5.3
Vì x; y  0 nên ta không chọn các ước âm của 15.
x  3 y  1  1
Ta có  nên không chọn các cặp tích có thừa số thứ hai nhỏ hơn 3.
x  y  3  3
• Cho các hệ số ở vế này bằng thừa số của vế kia tạo thành các hệ phương trình. Giải các
phương trình này tìm nghiệm.
Ta có các hệ phương trình sau:
x  3 y  1  1 x  3 y  0 x  18
  
x  y  3  15 x  y  12 x  6
  
  
x  3 y  1  3 x  3 y  2 x  2
     ,
x  y  3  5 x  y  2  y  0
  
x  3 y  1  5 x  3 y  4 x  2
  
x  y  3  3 x  y  0  y  2
  
kết hợp với điều kiện ta có được x  2 và y  0 .
Câu 3. Điều kiện: x  y  0 .
Phương trình thứ nhất tương đương với
4  x  y  1   3 x  y  x  y  1   0
2

 
2  x  y  1
  2  x  y  1  2  x  y  1  0
  
3  x  y  x  y  1
 
 1 
  2  x  y  1  2  x  y  1    0 (*).
  
 3  x  y   x  y  1 
Liên hệ tài liệu word toán sđt và zalo: 039.373.2038 TÀI LIỆU TOÁN HỌC
254
Website:tailieumontoan.com
Do x  y  0 , nên (*) tương đương với 2  x  y  1  0  2 y  2 x  1 , thế vào phương trình
thứ hai, ta có 6 x 2  x  1  0
 1
x   y  0
 2
 .
 1 5
x    y  
 3 6
 1   1 5 
Hệ phương trình có nghiệm:  x; y   ; 0 ,  ;   .
 2   3 6 
Nhận xét: Bài toán giải hệ phương trình bằng phương pháp nhân liên hợp.
Nhắc lại kiến thức và phương pháp:
• Điều kiện xác định: Biểu thức dưới mẫu khác 0; Biểu thức dưới dấu căn không âm.
x  y  1  0
Điều kiện xác định:   xy 0 .
x  y  0

• Nhẩm nghiệm để tìm lượng liên hợp.


1
+ Ta thấy x  y  thì phương trình thứ nhất của hệ bằng 0. Do đó phương trình đó
2
1
thứ nhất của hệ phương trình có nhân tử x  y  hay 2  x  y  1 .
2
x  y  1  1  4  x  y   3  x  y   4  x  y   1   3  x  y  x  y  1   0 .
2 2

   
+ Hằng đẳng thức hiệu hai bình phương a  b  a  ba  b .
2 2

2
4  x  y   1   2  x  y   1  2  x  y   1 .
  
+ Nhân lượng liên hợp.
 3 x  y  x  y  1  3 x  y  x  y  1
       
3  x  y  x  y  1  
3  x  y  x  y  1
2 2
 3 x y   x y 1
 

    3  x  y   x  y  1 2  x  y  1
   ,
3  x  y  x  y  1 3  x  y  x  y  1 3  x  y  x  y  1
2
suy ra x  y  1  1  4  x  y   3  x  y
 
  4  x  y   1   3  x  y  x  y  1   0
2

   
2  x  y  1
  2  x  y   1  2  x  y   1  0
  
3  x  y  x  y  1
 
   1 
  2  x  y  1  2  x  y  1   0.
  

3  x  y   x  y  1

 A1  0

A  0
• Phương trình tích A1 .A2 ..An  0   2 .

A  0
 n

Liên hệ tài liệu word toán sđt và zalo: 039.373.2038 TÀI LIỆU TOÁN HỌC
255
Website:tailieumontoan.com
2
x  y  1  1  4  x  y   3  x  y
 
   1 
  2  x  y  1  2  x  y  1  0
  

3  x  y   x  y  1

 2  x  y  1  0


 1 .
 2  x  y  1  0
 3  x  y  x  y  1

• Chứng minh một phương trình vô nghiệm.
1
Ta có x  y  0  2  x  y  0 và  0 , suy ra
3  x  y  x  y  1
1 1
2  x  y   0  2  x  y  1 0 .
3  x  y  x  y  1 3  x  y  x  y  1
1
Do đó phương trình 2  x  y  1   0 vô nghiệm.
3  x  y  x  y  1

• Giải hệ phương trình



y  x  1 
2  x  y  1  0  2  y  x  1
 2    2
4 x  2 xy  1  2  1   2
  4 x  2 x 
 x    1  4 x  2 x 2
x 1
 2 


 1 x  1
 x   2

6 x 2  x  1  0  2 
 y  0
  1 
   x     (nhận, thỏa mãn).
 y  x  1  3 x   1
 
2  1 
 3
 y  x   5
 2  y  
 6
Câu 4. Phương trình tương đương với x 2  y 2  4 x  2 (1).

Ta có x 2  4 x  2  y 2  0  x  6  2 x  6  2  0  
 2 6  x2 6
 10  4 6  4 x  2  10  4 6 (2).
Từ (1) và (2), suy ra 10  4 6  x 2  y 2  10  4 6 .
Nhận xét: Bài toán áp dụng biến đổi tương đương một phương trình, giải bất phương
trình bậc hai.
Nhắc lại kiến thức và phương pháp:
• Biến đổi tương đương một phương trình.
x2  y 2  4x  2  0  x2  y 2  4x  2 (1).
2
 x  4 x  2  y 2
(2).
• Bất đẳng thức.
Ta có y 2  0  y 2  0 kết hợp với (2) ta có x 2  4 x  2  0 .

Liên hệ tài liệu word toán sđt và zalo: 039.373.2038 TÀI LIỆU TOÁN HỌC
256
Website:tailieumontoan.com

• Giải bất phương trình bậc hai.


  
x2  4x  2  0  x  6  2 x  6  2  0  2  6  x  2  6

• Biến đổi tương đương bất phương trình.


2  6  x  2  6  10  4 6  4 x  2  10  4 6 .
Kết hợp với (1) ta có 10  4 6  x 2  y 2  10  4 6 (điều phải chứng minh).
Câu 5.
A

B
K C

P M

  MBC
1). Ta có MNC  (1) (cùng MC
 ).

  BCN
MBC  (do cùng phụ với góc ABC
 ) (2).
  BCN
Từ (1) và (2), ta có MNC  suy ra tam giác KNC cân tại K .

Nhận xét: Chứng minh một tam giác cân bằng cách chứng minh tam giác đó có hai góc
bằng nhau.
Nhắc lại kiến thức và phương pháp:
• Trong một đường tròn, hai góc nội tiếp cùng chắn một cung thì bằng nhau.
  MBC
MNC  (hai góc nội tiếp cùng chắn cug MC
 của (O) ).

• Góc nội tiếp chắn nửa đường tròn bằng 90


 là góc nội tiếp chắn nửa đường tròn được chia bởi đường kính AM nên
MBA

MBA   CBA
  90  MBC   90 .

• Hai góc cùng cộng với một góc bằng được hai góc bằng nhau thì hai góc ban đầu bằng

nhau.
  CBA
+ MBC   90 (chứng minh trên);

  ACB
+ ABC   90 (do CN  AB );

  ACB
Suy ra MBC 

• Tam giác có hai góc bằng nhau là tam giác cân.

Liên hệ tài liệu word toán sđt và zalo: 039.373.2038 TÀI LIỆU TOÁN HỌC
257
Website:tailieumontoan.com
 MNC
  MBC

  ACB
 MNC   KCN
 hay KNC  suy ra KCN cân tại K (điều phải

  ACB
 MBC 


chứng minh).

2). Ta có ON  OC (3).

Từ trên suy ra KN  KC (4).

Từ (3) và (4), ta có OK  NC .
Do NC  BM (cùng vuông góc với AB ).

Nhận xét: Chứng minh một tam giác cân bằng cách chứng minh tam giác đó có hai góc
bằng nhau.
Nhắc lại kiến thức và phương pháp:
• Hai điểm cùng thuộc một đường tròn thì khoảng cách đến tâm bằng nhau.

Ta có N ; C thuộc (O) nên ON  OC .

• Tam giác cân có hai cạnh bên bằng nhau

Tam giác KCN cân tại K nên KN  KC

• Khoảng cách từ một điểm đến hai đầu mút của một đoạn thẳng bằng nhau thì điểm
đó thuộc trung trực của đoạn thẳng đó.
+ ON  OC nên O thuộc trung trực của NC ;
+ KN  KC nên K thuộc trung trực của NC ;
Suy ra OK là trung trực của NC nên OK  NC .
• Hai đường thẳng cùng vuông góc với một đường thẳng thì song song với nhau.

NC  AB  NC  BM .
BM  AB


a  b
• Quan hệ từ vuông góc đến song song  thì b  c .

a  c


OK  NC

  OK  BM (điều cần chứng minh).

NC  BM

3). Ta có
  BAM
+ BNM  ( MB
 ) (5).
  BCN
+ BMN  ( NB
 ) (6).

  NCB
+ BAM  (do cùng phụ với góc ABC
 ) (7).
  BMN
Từ (5), (6) và (7), suy ra BNM  nên BM  BN .
Từ giả thiết ta có ON  OM và PM  PN nên 3 điểm P; B; O nằm trên đường trung trực
đoạn MN vậy P; B; O thẳng hàng.

Liên hệ tài liệu word toán sđt và zalo: 039.373.2038 TÀI LIỆU TOÁN HỌC
258
Website:tailieumontoan.com
Nhận xét: Chứng minh ba điểm thẳng hàng ta chứng minh chúng cùng thuộc một đường
thẳng cố định.
Nhắc lại kiến thức và phương pháp:
• Trong một đường tròn, hai góc nội tiếp cùng chắn một cung thì bằng nhau.
  BAM
+ BNM  (hai góc nội tiếp cùng chắn cung BM
 của đường tròn (O) ).

  BCN
+ BMN  (hai góc nội tiếp cùng chắn cung BN
 của đường tròn (O) ).

• Tam giác có hai góc bằng nhau là tam giác cân. Tam giác cân có hai cạnh bên bằng
nhau.
BNM   BAM


    
BMN  BCN  BNM  BMN  BMN cân tại B nên BM  BN .

  NCB
BAM 

• Khoảng cách từ một điểm đến hai đầu mút của một đoạn thẳng bằng nhau thì điểm
đó thuộc trung trực của đoạn thẳng đó.
+ BM  BN nên B nằm trên đường trung trực của MN ;
+ OM  ON (do M ; N cùng nằm trên (O) ) nên O nằm trên trung trực của MN ;
+ PM  PN (do tính chất hai tiếp tuyến cắt nhau) nên P nằm trên trung trực của MN ;
Suy ra B; O; P cùng nằm trên trung trực của MN hay ba điểm B; O; P thẳng hàng.
Câu 6.

B
H

Hạ CK vuông góc AB tại K (giải thích tam giác ABC không tù tại B hay C ).
Ta có CK  2 3a .
Nên ta có S ABC  3a 2 3 (đvdt).

Ta có BK  a , suy ra BC  BK 2  CK 2  a 13 .
2S ABC 6 a 39
AH   .
BC 13
Nhận xét: Chứng minh ba điểm thẳng hàng ta chứng minh chúng cùng thuộc một đường
thẳng cố định.
Nhắc lại kiến thức và phương pháp:
• Trong một tam giác, cạnh đối diện với góc lớn hơn thì lớn hơn.

Liên hệ tài liệu word toán sđt và zalo: 039.373.2038 TÀI LIỆU TOÁN HỌC
259
Website:tailieumontoan.com
 là góc tù thì C
+ Giả sử, ABC có C  là góc lớn nhất nên C  suy ra AB  AC mà
B

 là
AB  3a và AC  4 a do đó có 3a  4 a  3  4 (vô lý). Suy ra ABC không thể có C

góc tù;
 là góc tù;
+ Chứng minh tương tự ta có ABC không thể có B

Suy các đường cao của ABC đều nằm bên trong tam giác.

• Trong tam giác vuông, độ dài cạnh góc vuông bằng tích cách huyền với sin góc đối

diện với cạnh góc vuông đó.

3
AKC vuông tại K có AK  AC sin A  AC sin 60  4 a.  2a 3 .
2

• Định lý Py-ta-go trong tam giác vuông: “Bình phương cạnh huyền bằng tổng bình

phương hai cạnh góc vuông”.

+ AKC vuông tại K có AC 2  KA 2  KC 2


2

 KA 2  2 a 3  2
 4 a  KA 2  16 a 2  12 a 2  4 a 2  KA  2 a  KB  AB  KA  3a  2 a  a .

2
+ BKC vuông tại K có BC 2  KB2  KC 2  a 2  2 a 3  
 BC 2  a 2  12 a 2  13a 2  BC  a 13

• Diện tích tam giác bằng nửa tích đường cao với cạnh tương ứng
1 1
+ SABC  .CK.AB  .2 a 3.3a  3a 2 3 (đvdt);
2 2

1 1
+ SABC  .AH .BC  .AH .a 13 (đvdt);
2 2

1 6 a 2 3 69 a 3
Suy ra . AH .a 13  3a 2 3  AH   .
2 a 13 13

1 1 1
Câu 7. Đặt x  ; y  ; z  suy ra xyz  1 và x; y; z dương.
a b c

x y z 9 9
Bất đẳng thức tương đương với P   2 2  (*).
y 2
z x 2  x  y  z 2
1 x 2 1 y 2 1 z 2
Ta có  2 ;  2 ;  2
x y y y z z z x x
x y z 1 1 1
  2  2     xy  yz  zx .
y 2
z x x y z

Ta có x  y  z  xyz  x  y  z

Liên hệ tài liệu word toán sđt và zalo: 039.373.2038 TÀI LIỆU TOÁN HỌC
260
Website:tailieumontoan.com
2
2 2 2 xy  yz  zx
  xy zx   yz yx   zx( zy )   xy   yz   zx  x  y  z  .
3
27
P   xy  yz  zx  2
.
2  xy  yz  zx
27 9 9
Do  xy  yz  zx   P .
2  xy  yz  zx
2
2 2

Cô-Si cho 3 số .
Dấu “  ” xảy ra khi a  b  c  1 .
Nhận xét: Bài toán chứng minh bất đẳng thức bằng phương pháp đổi biến và áp dụng bất
đẳng thức Cô-si.
Nhắc lại kiến thức và phương pháp:
• Đổi biến, biến đổi để có giả thiết mới, điều kiện cho biến mới, điều cần chứng minh
mới.
1 1 1
Đặt x  ; y  ; z  suy ra xyz  1 và x; y; z dương.
a c b
x y z 9 9
Cần phải chứng minh P  2  2  2   .
y z x 2  x  y  z 2
• Bất đẳng thức Cô-si cho hai số dương: A 2  B2  2 AB với A , B dương.
1 x 1 x 2
+ Với x, y là các số dương có và 2 là các số dương nên ta có  2  .
x y x y y
Hoàn toàn tương tự ta có:
1 y 2
+  
y z2 z
1 z 2
+  
z x2 x
• Cộng vế theo vế của các bất đẳng thức cùng chiều ta được bất đẳng thức mới cùng
chiều.
Cộng vế theo vế của các bất đẳng thức vừa chứng minh trên ta được:
x y z 2 2 2 1 1 1
 2 2     
y 2
z x x y z x y z
x y z 1 1 1 xyz xyz xyz
  2 2       xy  yz  zx (vì xyz  1 ).
y 2
z x x y z x y z
• Biến biểu thức để có được đẳng thức đúng.
x  y  z  xyz  x  y  z (vì xyz  1 )   xy zx   yz yx   zx( zy ) .
• Áp dụng ngược chiều bất đẳng thức Cô-si.

 2 2
 2  xy zx   xy   zx



 2 2 2 2 2
2  yz yx   yz   xy   xy zx   yz yx   zx( zy )   xy   yz   zx




2
2  zx( zy )   yz   zx
2



2
xy  yz  zx
xyz .
3

Liên hệ tài liệu word toán sđt và zalo: 039.373.2038 TÀI LIỆU TOÁN HỌC
261
Website:tailieumontoan.com
• Kết hợp các bất đẳng thức nhỏ thành bất đẳng thức lớn.
27
P   xy  yz  zx  2
.
2  xy  yz  zx
27 9
Mà  xy  yz  zx   .
2  xy  yz  zx
2
2

• Với a  b ; b  c thì a  c .

 27

 P   xy  yz  zx 

 2  xy  yz  zx
2
 9
Ta có   P  (điều phải chứng minh).

 27 9 2

xy  yz  zx  
 2  xy  yz  zx
2
2

ĐỀ SỐ 30.

1 1 1 1
Câu 1. Ta có    ... 
1 3 3 5 5 7 2013  2015
1 2015  1

2

1  3  3  5  ...  2013  2015 
2
. 
2015  1
+ Xét  21,5  2015  44  2015  44 2  2015  1936 (luôn đúng).
2
1 1 1 1
Vậy    ...   21,5 .
1 3 3 5 5 7 2013  2015
2015  1
+ Có thể thay giá trị 21,5 bằng một giá trị khác lớn hơn là a ( a  21,5 ) sao cho a 
2
2015  1
(  21,9 ).
2
Nhận xét: Bài toán chứng minh bất đẳng thức số dựa vào các phương pháp biến đổi căn
thức, biểu thức đơn giản.
Nhắc lại kiến thức và phương pháp:
1 A B
• Trục căn thức ở mẫu  với A  B; A  0; B  0 .
A B AB
1 1 3 1
1 3

2
  1 3 ;
2
 
1 3 5 1
3 5

2

2
 3 5 ; 

1 2013  2015 1
2013  2015

2

2
 2013  2015 . 
• Cộng vế theo vế của các đẳng thức ta được đẳng thức mới
1 1 1 1
   ... 
1 3 3 5 5 7 2013  2015
1 1 1

  1 3 
2 2
  3  5   2
 2013  2015 
Liên hệ tài liệu word toán sđt và zalo: 039.373.2038 TÀI LIỆU TOÁN HỌC
262
Website:tailieumontoan.com
1

2

1  3  3  5  ...  2013  2015 
1 2015  1

  1  2015 
2 2
. 
• Chứng minh một bất đẳng thức bằng phương pháp biến đổi tương đương về một bất
đẳng thức luôn đúng.
1 1 1 1
   ...   21, 5
1 3 3 5 5 7 2013  2015
2015  1
  21, 5  2015  1  43  2015  44 .
2
2
  2015   44 2
 2015  1936 (luôn đúng)
• Chứng minh ý tiếp theo.
1 1 1 1
   ...   a  21, 5
1 3 3 5 5 7 2013  2015
2015  1
  a  21, 5  2015  1  2 a  43  2015  2 a  1  44
2
2
 1936  2 a  1  2015 .
2
Tồn tại các giá trị của 2 a  1 nên sẽ tồn tại các giá trị của a .
2 2
Câu 2. Ta có A   x  3 1    x3 2   x  3 1  x3 2 .

+ Nếu 4  x  7 thì A  x  3  1  2  x  3  1 .
+ Nếu x  7 thì A  x  3  1  x  3  2  2 x  3  3 .
Nhận xét. Bài toán rút gọn bằng các hằng đẳng thức, các quy tắc về dấu,…
Nhắc lại kiến thức và phương pháp.
2
• Hằng đẳng thức a 2  2 ab  b2  a  b .
2
x22 x3  x  3  2 x3 1   x  3 1
2
x  1 4 x  3   x  3  4 x3 4   x3 2  .

• Hằng đẳng thức a2  a


2
x22 x3   x  3 1   x  3 1
2
x  1 4 x  3   x3 2   x3 2 .

a khi a  0
• Quy tắc phá dấu giá trị tuyệt đối a   .

a khi a  0


 x  3  1  0 x  4  x  3  1  1  x  3


+ Với   x  7 ta có  , suy ra
 x  3  2  0   x  3  2  2  x  3
 x  3 

A  x  2  2 x  3  x  1 4 x  3  1 x  3  2  x  3  3  2 x  3 .

Liên hệ tài liệu word toán sđt và zalo: 039.373.2038 TÀI LIỆU TOÁN HỌC
263
Website:tailieumontoan.com

  x  3  1  x  3  1
 x  3 1  0  x  4 
+ Với  
 ta có  , suy ra

 x  3  2  0 

 x7  x  3  2  2  x  3

 

A  x  2  2 x  3  x  1 4 x  3  x  3 1  2  x  3  1 .
  x  3  1  x  3  1
 x  3 1  0  x  4 

+ Với   ta có  , suy ra

 x3 2 0  
x  7  x  3  2  x  3  2

 

A  x  2  2 x  3  x  1 4 x  3  x  3  1  x  3  2  2 x  3  3 .
Vậy: Với 3  x  4 thì A  3  2 x  3 ; Với 4 x7 thì A  1 ; Với x  7 thì
A  2 x3 3 .
Câu 3. Ta có
 x  1 x  2 x  3 x  4  35  x 2  5x  4x2  5x  6  35 .
t  5
Đặt t  x 2  5x  4 , ta được t t  2  35  t 2  2t  35  0   .
 t  7

 x  5  29
 2
+ Với t  5 , ta có x 2  5x  4  5   .

 x  5  29
 2

+ Với t  7 , ta có x  5x  11  0 (vô nghiệm).
2

5  29
Vậy phương trình có nghiệm: x  .
2
a  b  ck
ab bc ca 
Câu 4. Đặt    k  b  c  ak .
c a b 
c  a  bk
ab bc ca
Ta có B  . .  k3 .
c a b
Mà a  b  c  ck  c   k  1 c .
Tương tự a  b  c   k  1 b ; a  b  c   k  1 a ;
Suy ra 3a  b  c   k  1a  b  c
a  b  c  0
 a  b  c k  2  0   .
k  2

+ Nếu a  b  c  0 thì a  b  c  k  1  B  1 .
+ Nếu k  2 thì a  b  2c ; b  c  2 a  a  b  2 a  b  c  2c  a  c (trái giả thiết).
Vậy B  1 .
Câu 5. Do n2  n  7  0 , nên
2
a  n4  2n3  2n2  n  7  n4  2n3  n2  n2  n .
Giả sử a là số chính phương thì a  b2 ( b   ).
2 2
b2  n2  n  b  n2  n  b  n2  n  1  b2  n2  n  1
2
 a  n2  n  1  n4  2n3  2n2  n  7  n4  2n3  3n2  2n  1

Liên hệ tài liệu word toán sđt và zalo: 039.373.2038 TÀI LIỆU TOÁN HỌC
264
Website:tailieumontoan.com
 n2  n  6  0  n  2n  3  0  3  n  2 .
Ta có
n 3 2 1 0 1 2
4 3
a  n  2n  2n  n  7 2
49 13 7 7 13 49
Vậy n  3; n  2 .
Nhận xét: Bài toán tìm giá trị của một ẩn để thỏa mãn biểu thức là một số chính phương ta
sử dụng các tính chất của số chính phương cùng sự đánh giá bất đẳng thức số.
Nhắc lại kiến thức và phương pháp:
• Chứng minh một biểu thức dương.
2
 1 1  27  1 27
Ta có n  n  7  n2  2.n.     n    .
2

 2 4  4  2  4
2 2
 1  1 27 27
Ta thấy n    0  n      0 với mọi giá trị của n .
 2   2  4 4
Tức là n2  n  7  0
• Cộng cùng một lượng vào hai vế của một bất đẳng thức ta được một bất đẳng thức
mới tương đương.
n2  n  7  0
 n 4  2 n 3  n 2   n 2  n  7  n 4  2 n 3  n 2 
2 2
 n4  2n3  2n2  n  7  n2  n hay a  n2  n
• Số chính phương là bình phương của một số nguyên.
a là số chính phương nên a có dạng a  b2 với b là một số nguyên.
2
Khi đó ta có b2  n2  n  b  n2  n  b  n2  n  1 .
• Hai vế của bất đẳng thức không âm, bình phương hai vế ta được một bất đẳng thức
cùng chiều.
2 2
b  n2  n  1  b2  n2  n  1 hay  a  n2  n  1
 n 4  2 n 3  2 n 2  n  7  n 4  2 n 3  3n 2  2 n  1
 n2  n  6  0  n  2n  3  0  3  n  2
Tức là n  3;  2;  1; 0; 1; 2 .
• Với một số nguyên nằm trong một khoảng giá trị nào đó, ta lập bảng xét các giá trị đó.

n 3 2 1 0 1 2
4 3
a  n  2n  2n  n  7 2
49 13 7 7 13 49
Nhận Loại Loại Loại Loại Nhận
Vậy n  3 hoặc n  2 .

Câu 6.

Liên hệ tài liệu word toán sđt và zalo: 039.373.2038 TÀI LIỆU TOÁN HỌC
265
Website:tailieumontoan.com

1). Rõ ràng hai điểm M ; N khác phía đối với đường thẳng PQ .
  180  1 A   180  1 C
Ta có PMQ  B
 ; PNQ
2
 
 D
 , suy ra
2
 
  360  1 A
  PNQ
PMQ
2
 B
 C
 D
 
  180 , nên bốn điểm M ; N ; P ; Q cùng nằm trên một

đường tròn, điều phải chứng minh.


Nhận xét: Bài toán chứng minh bốn điểm cùng thuộc một đường tròn ta chứng minh cho
tứ giác tạo bởi bốn điểm đó là tứ giác nội tiếp.
Nhắc lại kiến thức và phương pháp:
• Tổng ba góc trong một tam giác bằng 180 .
+ Tam giác MAB có:
  MAB
MBA   PMQ
  180  PMQ
  180 MBA
  MAB
.

+ Tam giác NDC có:


  NDC
NCD   CND
  180  PNQ
  180 NCD .
  NDC

• Phân giác của một góc chia góc đó thành hai góc bằng nhau và bằng nửa góc ban đầu.

A;
 nên MAB
+ AM là phân giác của DAB
2

 nên MBA
+ BM là phân giác của ABC  B;
2
1 1   180  1 A
  180 MBA
Suy ra PMQ
2
  MAB
2 2
 B
 .
 

 nên NCD
+ CN là phân giác của DCB C;
2

 nên NDC
+ DN là phân giác của ADC D;
2
1 1 1
  180  C
Suy ra PNQ
2
 D
2
  180  C
2
 D .
 
• Tổng bốn góc trong một tứ giác bằng 360 .
Tứ giác ABCD có A  B C
 D  360
• Tứ giác có tổng hai góc trong đối diện bằng 180 là tứ giác nội tiếp.

Liên hệ tài liệu word toán sđt và zalo: 039.373.2038 TÀI LIỆU TOÁN HỌC
266
Website:tailieumontoan.com

  180  1 A   180  1 C
  PNQ
Tứ giác MPNQ có: PMQ  B
2

 D
  2
 
1     1
 360
2
 2

A  B  C  D  360 .360  360 180  180 Suy ra tứ giác MPNQ là tứ

giác nội tiếp hay bốn điểm M ; P; N ; Q cùng nằm trên một đường tròn (điều phải
chứng minh)
 , nên I P  I Q .
2). MI là đường phân giác của góc PMQ 1 1

 , nên K P  K Q .
NK là đường phân giác của góc PNQ 1 1

Nên I1K1 là trục đối xứng của đường tròn đi qua bốn điểm M ; N ; P; Q , suy ra I1K1 là
đường kính của đường tròn đi qua bốn điểm M ; N ; P; Q .
Tương tự: J1 H1 là đường kính của đường tròn đi qua bốn điểm M ; N ; P; Q .
Vậy I1K1  J1 H1 (điều phải chứng minh).
Nhận xét: Bài toán chứng minh hai đoạn thẳng bằng nhau ta sử dụng các tính chất hình
học chứng minh hai đoạn thẳng đó có độ dài cùng bằng một giá trị nào đó.
Nhắc lại kiến thức và phương pháp:
• Trong một đường tròn, phân giác của một góc nội tiếp chia chia chứa góc đó thành hai
cung bằng nhau. Hai cung bằng nhau thì hai dây cung bằng nhau.
 chắn cung P
+ MI là đường phân giác của góc nội tiếp PMQ  của đường tròn đi qua
NQ


bốn điểm M ; N ; P; Q cắt P tại I1 nên I P  IQ do đó I1 P  I1Q .
NQ 1 1

 chắn cung P
+ NK là đường phân giác của góc nội tiếp PNQ  của đường tròn đi
MQ

qua bốn điểm M ; N ; P; Q cắt P 
tại K1 nên K  Q do đó K1 P  K1Q .
MQ 1
PK 1

• Một điểm cách đều hai đầu của một đoạn thẳng thì nằm trên đường trung trực của
đường thẳng đó.
+ I1 P  I1Q nên I1 nằm trên đường trung trực của PQ ;
+ K1 P  K1Q nên K1 nằm trên đường trung trực của PQ ;
Suy ra I1K1 là đường trung trực của PQ .
• Trong một đường tròn, đường trung trực của một dây là trực đối xứng của đường
tròn.
Đường tròn đi qua bốn điểm M ; N ; P; Q có I1K1 là đường trung trực của dây PQ nên
I1K1 là trực đối xứng của đường tròn đi qua bốn điểm M ; N ; P ; Q .
• Trục đối xứng của một đường tròn là đường thẳng chứa một đường kính.
+ I1K1 là trực đối xứng của đường tròn đi qua bốn điểm M ; N ; P; Q nên I1K1 là đường
kính của đường tròn đi qua bốn điểm M ; N ; P; Q ;
+ Hoàn toàn tương tự ta có J1 H1 là đường kính của đường tròn đi qua bốn điểm
M ; N ; P; Q ;
Suy ra I1K1  J1H1 (điều phải chứng minh).

Liên hệ tài liệu word toán sđt và zalo: 039.373.2038 TÀI LIỆU TOÁN HỌC

You might also like